You are on page 1of 378

2000 FELADAT

AZ ELEMI MATEMATIKA KÖRÉBİL


Name: Jozsef Kolompar
Order: 26985340

Köszönjük a vásárlást a szerző és a kiadó, valamint a terjesztő nevében is.


Vásárlásával támogatta, hogy Magyarországon az elektronikus könyvkiadás
fejlődni tudjon, a digitális kereskedelemben kapható könyvek választéka
egyre szélesebb legyen. Köszönjük, és reméljük webáruházunkban
hamarosan viszontlátjuk.
Tehetségek példatára sorozat

A sorozat kötetei:
Pogáts Ferenc: Varga Tamás matematikai versenyek 1.
Pogáts Ferenc: Varga Tamás matematikai versenyek 2.
Pogáts Ferenc – Fazakas Tünde: Varga Tamás matematikai versenyek 3.
(Szerkesztı kollektíva): Bergengóc példatár 1.
(Szerkesztı kollektíva): Bergengóc példatár 2.
RÓKA SÁNDOR

2000 FELADAT
AZ ELEMI MATEMATIKA
KÖRÉBİL
Hatodik kiadás

Budapest, 2010
Kedves Olvasó!
Önre gondoltunk, amikor a könyv elıkészítésén munkálkodtunk.
Kapcsolatunkat szorosabbra főzhetjük, ha a kiadó honlapján, a www.typotex.hu
címen feliratkozik hírlevelünkre, melybıl értesülhet
új kiadványainkról, akcióinkról, programjainkról.
A honlapon megismerkedhet teljes kínálatunkkal, egyes könyveinknél pedig új
fejezeteket, bibliográfiát, hivatkozásokat találhat, illetve az esetlegesen
elıforduló hibák jegyzékét is letöltheti.
Kiadványaink egy része e-könyvként (is) kapható: www.interkonyv.hu

Kiadja a Typotex kiadó, az 1795-ben alapított Magyar Könyvkiadók és


Könyvterjesztık Egyesülésének tagja.
Felelıs kiadó: Votisky Zsuzsa
Felelıs szerkesztı: Fried Katalin
Tördelés: Könyvmővek Bt.
Borítóterv: Tóth Norbert
Terjedelem: 26,5 (A/5) ív
Készült a G-Print nyomdában
Felelıs vezetı: Wilpert Gábor
Bevezető

Ez a feladatgyűjtemény sokéves munka eredménye. 1992-ben jelent meg az 1000


feladat az elemi matematika köréből c. könyvem, melynek bővített, átdolgozott kiadá-
sát, az 1500 feladat . . . -ot 1996-ban adta ki a TypoTEX. Az azóta eltelt évek alatt a
feladatgyűjteményen elvégeztem a szükségesnek gondolt javításokat, kiegészítéseket, és
új fejezeteket állítottam össze. Remélem, hogy előnyére vált a könyvnek ez az éveken
át tartó bővítés és a korábbi feladatsorok csiszolgatása. Most vége az átdolgozásoknak;
már évekkel ezelőtt is úgy terveztem, hogy a 2000 feladat az elemi matematika köréből
lesz ennek a feladatgyűjteménynek az utolsó változata. Újabb átdolgozás, bővítés nem
várható.
A feladatokat zömmel a hazai és az orosz versenyfeladatok közül gyűjtöttem, fel-
használtam a KöMaL (Középiskolai Matematikai és Fizikai Lapok) és a Kvant feladatait,
és további sokféle más forrásra is támaszkodtam. A feladatgyűjtemény összeállításánál
arra törekedtem, hogy szép és érdekes feladatok kerüljenek bemutatásra, egy-egy téma-
kört minél gazdagabban tárjunk az olvasó elé; továbbá szándékom volt, hogy a feladat-
gyűjtemény segítse a versenyekre való felkészülést.
Ezt a gyűjteményt használhatják 14–18 éves diákok a tanulmányi versenyekre és a
KöMaL pontversenyére való felkészülésben. Megjegyezve azért, hogy noha vannak geo-
metriai feladatok, de a hagyományos geometriai témák hiányoznak a feladatgyűjtemény-
ből. A középiskolás korosztályt tanító tanárok számára szakköri feladatgyűjteményként
szolgálhat a könyv: egy-egy témakör feldolgozásához bőséges példaanyagot találunk. A
2000 feladat . . . használható segédkönyvként is a tanárjelöltek képzésében, az Elemi
matematika szemináriumokon.
A feladatokat témakörök szerint csoportosítottam, s legtöbbjükhöz megoldást adtam
(esetleg csak a megoldás vázlatát, vagy ötletet, útmutatást). A feladat sorszáma mel-
lett zárójelben feltüntetett szám (1-től 7-ig) a feladat nehézségét mutatja: kisebb szám
könnyebb feladatot jelez. A nehézségi fokot egyértelműen megállapítani nyilván nem
lehet, de a megadott skálázás némi segítséget adhat a feladat megoldójának. Érdemes
egy-egy fejezet feldolgozásánál a feladatok sorrendjét követni, mert a korábban szerep-
lő feladatok többnyire ötletet adnak és segítséget nyújtanak a megoldáshoz. Csábító lehet
a feladat megismerése után rögtön elolvasni a megoldáshoz adott útmutatást, de ez némi
akaraterővel halogatható. A feladattal barátkozni kell, néha napokig gondolkodni rajta.
Közelebb juthatunk a feladat megoldásához, ha megpróbáljuk az állítást élesíteni vagy
a feltételek változtatásával megfigyeljük, hogyan változik az állítás.
6 Bevezető

A problémamegoldásban jártas embereket az különbözteti meg a náluk kevésbé si-


keres kezdőktől, hogy az előbbieknek nagyobb a tudásuk. Az orvosnak ismernie kell az
anatómiát, a futballistának a saját sportágát, a szakácsnak az ételeket. Becslések szerint
még a legtehetségesebb sakkozóból sem válhat nagymester addig, amíg legalább tíz évet
el nem tölt sakkfeladványok megoldásával. Minden problémamegoldásnak van megta-
nulható része. Pólya György szerint: „A problémamegoldás csakúgy gyakorlat kérdése,
mint az úszás, sízés vagy zongorázás. Megtanulni is csak utánzás és gyakorlás útján
lehet . . . Aki úszni akar tanulni, annak vízbe kell ugrania, aki problémákat megol-
dani akar tanulni, annak problémák megoldását kell gyakorolnia.” Ahhoz, hogy valaki
matematika versenyeken jó eredményeket érjen el, bizony sok munka szükséges. Nem
elég a tehetség, érzék; a feladatmegoldást, matematikai gondolkodást tanulni is kell. A
versenyekre lehet készülni, a versenyekre fel lehet készülni. Ez a készülés, ez az edzés
feladatmegoldást jelent.
A 2000 feladat . . . lehetőséget biztosít a problémák megoldásának gyakorlására.
Aki egyszer megérezte az eredményes problémamegoldás örömét, egy életre megőrzi
azt; és amit ez kifejleszt, a hajlékony gondolkodás, az az embert egész életén végigkísérő
értékes képesség.
A feladatok megismerésekor a figyelmes olvasó megtapasztalhatja a matematika
örömét a meglepő tételekben, szép okoskodásokban, logikus gondolatmenetekben. Egy
bizonyítás nem ritkán izgalmas, érdekfeszítő olvasmány.
Hálás köszönettel tartozom a könyv lektorának, Hódi Endrének, aki alapos, körülte-
kintő munkájával nagyon sok segítséget nyújtott. Az ő ösztönzésére jelentősen csökkent
a megoldás nélküli feladatok száma; segített a hibák felderítésében; útmutatásai alapján
igyekeztem a megoldásokat minél érthetőbben, világosabban leírni; a feladatok megfo-
galmazásában pedig, ahol kellett, figyelmeztetett a pontosságra. A lektor lelkiismeretes
és gondos munkát végzett; nagyon sokat tett azért, hogy ez a könyv minél jobb legyen.
Feladatok

1. Fejtörők

1. [3] Az ábrán látható autóbusznak melyik az eleje?

2. [3] Az ábrán látható három egyfülű nyulat vágjuk ki papírból, majd helyezzük el a
nyulakat úgy, hogy mindegyik nyúlnak két füle legyen.

3. [3] Az asztalra letettünk 10 db egyforma pénzérmét az


ábrán látható módon. Ezekből 3 érmét helyezzünk
máshová úgy, hogy egy más helyzetű, az eredetivel
egybevágó szabályos háromszöget kapjunk.

4. [3] Rajzoljunk az ábrába a ceruza felemelése nélkül olyan


töröttvonalat úgy, amely négy, végpontjaival sorban
egymáshoz csatlakozó egyenesszakaszból áll, és mind
a kilenc ponton áthalad.
8 Feladatok

5. [2] Egy vadász medvenyomokat követve 1 km-t haladt déli irányban, majd 1 km-t
ment keletre, végül 1 km-t északra, és ekkor visszaért oda, ahonnan elindult.
Milyen színű a medve?
6. [2] A tengeren négy hajó halad együtt, közel egymáshoz: bármely két hajó távol-
sága 3 km. A hajók között van teherszállító, olajszállító és utasszállító hajó.
Milyen hajó a negyedik?
7. [3] Egy hajó hosszának, árbócmagasságának, a hajó kapitánya és kisfia életkorának
(a számok egész számok) szorzata 323 935. Hány éves a kapitány?
8. [3] Az ábrán látható nyolcszöget – mint a második ábra mutatja – fel lehet osztani
négy egybevágó részre. És hogyan lehet felosztani öt egybevágó részre?

9. [3] A felírt „egyenlőségben” egy számjegyet helyez-


zünk máshová úgy, hogy igaz egyenlőséghez jus- 101 − 102 = 1
sunk.
10. [5] Egy alkalommal Félix kiment a táblához, és felírta a következőt: 1995 +
+146 = 210 + 1117. Ez igaz egyenlőséggé változtatható úgy, hogy két számje-
gyet letörlünk, majd máshová írjuk. Hogyan? (Megengedett az, hogy egy szám-
jegyet két számjegy közé „beékeljünk”.)
11. [4] Tegyük igazzá a 76 = 24 egyenlőséget úgy, hogy a számjegyeket szabadon
mozgathatjuk.
12. [3] Elkészítettük az alábbi számkártyákat, és azokat az ábra
szerint helyeztük el. Mozgassunk el két számkártyát úgy, 1 3
hogy a két összeg egyenlő legyen.
2 4
7 5
+ 9 + 8
19 20
1. Fejtörők 9

13. [5] Az ábrát gyufaszálakból készítettük. He-


lyezzünk át egy szál gyufát úgy, hogy
igaz egyenlőséget kapjunk.

14. [3] Milyen számot kell írni az üres mezőkbe?

12 15 30 17 26 22
24 20 25 20 13 28 18
24 20 30 20 26 30
15. [2] Mi a sorozat következő eleme?

16. [2] Mi lehet a sorozat következő eleme?


a) 2, 3, 5, 7, 11, 13, 17, 19, ?
b) 1, 2, 6, 24, 120, 720, ?
c) 1, 1, 2, 3, 5, 8, 13, 21, 34, 55, ?
d) 100, 101, 103, 107, 115, 122, ?
e) 1, 4, 9, 16, 25, 36, ?
f) 77, 49, 36, 18, ?
17. [4] Mi lehet a sorozat következő eleme?
1000, 22, 20, 13, 12, . . .
18. [4] Fedezzük fel a szabályosságot a következő sorozatban: 111, 213, 141, 516, 171,
819, 202, 122, . . .
19. [5] Mi a sorozat következő eleme?
1, 11, 21, 1211, 111221, 312211, 13112221, . . .
20. [4] Vajon mi lesz a 461, 552, 663, 794, 846, . . . sorozat következő eleme?
21. [4] Vajon hogyan folytassuk a betűk sorozatát: NEKHNÖH . . . ?
22. [4] Mi lehet a három hiányzó betű: B, H , F , V , K, ?, ?, ?.
10 Feladatok

23. [4] Melyik szó következik a szavak sorozatában: segély, otthon, világ, birodalom,
hadoszlop, érzék, . . .
a) víz, b) zene, c) üdvözlet, d) mennyország.
24. [7] A számok között mindenhol ugyanaz az egyszerű összefüggés fedezhető fel.
Vajon mi ez az összefüggés?
0 0 0 0 0 0 0
1 1 1 1 1 1 1 1
1 2 5 2 1 3 2 2
1 9 9 1 2 5 3
1 4 16 4 1 3 7 7
1 3 7 7 3 1 4 16
1 2 5 3 5 2 1 9 9
1 3 2 2 3 1 2 5
1 1 1 1 1 1 1 1
0 0 0 0 0 0 0 0

25. [2] Írjunk a 7 ∗ ∗ ∗ ∗ ∗ ∗ 9 tízes számrendszerbeli számban a ∗-ok helyé-


re számjegyeket úgy, hogy bármely három szomszédos számjegy összege 20
legyen.
26. [3] A 231213 hatjegyű számban a két 1-es között 1 számjegy, a két 2-es között 2
számjegy, a két 3-as között 3 számjegy van. Írjunk fel olyan nyolcjegyű számot,
amelyben 2 db 1, 2 db 2, 2 db 3 és 2 db 4-es számjegy van, és a két 1-es között
1 számjegy, a két 2-es között 2 számjegy, a két 3-as között 3 számjegy, a két
4-es között 4 számjegy legyen.
27. [3] Írjuk a 0, 0, 1, 1, 2, 2, 3, 3 számjegyeket egymás után olyan sorrendbe, hogy a
2 db 0 között 0, a 2 db 1-es között 1, a 2 db 2-es között 2, a 2 db 3-as között
3 számjegy legyen.
28. [3] A kis négyzetekbe írjuk be az 1, 2, 3, 7, 8, 9, 13,
14, 15 számokat úgy, hogy mindegyik sorban,
mindegyik oszlopban és a két átlóban ugyan-
annyi legyen a számok összege.
1. Fejtörők 11

29. [3] Írjuk be a táblázat mezőibe az 1, 2, 3, 4,


5, 6, 7, 8, 9 számokat úgy, hogy minde-
gyik sorban és mindegyik oszlopban annyi 6
legyen a számok összege, mint amennyit a
sor mellett, ill. az oszlop alatt levő szám 16
mutat.

23
14 12 19
30. [3] Írjuk be a táblázat mezőibe az 1, 2, 3, 4,
5, 6, 7, 8, 9 számokat úgy, hogy a számok
szorzata minden sorban egyenlő legyen a 20
sor mellé írt számmal, s hasonlóan minden
oszlopban a számok szorzata legyen egyen- 108
lő az oszlop alá írt számmal.
168
42 80 108

31. [3] Írjunk a téglalapokba különböző pozitív


egészeket úgy, hogy mindegyik az alat-
ta levő két szám összege legyen (termé-
szetesen ez az alsó sorra nem vonatko-
zik), és a legfelső mezőben minél ki-
sebb szám álljon.

32. [3] Válasszunk ki a 4 × 4-es táblázat mezőiből tízet


úgy, hogy mindegyik sorban és mindegyik oszlop-
ban páros legyen a kiválasztott mezők száma.
12 Feladatok

33. [3] A kör kerületét 12 egyenlő részre osztottuk. Erre a 12 helyre írjunk nullától kü-
lönböző egész számokat úgy, hogy bármelyiktől indulva, minden harmadik szá-
mot véve, azok összege nulla legyen, és bárhonnan indulva, minden negyedik
számot véve, azok összege is nulla legyen. (Tehát, ha kiválasztunk egy szabá-
lyos sokszöget, melynek csúcsai a 12 pont közül valók, ott a csúcsokban levő
számok összege nulla.)

34. [3] A ∗ jelek helyére írjuk be a 0, 1, 2, 3, 4, 5, 6, 7, 8, 9



∗∗
számjegyeket úgy, hogy helyes legyen az összeadás.
(Minden számjegy egyszer szerepel egy összegben,

+ ∗∗∗
és az összegben szereplő számok egyike sem kezdő-
dik nullával.) Keressünk több megoldást.

∗∗∗∗
35. [3] A ∗ jelek helyére írjuk be az 1, 2, 3, 4, 5, 6, 7, 8, 9
∗∗∗
+ ∗∗∗
számjegyeket úgy, hogy helyes legyen az összeadás.
(Minden számjegy egyszer szerepel egy összegben.)

∗∗∗
Keressünk több megoldást.

36. [4] Válasszunk három számjegyet úgy, hogy azok különböző sorrendjével fel tud-
junk írni három olyan háromjegyű számot, melyek közül kettőnek az összege
lesz a harmadik szám.
37. [4] Válasszunk négy számjegyet úgy, hogy azok különböző sorrendjével fel tudjunk
írni három olyan négyjegyű számot, melyek közül kettőnek az összege lesz a
harmadik szám.
38. [4] Adjunk meg három olyan 9-jegyű számot, melyek közül az egyik a másik kettő
összege, és amely számok mindegyikét az 1, 2, 3, 4, 5, 6, 7, 8, 9 számjegyek
valamilyen sorrendjével írjuk fel.
39. [3] Határozzuk meg a törtkifejezés értékét, ha az azonos betűk azonos, a különböző
betűk különböző számjegyeket jelölnek.

C·S·O·D·Á·L·A·T·O·S
=?
B·A·L·A·T·O·N
1. Fejtörők 13

40. [4] Az alábbi számításokban a számjegyeket betűkkel helyettesítettük; azonos szám-


jegyeket azonos betűkkel, különböző számjegyeket különböző betűkkel. Milyen
számokat rejtenek a betűk?

a) b) c)

S E ND
S E ND S AVE MORE
+ MORE + MORE + G OL D
MO NE Y MO NE Y MO NE Y
d) e) f)

F ORTY
USA THRE E T EN
+USSR +F OUR + T EN
PE ACE SE VE N SI XTY
g) h) i)

SOK CB AB L A P K E P
SOK CB KAR
KI S CAA E K L
+ KOS ADEA P A O}
OKOS
K U L L O}
41. [4] Az alábbi szorzásokban csak néhány számjegyet ismerünk. Határozzuk meg az
ismeretlen számjegyeket.
a) b) c)

aa aa aa aa aa 7a
aa aaa aa
a7 aa aaa
aaaa 9aaa aaa
14 Feladatok

42. [5] Az alábbi szorzásokban mindegyik számjegy prímszám. Határozzuk meg az


ismeretlen számjegyeket.
a) b)
aaa aa aaa aaa
aaaa aaaa
aaaa aaaa
aaaaa aaaa
aaaaaa
43. [5] Az alábbi szorzásokban az a betű páros, a b betű páratlan számjegyet jelöl.
Határozzuk meg az ismeretlen számjegyeket.
a) b) c)
aaa bbb bbb aaa aab bb
aba aba abb
aaaa aaba abab
abba abba bbbbb
baabba baaba
44. [3] A 2 : 3 : 4 : 5 : 6 = 5 kifejezésbe írjunk zárójeleket úgy, hogy az így kijelölt
osztásokat elvégezve a felírt eredményt kapjuk.
45. [3] Az 1 : 2 : 3 : 4 : 5 : 6 : 7 : 8 : 9 : 10 = 7 kifejezésbe írjunk zárójeleket úgy,
hogy az így kijelölt osztásokat elvégezve a felírt eredményt kapjuk.
46. [2] Írjunk az 1 2 3 4 5 6 7 8 9 = 100 kifejezésben a számjegyek közé +
és − jeleket vagy semmit úgy, hogy igaz legyen az egyenlőség. Egy megoldás
a következő: 123 − 4 − 5 − 6 − 7 +8 −9 = 100. Keressünk minél több megoldást.
47. [2] Írjunk a 9 8 7 6 5 4 3 2 1 = 100 kifejezésben a számjegyek közé
+ és − jeleket vagy semmit úgy, hogy igaz legyen az egyenlőség. Keressünk
minél több megoldást.
48. [2] Írjunk a 9 8 7 6 5 4 3 2 1 = 99 kifejezésben a számjegyek közé +
jeleket vagy semmit úgy, hogy igaz legyen az egyenlőség. Keressünk minél
több megoldást.
49. [2] Az 1, 2, 3, . . . , 15 számokat osszuk szét öt csoportba úgy, hogy mindegyik cso-
portban három szám legyen, s a három szám közül a legnagyobb egyenlő legyen
a másik kettő összegével.
50. [3] Az 1, 2, 3, . . . , 13 számokat osszuk szét három csoportba úgy, hogy egyik cso-
portban se szerepeljen két olyan szám, melyek összege is a csoportban van, és
ne legyen ugyanabban a csoportban egy szám és a kétszerese.
1. Fejtörők 15

51. [3] A 10, 11, 12, . . . , 49 számokat osszuk szét két csoportba úgy, hogy egyik cso-
portban se szerepeljen két olyan szám, melyek összege is a csoportban van, és
ne legyen ugyanabban a csoportban egy szám és a kétszerese.
52. [3] Válasszunk ki az 1, 2, 3, . . . , 100 számokból minél többet úgy, hogy a kiválasz-
tott számok között ne legyen két olyan, melyek egyike osztója a másiknak.
53. [3] Válasszunk ki az 1, 2, 3, . . . , 100 számokból minél többet úgy, hogy a kiválasz-
tott számok között ne legyen két olyan, melyek összege is a kiválasztott számok
között van, és a kiválasztott számok között ne legyen olyan sem, melynek a két-
szerese is a kiválasztottak között van.
54. [3] Egy zár, melyen három nyomógomb van (1-es, 2-es, 3-as), akkor nyílik, ha
a gombokat egy előírt sorrendben nyomjuk meg közvetlenül egymás után. (A
megfelelő három gombnyomást esetlegesen megelőző gombnyomások soroza-
tának nincs hatása a zár szerkezetére.) A zár biztosan kinyílik például a követ-
kező sorozatra: 123 132 213 231 312 321. Adjunk meg olyan 9 gombnyomásból
álló sorozatot, mely biztosan kinyitja a zárat.
55. [2] Választunk egy háromjegyű számot és elosztjuk számjegyeinek összegével. Az
így kapott hányados melyik háromjegyű szám esetén lesz a legkisebb, és mikor
lesz a legnagyobb?
56. [1] Melyik az a legkisebb természetes szám, amelyben a számjegyek összege 1994?
57. [2] Melyik az a legkisebb természetes szám, amelyben a számjegyek szorzata 200?
58. [2] Melyik az a legnagyobb négyjegyű szám, amelyben nincs két azonos számjegy,
és a számjegyek szorzata 216?
59. [3] Adjunk meg olyan pozitív egész számot, mely felírható 3 egymást követő egész
szorzataként is, és felírható 6 egymást követő egész szorzataként is.
60. [3] Egy futóversenyen 12 rajtszámmal jelzett versenyző indult. Mi volt a beérkezés
sorrendje, ha a rajtszám és a helyezési szám szorzata mindig 1-gyel nagyobb
egy 13-mal osztható számnál?
61. [3] Az 1, 2, 3, 4, 5, 6 számokat felhasználva (egy szám többször is előfordulhat)
egy négyjegyű számot írtunk fel. Többen megpróbálják kitalálni ezt a számot.
Az első tipp: 4215. Két számot eltalált, de csak egy van jó helyi értéken. A
második tipp: 2365. Ismét két számot talált el, de csak az egyik van jó helyi
értéken. A harmadik tipp: 5525. Itt még a számokat sem találta el. Mi lehet a
keresett szám?
62. [3] Rajzoljunk le 8 szakaszt úgy, hogy mindegyik három másikat metsszen.
63. [3] Egy háromszög és egy négyszög legfeljebb hány pontban metszi egymást, ha
nincs közös oldalegyenesük?
16 Feladatok

64. [3] Vegyünk fel 6 pontot a síkon, és kössünk össze azokból néhányat szakaszokkal
úgy, hogy mindegyik pontból 3 szakasz induljon, és a szakaszok ne keresztez-
zék egymást.
65. [3] Vegyünk fel 6 pontot a síkon, és kössünk össze azokból néhányat szakaszokkal
úgy, hogy mindegyik pontból 4 szakasz induljon, és a szakaszok ne keresztez-
zék egymást.
66. [3] Vegyünk fel 7 pontot a síkon úgy, hogy ha azokat páronként összekötjük egye-
nesekkel, akkor összesen 14 különböző egyenest kapjunk.
67. [4] Adott néhány egyforma méretű tégla. Ezek segítségével csak egy vonalzót hasz-
nálva, hogyan lehetne megmérni a tégla testátlóját?
68. [4] Tudor egy 200-jegyű számra gondolt, amelynek számjegyösszegét megsúgta
Vidornak. Vidor kiszámította a neki megsúgott szám számjegyösszegét, s ezt
megsúgta Morgónak. Morgó meghatározta ezen szám számjegyösszegét, ami
kétjegyű szám volt, s ezt megsúgta Hapcinak. Hapci is meghatározta a számje-
gyösszeget, s ezt megsúgta Kukának.
Milyen számot súgott Vidor Morgónak? Milyen számot súgott Hapci Kukának?
69. [3] Két pohár közül az egyikben 1 dl bor, a másikban ugyanennyi víz van. Egy ka-
nál bort átöntünk a vízbe, ezt jól megkavarjuk, majd innen egy kanál folyadékot
(víz-bor keveréket) átöntünk a borba. A vízben lesz-e több bor, vagy a borban
lesz-e több víz?
70. [3] Igaz-e, hogy nagyapáim dédapjai ugyanazok a személyek, mint dédapáim nagy-
apjai?
71. [3] A mesebeli róka a következő egyezséget kötötte egy legénnyel: ahányszor át-
megy a hídon, a róka mindannyiszor megkétszerezi a pénzét, s ebből a pénzből
24 krajcár vámot kell fizetnie. A legény azt hitte, hogy ily módon sok pénzhez
juthat, de nagyot csalódott. Amikor ugyanis harmadszor ment át a hídon, és ki-
fizette a vámot, nem maradt egy krajcárja sem. Mennyi pénze volt a legénynek,
mikor az egyezséget megkötötte?
72. [3] Öt játékos megegyezett, hogy a vesztes minden játszma után megkétszerezi a
többi pénzét. Összesen öt játszmát játszottak, mindegyik játékos egyszer ve-
szített. A játék végén mindegyiküknek 128 F t-ja volt. Mennyi pénze volt a
játékosoknak külön-külön a játék megkezdése előtt?
73. [4] Egy szobában 10 szék van sorban egymás mellett. A székek kezdetben üresek.
Időnként valaki bejön a szobába, leül egy üres székre, és ugyanekkor egyik
szomszédja (ha van neki) föláll és kimegy. Legfeljebb hány szék lehet foglalt
egyszerre a szobában?
1. Fejtörők 17

74. [3] Három ládikó mindegyikében 2-2 golyó van: fehér-fehér, piros-piros, fehér-
piros, és mindegyik ládán felirat: F F , P P , F P , ami a ládában levő golyók
színét jelzi. Azonban mindegyik felirat hamis. Valamelyik – általunk választott
– ládikóból kivehetünk egy golyót, s ezek után meg kell mondani, hogy az egyes
ládikókban milyen golyók vannak. Hogyan lehet ezt megtenni?
75. [3] Egy papírlapon az alábbi öt állítás olvasható.
Ezen a lapon 1 állítás hamis.
Ezen a lapon 2 állítás hamis.
Ezen a lapon 3 állítás hamis.
Ezen a lapon 4 állítás hamis.
Ezen a lapon 5 állítás hamis.
Az állítások közül melyik igaz, melyik hamis?
76. [3] Egy papírlapon az alábbi öt állítás olvasható.
Ezen a lapon legfeljebb 1 állítás igaz.
Ezen a lapon legfeljebb 2 állítás igaz.
Ezen a lapon legfeljebb 3 állítás igaz.
Ezen a lapon legfeljebb 4 állítás igaz.
Ezen a lapon legfeljebb 5 állítás igaz.
Az állítások közül melyik igaz, melyik hamis?
[3]
77. A három ládikón levő állításból legfeljebb egy igaz.

Hol van az arany?


[3]
78. A három ládikón levő állítások között van igaz és van hamis állítás.

Hol van az arany?


18 Feladatok

79. [2] Az asztalon kártyalapok vannak, melyek egyik oldalán betű, másik oldalán szám
áll. Mi az előttünk levő négy kártyalapon az A, B, 1, 2 felírást látjuk. Legkeve-
sebb hány lapot kell megfordítani, ha el akarjuk dönteni, hogy igaz-e a követ-
kező állítás: „Magánhangzó mögött páros szám áll.”?
80. [3] Az alábbi feladatokban szereplők mindegyike vagy az igazmondók, vagy a ha-
zugok szektájába tartozik.
a) Egy szobában ketten vannak, A és B. Azt mondja A: „Legalább egyikünk
hazug.” Ki melyik szektába tartozik?
b) Egy szobában ketten vannak, A és B. Azt mondja A: „Én hazug vagyok, de
B nem az.” Ki melyik szektába tartozik?
c) Egy szobában hárman vannak, A, B és C. A következők hangzanak el.
A: „Mindhárman hazugok vagyunk.”
B: „Pontosan egy igaz van köztünk.”
Ki melyik szektába tartozik?
d) Egy szobában hárman vannak, A, B és C. A következők hangzanak el.
A: „Mindhárman hazugok vagyunk.”
B: „Pontosan egy hazug van köztünk.”
C melyik szektába tartozik?
81. [3] Egy vándor egy útelágazáshoz ér, egyik út a sivatagba, a másik Mekkába vezet.
Az útelágazásnál ketten álldogálnak, egy igazmondó és egy hazug. Mit kell
kérdeznie a vándornak, hogy megtudja a Mekkába vezető utat, ha csak egyetlen
kérdést tehet fel, és nem tudja, hogy igazmondóval vagy hazuggal beszél-e?
82. [3] Két szomszédos város egyikében igazmondók, a másikban hazugok laknak. A
városlakók természetesen eljárnak látogatóba a szomszédos városba. Mit kér-
dezzen egy turista, hogy megtudja, melyik városban tartózkodik (az igazmon-
dókéban vagy a hazugokéban), ha csak egy kérdést tehet fel, és nem tudja, hogy
igazmondót vagy hazugot kérdez?
83. [3] Hárman beszélgetnek: A, B és C.
A azt mondja: „B hazudik.”
B azt mondja: „C hazudik.”
C azt mondja: „A és B hazudik.”
Ki mond igazat és ki hazudik?
84. [4] Olvassuk el figyelmesen a következő 5 állítást:
(A): A (B) állítás igaz.
(B): Az (A), (B), (C), (D), (E) állítások közül legfeljebb az egyik igaz.
(C): Az (A), (B), (C), (D), (E) állítások mindegyike igaz.
(D):
(E):
2. Páros vagy páratlan? 19

A (D) és (E) állításokat varázstintával írtuk, láthatatlanok azok számára, akik


nem mindig mondanak igazat. Állapítsuk meg, hogy az (A), (B), (C), (D), (E)
állítások közül melyik igaz, melyik hamis.

2. Páros vagy páratlan?

85. [1] El lehet-e osztani 100 nyulat 5 gyerek között úgy, hogy mindegyik gyereknek
páratlan számú nyúl jusson?
86. [1] Meg lehet-e adni négy egész számot úgy, hogy összegük és szorzatuk is páratlan
szám legyen?
87. [1] Három egész szám összege 1994. Lehet-e 1 a három szám szorzatának utolsó
jegye?
88. [2] Szét lehet-e osztani az 1, 2, 3, . . . , 1994 számokat két csoportba úgy, hogy
mindegyik csoportban páratlan legyen a számok összege?
89. [2] Adjunk meg néhány egész számot úgy, hogy szorzatuk és összegük is 9 legyen.
90. [2] Lehet-e 9 egész szám összege is, szorzata is 9?
91. [3] Lehet-e 10 egész szám összege is, szorzata is 10?
92. [2] Lehet-e 8 egész szám összege is, szorzata is 8?
93. [2] Lehet-e 9 egész szám szorzata 9, összege pedig 0?
94. [3] Lehet-e 10 egész szám szorzata 10, összege pedig 0?
95. [2] Lehet-e 8 egész szám szorzata 8, összege pedig 0?
96. [3] Az 1, 2, 3, . . . , 9 számok valamilyen más sorrendjét a1 , a2 , a3 , . . . , a9 jelöli.
Lehet-e páratlan az (a1 − 1) · (a2 − 2) · (a3 − 3) · . . . · (a9 − 9) szorzat?
97. [3] Az a1 , a2 , a3 , . . . , a9 számok valamilyen más sorrendjét b1 , b2 , b3 , . . . , b9
jelöli. Lehet-e páratlan az (a1 − b1 ) · (a2 − b2 ) · (a3 − b3 ) · . . . · (a9 − b9 ) szorzat?
98. [3] Egy négyzet alapú tortát összesen 13 vízszintes és függőleges irányú vágással
szeletekre vágtunk. Lehet-e páratlan szám a kapott szeletek száma?
99. [2] A ±1 ± 3 ± 5 ± 7 ± 9 ± 11 kifejezésben megválaszthatók-e úgy a + és − jelek,
hogy ezt követően kiszámolva az összeget, a végeredmény 13 legyen?
100. [2] A ±1 ± 2 ± 3 ± · · · ± 100 kifejezésben megválaszthatók-e úgy a + és − jelek,
hogy ezt követően kiszámolva az összeget, a végeredmény 1991 legyen?
101. [3] Miért nem lehet 100 különböző páratlan szám reciprokának összege 1? Lehet-e
100 különböző egész szám reciprokának összege 1?
20 Feladatok

102. [3] Lehet-e egész szám az első 100 prímszám reciprokának összege?
103. [3] Összeadtunk három egymást követő egész számot, majd összeadtuk a következő
három számot is. Lehet-e 111 111 111 az így kapott két szám szorzata?
Megjegyzés: A párosság vizsgálatával oldható meg a Diofantoszi egyenletek, az Invariáns tulaj-
donságok, a Feladatok a sakktáblán és a Különböző kombinatorikai feladatok c. fejezetek több
feladata.

3. Párbaállítás

104. [1] Számoljuk ki a következő kifejezések értékét.


a) 1 + 3 + 5 + · · · + 95 + 97 + 99 =?
1 + 2 + 3 + · · · + 99 + 100 + 101
b) =?
1 − 2 + 3 − · · · + 99 − 100 + 101
105. [3] Mennyi az
a) ±1 ± 2 ± 3 ± 4 ± · · · ± 1993, illetve

b) ±12 ± 22 ± 32 ± 42 ± · · · ± 19932 összeg legkisebb pozitív értéke? (A ± jel


helyére a megfelelően választott + és − jelek valamelyikét írjuk be.)
106. [3] Mivel egyenlő az 1, 2, 3, . . . , 998, 999, 1000 számok számjegyeinek összege?
107. [1] Osszuk szét az 1, 2, 3, . . . , 100 számokat
a) két, egyaránt ötven számból álló csoportba
b) öt, egyaránt húsz-húsz számból álló csoportba
úgy, hogy mindegyik csoportban ugyanannyi legyen a számok összege.
108. [2] Az autóbuszjegyen 4 vagy 5 lyuk lyukasztásával lehet többféle lyukkombinációt
megvalósítani?
109. [2] Hány részre osztják a teret a kocka lapsíkjai?
110. [3] Hány részre osztják a teret a tetraéder lapsíkjai?
111. [3] Szabályos tetraéder minden élén át síkot fektetünk, mely két egybevágó részre
osztja a tetraédert. Hány részre bontottuk fel így a tetraédert?
112. [2] 1992 asztaliteniszező között kieséses versenyt szervezünk. Minden forduló után
mindegyik párból a győztes jut tovább. Ha egy fordulóban valamelyik játékos-
nak nem jut ellenfél, úgy ő automatikusan (erőnyerőként) tovább jut. Hány mér-
kőzést játszanak le, míg megtalálják a legjobb játékost?
3. Párbaállítás 21

113. [2] Egy 6 × 10-es négyzetrácsos papírt rácsegyenesek mentén két részre vágunk,
majd az egyik darabot ismét csak rácsegyenesek mentén két részre vágjuk és így
tovább: minden alkalommal valamelyik darabot két részre vágjuk. Legkevesebb
hány vágásra van szükség, amíg a papirost 1 × 1-es négyzetekre daraboljuk? És
legfeljebb?
114. [3] Mutassuk meg, hogy egy természetes szám (pozitív) osztóinak száma pontosan
akkor páratlan, ha a szám négyzetszám.
115. [2] Miért nem lehet a 7 777 777 szám osztóinak száma 1991?
116. [3] A szultán születésnapján néhány rabot szabadon akar bocsátani. A 100 cellás
börtönben 100 börtönőr van. Az 1. őr minden ajtót kinyit. A 2. őr minden 2.
ajtót bezár. A 3. őr minden 3. ajtót kinyit, ha zárva volt, s bezár, ha nyitva volt.
Hasonlóan nyit-zár a többi őr is. Mely cellák ajtaja marad nyitva?
117. [3] Az n számnak 100 osztója van. Szorozzuk össze a szám osztóit! Mi lesz a
végeredmény?
118. [5] Mely n-re lesz n összes pozitív osztójának szorzata 2120 · 360 · 590 ?
119. [4] Az n természetes számot tökéletesnek nevezzük, ha egyenlő a nála kisebb pozi-
tív osztóinak összegével (ilyen pl. a 6, a 28). Számoljuk ki az n tökéletes szám
pozitív osztói reciprokának összegét.
120. [4] Egy n természetes szám pozitív osztóinak összegét elosztottuk az osztók recip-
rokainak összegével. Igaz-e, hogy a kapott hányados értéke n?
[4] √
121. Az n összetett szám legkisebb prímosztója p. Mutassuk meg, hogy p ≤ n.
n
122. [4] Az n szám legkisebb prímosztója p, és összetett szám. Mutassuk meg, hogy
√ p
p ≤ 3 n.
123. [4] Mutassuk
√ meg, hogy az n természetes szám pozitív osztóinak száma kisebb,
mint 2 n.
124. [4] Mutassuk meg, hogy egy páros szám páros osztóinak összege nagyobb, mint a
páratlan osztók összege.
125. [4] Mutassuk meg, hogy tetszőleges n > 2 természetes számnál kisebb, n-hez rela-
tív prím számok száma páros.
126. [4] Mutassuk meg, hogy tetszőleges n > 2 természetes számnál kisebb, n-hez rela-
tív prím természetes számok összege többszöröse n-nek.
127. [5] Mutassuk meg, hogy ha k > 1 egész szám, akkor a 4k-nál kisebb, 4k-hoz relatív
prímek összege osztható 8k-val.
128. [4] Hány olyan ötjegyű szám van, melyben a számjegyek nemcsökkenő sorrendben
követik egymást?
22 Feladatok

129. [4] Legfeljebb hány pontban metszik egymást egy n oldalú konvex sokszög átlói?
130. [4] Legfeljebb hány háromszöget fognak közre a háromszög belsejében egy n ol-
dalú konvex sokszög átlói?
131. [3] Egy n-oldalú konvex sokszög csúcsai A1 , A2 , . . . , An . Tekintsük azokat a
sokszögeket, melyek csúcsai A1 , A2 , . . . , An közül valók. Mely sokszögekből
van több: azokból melyeknek egyik csúcsa A1 vagy azokból, amelyeknek A1
nem csúcsa?
132. [4] Hány téglalapot lehet kijelölni egy 6 × 10-es négyzetrácsban úgy, hogy oldalaik
rácsegyenesek legyenek?
133. [4] Egy 27 db egybevágó kis kockából kirakott 3 × 3 × 3-as kockában hány olyan
téglatest jelölhető ki, mely ezen kis kockákból áll?
134. [4] A számegyenesen egyesével lépkedünk az egész számokon; hol jobbra, hol
balra lépünk. Hányféleképp juthatunk a 0-ból az 1-be
a) 6, illetve
b) 7 lépésben?
135. [4] A sakktábla bal alsó sarkából a jobb felső sarokba hányféle útvonalon juthat el
a bástya, ha a bástyát csak előre és fölfelé lehet tolni?
136. [3] Igazoljuk, hogy 13 + 23 + 33 + · · · + 19903 osztható 1991-gyel.

sin 0◦ + sin 1◦ + sin 2◦ + · · · + sin 90◦


137. [3] Számítsuk ki a tört értékét.
cos 0◦ + cos 1◦ + cos 2◦ + · · · + cos 90◦

138. [3] sin2 1◦ + sin2 2◦ + sin2 3◦ + · · · + sin2 89◦ + sin2 90◦ =?


139. [4] Igazoljuk a
lg tg 1◦ · lg tg 2◦ · lg tg 3◦ · . . . · lg tg 89◦ = lg tg 1◦ + lg tg 2◦ + lg tg 3◦ + · · · + lg tg 89◦
egyenlőséget.
140. [4] Mutassuk meg, hogy az 1, 2, . . . , 10k számokban levő számjegyek száma egyen-
lő az 1, 2, . . . , 10k+1 számokban levő 0-k számával.
141. [4] A 3 számot négy különböző módon bonthatjuk fel pozitív egészek összegére:
3; 2 + 1; 1 + 2; valamint 1 + 1 + 1. Az összeadásban a tagok sorrendje lényeges,
így az 1 + 2 és a 2 + 1 különböző felbontásnak számít. Hány különböző módon
bonthatjuk fel az n természetes számot?
142. [4] Egy 11 × 6 × 5 = 330 db egységkockából álló, fából készült téglatestet egy
szú az egyik testátlója mentén végigrágott. Hány egységkockát károsított meg
a szú?
4. Miért nem négyzetszám? 23

143. [4] Egy városban az autóbuszjegyeken a sorszámozás 000 000-tól 999 999-ig megy.
Nevezzünk érdekesnek egy sorszámot, ha az első három számjegy összege meg-
egyezik az utolsó három jegy összegével. Mutassuk meg, hogy az érdekes sor-
számok száma megegyezik az olyan sorszámok számával, amelyekben a szám-
jegyek összege 27.
144. [4] Adjuk össze azokat a 2000-nél kisebb természetes számokat, amelyekben a
számjegyek összege páros.
145. [4] Csupa 1-esekből és 2-esekből álló n-jegyű számokat írunk egy táblára úgy, hogy
bármely két szám legalább három helyen különbözzék. Bizonyítsuk be, hogy
2n
legfeljebb db szám kerülhet a táblára.
n+1
Megjegyzés: A párbaállítás ötletét használja a Területátalakítások és a Matematikai játékok c.
fejezetek több feladatának megoldása, ill. a 1756., 1757. feladatok is.

4. Miért nem négyzetszám?

Miért nem négyzetszámok a 146–187. feladatokban felírt számok?


146. [3] 117 + 116 + 115 + 114 + 113 + 112 + 11 + 1?
147. [3] 100! + 7?
148. [3] 1! + 2! + 3! + · · · + 100!?
149. [3] 3 + 32 + 33 + · · · + 3100 ?
150. [3] 1010 + 3?
151. [3] 1010 + 5?
152. [3] 10100 + 1050 + 1?
153. [3] abab alakú négyjegyű szám?
154. [3] ababab alakú hatjegyű szám?
155. [3] abcabc alakú hatjegyű szám?
156. [3] abc + bca + cab alakú háromjegyű számok összege?
157. [3] Az 1, 2, 3, 4, 5, 6 számjegyek valamilyen sorrendjével felírt hatjegyű szám?
158. [3] A 2, 3, 4, 5, 6 számjegyek valamilyen sorrendjével felírt ötjegyű szám?
159. [3] 1 db 1-es, 2 db 2-es, 3 db 3-as, . . . , 9 db 9-es számjegy valamilyen sorrendjével
felírt 45-jegyű szám?
24 Feladatok

160. [3] 12 + 22 + 32 + · · · + 562 ?


161. [3] 30 db 1-es és néhány 0 segítségével felírt szám?
162. [4] 600 db 6-os és néhány 0 segítségével felírt szám?
163. [3] Az 1, 2, 3, . . . , 1991 számok valamilyen sorrendben történő, egymás után
írásával kapott szám?
164. [3] 4 egyforma, 0-tól különböző számjegyre végződő szám?
165. [3] Az a szám, amelynek utolsó két számjegye páratlan?
166. [3] 111 . . . 111 (100 db 1-es)?
167. [3] 444 . . . 444 (100 db 4-es)?
168. [3] 144 . . . 444 (99 db 4-es)?
169. [3] 19948 + 7?
170. [3] 2 egymást követő pozitív egész szám szorzata?
171. [3] 3 egymást követő pozitív egész szám szorzata?
172. [4] 4 egymást követő pozitív egész szám szorzata?
173. [3] 2 páratlan négyzetszám összege?
174. [3] 3 egymást követő négyzetszám összege?
175. [3] 4 egymást követő négyzetszám összege?
176. [3] 5 (ill. 6, 7, 8, 9, 10) egymást követő négyzetszám összege?
177. [3] p1 p2 p3 . . . pk − 1, ahol p1 , p2 , p3 , . . . , pk az első k db (k > 1) prímszámot
jelöli?
178. [3] p1 p2 p3 . . . pk + 1, ahol p1 , p2 , p3 , . . . , pk az első k db (k ≥ 1) prímszámot
jelöli?
179. [3] Az első 99 prímszám négyzetének összege?
180. [5] Egy pozitív négyzetszám és valamelyik osztójának összege?
181. [5] Egy pozitív négyzetszám és valamely osztója kétszeresének összege?
182. [3] x 2 + y és y 2 + x számok mindegyike, ahol x és y pozitív egészek?
183. [3] x 2 + 4y és y 2 + 4x számok mindegyike, ahol x és y pozitív egészek?
184. [3] n4 + 2n3 + 2n2 + 2n + 1, ahol n pozitív egész számot jelöl?
185. [3] n3 + 2n2 + 2n + 1, ahol n pozitív egész számot jelöl?
186. [5] 11571990 + 341990 ?
187. [5] 2n + 3n , ahol n pozitív egész számot jelöl?
5. Négyzetszámok 25

5. Négyzetszámok

188. [1] Lehet-e két négyzetszám szorzata is, hányadosa is négyzetszám?


189. [2] Van-e különböző négyzetszámokból álló, végtelen sok elemű számtani sorozat?
190. [2] 190 246 849 és 190 302 025 két egymás utáni páratlan szám négyzete. Számítsuk
ki a köztük levő páros szám négyzetét.
191. [3] Mely pozitív egész n-ekre lesz n2 + n + 41 négyzetszám?
192. [3] Az alább felsorolt számok közül az egyik négyzetszám, a többi nem. Melyik a
négyzetszám:
a) 3 669 517 136 205 224,
b) 1 898 732 825 398 318,
c) 4 751 006 864 295 101,
d) 5 901 643 220 186 100,
e) 7 538 062 944 751 882,
f) 2 512 339 789 576 516?
193. [3] Miért nem lehet egy négyzetszám jegyeinek összege 1991?
194. [2] Van-e olyan négyzetszám (köbszám stb.), melynek első négy számjegye 1991
(ebben a sorrendben)?
195. [4] Adott egy 99 jegyű, 9-esekből álló szám. Írhatunk-e mögé 100 számjegyet úgy,
hogy a kapott 199-jegyű szám négyzetszám legyen?
196. [2] Van-e minden n-re n-jegyű négyzetszám?
197. [2] Mutassuk meg, hogy négyzetszám 16-tal osztva mindig négyzetszám maradékot
ad.
198. [4] Mutassuk meg, hogy ha egy pozitív egészekből álló, végtelen sok elemű szám-
tani sorozat elemei közt van egy négyzetszám (köbszám stb.), akkor végtelen
sok négyzetszám (köbszám stb.) van.
199. [3] Adjunk meg különböző pozitív egészekből álló végtelen számtani sorozatot,
amelynek elemei közt nincs négyzetszám.
200. [4] Adjunk meg végtelen sok természetes számot úgy, hogy közülük akárhányat is
választunk, azok összege nem négyzetszám.
201. [4] Mutassuk meg, hogy végtelen sok olyan természetes szám van, mely nem állít-
ható elő
a) két négyzetszám, illetve
b) két köbszám
összegeként.
26 Feladatok

202. [3] Mutassuk meg, hogy 10 bármely pozitív egész kitevős hatványa előállítható két
négyzetszám összegeként.
203. [3] Mutassuk meg, ha az a, b, c pozitív egész számok olyanok, hogy a · b és b · c
is négyzetszám, akkor a · c értéke is négyzetszám.
204. [3] Megoldhatók-e az egész számok körében a következő egyenletek:
a) x 2 + y 2 = 1987,
b) x 2 + y 2 = 1986,
c) x 2 + y 2 = 1989,
d) x 2 − y 2 = 1989,
e) x 2 − y 2 = 1990?
205. [3] Mutassuk meg, hogy ha x is, y is
a) a 2 + b2 , illetve
b) a 2 + 2b2
alakú, akkor xy is ilyen alakú.
206. [4] Mutassuk meg, hogy x akkor és csak akkor a 2 + b2 alakú, ha
a) 4x, illetve
b) 2x
is ilyen alakban írható.
207. [4] Mutassuk meg, hogy x akkor és csak akkor a 2 +2b2 alakú, ha 3x is ilyen alakban
írható.
208. [5] Tekintsük az a 2 + 5b2 alakú számok M halmazát, ahol a és b nemnegatív egész
számok.
a) Bizonyítsuk be, hogy ha x és y eleme az M halmaznak, akkor xy is eleme
az M halmaznak.
b) Alapszámoknak nevezzük azokat az M-beli számokat, melyek nagyobbak
1-nél és nem oszthatók M-beli számokkal (1-et és önmagukat kivéve).
Van-e olyan M-beli szám, mely két különböző módon állítható elő alap-
számok szorzataként?
c) Bizonyítsuk be, hogy végtelen sok alapszám van.
209. [5] Bizonyítsuk be, hogy ha két egész szám összege a 2 + 3b2 alakú, ahol a és b
egész számok, akkor a köbösszegük is ilyen.
210. [6] Nevezzük az n számot érdekesnek, ha előáll 3x 2 + 32y 2 alakban, ahol x és y
egész számok. Mutassuk meg, hogy ha n érdekes szám, akkor 97n is érdekes
szám.
5. Négyzetszámok 27

1 1 1
211. [5] Az x, y, z pozitív egész számok legnagyobb közös osztója 1, továbbá + = .
x y z
Bizonyítsuk be, hogy x + y négyzetszám.
[5]
212. Bizonyítsuk be, hogy ha az x, y egészekre teljesül a 2x 2 +x = 3y 2 +y egyenlőség,
akkor 2x + 2y + 1 és 3x + 3y + 1 négyzetszámok.
213. [5] Legyen N olyan 16-jegyű pozitív egész szám, amelynek jegyei között a 0, 1,
4, 9 nem fordul elő. Bizonyítsuk be, hogy N-nek van néhány olyan egymást
követő számjegye, amelyek szorzata négyzetszám.
Igaz-e a feladat állítása a fenti típusú 15-jegyű számokra is?
214. [4] Mutassuk meg, hogy 12 egymást követő egész szám négyzetének összege nem
osztható ezeknek a számoknak az összegével.
215. [4] Milyen pozitív egész n-ekre lesz négyzetszám
a) n2 − n + 2,
b) n3 − n + 2,
c) n4 − n + 2,
d) n5 − n + 2?
216. [4] Mutassuk meg, hogy a következő számok négyzetszámok.
a) 999
. . . 9 000
 . . . 0 25 m) 1 777
. . . 7 92 888
. . . 8 921
n n n n−1
b) 999
. . . 9 8 000
 . . . 0 1 n) 3 999
. . . 9 6 000
. . . 0 1
n n n n
c) 111
. . . 1 222
 . . . 2 5 o) 7 111
. . . 1 0 222
. . . 2 5
n n+1 n−2 n
d) 444
 . . . 4 888
 . . . 8 9 p) 15 999
. . . 9 2 000
. . . 0 1
n n−1 n n
e) 111
. . . 1 555
 . . . 5 6 q) 63 999
. . . 9 84 000
. . . 0 1
n n−1 n−1 n
f) 444
. . . 4 6 222
 . . . 2 4 r) 255 999
. . . 9 68 000
. . . 0 1
n n n−1 n
g) 5 444
 . . . 4 7 555
 . . . 5 6 s) 143 999
. . . 9 76 000
. . . 0 1
n n n−1 n
h) 2 777
 . . . 7 888
 . . . 8 9 t) 783 999
. . . 9 44 000
. . . 0 1
n−1 n n−1 n
i) 1 777
 . . . 7 9 555
 . . . 5 6 u) 575 999
. . . 9 52 000
. . . 0 1
n n n−1 n
j) 7 111
 . . . 1 2 888
 . . . 8 9 . . . 1 ·1 000
v) 111  . . . 0 5 + 1
n n n n−1
k) 224 999
 . . . 9 1 000
 . . . 0 9 w) 444
 . . . 4 −11 · 444
 . . . 4 +9
n−2 n 2n n
l) 1 777
 . . . 7 6 888
 . . . 8 9
n n
28 Feladatok

217. [4] Igazoljuk az alábbi összefüggéseket.


. . . 3 · 33
a) 33  . . . 3 = 11
  . . . 1 0 99 
  . . . 9 88 . . . 8 9
 
m n n−1 m−n n−1
b) 66 . . . 6 · 66
  . . . 6 = 44
  . . . 4 3 99 
  . . . 9 55 . . . 5 6
 
m n n−1 m−n n−1
. . . 9 · 99
c) 99  . . . 9 = 99
  . . . 9 8 99 
  . . . 9 00 . . . 0 1
 
m n n−1 m−n n−1
d) 33 . . . 3 · 66
  . . . 6 = 22
  . . . 2 1 99 
  . . . 9 77 . . . 7 8
 
m n n−1 m−n n−1
. . . 3 · 99
e) 33  . . . 9 = 33
  . . . 3 2 99 
  . . . 9 66 . . . 6 7
 
m n n−1 m−n n−1
. . . 6 · 99
f) 66  . . . 9 = 66
  . . . 6 5 99 
  . . . 9 33 . . . 3 4
 
m n n−1 m−n n−1

218. [4] Igazoljuk az alábbi összefüggéseket.


a) (666 . . . 6)2 + 888 . . . 8 = 444 . . . 4
        
n n 2n

b) 111
  . . . 1 − 222
 . . . 2 = 333
 . . . 3
2n n n
[4]
219. Az A számot úgy képezzük, hogy a 16 számjegyei közé 2k + 1 db 7-est írunk;
a B számot pedig úgy kapjuk, hogy a 32 számjegyei közé k db 5-öst írunk.
Mutassuk meg, hogy A − B négyzetszám!

6. Két szomszédos egész szám szorzata

220. [2] Lehet-e 1! + 2! + 3! + ... + 100! két egymást követő egész szám szorzata?
221. [3] Lehet-e 3100 + 1 két egymást követő egész szám szorzata?
222. [3] Lehet-e 100! + 4 két egymást követő egész szám szorzata?
223. [3] Lehet-e 2(6n + 1) két egymást követő egész szám szorzata?
224. [3] Mutassuk meg, hogy bármely egész n-re 3n3 + 2n2 + n + 1 nem lehet két szom-
szédos egész szorzata.
1
225. [3] Mutassuk meg, hogy minden egész n-re (n4 + 2n3 + 3n2 + 2n) két szomszédos
4
egész szorzata.
226. [3] Mutassuk meg, hogy minden egész n-re 2(9n + 9n−1 + · · · + 9 + 1) két szomszédos
egész szorzata.
7. Diofantoszi egyenletek 29

227. [3] Bizonyítsuk be, hogy bármely nemnegatív n egészre n2 + 7n + 8 nem lehet két
szomszédos egész szorzata. Mi a válasz negatív egész n számok esetén?
228. [3] Mely egész n-ekre lesz n2 + 2n + 12 két szomszédos egész szorzata?
229. [4] Mely egész n-ekre lesz 15-tel osztható n2 + n + 2?
230. [4] Igazoljuk, hogy n2 + n + 1 egyetlen egész n-re sem többszöröse 1990-nek.
231. [4] Oldjuk meg a 6x = y 2 + y − 2 egyenletet az egész számok körében.
232. [4] A 2, 3, 4, 5, 6, 7, 8, 9, 0 számok valamilyen sorrendjével fel lehet-e írni olyan
kilencjegyű számot, mely két szomszédos egész szorzata?
233. [4] Mutassuk meg, hogy 9n + 2 pontosan akkor két szomszédos egész szorzata, ha
n is ilyen.
234. [4] Mutassuk meg, hogy bármely egész n-re és természetes k-ra 2n3k + 4nk + 10
nem lehet két szomszédos egész szorzata.
235. [4] Mutassuk meg, hogy a következő számok mindegyike két szomszédos egész
szorzata.
a) 8999 . . . 91000 . . . 02 (a számban ugyanannyi 9 és 0 van)
b) 111 . . . 1222 . . . 2 (a számban ugyanannyi 1 és 2 van)

7. Diofantoszi egyenletek

Mutassuk meg, hogy a következő egyenletek nem oldhatók meg a pozitív egészek kö-
rében. (236–269. feladatok)
236. [2] ab(a + b) = 1993.
237. [2] a 2 + b2 + a + b = 1993.
238. [2] nk+1 = (n + 1)k .
239. [2] 1992x + 1993y = 1994z .
240. [2] x 7 + 45x − 1993 = 0.
241. [2] n4 + n3 + n2 + n = 3100 .
242. [2] a 5 b = ab5 + 1993.
243. [2] a 2 − 2ab = 1994.
244. [2] 6x − 15y = 14.
245. [3] a 2 − b2 + c2 = 100, 5 | abc.
30 Feladatok

246. [3] a 2 − 3b = 17.


247. [3] 3a 2 − 4b2 = 13.
248. [3] a 3 − a = 3b2 + 1.
249. [3] a 2 + 4a − 8b = 11.
250. [3] a 3 + 2 = 4b(b + 1).
251. [3] a 2 − 2b2 = 11.
252. [3] a 2 − 2b2 + 8c = 3.
253. [4] a 2 + b2 + c2 = 73 .
254. [4] a 3 + b3 + c3 = 9abc + 4.
255. [4] 19a 3 − 17b3 = 50.
256. [3] a 2 + b2 = 13013.
257. [4] a 3 + b3 = 1991.
258. [4] a 3 + b3 + c3 = 1993.
259. [4] a 3 + b3 + c3 = 19872 .
260. [4] a14 + a24 + · · · + a14
4
= 1599.
261. [4] 19a 3 − 84b2 = 1984.
262. [4] a 2 − 2b2 + 363c2 = 77.
263. [4] a 2 + 9b2 − 2c2 = 3.
264. [3] 5n + 11n = 12n .
265. [3] x 2 + y 2 = 2x + 2y − 3.
266. [3] x 4 − 2x 2 y + 3y 2 + 2 = 0.
267. [4] a 2 − 2ab + 2b2 − 4b3 = 0.
1 1 1
268. [3] 2 + + = 1.
a ab b2
1 1 1 1 3
269. [3] 2 + 2 + 2 + 2 = , ahol a, b, c, d különböző egész számok.
a b c d 2
7. Diofantoszi egyenletek 31

Oldjuk meg a következő egyenleteket az egész számok körében. (270–297. feladatok)


270. [2] ab = a + b.
271. [2] ab + a + b = 6.
272. [2] ab + 2a + 3b = 36.
273. [2] abc + ab + bc + ca + a + b + c = 1000.
1 1 1
274. [2] + = .
a b 7
1 1 1 1
275. [2] + − = .
a b ab 7
276. [2] 2a 3 + ab − 7 = 0.
277. [2] ab + 3a − 5b + 3 = 0.
278. [2] a 2 − b2 = 100.
279. [2] a 2 − 4b2 = 116.
280. [2] 2a 2 + 5ab − 12b2 = 28.
281. [3] a(b + 1)2 = 243b.
282. [3] a 2 = b2 + 2b + 13.
283. [3] a 2 = b3 + 1.
284. [3] a 2 + b2 + c2 = ab + bc + ca.
285. [4] a 2 − ab + b2 = a + b.
286. [3] a 3 + b3 − 3a 2 + 6b2 + 3a + 12b + 6 = 0.
287. [2] (n! + 1) · (m! + 1) = (n + m)!, n ≥ 1, m ≥ 1.
288. [3] 1! + 2! + 3! + ... + n! = m2 , n ≥ 0.
289. [3] abc = a! + b! + c!, a > 0, b > 0, c > 0.
290. [3] a + b + c = 1,
a 3 + b3 + c3 = 1 és
abc = −16.
291. [4] x 3 − 5x 2 + 8x − 6 = 0.
292. [4] a 3 + (a + 1)3 = (a + 2)3 .
293. [4] a 3 + (a + 1)3 + (a + 2)3 = (a + 3)3 .
294. [3] abc = a + b + c, a > 0, b > 0, c > 0.
295. [3] abc = ab + bc + ca, a > 0, b > 0, c > 0.
32 Feladatok

296. [3] abc = 2(ab + bc + ca), a > 0, b > 0, c > 0.


1 1 1 1
297. [3] + + + = 2, a > 0, b > 0, c > 0.
a b c abc
298. [2] Egy téglalap kerületének és területének mérőszáma megegyezik, oldalainak mé-
rőszáma egész szám. Mekkora a területe?
299. [2] Melyek azok a kétjegyű számok, amelyek 17-tel nagyobbak számjegyeik szor-
zatánál?
300. [3] Három prímszám szorzata ötszöröse összegüknek. Melyek ezek a prímek?
301. [4] Bizonyítsuk be, hogy sem 1994, sem 1995 nem lehet az egész együtthatós ax 2 +
+bx + c = 0 másodfokú egyenlet diszkriminánsa. Lehet-e 1996?
302. [4] Mutassuk meg, hogy ha az a, b, c nemnegatív egészekre 28a + 30b + 31c = 365
teljesül, akkor a + b + c = 12. Oldjuk meg az egyenletet.
303. [4] Mutassuk meg, hogy az [x]+[2x]+[4x]+[8x]+[16x]+[32x] = 12345 egyenletnek
nincs megoldása.
304. [5] Megoldható-e az egész számok körében az x 2 + 3xy − 2y 2 = 122 egyenlet?
305. [5] Oldjuk meg az egész számok körében az a + b + c = 7, a 3 + b3 + c3 = 1
egyenletrendszert.
306. [5] Oldjuk meg az egész számok körében az a + b + c = 31, a 2 + b2 + c2 = 325
egyenletrendszert.
307. [5] Mutassuk meg, hogy az a 2 + b2 = c2 egyenletnek végtelen sok különböző meg-
oldása van az egész számok körében, melyre (a, b, c) = 1.
308. [5] Mutassuk meg, hogy az a 2 + b2 = c3 egyenletnek végtelen sok különböző meg-
oldása van az egész számok körében, melyre (a, b, c) = 1.
309. [5] Mutassuk meg, hogy az a 2 + b2 = c4 egyenletnek végtelen sok különböző meg-
oldása van az egész számok körében, melyre (a, b, c) = 1.
310. [4] Mutassuk meg, hogy az a 2 = 2b2 egyenlet nem oldható meg a nullától külön-
böző egészek körében.
311. [4] Mutassuk meg, hogy az a 2 = 7b2 egyenlet nem oldható meg a nullától külön-
böző egészek körében.
312. [4] Mutassuk meg, hogy a 6(a 2 + b2 ) = c2 + d 2 egyenlet nem oldható meg a nullától
különböző egészek körében.
313. [4] Mutassuk meg, hogy a 7(a 2 + b2 ) = c2 + d 2 egyenlet nem oldható meg a nullától
különböző egészek körében.
314. [4] Mutassuk meg, hogy az a 2 + b2 = 3c2 egyenlet nem oldható meg a nullától
különböző egészek körében.
8. Prímszámok 33

315. [4] Mutassuk meg, hogy az a 3 + 3b3 + 9c3 = 9abc egyenlet nem oldható meg a
nullától különböző egészek körében.
316. [4] Mutassuk meg, hogy az a 3 − 2b3 − 4c3 = 0 egyenlet nem oldható meg a nullától
különböző egészek körében.
317. [4] Mutassuk meg, hogy az a 2 + b2 + c2 = a 2 b2 egyenlet nem oldható meg a nullától
különböző egészek körében.
318. [4] Mutassuk meg, hogy az a 2 +b2 +c2 = 2abc egyenlet nem oldható meg a nullától
különböző egészek körében.
319. [4] Mutassuk meg, hogy az a 2 + b2 + c2 + d 2 = 2abcd egyenlet nem oldható meg a
nullától különböző egészek körében.
320. [6] Mutassuk meg, hogy az a 4 + b4 = c4 egyenlet nem oldható meg a nullától
különböző egészek körében.
321. [5] Mutassuk meg, hogy az a 2 + b2 + c2 + a + b + c = 1 egyenlet nem oldható meg
a racionális számok körében.
322. [5] Van-e az a 2 + ab + b2 = 2 egyenletnek racionális számokból álló megoldása?

8. Prímszámok

323. [1] Páros-e vagy páratlan az első 100 prímszám összege?


324. [1] Adjunk meg két egész számot, melyek összege is, szorzata is prím.
325. [1] Három egész szám számtani sorozatot alkot, s a három szám szorzata prímszám.
Melyek ezek a számok?
326. [1] Adjunk meg két olyan p és q prímszámot, melyekre igaz, hogy p + q és p − q
is prím.
327. [1] Miért nem lehet két prím összege 1991?
328. [1] Három prímszám összege 1992. E három prímszám közül melyik a legkisebb?
329. [2] Van-e három olyan prímszám, amelyek összege 1234, szorzata 87654321?
330. [4] Két prímszám különbsége 100. A tízes számrendszerbeli alakjukat egymás után
írva, egy újabb prímszámot kapunk. Melyek ezek a számok?
331. [1] Van-e hat egymást követő prímszám, melyek összege is prím?
332. [1] Miért nincs olyan prímszám, mely után pontosan 10 összetett szám következik?
333. [2] Lehet-e 15 egymást követő egész szám összege prímszám?
34 Feladatok

334. [2] Lehet-e 16 egymást követő egész szám összege prímszám?


335. [2] Lehet-e 3 × 3-as bűvös négyzetet készíteni az első kilenc prímszámból?
336. [2] Oldjuk meg a prímszámok körében a 2x + 3y + 6z = 78 egyenletet.
337. [3] Oldjuk meg a prímszámok körében a pq + 1 = r egyenletet.
338. [3] Oldjuk meg a prímszámok körében a p2 − 2q 2 = 1 egyenletet.
339. [3] Oldjuk meg a prímszámok körében a p2 + q 3 = r 4 egyenletet.
340. [2] Egy apa és két különböző korú kisgyermekének életkora ugyanannak a prím-
számnak pozitív egész kitevős hatványai. Egy évvel ezelőtt mindhármuk élet-
kora prímszám volt. Hány évesek most?
341. [3] Egy háromjegyű szám jegyeinek összege egy prím négyzete, a jegyek szorzata
egy prím köbe. Melyik ez a szám?
342. [3] Van-e olyan n egész szám, melyre 2n − 1 és 2n + 1 is prímszám?
343. [3] Legyen p > 3 prímszám. Mutassuk meg, hogy 3 | p2 − 1, valamint 24 | p2 − 1.
344. [3] Lehetnek-e az n + 5, n + 7 és n + 15 számok egyszerre prímszámok, ahol n egész
számot jelöl?
345. [3] Mely p prímszámra lesz
a) p + 5 és p + 10,
b) p + 10 és p + 14,
c) 8p − 1 és 8p + 1,
d) 8p2 − 1 és 8p2 + 1,
e) p2 + 2,
f) 14p2 + 1, illetve
g) 2p + 1, 3p + 2, 4p + 3 és 6p + 1
egyszerre prímszám?
346. [3] Mutassuk meg, hogy ha p és p2 + 8 prímszámok, akkor p2 + p + 1 is prímszám.
347. [3] Mutassuk meg, hogy ha p és 8p2 +1 prímszámok, akkor 8p2 +2p+1 is prímszám.
348. [4] Mely p és q prímszámokra lesz p + q és p2 + q 2 − q is prímszám?
349. [4] Mely p és q prímszámokra lesz pq + q p is prímszám?
350. [4] Mely p és q prímszámokra lesz pq − 1 és pq + 1 is prímszám?
351. [4] Mely p prímszámra lesz 4p + 1 négyzetszám?
352. [4] Legyen p olyan prím, amelyre p2 + p + 1 és p2 − p + 1 is prím. Bizonyítsuk be,
hogy ha p4 + p3 + p2 + p + 1 nem prímszám, akkor négyzetszám.
8. Prímszámok 35

353. [4] Határozzuk meg azokat a p prímeket, amelyekre 4p2 +1 és 6p2 +1 is prímszám.
354. [3] Mutassuk meg, hogy ha a p, q és r, s számpárok ikerprímek, és mindegyik
nagyobb, mint 3, akkor pr −qs osztható 12-vel. (Két prím iker, ha különbségük
2.)
355. [4] Mutassuk meg, hogy két egymás utáni páratlan prím összege legalább három
(nem feltétlenül különböző) prím szorzata.
356. [3] Lehetnek-e valamely n természetes számra prímszámok a következő kifejezé-
sek:
a) n4 + 2n3 + 2n2 + 2n + 1,
b) 6n + 3n + 2n+1 + 2,
c) 32n+1 − 22n+1 − 6n ,
d) 8n2 + 10n + 3,
e) 25n4 + 9n2 + 1,
f) 4n3 + 6n2 + 4n + 1,
g) |n4 + n3 − 2n2 − 3n − 3|?
357. [4] Adjunk meg olyan, 1-nél nagyobb egész számot, mely nem osztható a 2, 3, 5,
7, . . . , p prímek egyikével sem.
358. [4] A következő polinomok néhány egymást követő természetes számra prímszá-
mot adnak értékül:
n2 + n + 41, n = 0, 1, 2, . . . , 39 (Euler, 1772),
n2 − 79n + 1601, n = 0, 1, 2, . . . , 79,
n2 − 5n + 47, n = 0, 1, 2, . . . , 42,
2n2 − 2n + 19, n = 0, 1, 2, . . . , 18,
3n2 − 3n + 23, n = 0, 1, 2, . . . , 22,
8n2 − 326n + 2659, n = 0, 1, 2, . . . , 39,
36n2 − 810n + 2753, n = 0, 1, 2, . . . , 44.
Keressünk további ilyen polinomokat — számítógéppel.
359. [4] Keressünk — számítógéppel — különböző prímszámokból álló 3 × 3-as bűvös
négyzetet.
360. [4] Miért nincs olyan egész együtthatós polinom, mely az x változó minden egész
értékére prímszámot vesz fel értékül?
361. [4] Miért nincs csupa különböző prímszámból álló végtelen számtani sorozat?
362. [3] Keressünk 200-nál kisebb prímekből álló — minél több elemű — számtani
sorozatot.
363. [3] Keressünk 100-nál kisebb, páronként relatív prímekből álló — minél több elemű
— számtani sorozatot.
36 Feladatok

364. [4] Adjunk meg olyan, természetes számokból álló végtelen számtani sorozatot,
amelynek egyik eleme sem állítható elő két prím összegeként.
365. [3] Igaz-e, hogy bármely természetes szám egy jegyének megváltoztatásával prím-
számmá alakítható?
366. [4] Tudjuk, hogy az a, b, c, d természetes számokra ab = cd. Mutassuk meg, hogy
az a 2000 + b2000 + c2000 + d 2000 szám összetett szám.
367. [4] Legyen p 1-nél nagyobb egész szám. Bizonyítsuk be, hogy p pontosan akkor
prímszám, ha p bármely négy pozitív egész szám összegére való felbontásában
semelyik két tag szorzata sem egyenlő a másik két tag szorzatával.
368. [6] Mutassuk meg, hogy a prímszámok száma végtelen.

9. Oszthatósági feladatok

369. [2] Igazoljuk a következő oszthatóságokat:


a) 9 | 1033 + 8,
b) 6 | 1010 + 14,
c) 72 | 1020 + 8.
370. [2] Határozzuk meg a felírt szám hiányzó számjegyeit úgy, hogy teljesüljön az
oszthatóság. Keressük meg az összes megoldást.
a) 36 | 52x2y,
b) 72 | x378y,
c) 45 | 24x68y,
d) 48 | 24x68y,
e) 99 | 62xy427.
371. [2] Írjunk az 1994 elé is, után is egy-egy számjegyet úgy, hogy a kapott hatjegyű
szám osztható legyen
a) 88-cal;
b) 99-cel.
372. [1] Igazak-e a következő állítások:
a) Ha egy szám osztható 6-tal és 8-cal, akkor osztható 48-cal is.
b) Ha egy szorzat osztható 6-tal, akkor valamelyik tényezője osztható 6-tal.
9. Oszthatósági feladatok 37

373. [2] Mutassuk meg, hogy a következő számok összetett számok.


a) 106 − 57 ,
b) 10100 − 7,
c) 420 − 1,
d) 1 000 027,
e) 1000 . . . 001 (1991 db 0),
f) 111 . . . 111 (1989 db 1-es),
g) 111 . . . 111 (1990 db 1-es),
h) 111 . . . 121 . . . 111 (a 2 előtt is, után is ugyanannyi db 1-es),
i) 10300 − 2 · 10100 + 1,
j) 1! + 2! + 3! + · · · + 100!.
374. [3] Mutassuk meg, hogy 347 777 743 összetett szám.
375. [3] Mutassuk meg, hogy a 49 + 610 + 320 szám összetett szám.
376. [3] Mutassuk meg, hogy a 210 + 512 szám összetett szám.
377. [4] Mutassuk meg, hogy az 53 · 83 · 109 + 40 · 66 · 96 szám összetett szám.
378. [4] Mutassuk meg, hogy a 989 · 1001 · 1007 + 320 szám összetett szám.
379. [4] Mutassuk meg, hogy az 5123 + 6753 + 7203 szám összetett szám.
380. [4] Az a és b természetes számokra teljesül a 34a = 43b összefüggés. Mutassuk
meg, hogy ekkor a + b összetett szám.
381. [3] Mutassuk meg, hogy minden, 0-tól különböző n természetes számhoz van olyan
m természetes szám, melyre n · m + 1 összetett szám.
382. [4] Mutassuk meg, hogy 19·8n +17 az n minden nemnegatív egész értékére összetett
szám.
383. [1] Melyik az a legkisebb természetes szám, mely osztható az 1, 2, 3, 4, 5, 6, 7, 8,
9, 10 számok mindegyikével?
384. [1] Mutassuk meg, hogy 9 egymást követő egész szám összege osztható 9-cel.
385. [1] Mutassuk meg, hogy 10 egymást követő egész szám összege sohasem osztható
10-zel.
386. [1] Öt egymást követő egész számot összeszorzunk. Milyen jegyre végződik az így
kapott szám?
387. [1] Öt egymást követő páratlan számot összeszorzunk. Milyen jegyre végződik az
így kapott szám?
388. [1] Négy egymást követő pozitív egész szorzata 3024. Melyek ezek a számok?
38 Feladatok

389. [2] Adjuk meg 15-nek az első három olyan pozitív többszörösét, melyben a szám-
jegyek összege 15.
390. [3] Adjuk meg 45 legkisebb olyan pozitív többszörösét, melyben csak 0 és 8-as
számjegyek vannak.
391. [2] Adjuk meg 1991 legkisebb olyan pozitív többszörösét, mely 1992-re végződik.
392. [2] Miért nem lehet egy természetes szám számjegyeinek szorzata 111?
n+1 n+8
393. [3] Miért nem lehet egyszerre egész és , n ∈ N?
15 21
394. [3] Miért nem lehet az 1, 2, 3, . . . , 10 számokat két csoportra osztani úgy, hogy a
két csoportban levő számok
a) összege vagy
b) szorzata egyenlő legyen?
395. [3] Miért nem lehet 5 egymást követő természetes számot két csoportba osztani
úgy, hogy a két csoportban levő számok szorzata egyenlő legyen?
396. [5] Miért nem lehet 6 egymást követő természetes számot két csoportba osztani
úgy, hogy a két csoportban levő számok szorzata egyenlő legyen?
397. [2] Írjuk fel az 1, 2, 3, 4, 5, 6 számjegyek valamilyen sorrendjével a legnagyobb,
12-vel osztható számot.
398. [2] Írjuk fel azt a legkisebb 9-jegyű számot, melynek első 2 jegyéből álló szám
osztható 2-vel, az első 3 jegyből álló szám osztható 3-mal, . . . , az első 8 jegyből
álló szám osztható 8-cal, és maga a szám osztható 9-cel.
399. [3] Írjunk fel olyan, az 1, 2, . . . , 6 számjegyek valamilyen sorrendjéből álló 6-jegyű
számot, melynek első 2 jegyéből álló szám osztható 2-vel, az első 3 jegyből
álló szám osztható 3-mal, az első 4 jegyből álló szám osztható 4-gyel, az első
5 jegyből álló szám osztható 5-tel, és maga a szám osztható 6-tal.
400. [4] Írjuk fel azt az 1, 2, . . . , 9 számjegyek valamilyen sorrendjéből álló 9-jegyű
számot, melynek első 2 jegyéből álló szám osztható 2-vel, az első 3 jegyből
álló szám osztható 3-mal, . . . , az első 8 jegyből álló szám osztható 8-cal, és
maga a szám osztható 9-cel.
401. [3] Írjuk fel azt az 1, 2, . . . , 9 számjegyek valamilyen sorrendjéből álló legkisebb
9-jegyű számot, mely osztható 99-cel.
402. [5] Határozzuk meg a 999-cel osztható, 9-es számjegyet nem tartalmazó legkisebb
pozitív egész számot.
403. [4] Adjunk meg olyan 7-jegyű számot, melynek számjegyei különbözők és a szám
osztható mindegyik számjegyével.
9. Oszthatósági feladatok 39

404. [4] Határozzuk meg azokat a 4-jegyű, 9-re végződő számokat, amelyek oszthatók
számjegyeik mindegyikével.
405. [4] Keressük meg azt a legnagyobb számot, amelyre teljesül, hogy az első n szám-
jegyéből álló n-jegyű szám osztható az n-edik prímszámmal, n = 1, 2, 3, . . . ,
N esetén, ahol N a keresett szám számjegyeinek száma.
406. [5] Melyik az az ötjegyű szám, amelyik egyenlő számjegyei szorzatának 45-szö-
rösével?
407. [5] Melyik az a legkisebb, 7-esekből és 3-asokból álló szám, melyben a számjegyek
összege s maga a szám is osztható 7-tel és 3-mal?
408. [4] Írjunk fel 6 db 2-es és 2 db 1-es segítségével 7-tel osztható 8-jegyű számot.
409. [4] Melyik az a legkisebb, 28-cal osztható pozitív egész szám, amelynek a 10-es
számrendszerbeli alakja 28-ra végződik, és számjegyeinek összege 28?
410. [4] Melyik az a legkisebb, 56-tal osztható pozitív egész szám, amelynek a 10-es
számrendszerbeli alakja 56-ra végződik, és számjegyeinek összege 56?
411. [4] Adjunk meg két olyan 6-jegyű számot, melyeket egymás után írva olyan 12-
jegyű számot kapunk, amely osztható a két 6-jegyű számmal.
412. [4] Meg lehet-e adni két ötjegyű számot úgy, hogy azok mindegyik számjegye
páratlan legyen, és a két szám szorzataként kapott számban is minden számjegy
páratlan?
413. [4] Határozzuk meg a legkisebb pozitív egész számot, amely legalább 600-szor
akkora, mint bármelyik prímosztója.
414. [5] Egy ötjegyű számot nevezzünk felbonthatatlannak, ha a szám nem áll elő két
háromjegyű szám szorzataként. Legfeljebb hány egymást követő felbonthatatlan
szám van?
415. [5] 49 pozitív egész szám összege 999. Határozzuk meg ezen számok legnagyobb
közös osztójának lehetséges legnagyobb értékét.
416. [5] 10 pozitív egész szám összege 1001. Határozzuk meg ezen számok legnagyobb
közös osztójának lehetséges legnagyobb értékét.

Bizonyítsuk be a következő oszthatóságokat. (417–440. feladatok)


417. [2] 5 | 119 + 118 + 117 + · · · + 11 + 1.
418. [3] 100 | 1110 − 1.
419. [2] 200 | 1993 − 199.
420. [2] 13 | 270 + 370 .
421. [2] 13 | 31974 + 51974 .
40 Feladatok

422. [3] 1976 | 19751975 − 19771977 + 1978.


1977
423. [3] 19771977 + 1 | 1977(1977 )
+ 1.
[3]
424. 18 | 17 + 19
19 17
(Hogyan lehetne általánosítani?).
425. [4] 1897 | 2903n − 803n − 464n + 261n , n ∈ N.
[4]
426. 1946 | 1492 − 1770 − 1863 + 2141 ,
n n n n
n ∈ N.
427. [4] 1957 | 17212n − 732n − 5212n + 2122n , n ∈ N.
[4]
428. 1958 | 2225 − 2082 + 1815 ,
n n n
n ∈ N , n ≥ 1.
429. [4] 1995 | 2801n − 2696n − 2269n + 169n , n ∈ N.
[4]
430. 5 | 1 + 2 + 3 + 4 pontosan akkor, ha 4 - n, n ∈ N .
n n n n

[3]
431. 18 | 22n + 24n − 10, n > 0, n ∈ N .
432. [3] 64 | 32n+3 − 24n + 37, n ∈ N .
433. [3] 11 | 32n+2 + 26n+1 , n ∈ N .
434. [3] 16 | 5n − 4n − 1, n ∈ N .
435. [3] 36 | 7n − 6n − 1, n ∈ N (Hogyan lehetne általánosítani?).
436. [3] 4003 | 1 · 2 · . . . · 2001 + 2002 · 2003 · . . . · 4002.
437. [7] 71 | 61! + 1.
438. [7] 71 | 63! + 1.
439. [4] 7 | 22225555 + 55552222 .
440. [4] 19922 + 1993 | 19921993 − 1992.
441. [3] Mutassuk meg, hogy ha 2n + 1 prímszám, akkor n = 2k (n, k ∈ N ).
442. [3] Mutassuk meg, hogy ha 2n − 1 prímszám, akkor n is prímszám (n ∈ N , n > 1).
443. [4] Mutassuk meg, hogy ha m páratlan, akkor (2m − 1, 2n + 1) = 1.
n m
444. [4] Mutassuk meg, hogy (22 + 1, 22 + 1) = 1.
445. [5] Mutassuk meg, hogy ha m > 2, akkor 2n + 1 sohasem osztható (2m − 1)-gyel.
22 −1 − 2
n
2n − 2
446. [5] Mutassuk meg, hogy ha egész szám, akkor is egész szám.
n 2n − 1
22p − 1
447. [6] Legyen p > 3 prímszám és n = . Mutassuk meg, hogy n|2n − 2.
3
448. [3] Adott néhány egész szám, melyek összege osztható 6-tal. Mutassuk meg, hogy
ekkor a számok köbeinek összege is osztható 6-tal.
9. Oszthatósági feladatok 41

449. [4] Mutassuk meg, hogy a következő oszthatóságok sohasem teljesülnek.


a) n + 4 | n2 + 8n + 15, n ∈ N ,
b) 121 | n2 + 3n + 5, n ∈ N .
450. [4] Igazoljuk, hogy x 5 + 3x 4 y − 5x 3 y 2 − 15x 2 y 3 + 4xy 4 + 12y 5 semmilyen x, y
értékre nem veszi fel értékül a 33-at.
451. [3] Határozzuk meg 1989 + 8189 utolsó két számjegyét.
452. [2] Igaz-e, hogy ha egy szám számjegyeinek összege osztható 27-tel, akkor a szám
is osztható 27-tel?
453. [4] Tudjuk, hogy az a, b, c egész számokra a + b + c = (a − b)(b − c)(c − a).
Mutassuk meg, hogy ekkor a + b + c osztható 27-tel.
454. [3] Mutassuk meg, hogy 111 . . . 111 (81 db 1-es) osztható 81-gyel.
455. [4] Osztható-e 1980-nal a 19 20 21 22 23 . . . 78 79 80 szám?
456. [3] Mutassuk meg, hogy ha egy szám osztható 99-cel, akkor a szám jegyeinek
összege legalább 18.
457. [3] Mutassuk meg, hogy a 37, 537, 5537, 55537, 555537, . . . sorozatban végtelen
sok összetett szám van.
458. [3] Mutassuk meg, hogy a 13, 149 13, 149 149 13, 149 149 149 13, . . . sorozatban
végtelen sok összetett szám van.
459. [4] Mutassuk meg, hogy a 7, 37, 337, 3337, 33 337, . . . sorozatban végtelen sok
összetett szám van.
460. [4] Mutassuk meg, hogy a 31, 331, 3331, 33 331, . . . sorozatban végtelen sok
összetett szám van.
 √ 
461. [6] Mutassuk meg, hogy a 2k · 2 k = 1, 2, . . . számok között végtelen sok
összetett szám van.
462. [3] Mutassuk meg, hogy ha egy tetszőleges háromjegyű számot a szám után újra
leírunk, akkor az így kapott abcabc alakú hatjegyű szám osztható 13-mal.
463. [3] Mutassuk meg, hogy ha 27 | abc, akkor 27 | bca.
464. [4] Felírtunk egy kör kerületére 1989 db számjegyet. Tegyük fel, hogy egy szám-
jegytől kezdve egy irányban végigolvasva, a kapott 1989-jegyű szám osztható
27-tel. Mutassuk meg, hogy ez esetben bárhonnan kezdve az olvasást, a kapott
szám osztható lesz 27-tel.
465. [3] Mutassuk meg, hogy egy 41-gyel osztható ötjegyű szám első számjegyét áttéve
utolsó számjegynek, az így kapott ötjegyű szám is osztható lesz 41-gyel.
42 Feladatok

466. [4] Mutassuk meg, hogy egy 7-tel osztható hatjegyű szám utolsó jegyét elsőnek
írva, az így kapott hatjegyű szám is osztható lesz 7-tel.
467. [4] Bizonyítsuk be, hogy ha egy nyolcjegyű szám osztható 101-gyel, akkor a szám
jegyeinek valamely más sorrendjével felírt nyolcjegyű szám is osztható lesz
101-gyel.
468. [3] Mutassuk meg, hogy ha 13 | 2a + b és 13 | 5a − 4b, akkor 13 | a − 6b.
469. [3] Mutassuk meg, hogy ha 7 | 100a + b, akkor 7 | a + 4b.
470. [3] Mutassuk meg, hogy 7 | 10a + b pontosan akkor, ha 7 | a − 2b.
471. [3] Mutassuk meg, hogy ha 11 | 3a + 4b, akkor 11 | a + 5b.
472. [3] Mutassuk meg, hogy ha 17 | 5a + 2b, akkor 17 | 9a + 7b.
473. [2] Mely n egészekre lesz egész szám
n + 11
a) ,
n−9
3n + 5
b) ,
n+3
n2 + 1
c) ,
n+1
n2 + 1992
d) ?
n + 1992
n + 11
474. [3] Mely n egész szám esetén lehet az törtet egyszerűsíteni?
n−9
475. [3] Bizonyítsuk be, hogy a következő törteket egyetlen n egész szám esetén sem
lehet egyszerűsíteni.
3n + 1
a) ,
5n + 2
12n + 1
b) ,
30n + 2
21n + 4
c) ,
14n + 3
n3 + 2n
d) .
n4 + 3n2 + 1
10. Különféle számelméleti feladatok 43

476. [3] Az
a) 5n + 6 és 8n + 7;
b) 7n + 1 és 8n + 3;
c) n + 7 és 2n + 3;
d) n2 + 1 és (n + 1)2 + 1
számoknak bizonyos n természetes számok esetén van 1-nél nagyobb közös
osztója. Mi lehet ez a közös osztó?
477. [3] Mutassuk meg, hogy az 19931994 + 19941993 és az 1993 · 1994 számok relatív
prímek.
478. [3] Egy 6-ra végződő szám utolsó jegyét elhagyjuk és ezt a jegyet a szám első jegye
elé írjuk. Az így kapott szám négyszerese az eredetinek. Melyik ez a szám?
479. [3] Egy 2-re végződő szám utolsó jegyét elhagyjuk és ezt a jegyet a szám első jegye
elé írjuk. Az így kapott szám kétszerese az eredetinek. Melyik ez a szám?
480. [3] Van-e olyan 3-ra végződő szám, melynek utolsó jegyét letörölve, s a szám elé
írva olyan számot kapunk, mely az eredeti számnak 3-szorosa?
481. [3] Egy 1-essel kezdődő szám első jegyét elhagyjuk és ezt a jegyet a szám utolsó
jegye után írjuk. Az így kapott szám háromszorosa az eredetinek. Melyik ez a
szám?
482. [3] Van-e olyan n egész szám, mely elé, ha alkalmasan választott számjegyet írunk,
az n szám 58-szorosát kapjuk? Vajon van-e olyan n egész szám, mely elé al-
kalmasan választott számjegyet írva, az n szám 57-szeresét kapjuk?

10. Különféle számelméleti feladatok

483. [1] Lehet-e három egymást követő egész szám


a) összege 1991, illetve
b) szorzata 1990?
484. [3] Mutassuk meg, hogy három egymást követő pozitív egész szám szorzata nem
lehet köbszám.
485. [3] Bizonyítsuk be, hogy négy egymást követő egész szám mindegyike nem lehet
hatványszám.
486. [3] Mutassuk meg, hogy hat egymást követő, 6-nál nagyobb egész között
a) van négy összetett szám;
b) van olyan, mely relatív prím a többihez.
44 Feladatok

487. [3] Mutassuk meg, hogy 12 egymást követő egész szám között van olyan, mely
kisebb, mint valódi osztóinak összege.
488. [3] Néhány egymást követő egész szám összege 100. Melyek ezek a számok?
489. [2] Lehet-e a 0, 1, 2, . . . , 9 számok alkalmas sorrendjével olyan 10-jegyű számot
felírni, amely szám 2-nek pozitív egész kitevőjű hatványa?
490. [4] Kártyalapokra felírtuk 11 111-től 99 999-ig az egész számokat (minden lapra
egy számot). Ezeket a lapokat valamilyen sorrendben egymás után helyezve a
kapott 444 445-jegyű szám lehet-e 2-nek pozitív egész kitevőjű hatványa?
491. [4] Lehet-e egymást követő pozitív egész számok összege 2100 ?
492. [4] Végződhet-e 2 valamelyik hatványa négy egyforma számjegyre?
493. [3] Hány számjegyből áll a 2100 tízes számrendszerben felírt alakja?
494. [4] Valaki a tízes számrendszerben pontosan kiszámította 21996 és 51996 értékét.
Hány számjegy van a két számban összesen?
495. [3] Van-e olyan kétjegyű szám, melynek minden hatványa ugyanerre a két jegyre
végződik?
496. [3] Van-e olyan 3-jegyű szám, melynek minden hatványa ugyanerre a három jegyre
végződik?
497. [2] 13 különböző pozitív egész szám összege 92. Melyek ezek a számok?
498. [3] Melyik az a csupa különböző jegyből álló legnagyobb természetes szám, amely-
ben bármely három számjegy összege nem osztható 19-cel?
499. [3] Egy matematikaórán a tanár felírt egy számot a táblára. Az egyik diák így
szólt: „A szám osztható 31-gyel”. A második: „A szám 30-cal is osztható.”
Egy harmadik diák szerint a szám osztható 29-cel, a negyedik szerint 28-cal,
és így tovább, végül a harmincadik diák azt mondta, hogy a szám osztható 2-
vel. A tanár ezek után közölte, hogy csak két állítás nem volt igaz, s ez a kettő
egymás után hangzott el. Melyik volt a két téves állítás?
500. [4] A tanár egy 50 000-nél kisebb természetes számot írt fel a táblára. Az első
tanuló szerint a szám osztható 2-vel, a második diák szerint osztható 3-mal,
. . . , a 12. tanuló szerint osztható 13-mal. Két egymás után megszólaló diák
kivételével mindenki igazat mondott.
A tanár melyik számot írta fel a táblára?
501. [3] Egy 80 tagú számsorozatról tudjuk, hogy bármely közbülső tagja egyenlő szom-
szédainak szorzatával. Továbbá az első 40 tag szorzata is 8, valamint összes
tagjának szorzata is 8. Határozzuk meg a sorozat első és második elemét.
502. [5] Mutassuk meg, hogy a Fibonacci-sorozat elemei között nem találunk 7k alakú
számot.
10. Különféle számelméleti feladatok 45

503. [5] Az a1 , a2 , . . . , an , . . . sorozat elemei a 0, 1, 2 számok. Tudjuk, hogy a sorozat


nem periodikus. Legyen bn = 0, ha an = 0, különben pedig bn = 1. Legyen
cn = 1, ha an = 2, különben pedig cn = 0. Mutassuk meg, hogy a bn és cn
sorozat egyidejűleg nem lehet periodikus.
504. [4] Mutassuk meg, hogy az a1 = 2, an+1 = an2 − an + 1, n = 1, 2, 3, . . . sorozat
elemei páronként relatív prímek.
505. [3] Van-e olyan abcd négyjegyű szám, melyre abcd − dcba = 1008?
506. [3] Lehet-e egyenlő egy kétjegyű szám a számjegyeinek szorzatával?
507. [2] Egy számot nevezzünk szépnek, ha egyenlő valódi osztóinak szorzatával (ilyen
pl. a 10). Melyik a 10. szép szám?
508. [3] Miért nem lehet köbszám 10150 + 5 · 1050 + 1?
509. [3] Jelöljön n pozitív egész számot. Mutassuk meg, hogy az n + 1 és 8n + 1 számok
nem lehetnek egyszerre köbszámok.
510. [3] Mutassuk meg, hogy ha a, b, c olyan pozitív egészek, melyekre ab, bc, ca
számok mindegyike köbszám, akkor az a, b, c számok is köbszámok.
511. [3] A 0, 1, 2, 3, 4, 5, 6, 7, 8, 9 számjegyek pontosan egyszeri felhasználásával
számokat írtunk fel. Lehet-e 100 ezeknek a számoknak az összege?
512. [3] 7 rabló a zsákmányolt aranyat úgy osztja el, hogy névsor szerint vesznek belőle
annyit, amennyi az ott levő aranyak számának számjegyösszege (pl. ha a soron
következő zsivány előtt 156 db arany van, akkor ő 1 + 5 + 6 = 12 db-ot vesz
el). Két teljes kör után az arany elfogy. Mindenkinek ugyanannyi jutott, csak a
bandavezérnek lett több. Hányadik a névsorban a főnök, s mennyi arany jutott
neki?
513. [3] Két padon 6-6 gyerek ül. Valamennyien különböző életkorúak (az életkorok
egész számok), és az egyik padon ülő gyerekek életkorának összege és szorzata
is megegyezik a másik padon ülők életkorának összegével, ill. szorzatával. A
legidősebb gyerek 16 éves. Hány évesek azok a gyerekek, akik vele egy padon
ülnek?
514. [4] — Nem tudod — kérdezte a lottóhúzás napján egy matematikus a kollégáját
—, hogy milyen számokat húztak ki?
— Képzeld — felelte az —, van köztük olyan szám, amellyel bármely két
kihúzott szám összege osztható!
— Mi ez a szám?
— Ha megmondanám, kitalálnád a nyerőszámokat.
— Legalább azt mondd meg, páros-e vagy páratlan-e ez a szám? — kérte a
matematikus, majd a válasz után felkiáltott:
— Ötösöm van!
Mi volt az öt nyerőszám, ha a telitalálatot elért matematikus csak egy szel-
vénnyel játszott? (Feltesszük, hogy mindketten jól okoskodtak és persze igazat
mondtak.)
46 Feladatok

515. [4] Kovács úr Seholsincs utcában lakik egy házban, amelynek száma 13 és 1300
közötti szám. Horváth úr kíváncsi Kovács úr házának számára, ezért kérdezgetni
kezdi. Horváth: „Nagyobb, mint 500?” Kovács válaszol, de hazudik. (Horváth
ezt nem tudja.) Horváth: „Négyzetszám?” Kovács válaszol, de hazudik. (Hor-
váth ezt nem tudja.) Horváth: „Köbszám?” Kovács válaszol és igazat mond.
Horváth: „Ha most megmondod, hogy a szám utolsó előtti számjegye 1-es vagy
sem, akkor meg tudom mondani, hogy hol laksz.” Kovács válaszol. Erre Hor-
váth megmondja az általa helyesnek tartott házszámot, amire Kovács azt mond-
ja: „Tévedtél.”
Mi Kovács úr házszáma?
516. [5] Gyermekeim években kifejezett életkorának szorzata 1664. A legfiatalabb leg-
alább fele annyi idős, mint a legidősebb. Én 50 éves vagyok. Hány gyermekem
van?
517. [5] X-né dicsekszik a munkatársainak: „Mind a két fiamnak ma van a születésnapja.
Egyik sincs még 10 éves. Találjátok ki, hogy mennyi idősek! Anna, neked meg-
súgom az életkoruk szorzatát.” (Megsúgja.) Anna: „Ebből még nem lehet meg-
határozni.” „Akkor Borinak az életkoruk hányadosát súgom meg.” (Megsúgja.)
Bori: „Még most sem egyértelmű.” „Igazatok van, Borinak a korkülönbséget
kellett volna megmondanom, akkor kitalálhatta volna.”
Hány évesek X-né fiai?
518. [5] Egy reggel a pap azt mondja a sekrestyésnek:
— Ma találkoztam három emberrel. Az években kifejezett életkoruk szorzata
egyenlő 2450-nel, összege pedig kétszerese az ön életkorának. Milyen idősek
azok az emberek?
Délután a sekrestyés bevallja, hogy nem tud válaszolni a kérdésre. Erre a pap
kisegíti:
— Megjegyzem, hogy a három ember közül az egyik idősebb nálam.
Hány éves a pap?
519. [5] „Hányan laknak ebben a házikóban?” — tette fel a kérdést a népszámlálási
összeíró. „Hárman.” „Hány évesek?” „Életkoruk szorzata 225, éveik összege
azonos a házszámmal.” Az összeíró megnézte a házszámot, majd így felelt:
„Nekem ez elég is. Ön a legidősebb?” „Igen” — hangzott a válasz.
Határozzuk meg a lakosok életkorát.
520. [5] Egy matematikai kongresszus szünetében történt. Mikor az egyik résztvevő pro-
fesszortól azt kérdezték kollégái, hány gyereke van és milyen idősek, így felelt:
— Három fiam van; a véletlen úgy hozta, hogy mind a háromnak épp ma van a
születésnapja. Ha években kifejezett életkorukat összeszorzom, 36-ot kapok; ha
viszont összeadom ugyanezt a három számot, akkor pontosan annyit, ahányadi-
ka ma van.
Kisvártatva így hangzott a viszontválasz:
10. Különféle számelméleti feladatok 47

— Ebből még nem tudhatjuk, hogy hány évesek a gyerekek.


— Igaz is, elfelejtettem megmondani: amikor a legkisebb gyerek születését
vártuk, a két idősebbet elküldtük vidékre a nagyszüleikhez.
— Köszönjük, most már tudjuk mind a három gyerek életkorát.
Állapítsuk meg mi is, hány évesek a gyerekek, és azt is, hogy a hónapnak
hányadik napján hangzott el ez a beszélgetés.
521. [5] András és Tamás is kiválaszt magának egy-egy pozitív egész számot, s ezt a
számot megsúgják Petinek. Peti közli velük, hogy a két szám különbsége 1995.
Erre András kijelenti, hogy ő nem tudja megmondani a másik számot, majd
Tamás is ugyanezt mondja. Ezek után András kijelenti, hogy most már tud-
ja, milyen számot választott Tamás; de ha mindketten 1-gyel nagyobb számra
gondoltak volna, akkor még nem tudta volna kitalálni.
Mi volt a két szám?
522. [5] Peti két egymást követő pozitív egész számot választ, ezeket külön-külön felírja
egy-egy cédulára. Egyiket Andrásnak, másikat Tamásnak adja oda (akik tudják,
hogy a két szám szomszédos), mindketten megnézték, hogy milyen szám van a
papírjukra írva, de ezt nem közölték a másikkal. Ezután a következő tartalmas
párbeszéd zajlott le közöttük:
András: „Én nem tudom, hogy nálad milyen szám van.”
Tamás: „Én sem tudom, hogy nálad milyen szám van.”
András: „Én nem tudom, hogy nálad milyen szám van.”
Tamás: „Én sem tudom, hogy nálad milyen szám van.” . . .
András összesen tízszer mondta el, hogy nem tudja, milyen szám van Tamásnál,
és Tamás is tízszer válaszolja azt, hogy ő nem tudja, milyen szám van András-
nál. Ám a tizenegyedik alkalommal András azt mondta: „Most már tudom, hogy
milyen szám van nálad.”
Milyen szám van Tamásnál?
523. [7] Egy tanár így szól két kedves tanítványához, A-hoz, B-hez: „Gondoltam két, 1-
nél nagyobb, nem feltétlenül különböző egész számot. A-nak megsúgom a két
szám szorzatát, B-nek a két szám összegét. Ez az összeg kisebb 100-nál. Anél-
kül, hogy egymással beszélnétek, próbáljátok meg kitalálni holnapig a gondolt
számokat.”
A következő napon a tanítványok így szólnak a tanárhoz:
A: „Nem jöttem rá, hogy melyek a gondolt számok.”
B: „Én tudtam, hogy nem tudod meghatározni a gondolt számokat.”
A: „Akkor már tudom, hogy melyek a gondolt számok.”
B: „Már én is tudom.”
Melyek voltak a gondolt számok?
524. [3] Határozzuk meg a legkisebb x pozitív számot, amelyre [x] · {x} ≥ 3.
48 Feladatok

525. [3] Mely n-re lesz n pozitív osztóinak összege


a) n + 3,
b) n + 6?
526. [3] Melyek azok a háromjegyű számok, amelyek pozitív osztóinak száma 5?
527. [3] Melyik az a legkisebb természetes szám, melynek
a) 10 pozitív osztója, illetve
b) 12 pozitív osztója van?
528. [4] Melyek azok a kétjegyű természetes számok, melyeknek legtöbb osztója van?
529. [4] Tamás felírta egymás után 1-től 2000-ig az egész számokat. Az így kapott N =
= 123 . . . 19992000 számról azt állítja, hogy annak 25 323 osztója van. Andris
szerint 25 322 osztója van. Bizonyítsuk be, hogy mindketten tévedtek.
530. [4] Melyik az a legkisebb pozitív egész szám, melynek fele négyzetszám, harmada
köbszám?
531. [3] Van-e olyan pozitív egész szám, melynek fele négyzetszám, negyede negyedik
hatvány?
532. [4] Melyik az a legkisebb pozitív egész szám, melynek kétszerese négyzetszám,
háromszorosa köbszám, ötszöröse ötödik hatvány?
533. [3] Van-e olyan n pozitív egész szám, amelyre az n! szám pontosan 11 db nullára
végződik?
534. [3] Hány nullára végződik a 100! szám?
535. [4] Határozzuk meg (2n )! prímtényezős alakjában 2 kitevőjét.
100!
536. [4] Mi lesz a 100 tört nevezője az egyszerűsítések után?
6

[3] 1000
537. Osztható-e 7-tel ?
500
538. [4] Van-e olyan n pozitív egész szám, melyre n! osztható 2n -nel?
539. [2] Hány olyan szám van az első 1000 természetes szám között, mely sem 2-vel,
sem 3-mal nem osztható?
540. [2] Hány olyan szám van az első 1000 természetes szám között, mely relatív prím
a 2, 3, 5 számok mindegyikéhez?
541. [2] Hány olyan szám van az első 1000 természetes szám között mely a 2, 3, 5
számok közül
a) legalább az egyikkel,
b) pontosan eggyel,
c) legfeljebb kettővel,
d) pontosan kettővel osztható?
10. Különféle számelméleti feladatok 49

542. [2] 327-et és 539-et elosztjuk ugyanazzal a háromjegyű számmal, s mind a kétszer
ugyanaz a maradék. Mennyi ez a maradék?
543. [1] Egy számot 5-tel osztva 2, 6-tal osztva 5 a maradék. Mennyi a maradék, ha
30-cal osztunk?
544. [1] Melyik az a legkisebb, 7-tel osztható természetes szám, mely 2-vel, 3-mal,
4-gyel, 5-tel vagy 6-tal osztva mindig 1 maradékot ad?
545. [1] Melyik az a legkisebb természetes szám, melyet ha 2-vel, 3-mal, 4-gyel, 5-tel,
ill. 6-tal osztunk, rendre az 1, 2, 3, 4, ill. 5 maradékot kapjuk?
546. [2] Melyik az a legkisebb természetes szám, melyet ha 5-tel, 7-tel, 9-cel osztunk,
rendre a 3, 4, ill. 5 maradékot kapjuk?
547. [2] Mutassuk meg, hogy minden, 10-nél nagyobb egész szám előállítható prímszá-
mok összegeként.
548. [2] Mutassuk meg, hogy minden, 10-nél nagyobb egész szám előállítható összetett
számok összegeként.
549. [3] Melyek azok a számok, amelyek előállíthatók két összetett szám összegeként?
550. [4] Melyek azok a számok, amelyek előállíthatók két, 1-nél nagyobb, egymáshoz
relatív prím szám összegeként?
551. [4] Melyek azok a számok, amelyek előállnak három, 1-nél nagyobb, páronként
relatív prím szám összegeként?
552. [5] Adva van 13 racionális szám, melyek közül bármelyiket hagyjuk is el, a mara-
dék 12 beosztható két hatos csoportba úgy, hogy az egyes csoportokban levő
számok összege megegyezik. Mutassuk meg, hogy mind a 13 szám egyenlő.
553. [6] Adott 10 pozitív egész, amelyek egyikének sincs 20-nál nagyobb prímosztója.
Mutassuk meg, hogy kiválasztható közülük néhány (esetleg csak egy, esetleg
az összes) úgy, hogy a szorzatuk négyzetszám.
554. [6] Adott 11 pozitív egész, amelyek egyikének sincs 30-nál nagyobb prímosztója.
Mutassuk meg, hogy kiválasztható közülük néhány (esetleg csak egy, esetleg
az összes) úgy, hogy a szorzatuk négyzetszám.
555. [6] Adott 1994 db pozitív egész szám, melyek szorzatának 1993 különböző prím-
osztója van. Mutassuk meg, hogy a számok közül néhánynak a szorzata négy-
zetszám.
556. [6] Adott 48 pozitív egész szám, melyek szorzatának 10 különböző prímosztója
van. Mutassuk meg, hogy a számok között van négy olyan, melyek szorzata
négyzetszám.
50 Feladatok

557. [6] Adott 1985 pozitív egész szám, melyeknek egyikének sincs 26-nál nagyobb
prímosztója. Mutassuk meg, hogy a számok között van négy olyan, melyek
szorzata negyedik hatvány.
558. [7] Adott 2k + 1 db pozitív egész szám, melyek szorzatának k különböző prím-
osztója van. Mutassuk meg, hogy a számok között van néhány olyan, melyek
szorzata köbszám.
559. [7] Adott 3k + 1 db pozitív egész szám, melyek szorzatának k különböző prím-
osztója van. Mutassuk meg, hogy a számok között van néhány olyan, melyek
szorzata negyedik hatvány.

11. Számok reciprokainak összege

Jelölések: Legyen a1 , a2 , a3 , . . . , egy számsorozat. Gyakran használjuk az


olyan összegeket, mint a1 + a2 + a3 + · · · + an . Ez az összeg tömörebben is
n
felírható a következő jelöléssel: ak , ill. ak . (Ezt így olvassuk: szumma
k=1 1≤k≤n
k egyenlő egytől n-ig ak .)
Az a1 ·a2 ·a3 ·. . .·an szorzatot szokás a következő — rövidebb — módon is írni:
n
ak , ill. ak . (Ezt így olvassuk: produktum k egyenlő egytől n-ig ak .)
k=1 1≤k≤n


n
1
560. [4] Mutassuk meg, hogy a összeg értéke nem lehet egész szám.
k=1
k
n
1
561. [3] Mutassuk meg, hogy a 2
összeg értéke nem lehet egész szám.
k=1
k
1 1 1
562. [4] Mutassuk meg, hogy az + +···+ összeg értéke nem lehet egész
n n+1 n+m
szám.
1 1 1
563. [5] Mutassuk meg, hogy az + + · · · + összeg értéke nem lehet egész
3 5 2n + 1
szám.
1
564. [6] Mutassuk meg, hogy a összeg értéke nem lehet egész szám.
1≤a<b≤18
ab
1
565. [6] Mutassuk meg, hogy a összeg értéke nem lehet egész szám.
1≤a<b<c≤18
abc

566. [4] Mutassuk meg, hogy a hatványszámok reciprokainak összege 2-nél kisebb.
11. Számok reciprokainak összege 51

567. [5] Mutassuk meg, hogy nincs olyan, különböző számokból álló végtelen számtani
sorozat, melynek mindegyik eleme hatványszám.
568. [5] Tekintsük azokat a természetes számokat, amelyeknek nincs 3-nál nagyobb prí-
mosztója. Mutassuk meg, hogy ezek reciprokainak összege egész szám.

1
569. [5] a1 = 1, an+1 = 1 + a1 · a2 · . . . · an , n ≥ 1. Mutassuk meg, hogy = 2.
a
n=1 n

570. [5] Tekintsük a 2, 3, . . . , n + 1 számokat, valamint az ezekből képezhető páronként


vett szorzatokat, a hármasával vett szorzatokat, a négyesével vett szorzatokat,
. . . , majd adjuk össze ezek reciprokait. Bizonyítsuk be, hogy a kapott vége-
n
redmény .
2
571. [7] Mutassuk meg, hogy a 9-es számjegyet nem tartalmazó, különböző pozitív egé-
szek reciprokainak összege nem lehet nagyobb 30-nál.
[5] √
572. Jelölje an azt az egész számot, melyhez n legközelebb van. Mutassuk meg,

1980
1
hogy a összeg értéke egész szám.
a
k=1 k

573. [4] Meg lehet-e választani az


a) 1, 2, 3, . . . , 13;
1 1 1
b) 1, , , . . . ,
2 3 13
számok előjelét úgy, hogy azok összege 0 legyen?

1 1 1 m
574. [4] 1 + + +···+ = , ahol p páratlan prím. Mutassuk meg, hogy p | m.
2 3 p−1 n

1 1 1 m
575. [4] 1 + + + ···+ = . Mutassuk meg, hogy 5 | m.
2 3 100 n
1 1 1 1 1 m
576. [5] 1 − + − + ···− + = . Mutassuk meg, hogy 1979 | m.
2 3 4 1318 1319 n

1 1 1 1 
577. [4] Lássuk be, hogy = 1 + + 2 + 3 + ... .
n=1
n p prím p p p


1 1 1 1 
578. [4] Lássuk be, hogy = 1 + + + + . . . .
n=1
n2 p prím p2 p4 p6

1 1 1 1
579. [4] Mutassuk meg, hogy + + ···+ 3 < .
33 43 n 12
52 Feladatok

12. Számok és számjegyek

Jelölések: A tízes számrendszerben felírt n természetes szám számjegyeinek összegét


S(n), számjegyeinek számát k(n), a számjegyek szorzatát P (n) jelöli.
580. [3] Mutassuk meg, hogy ha S(n) = S(2n), akkor n osztható 9-cel.
581. [3] Van-e olyan n pozitív egész szám, melyre S(n) = 16 és S(2n) = 17?
582. [4] Az A és B természetes számok bizonyos számjegyek átrendezésével jönnek
létre egymásból. Bizonyítsuk be, hogy S(2A) = S(2B).
583. [2] Bizonyítsuk be, hogy nincs olyan n természetes szám, amelyre
S(n(n − 1)) = S((n + 1)2 ).
584. [3] Van-e olyan n pozitív egész szám, melyre n − S(n) = 1994?
585. [3] Van-e olyan n pozitív egész szám, melyre n · S(n) = 300 003?
586. [3] A pozitív egészeken értelmezett f (n) függvényről tudjuk, hogy
f (n) = 0, ha n < 10, és tetszőleges n ≥ 10 természetes szám mellett f (n) =
= f (S(n)) + 1. Igaz-e, hogy ha 0 < n < m, akkor f (n) ≤ f (m)?
587. [3] Fel lehet-e írni az 1, 2, 3, 4, 5, 6, 7 számjegyek két különböző sorrendjével két
olyan hétjegyű számot, hogy egyik szám a másik kétszerese legyen?
588. [4] Fel lehet-e írni az 1, 2, 3, 4, 5, 6, 7 számjegyek két különböző sorrendjével két
olyan hétjegyű számot, hogy egyik szám a másik osztója legyen?
589. [4] Egy 9-cel osztható 1994-jegyű szám számjegyeinek összege A, az A szám je-
gyeinek összege B, és B számjegyeinek összege C. Állapítsuk meg C értékét.
[4]    
590. S S S 21994 =?
591. [4] Mutassuk meg, hogy két egymás utáni természetes számból legalább az egyik
előállítható n + S(n) alakban.
592. [6] Mutassuk meg, hogy végtelen sok olyan természetes szám van, mely nem állít-
ható elő n + S(n) alakban.
593. [4] Van-e olyan n természetes szám, amelyre S(n) és S(n + 1) is osztható 7-tel?
594. [4] Van-e olyan n természetes szám, amelyre S(n) és S(n + 1) is osztható 13-mal?
595. [4] Van-e olyan n természetes szám, amelyre S(n) és S(n + 1) is osztható 125-tel?
596. [4] Van-e olyan n természetes szám, melyre
a) S(n) = S(n + 1);
b) S(n) = S(n + 1) = S(n + 2)?
12. Számok és számjegyek 53

597. [4] k(21994 ) + k(51994 ) =?


598. [4] Mutassuk meg, hogy k(51090701 ) − k(21090701 ) páros.
599. [4] Kaphatunk-e 2-hatványt úgy, hogy más sorrendben írjuk fel egy legalább két-
jegyű 2-hatvány számjegyeit?
600. [4] A 21994 szám első jegyét letöröljük, s ezt hozzáadjuk a megmaradt számhoz.
A kapott számon megismételjük ezt az eljárást, és mindaddig folytatjuk ezt,
míg eredményül 10-jegyű számot nem kapunk. Tartalmazhatja-e mind a tíz
számjegyet ez a szám?
601. [4] Mutassuk meg, hogy minden n pozitív egész számnak van olyan többszöröse,
melyben mind a tíz számjegy szerepel.
602. [3] Mutassuk meg, hogy egy különböző természetes számokból álló végtelen szám-
tani sorozatnak mindig van olyan eleme, melynek első számjegye 1-es.
603. [4] Mutassuk meg, hogy egy különböző természetes számokból álló végtelen szám-
tani sorozatnak mindig van két olyan eleme, melyekben a számjegyek összege
egyenlő.
604. [5] Mely n pozitív egész számokra lesz P (n) = n2 − 10n − 22?
605. [5] Mutassuk meg, hogy végtelen sok pozitív egész n-re S(n) = P (n).
606. [3] Mutassuk meg, hogy végtelen sok pozitív egész n-re S(n) = P (n).
607. [5] Mutassuk meg, hogy minden pozitív egész n-re k(2n + 1974n ) = k(1974n ).
608. [4] Adjunk meg olyan, 100-jegyű n számot, amelyre S(n)|n.
609. [5] Mutassuk meg, hogy 18 egymást követő háromjegyű szám között van olyan n,
melyre S(n) | n.
610. [5] Mutassuk meg, hogy 39 egymást követő pozitív egész szám között van olyan
n, melyre 11 | S(n).
611. [5] Mutassuk meg, hogy 59 egymást követő pozitív egész szám között van olyan
n, melyre 12 | S(n).
612. [5] Mutassuk meg, hogy 78 egymást követő pozitív egész szám között van olyan
n, melyre 13 | S(n).
613. [5] Mutassuk meg, hogy 99 egymást követő pozitív egész szám között van olyan
n, melyre 14 | S(n).
614. [5] Legyen m tetszőleges természetes szám, x1 = 1, xn+1 = S(m · xn ),
n = 1, 2, 3, . . . . Mutassuk meg, hogy az így kapott sorozatnak mindig van olyan
eleme, mely később is előfordul a sorozatban.
54 Feladatok

615. [6] Mutassuk meg, hogy minden k természetes számhoz található végtelen sok
olyan t természetes szám, mely jegyei között nem tartalmaz 0-t, és S(t) = S(kt).
616. [7] Melyek azok az m pozitív egész számok, amelyekre S(km) = S(m) minden
olyan k egészre, amelyre 1 ≤ k ≤ m?
617. [4] Van-e olyan n természetes szám, melyre k(n) = 20, S(n) = 10, S(7n) = 70,
S(19n) = 19?
618. [4] Van-e olyan n természetes szám, melyre S(n2 ) = 100?
619. [4] Van-e olyan n természetes szám, melyre
S(n) = 1000, S(n2 ) = 10002 ?
620. [5] Mutassuk meg, hogy S(a + b) ≤ S(a) + S(b).
621. [5] Mutassuk meg, hogy S(a · b) ≤ S(a) · S(b).
622. [5] Mutassuk meg, hogy S(n) ≤ 8 · S(8n).
623. [5] Mutassuk meg, hogy S(n) ≤ 5 · S(55 n).
624. [5] Mutassuk meg, hogy S(n) ≤ 5 · S(2n).
625. [6] Mutassuk meg, hogy végtelen sok n természetes számra teljesül az
S(n) > 1996 · S(3n) egyenlőtlenség.
626. [5] Mutassuk meg, hogy végtelen sok olyan n pozitív egész szám van, melyre
S(2n+1 ) ≤ S(2n ).
627. [7] Mutassuk meg, hogy minden k természetes számhoz található olyan n termé-
szetes szám, melyre S(n) = k és k | n.
628. [7] Mutassuk meg, hogy ha n → ∞, akkor S(1994n ) → ∞.
629. [5] Keressük meg azt a legkisebb pozitív egész n számot, mely egyenlő számjegyei
összegének 1995-szörösével: n = 1995 · S(n).
630. [5] Vannak-e olyan a, b, c természetes számok, amelyekre
S(a + b) < 5, S(b + c) < 5, S(c + a) < 5, de S(a + b + c) > 50?
631. [6] Vannak-e olyan x, y, z pozitív egészek, amelyekre
x + S(x) = y + S(y) = z + S(z)?

13. Racionális és irracionális számok

√ 1 1 1
632. [1] Miért nem lehet 2=1+ + − ?
3 3 · 4 3 · 4 · 34
633. [2] Lehet-e két irracionális szám összege és szorzata is racionális?
13. Racionális és irracionális számok 55

634. [2] Lehet-e két irracionális szám összege és különbsége is racionális?


635. [3] Van-e három olyan irracionális szám, melyek közül bármely kettő összege ra-
cionális?
636. [5] Van-e olyan a és b irracionális szám, melyre a b racionális?
637. [3] Mutassuk meg, hogy log2 5 irracionális szám.
638. [4] Mutassuk meg, hogy tg 5◦ irracionális szám.
639. [4] Mutassuk meg, hogy sin 5◦ irracionális szám.
640. [4] Mutassuk meg, hogy minden n pozitív egész számra logn (n + 1) irracionális
szám.
641. [5] Mutassuk meg, hogy 0,123456789101112 . . . irracionális szám (a tizedes-
vessző után 1-gyel kezdve egymást követően a természetes számokat írtuk).
642. [6] Mutassuk meg, hogy 0,23571113171923 . . . irracionális szám (a tizedesvessző
után egymást követően a prímszámokat írtuk).

1
643. [6] Mutassuk meg, hogy a összeg értéke irracionális szám.
n=1
n!

1
644. [6] Mutassuk meg, hogy a 2
összeg értéke irracionális szám.
n=1
(n!)

1
645. [6] Mutassuk meg, hogy a összeg értéke irracionális szám.
n=1
22n

646. [7] Az a1 < a2 < · · · < an < . . . egészek végtelen sorozata olyan, hogy
ak
lim = ∞.
k→∞ a1 · a2 · . . . · ak−1
1 1 1
Mutassuk meg, hogy az + + + . . . összeg értéke irracionális szám.
a1 a2 a3
647. [4] Igazoljuk, hogy minden n pozitív egész számra lg n és lg(n + 1) számok közül
legalább az egyik irracionális.
648. [3] Mutassuk meg, hogy egy végtelen sok elemű számtani sorozat elemei között
nem lehet pontosan két racionális szám.
649. [3] Mutassuk meg, hogy egy végtelen sok elemű számtani sorozat elemei között
nem lehet pontosan két irracionális szám.
√ √ √ √
650. [3] Mutassuk meg, hogy ha x, y, x + y racionális számok, akkor x, y is
racionális.
56 Feladatok

√ √ √ √
651. [5] Mutassuk meg, hogy ha x, y, z, x + y + z racionális számok, akkor x,
√ √
y, z is racionális.
[6]
√ √ √ √
652. Igaz-e, hogy a 1 + 2 + 3 + · · · + n kifejezés értéke mindig irracionális
szám, ha n = 2, 3, 4, . . . ?
653. [6] Mutassuk meg, hogy ha a1 , a2 , . . . , an páronként különböző pozitív egészek,
melyek egyikének sincs négyzetszám
√ osztója,
√ és b1 , √
b2 , . . . , bn nullától külön-
böző egész számok, akkor b1 a1 + b2 a2 + · · · + bn an = 0.
654. [6] Mutassuk meg, hogy tetszőleges x1 , x2 , x3 , . . . , xn valós számokhoz található
olyan a, hogy az ax1 , ax2 , ax3 , . . . , axn számok mindegyike irracionális.
655. [6] Igazoljuk, hogy tetszőleges x1 , x2 , x3 , . . . , xn valós számokhoz található olyan
a, hogy az a + x1 , a + x2 , a + x3 , . . . , a + xn számok mindegyike irracionális.
656. [5] Egy kör középpontjának mindkét koordinátája irracionális. Mutassuk meg, hogy
nincs a körön három olyan pont, melyek mindegyik koordinátája racionális.
[5]
√ √
657. Egy kör középpontja ( 2; 3). Mutassuk meg, hogy nincs a körön két olyan
pont, melyek mindegyik koordinátája racionális.
658. [5] Egy háromszög oldalai 2, n, n + 1 (ahol n 1-nél nagyobb egész), és valamely
belső pontja az oldalaktól x, y, z távolságra van. Bizonyítsuk be, hogy x, y, z
nem lehet mind racionális.
659. [4] Mutassuk meg, hogy egy körbe írt négyzet csúcsainak a kör egy pontjától vett
távolságai között mindig van irracionális.
660. [5] Mutassuk meg, hogy nincs olyan konvex négyszög, melynek átlói 45◦ -os szöget
zárnak be egymással, az egyik átló a másik kétszerese, s a csúcsok koordinátái
racionális számok.
661. [5] Adott az ABC egységnyi oldalú szabályos háromszög. Van-e a háromszög sík-
jában olyan, a csúcsoktól különböző P pont, amelyre a P A, P B, P C szakaszok
mind racionálisak?
662. [4] Adott a koordináta-rendszerben egy négyzet úgy, hogy csúcsainak koordinátái
egész számok és a négyzet oldala egységnyi. Mutassuk meg, hogy tetszőleges
rácspontnak (olyan pontnak, melynek koordinátái egész számok) a négyzet va-
lamelyik csúcsától mért távolsága irracionális.
663. [5] Van-e olyan, egységnyi területű ABCD négyszög, hogy bármely O belső pont-
ra az OAB, OBC, OCD, OAD háromszögek közül legalább az egyiknek a
területét irracionális szám méri?
664. [6] Legyenek α és β olyan pozitív számok, hogy [α · k], k = 1, 2, . . . ; [β · k],
k = 1, 2, . . . együttesen előállítják a természetes számsor minden elemét
ismétlés nélkül. Bizonyítsuk be, hogy ez akkor és csak akkor lehetséges, ha
1 1
α irracionális és + = 1.
α β
14. Egész együtthatós polinomok 57

14. Egész együtthatós polinomok

665. [3] Határozzuk meg a-t, b-t és c-t úgy, hogy az x 4 + x 3 + ax 2 + bx + c polinom
(x − 1)-gyel, (x − 2)-vel, (x − 3)-mal való osztási maradéka rendre 1, 2, 3
legyen.
666. [3] Határozzuk meg a értékét úgy, hogy az ax 3 + 11x 2 + 7x + a polinom osztható
legyen (2x + 3)-mal.
667. [3] Határozzuk meg a és b értékét úgy, hogy a 2x 4 + 5x 3 − 17x 2 + ax + b polinom
osztható legyen (2x 2 − x − 6)-tal.
668. [4] Bizonyítsuk be, hogy az f (x) = (x − 2)100 + (x − 1)50 − 1 polinom osztható
(x 2 − 3x + 2)-vel.
669. [3] Mennyi az együtthatók összege a (2 − 3x + x 2 )1991 (2 + 3x + x 2 )1992 polinomban?
670. [4] Mutassuk meg, hogy az (x 6 − 3x 3 + 1)1993 + (x 7 + x 5 − 1)1993 polinomban x
páratlan kitevőjű hatványai együtthatóinak összege páratlan szám.
671. [4] Az (x −a)(x −b)(x −c)(x −d)−4 = 0 egyenletnek — ahol a, b, c, d különböző
egész számok — egész gyöke r. Mutassuk meg, hogy 4r = a + b + c + d.
672. [4] Mutassuk meg, ha az f (x) egész együtthatós polinom olyan, hogy f (7) = 10,
akkor f (11) = 13 nem lehetséges.
673. [4] Mutassuk meg, ha az f (x) egész együtthatós polinom olyan, hogy |f (3)| =
= |f (7)| = 1, akkor f (x)-nek nincs egész gyöke.
674. [4] Mutassuk meg, ha az f (x) egész együtthatós polinom olyan, hogy adott külön-
böző a, b, c egészekre |f (a)| = |f (b)| = |f (c)| = 1, akkor f (x)-nek nincs egész
gyöke.
675. [4] Mutassuk meg, ha az f (x) egész együtthatós polinom olyan, hogy f (1989) = 1991
és f (1992) = 1993, akkor f (x)-nek nincs egész gyöke.
676. [4] Mutassuk meg, ha az f (x) egész együtthatós polinom olyan, hogy f (19)·f (92) =
= 1991, akkor nincs olyan a egész szám, melyre f (a) = 1992 lenne.
677. [4] Mutassuk meg, ha az f (x) egész együtthatós polinom olyan, hogy öt különböző
egész helyen 1-et vesz fel értékül, akkor nincs olyan a egész szám, melyre
f (a) = −1 lenne.
678. [4] Miért nem vehet fel egy egész együtthatós polinom három különböző egész
helyen 2-t, s valamely más egész helyen 3-at?
679. [5] Mutassuk meg, hogy ha az f (x) egész együtthatós polinom a k, k + 1, k + 2
egész helyeken felvett értékei 3-mal oszthatók, akkor minden egész helyen 3-
mal osztható értéket vesz fel.
58 Feladatok

680. [6] Az f (x) egész együtthatós polinom bármely egész helyen olyan értéket vesz
fel, mely osztható az a1 , a2 , . . . , an egész számok valamelyikével. Mutassuk
meg, hogy ekkor van olyan ai , hogy az f (x) minden egész helyen osztható lesz
ai -vel.
681. [7] Igazoljuk, ha az f (x) és g(x) egész együtthatós polinom olyan, hogy bármely n
egész számra g(n) | f (n), akkor van olyan h(x) polinom, hogy f (x) = g(x) · h(x).
682. [6] Mutassuk meg, ha az f (x) egész együtthatós polinomhoz van olyan k termé-
szetes szám, hogy f (1), f (2), . . . , f (k) egyike sem osztható k-val, akkor a
polinomnak nincs egész gyöke.
683. [4] Miért nem lehet egy legalább negyedfokú, egész együtthatós polinomnak négy
különböző egész gyöke, ha valamely egész helyen 7 az értéke?
684. [4] Miért nem lehet egy egész együtthatós polinomnak négy különböző egész he-
lyen 7 az értéke, ha valamely egész számra 14 az értéke?
685. [5] Mutassuk meg, hogy nem létezik olyan f (x) egész együtthatós polinom, amely-
re különböző a, b, c egész számok mellett f (a) = b, f (b) = c, f (c) = a lenne.
686. [5] Legyen f (x) = g(x) · h(x), ahol f (x), g(x), h(x) egész együtthatós polinomok.
Mutassuk meg, hogy f (x) értéke nem lehet végtelen sok egész helyen prím-
szám.
687. [5] Bizonyítsuk be, hogy ha a hetedfokú egész együtthatós f (x) polinom értéke
hét különböző egész helyen +1 vagy −1, akkor a polinom nem bontható fel két
egész együtthatós polinom szorzatára.
688. [6] Mutassuk meg, hogy az (x − a1 ) · (x − a2 ) · . . . · (x − an ) − 1 polinom nem írható
fel két polinom szorzataként.
689. [7] Mutassuk meg, hogy az (x − a1 )2 · (x − a2 )2 · . . . · (x − an )2 + 1 polinom nem
írható fel két polinom szorzataként.

15. Kombinatorika a számelméletben

690. [3] Mutassuk meg, hogy ha az 1, 2, 3, . . . , 2n számok közül kiválasztunk n + 1-et,


azok között mindig lesz kettő, melyek relatív prímek.
(Igaz marad-e az állítás n + 1 helyett n-re megfogalmazva?)
691. [4] Mutassuk meg, hogy ha az 1, 2, 3, . . . , 2n számok közül kiválasztunk n + 1-et,
azok körében az x + y = z egyenlet megoldható lesz.
(Igaz marad-e az állítás n + 1 helyett n-re megfogalmazva?)
15. Kombinatorika a számelméletben 59

692. [4] Bizonyítsuk be, hogy ha 52 különböző pozitív egész szám egyike sem nagyobb,
mint 100; akkor kiválasztható a számok közül három úgy, hogy közülük kettő-
nek az összege egyenlő a harmadikkal.
693. [4] Bizonyítsuk be, hogy ha az 1, 2, 3, . . . , 100 számok közül kiválasztunk 69
különböző számot, akkor ezek között van három olyan szám, melyek összege
is a kiválasztottak között van.
694. [4] Mutassuk meg, hogy ha az 1, 2, 3, . . . , 2n számok közül kiválasztunk n + 1-et,
akkor azok között mindig lesz olyan kettő, hogy az egyik osztója a másiknak.
(Igaz marad-e az állítás n + 1 helyett n-re megfogalmazva?)
695. [5] Mutassuk meg, hogy ha az 1, 2, 3, . . . , 2n számok közül kiválasztunk n + 1-et,
akkor lesz olyan k egész szám (1 ≤ k ≤ 2n), amely a kiválasztott számok közül
kettővel is osztható.
(Igaz marad-e az állítás n + 1 helyett n-re megfogalmazva?)
696. [6] Mutassuk meg, hogy az 1 ≤ a1 < a2 < · · · < an ≤ 2n számok körében képzett
ai + aj összegek között (i ≤ j ) van olyan, amely 2-nek pozitív egész kitevős
hatványa.
697. [4] Mutassuk meg, hogy ha az 1 ≤ a1 < a2 < · · · < am ≤ N számok közül
m
1
bármely két különböző számra [ai , aj ] > N, akkor < 2.
a
i=1 i

698. [6] Mutassuk meg, hogy ha az 1 ≤ a1 < a2 < · · · < am ≤ N számok közül

bármely két különböző számra [ai , aj ] ≤ N, akkor m ≤ 2 N .
699. [5] Mutassuk meg, hogy ha az 1 ≤ a1 < a2 < · · · < an ≤ 2n számok közül
 2n 
bármely két különböző számra [ai , aj ] > 2n, akkor a1 > .
3
700. [4] Mutassuk meg, hogy az 1 ≤ a1 < a2 < · · · < am számok legkisebb közös
többszöröse legalább m · a1 .
701. [5] Mutassuk meg, hogy ha az N ≥ a1 > a2 > · · · > am > 0 egész számok közül
bármely két különböző számra [ai , aj ] ≤ N, akkor iai ≤ N.
702. [6] Az 1 < a1 < a2 < · · · < ak ≤ n számokra igaz, hogy egyik ai sem osztója a
többi szorzatának. Mutassuk meg, hogy k ≤ π(n), ahol π(n) a prímek száma
n-ig.
703. [7] [Erdős–Szekeres tétel.] Mutassuk meg, hogy a különböző valós számokból álló
a1 , a2 , . . . , amn+1 sorozatnak vagy létezik egy m-nél hosszabb csökkenő részso-
rozata, vagy van egy n-nél hosszabb növekvő részsorozata.
60 Feladatok

704. [5] Az a1 , a2 , . . . , ak pozitív egész számok nem nagyobbak n-nél, s az ai + aj


összegek mind különbözők. Jelölje f (n) az ilyen tulajdonságú sorozatok elemei
számának (tehát k-nak) a maximumát. Mutassuk meg, hogy

a) f (n) < 2n + 1;

b) f (n) ≥ 3 n.
705. [4] Bizonyítsuk be, hogy 101 különböző, 99-nél nem nagyobb abszolútértékű egész
szám közt van olyan három, melyek összege 0.
706. [4] Száz szám összege 0. Bizonyítsuk be, hogy a közülük kiválasztható számpárok
között legalább 99 van, amelyben a két tag összege nem negatív.
707. [5] n darab valós szám összege 99. Tudjuk, hogy a számok abszolút értéke leg-
feljebb 3. Bizonyítsuk be, hogy kiválasztható közülük néhány olyan, amelyek
összege 32 és 34 közé esik.
708. [4] Adott 50 szám, melyek összege 100. Bizonyítsuk be, hogy a számok közül
kiválasztható 3 úgy, hogy az összegük legalább 6 legyen.
709. [4] Adott hét különböző pozitív egész szám, melyek összege 100. Bizonyítsuk be,
hogy van köztük három, melyek összege legalább 50.
710. [4] Adott 101 szám. Bárhogyan választunk ki közülük 50-et, ezek összege kisebb,
mint a ki nem választottak összege. Bizonyítsuk be, hogy a számok mind pozi-
tívak.
711. [4] a1 , a2 , . . . , a16 száznál nem nagyobb, különböző pozitív egész számok. Bizo-
nyítsuk be, hogy van köztük négy különböző, amelyekre ai + aj = ak + al .
712. [4] Legyen a1 , a2 , . . . természetes számokból álló, szigorúan monoton növekvő so-
rozat. Bármely természetes szám vagy eleme a sorozatnak, vagy egyértelműen
előállítható a sorozat két különböző tagjának összegeként. Mutassuk meg, hogy
an < n2 .

16. Számkonstrukciók

713. [3] Melyik az a legnagyobb ötjegyű szám, amelynek minden számjegye nagyobb,
mint a mögötte álló számjegyek összege?
714. [3] Melyik az a legnagyobb egész szám, amelyben a harmadik jeggyel kezdődően
minden számjegy az előző két számjegy összege?
715. [3] Melyik az a legnagyobb hatjegyű szám, amelyben a harmadik jeggyel kezdő-
dően minden számjegy az előző két számjegy szorzata?
16. Számkonstrukciók 61

716. [3] Melyik az a legnagyobb hatjegyű szám, amelynek minden számjegye legalább
akkora, mint a mögötte álló számjegyek szorzata?
717. [3] Van-e olyan háromjegyű szám, amelyet számjegyei összegével akár növelünk,
akár csökkentünk, csupa egyenlő jeggyel írt számot kapunk?
718. [3] Az összes számjegyet pontosan egyszer felhasználva írjunk fel öt olyan számot,
melyek közül az egyik kétszerese, a másik háromszorosa, a harmadik négysze-
rese, a negyedik ötszöröse a legkisebbnek.
719. [3] Az összes számjegyet pontosan egyszer felhasználva írjunk fel három olyan
számot, melyek közül az egyik háromszorosa, a másik ötszöröse a legkisebbnek.
720. [3] Az összes, 0-tól különböző számjegyet pontosan egyszer felhasználva írjunk fel
három olyan számot, melyek közül az egyik háromszorosa, a másik ötszöröse
a legkisebbnek.
721. [4] Az összes, 0-tól különböző számjegyet pontosan egyszer felhasználva írjunk fel
három olyan számot, melyek úgy aránylanak egymáshoz, mint 1 : 2 : 3.
722. [3] Az összes számjegyet pontosan egyszer felhasználva írjunk fel három olyan
számot, melyek úgy aránylanak egymáshoz, mint 2 : 3 : 4.
723. [3] Keressük meg azt a számot, melynek köbe és a negyedik hatványa együtt mind
a tíz számjegyet tartalmazza, és mindegyik számjegy csak egyszer szerepel a
két szám valamelyikében.
724. [5] Keressünk olyan számot, amely és a négyzete az összes, 0-tól különböző szám-
jegyet tartalmazza, és mindegyik számjegy csak egyszer szerepel a két szám
valamelyikében.
725. [4] Az összes, 0-tól különböző számjegyet pontosan egyszer felhasználva írjunk fel
négy olyan számot, melyek közül kettőnek-kettőnek ugyanannyi a szorzata (pl.:
174 · 32 = 96 · 58 = 5568). Keressünk több megoldást.
726. [4] Írjunk fel két olyan számot, melyek szorzatukkal együtt mind a tíz számjegyet
pontosan egyszer tartalmazzák (pl.: 39 · 402 = 15 678). Keressünk több megol-
dást.
727. [4] Írjunk fel két olyan számot, melyek szorzatukkal együtt az összes, 0-tól külön-
böző számjegyet pontosan egyszer tartalmazzák (pl.: 48 · 159 = 7632). Keres-
sünk több megoldást.
728. [4] Írjunk fel két olyan számot, melyek ugyanazokat a számjegyeket tartalmazzák,
mint a két szám szorzata (pl.: 21 · 87 = 1827). Keressünk több megoldást.
729. [4] Keressük meg azokat a háromjegyű számokat, melyek egyenlők számjegyeik
köbének összegével.
62 Feladatok

730. [4] Keressük meg azokat a négyjegyű számokat, melyek egyenlők számjegyeik ne-
gyedik hatványának összegével.
731. [4] Keressük meg azokat az ötjegyű számokat, melyek egyenlők számjegyeik ötö-
dik hatványának összegével.
732. [3] Keressük meg a legnagyobb olyan 8-jegyű számot, melyben bármely számjegy
(a két szélső kivételével) kisebb két szomszédjának számtani közepénél.
733. [4] Keressünk olyan hatjegyű számot, melyre az jellemző, hogy akár 2-vel, akár 3-
mal, akár 4-gyel, akár 5-tel, akár pedig 6-tal szorozzuk meg, mindegyik esetben
olyan számot kapunk eredményül, amely az eredeti szám számjegyeinek más
sorrendjével áll elő.
734. [3] Egy számot nevezzünk szerencsésnek, ha jegyei két csoportba oszthatók úgy,
hogy a két csoportban a számok összege ugyanannyi (pl. 32 843 szerencsés,
hiszen 8 + 2 = 3 + 4 + 3). Melyik a legkisebb szerencsés szám, melynek a szom-
szédja is szerencsés? Melyik a legkisebb négyjegyű szerencsés szám, melynek
a szomszédja is szerencsés?
735. [2] Megadható-e néhány természetes szám úgy, hogy összegük is, szorzatuk is 203
legyen?
736. [3] Adjunk meg 100 (ill. 200) egész számot, melyek összege 0, szorzata 100 (ill.
200).
737. [3] Adjunk meg 1000 darab pozitív egész számot úgy, hogy azok összege ugyan-
annyi legyen, mint a szorzatuk.
738. [3] Készítsünk 3 × 3-as, 5 × 5-ös bűvös négyzeteket.
739. [3] Egy 3 × 3-as táblázat mezőibe írjunk különböző egész számokat úgy, hogy
minden sorban és minden oszlopban ugyanannyi legyen a számok szorzata.
740. [4] Az 1, 2, 3, . . . , 29, 30 számok közül válasszunk ki alkalmasan kilenc különbö-
zőt, és rendezzük el egy 3 × 3-as táblázat mezőibe úgy, hogy minden sorban és
minden oszlopban 270 legyen a számok szorzata.
741. [5] Az 1, 2, 3, . . . , 39, 40 számok közül válasszunk ki alkalmasan kilenc különbö-
zőt, és rendezzük el egy 3 × 3-as táblázat mezőibe úgy, hogy minden sorban,
minden oszlopban és mindkét átlóban ugyanannyi legyen a számok szorzata.
742. [4] Egy 3 × 3-as táblázat mezőibe írjunk különböző egész számokat úgy, hogy
minden sorban, minden oszlopban és a két átlóban ugyanannyi legyen a számok
szorzata. Mutassuk meg, hogy ekkor ez a szorzat köbszám lesz.
743. [5] Az 1, 2, 3, . . . , 29, 30 számok közül válasszunk ki alkalmasan 16 különbözőt és
rendezzük el egy 4×4-es táblázat mezőibe úgy, hogy minden sorban és minden
oszlopban ugyanannyi legyen a számok szorzata.
16. Számkonstrukciók 63

744. [3] Írjunk egy 3 × 4-es táblázat mezőibe számjegyeket úgy, hogy a sorokban ál-
ló négyjegyű számok, és az oszlopokban álló háromjegyű számok mindegyike
osztható legyen 92-vel.
745. [3] Írjunk egy 4 × 4-es táblázat mezőibe nullától különböző egész számokat úgy,
hogy a táblázat bármely 2 × 2-es, 3 × 3-as részének, ill. magának a 4 × 4-es
táblázatnak a csúcsaiban álló négy szám összege nulla legyen.
746. [4] Töltsük ki az 5 × 5-ös táblázat üres mezőit pozitív egész számokkal úgy, hogy
mindegyik sorban, mindegyik oszlopban egy-egy számtani sorozat egymást kö-
vető elemei álljanak.

74
186
103
0

747. [5] Mutassuk meg, hogy egy 2n × 2n-es táblázat mezői kitölthetők a −1, 0, 1
számokkal úgy, hogy a 4n db sor- és oszlopösszeg között ne legyenek egyenlők.
748. [5] Mutassuk meg, hogy ha egy (2n + 1) × (2n + 1)-es táblázat mezőibe a −1, 0, 1
számokat írjuk, akkor a 4n + 2 db sor- és oszlopösszeg között lesznek egyenlők.
749. [3] Az 1, 2, 3, . . . , 20 számokat írjuk fel körbe egy kör kerületén úgy, hogy az egy-
más mellett levő számok különbségének abszolútértéke mindenhol más legyen.
750. [5] Az 1, 2, 3, . . . , 12 számokat írjuk fel körbe egy kör kerületén úgy, hogy bár-
mely, egymás után álló a, b, c számra b2 − ac osztható legyen 13-mal.
751. [3] Az 1, 2, 3, . . . , 3n számokat osszuk szét n db, 3–3 számból álló csoportba úgy,
hogy mindegyik csoportban más-más legyen a számok összege.
752. [4] Az 1, 2, 3, . . . , 60 számok közül válasszunk ki minél többet úgy, hogy a kivá-
lasztott számok között bármely kettőnek a különbsége más-más legyen.
753. [4] Igaz-e a következő állítás: bármely hat természetes számból vagy kiválasztható
három, páronként relatív prím, vagy három olyan választható ki, melyeknek van
1-nél nagyobb közös osztója?
754. [5] Igaz-e a következő állítás: 17 egymást követő egész szám között mindig van
olyan, mely a többihez relatív prím?
755. [3] Az első 1983 pozitív egész közül válasszunk ki minél több számot úgy, hogy
semelyik két kiválasztott szám szorzata ne szerepeljen a kiválasztott számok
között.
756. [3] Igaz-e a következő állítás: ha az 1, 2, 3, . . . , 50 számokból kiválasztunk 30-at,
akkor mindig lesz köztük két olyan, amelyeknek az egyike a másik kétszerese?
64 Feladatok

757. [4] Osszuk szét a természetes számokat két halmazba úgy, hogy egyikben se legyen
végtelen sok elemű mértani sorozat.
758. [4] Osszuk szét a természetes számokat két halmazba úgy, hogy egyikben se legyen
végtelen sok elemű számtani sorozat.
759. [4] Osszuk szét a természetes számokat két halmazba úgy, hogy az egyikben ne
legyen végtelen sok elemű számtani sorozat, a másikban pedig ne legyen három
elemű számtani sorozat.
760. [4] Adjunk meg néhány, páronként különböző differenciájú számtani sorozatot úgy,
hogy bármelyik természetes szám eleme legyen legalább az egyik sorozatnak.
(A differenciák 1-nél nagyobb egészek.)
761. [4] Képezzünk 5 számból álló számtani sorozatot a 0, 1, 2, . . . , 9 számjegyek
mindegyikének egyszeri felhasználásával.
762. [4] Adjunk meg
a) 3, illetve
b) 5 pozitív egész számból álló számtani sorozatot úgy, hogy a számok szorzata
négyzetszám legyen.
763. [4] Adjunk meg végtelen sok szomszédos számpárt úgy, hogy mindegyiküknek
legyen 1-nél nagyobb négyzetszám osztója.
764. [4] Adjunk meg négy szomszédos természetes számot úgy, hogy mindegyiküknek
legyen 1-nél nagyobb négyzetszám osztója.
765. [5] Van-e 21 olyan, egymás után következő pozitív egész szám, hogy a számok
mindegyike osztható a 2, 3, 5, 7, 11, 13 prímek közül legalább eggyel?
766. [7] Adjunk meg végtelen sok olyan m, n számpárt, melyekre az m és n számoknak
ugyanazok a prímosztói, továbbá az m + 1 és az n + 1 számoknak is ugyanazok
a prímosztói.
767. [3] Adjunk meg három olyan, 1-nél nagyobb egész számot, hogy bármely kettő
szorzatához 1-et adva, az így kapott szám osztható legyen a harmadik számmal.
768. [3] Adjunk meg négy különböző pozitív egész számot úgy, hogy bármely három
szorzatához 1-et adva, az így kapott szám osztható legyen a negyedik számmal.
769. [3] Adjunk meg négy különböző egész számot úgy, hogy bármely három összege
osztható legyen a negyedikkel.
770. [4] Osszuk szét az 1, 2, 3, 4, . . . , 14, 15, 16 számokat két 8-as csoportba úgy, hogy
ha az egyik csoportban kiszámítjuk bármelyik két (különböző) szám összegét,
akkor pontosan ugyanazokat a számokat kapjuk eredményül (és mindegyiket
ugyanannyiszor), mint ha a másik csoportban végezzük el ugyanezeket az össze-
adásokat.
16. Számkonstrukciók 65

771. [2] Adjunk meg négy különböző pozitív egész számot úgy, hogy képezve belőlük
a páronkénti összegeket, hat egymást követő számot kapjunk.
772. [2] Adjunk meg négy különböző pozitív egész számot úgy, hogy képezve belőlük
a páronkénti különbségeket, hat egymást követő számot kapjunk.
773. [5] Adjunk meg n db páronként relatív prím számot úgy, hogy közülük akárhány-
nak az összege összetett szám legyen.
774. [3] Adjunk meg n db különböző természetes számot úgy, hogy közülük bármelyik
szám osztója legyen a többi szorzatának.
775. [5] Adjunk meg 8 természetes számot úgy, hogy egyik sem osztója valamely má-
siknak, de bármelyik négyzete osztható a többi szám bármelyikével.
776. [5] Az 1, 2, 3, . . . , 1994 számokból válasszuk ki a legtöbb számot úgy, hogy
ne legyen köztük két olyan szám, melyek összege osztható lenne a két szám
különbségével.
777. [6] Adjunk meg 100 különböző természetes számot úgy, hogy bármely kettő össze-
ge osztható legyen különbségükkel.
778. [6] Mutassuk meg, hogy a 0, 1, 2, . . . , 3k − 1 számok közül megadható 2k db kü-
lönböző szám úgy, hogy közülük semelyik ne legyen valamely másik kettőnek
a számtani közepe.
779. [3] Adjuk meg az {1, 2, 3, 4, 5, 6} halmaznak hat olyan részhalmazát, hogy közülük
bármely kettőnek egy közös eleme legyen.
780. [3] Adjuk meg a természetes számoknak három olyan végtelen részhalmazát úgy,
hogy a részhalmazok közül bármely kettőnek végtelen sok közös eleme legyen,
ám a három részhalmaznak ne legyen közös eleme.
781. [6] Van-e nemnegatív egész számokból álló két olyan végtelen A és B halmaz,
hogy bármely pozitív egész szám pontosan egyféleképpen írható fel egy A-hoz
és egy B-hez tartozó szám összegeként?
782. [6] Van-e nemnegatív egész számokból álló három olyan végtelen A, B és C hal-
maz, hogy bármely pozitív egész szám pontosan egyféleképpen írható fel egy
A-hoz, egy B-hez és egy C-hez tartozó szám összegeként?
783. [6] A nemnegatív egészek halmazát osszuk fel végtelen sok, közös elem nélküli
részhalmazra úgy, hogy mindegyiknek végtelen sok eleme legyen, és a rész-
halmazok hasonlók egymáshoz, azaz egyik részhalmazból megkapjuk bármely
másikat, ha egy alkalmasan választott egész számot hozzáadunk a részhalmaz
elemeihez („eltoljuk” a részhalmazt).
784. [3] Osszuk két csoportba az 1, 2, 3, 4, 5, 6, 7, 8 számokat úgy, hogy egyik cso-
portban se legyen két olyan szám, melyek összegének fele is ugyanabban a
csoportban lenne.
66 Feladatok

785. [3] Osszuk két csoportba az első 1000 négyzetszámot úgy, hogy a két csoportban
a négyzetszámok összege egyenlő legyen.
786. [5] Osszuk az első 27 négyzetszámot két csoportba úgy, hogy azokban egyenlő
legyen a számok összege.
787. [4] Osszuk három csoportba az első 81 négyzetszámot úgy, hogy a három csoport-
ban a négyzetszámok összege egyenlő legyen, és mindegyik csoportban 27–27
szám legyen.
788. [3] Keressünk olyan n-jegyű számot, amelynek első számjegye egyenlő a szám-
jegyei között található 0-k számával, második számjegye az 1-esek számával,
harmadik számjegye a 2-esek számával és így tovább.
789. [3] Írjunk a pontok helyébe számokat úgy, hogy a keretben igaz állítás legyen.

Ebben a keretben a 0 számjegyből pontosan . . . darab van, az 1-ből


. . . darab, a 2-ből . . . darab, a 3-ból . . . darab, a 4-ből . . . darab,
az 5-ből . . . darab, a 6-ból . . . darab, a 7-ből . . . darab, a 8-ból
. . . darab, s végül a 9-ből . . . darab.

790. [3] Adjunk meg olyan x1 , x2 , . . . , x10 számokat, melyekre teljesülnek a következő
feltételek: x1 a [0;1] intervallumba esik; x1 és x2 közül az egyik a [0;1] inter-
vallum egyik felébe, a másik a másik felébe esik; x1 , x2 , x3 közül az egyik a
[0;1] intervallum egyik harmadába, a másik egy másik harmadába, a harmadik
a harmadik harmadába esik és így tovább.

17. Melyik szám a nagyobb?

791. [1] 2300 vagy 3200 ?


792. [1] 2100 vagy 1030 ?
793. [1] 321 vagy 231 ?
794. [2] 202303 vagy 303202 ?
795. [2] 9920 vagy 999910 ?
796. [2] 920 vagy 2713 ?
797. [2] 3500 vagy 7300 ?
798. [2] 199110 vagy 199010 + 19909 ?
799. [3] 792 vagy 891 ?
17. Melyik szám a nagyobb? 67

800. [2] 242 vagy 327 ?


801. [3] 2100 + 3100 vagy 4100 ?
999 1000
802. [2] vagy ?
1000 1001
210 − 1 220 − 1
803. [2] 10
vagy ?
2 220
434343 434343434343
804. [2] vagy ?
575757 575757575757
22222221 33333331
805. [2] vagy ?
22222223 33333334
222221 444443
806. [2] vagy ?
333332 666665
3 300000001
807. [1] vagy ?
5 500000001
√ √
808. [3] 2 3
vagy 3 2 ?
√ √
809. [3] 7 5
vagy 5 7 ?
810. [2] 3111 vagy 1714 ?
811. [2] 12723 vagy 51318 ?
812. [2] 111979 vagy 371320 ?
100 150
813. [4] 23 vagy 32 ?

814. [3] 2 log12 145 vagy 15?
815. [3] log4 9 vagy log9 25?
816. [4] log5 6 vagy log6 7?
817. [4] lg2 11 vagy lg 12?

818. [4] sin 1 vagy log3 7?
819. [4] 1, 011000 vagy 1000?
820. [4] Mutassuk meg, hogy 479 < 2100 + 3100 < 480 .
68 Feladatok

18. Egyenletek és egyenletrendszerek

821. [3] Miért nem lehet megoldani az alábbi egyenleteket a valós számok körében?
√ √
a) x − 2 + 3 − x = x − 4.
√ √
b) x 2 + 1 + x 2 + 2 = 2.
2
c) x 2 + 2 = 2.
x
2
d) 2sin x = sin x.
e) sin x · cos x = sin 60◦ .
π
f) sin x · sin x + = 1.
3
1
g) sin x · sin(x + π) = .
3
h) x 10 − x 7 + x 2 − x + 1 = 0.

Oldjuk meg az alábbi egyenleteket a valós számok halmazán. (822–881. feladatok)



822. [1] (x + 2) x − 1 = 0.
1 1
823. [1] x + = 1991 + .
x − 1991 x − 1991
x2 − 1
824. [1] = 0.
x3 − 1
2 2
825. [2] (x 3 − 1) + (x 4 − 1) = 0.
826. [3] 2x + 3x = 35.
827. [3] 2x + 3x = 5x .
  
828. [3] x 2 − 9 + x 2 − 16 + x 2 − 25 = 7.
   35
829. [3] x 2 − 9 + x 2 − 16 + x 2 − 25 = .
x
830. [3] 3x + 27 = 18x.
831. [3] 4x = 1 + 2x.
832. [3] 8x (3x + 1) = 4.
√ x √ x
833. [4] (2 + 2) + (2 − 2) = 2−x .
834. [4] log1991 (x − 3) + log1992 (x − 3) = 3 − lg(x 5 − 24).
√ √ √ √ √ √
835. [4] sin x + sin 3x + sin 5x = cos x + cos 3x + cos 5x.
18. Egyenletek és egyenletrendszerek 69

836. [3] cos x = y 2 − 2y + 2.


 √  √
837. [3] (2x + 3) 2x + 3 = (6 − x) 6 − x.
x − 5 x − 4 x − 3 x − 100 x − 101 x − 102
838. [3] + + = + + .
100 101 102 5 4 3
x−a x−b x−c
839. [3] + + = 3, (a + b)(b + c)(c + a) = 0.
b+c c+a a+b
x−a x−b x−c 3x
840. [3] + + = , (a + b)(b + c)(c + a) = 0.
b+c c+a a+b a+b+c
x − bc x − ca x − ab
841. [3] + + = a + b + c, (a + b)(b + c)(c + a) = 0.
b+c c+a a+b
x x(x − 1) x(x − 1)(x − 2) · · · (x − n + 1)
842. [4] 1− + − · · · + (−1)n · = 0.
1 1·2 n!
√ √
843. [4] x − 2 + 3 − x = x 2 − 5x + 7.
 
844. [4] x 2 + x − 1 + x − x 2 + 1 = x 2 − x + 2.
845. [3] x 2 + y 2 = 2x + 2y − 2.
846. [3] 8(x 4 + y 4 ) − 4(x 2 + y 2 ) + 1 = 0.
36 4 √ 
847. [4] √ +√ = 28 − 4 x − 2 − y − 1.
x−2 y−1
848. [3] (x + y)2 = (x + 1)(y − 1).
√ √
849. [4] 3x 2 + 6x + 7 + 5x 2 + 10x + 14 = 4 − 2x − x 2 .

850. [3] x 2 + 2y 2 − 2yz = 100
.
2xy − z2 = 100

851. [4] (x 2 − a 2 )2 = 4ax + 1.


852. [3] x 2 + y 2 + z2 + t 2 = x(y + z + t).

853. [3] x 2 + 10y + 41 = 0 ⎪ ⎬
y 2 − 2z − 23 = 0 .


z2 − 6x + 17 = 0

854. [3] x 2 − yz = y − x ⎪ ⎬
y 2 − zx = z − y .


z2 − xy = x − z
70 Feladatok


855. [3] x 2 − xy − xz + z2 = 0 ⎪⎬
x 2 − xz − yz + 3y 2 = 2 .


y 2 + xy + yz − z2 = 2
2x 2 2y 2 2z2
856. [4] = y, = z, = x.
1 + x2 1 + y2 1 + z2



1 1 1 1 1 1 1 1
857. [4] x= y+ ,y= z+ ,z= t+ ,t= x+ .
2 y 2 z 2 t 2 x
xy 2 yz 6 zx 3
858. [3] = , = , = .
x+y 3 y+z 5 z+x 4
xyz xyz 6 xyz 3
859. [4] = 2, = , = .
x+y y+z 5 z+x 2

860. [3] x 2 + xy = 10
.
y 2 + xy = 15
1
861. [4] x 3 − x 2 − x = .
3 ⎫
1⎪
862. [5] x3 − y2 − y = ⎪ ⎪
3⎪ ⎪


1
y −z −z =
3 2 .
3⎪ ⎪

1⎪ ⎪
z3 − x 2 − x = ⎪ ⎭
3
863. [3] (x − y + z)2 = x 2 − y 2 + z2 .
√ √ √ √
864. [4] x − y + z = x − y + z.
865. [4] x 3 − 3x 2 + 3x + 7 = 0.
866. [4] 4x 3 + 18x 2 + 27x + 10 = 0.
867. [4] x 4 + 4x 3 + 6x 2 + 4x − 2 = 0.
868. [4] 9x 3 + 3x 2 + 3x + 1 = 0.
869. [4] 3x 4 + 4x 3 − 6x 2 + 4x − 1 = 0.
870. [4] x 4 − x 2 + 2x − 1 = 0.
871. [4] x 4 + 4x 3 + 10x 2 + 12x − 16 = 0.
√ √
872. [4] 2 − x + x − 1 − 1 = 0.
3

√ √
873. [4] 1 + x + 1 − x = 2.
3 3
18. Egyenletek és egyenletrendszerek 71
√ √ √
874. [4] x−1+ x − 2 = 2x − 3.
3 3 3

[4] x5
875. x = 5.
[4]
876. [x ] + [x 2 ] + [x] = {x} − 1.
3

877. [4] [x] = 1 + {x 2 − 1}.


878. [4] x 3 − [x] = 3.
 
879. [4] lg x = [x] − x .
 
 √

880. [4] x x x x . . . = 1994.
 
[4]

881. x= −3 + 4 −3 + 4 −3 + 4x.

882. [4] Tudjuk, hogy a 2 + b2 + c2 = ab + bc + ca. Mutassuk meg, hogy a = b = c. Oldjuk


meg a 4x + 9x + 25x = 6x + 10x + 15x egyenletet.
883. [3] Mutassuk meg, hogy sin2n x + cos2n x ≤ 1, n ∈ N , n > 0.
884. [4] Oldjuk meg a következő egyenleteket.
a) sin10 x + cos10 x = 1, b) sin9 x + cos11 x = 1,
c) sin10 x − cos10 x = 1, d) sin9 x − cos11 x = 1.
885. [4] Oldjuk meg az 
x3 + y3 = 1
x4 + y4 = 1
egyenletrendszert.
886. [4] Mutassuk meg, hogy az x 3 + 4x 2 + 6x + c = 0 egyenletnek semmilyen c valós
szám esetén sem lehet három különböző valós gyöke.
[5]

887. Oldjuk meg az (x 3 − 6)3 = 6 + x + 6 egyenletet.
3

888. [5] Oldjuk meg az (x 2 + x − 2)3 + x 2 − 2 = x 3 egyenletet.


889. [3] Az x 2 + px + q = 0 egyenlet gyökei p és q. Határozzuk meg p és q értékét.
890. [3] Igazoljuk, hogy az x 10 + px 7 + q = 0 egyenletnek nincs egész gyöke, ha p és q
páratlan egészek.
72 Feladatok

19. Egyenlőtlenségek

1
891. [1] Néhány szám összege 1. Lehet-e négyzetösszegük kisebb, mint ?
100
892. [1] Néhány szám összege 1. Lehet-e négyzetösszegük nagyobb, mint 100?
893. [3] 100 pozitív szám összege 1. Közülük minden lehetséges módon kiválasztunk
50-et és kiszámoljuk azok szorzatát. Mutassuk meg, hogy az 50 tényezős szor-
zatok összege kisebb 1-nél.

894. [2] Határozzuk meg a 0, 999 . . . 9 (60 db 9-es) szám első 60 tizedesjegyét.
√ √ √ √
895. [3] Mutassuk meg, hogy 1 + 2 + 3 + · · · + n > n, (n > 1).
1 1 1 1 √
896. [3] Mutassuk meg, hogy √ + √ + √ + · · · + √ > n, (n > 1).
1 2 3 n
1 1 1 1 1
897. [3] Mutassuk meg, hogy + + + ···+ > , (n > 1).
n+1 n+2 n+3 2n 2
1 1 1 1 1
898. [3] Mutassuk meg, hogy + + + · · · + 2 > 1 − , (n > 1).
n+1 n+2 n+3 n n
1 1 1 1
889. [3] Mutassuk meg, hogy 1 − < √ +√ + ···+ √ < 1.
n 2
n +1 2
n +2 2
n +n
1 1 1 1
900. [3] Mutassuk meg, hogy + + + ··· + > 1.
51 52 53 200
1 1 1 1
901. [3] Mutassuk meg, hogy 1 + + + + ···+ > 4.
2 3 4 64
1 1 1 1
902. [3] Mutassuk meg, hogy 1 < + + + ···+ < 2.
5 6 7 17
1 1 1 1
903. [4] Mutassuk meg, hogy 1+ + + + ··· + < 3.
1! 2! 3! n!
1 1 1 1
904. [4] Mutassuk meg, hogy 1 + 2 + 2 + 2 + · · · + 2 < 2.
2 3 4 n
 


905. [3] Mutassuk meg, hogy 6 + 6 + 6 + 6 < 3.

906. [3] Mutassuk


 meg, hogy 
  
√ √ √
12 + 12 + 12 + 20 + 20 + 20 + 30 + 30 + 30 < 15.
19. Egyenlőtlenségek 73
 
 
 
 √
907. [4] Bizonyítsuk be, hogy 100 + 99 + 98 + · · · + 2 + 1 < 11.
 
 
 
 √
908. [4] Bizonyítsuk be, hogy 1 + n + n + n3 + · · · + nn < n
2 (n > 1).
  
[5] √ 9
909. Bizonyítsuk be, hogy 1+ 2+ 3 + ···+ n < .
5
 
  

 √
910. [7] Mutassuk meg, hogy 2 3 4 . . . n < 3, ahol n = 2, 3, 4, . . .

911. [3] Mutassuk meg, hogy ha a és b pozitív számok, akkor


a+b a b
≤ + .
1+a+b 1+a 1+b
1 1 1
912. [3] Legyen a > b > c. Mutassuk meg, hogy + + > 0.
a−b b−c c−a
913. [3] Legyen a1 ≤ a2 ≤ . . . ≤ a10 . Mutassuk meg, hogy
a1 + a2 + · · · + a6 a1 + a2 + · · · + a10
≤ .
6 10
914. [4] Legyen a > b > c. Mutassuk meg, hogy

a 2 (b − c) + b2 (c − a) + c2 (a − b) > 0.
915. [4] Mutassuk meg, hogy tetszőleges a és b pozitív valós számokra teljesül az

a 3 + b3 ≥ a 2 b + ab2
egyenlőtlenség.
916. [4] Mutassuk meg, hogy tetszőleges a és b valós számokra teljesül az

a 4 + b4 ≥ ab(a 2 + b2 )
egyenlőtlenség.
[4]
917. Mutassuk meg, hogy tetszőleges a és b pozitív valós számokra teljesül az

a 3 b3
a 2 + b2 ≤ +
b a
egyenlőtlenség.
74 Feladatok

918. [4] Mutassuk meg, hogy tetszőleges a és b valós számokra teljesül az

a + b a 2 + b2 a 3 + b3
· ≤
2 2 2
egyenlőtlenség.
919. [4] Mutassuk meg, hogy tetszőleges a és b valós számokra teljesül az

a + b a 3 + b3 a 4 + b4
· ≤
2 2 2
egyenlőtlenség.
[4]
920. Mutassuk meg, hogy tetszőleges a és b valós számokra teljesül az

a + b a 2 + b2 a 3 + b3 a 6 + b6
· · ≤
2 2 2 2
egyenlőtlenség.
[4]
921. Határozzuk meg a következő kifejezések minimumát.
a) x 2 + y 2 + 2x − 4y + 6,
b) x 6 + 2x 4 + 2x 3 + x 2 + 2x − 1,
c) 2x 2 − 8xy + 17y 2 − 16x − 4y + 2062.

Igazoljuk az alábbi egyenlőtlenségeket. (922–933. feladatok)


922. [3] a 2 + b2 + c2 ≥ ab + bc + ca.
3
923. [4] a 2 + b2 + c2 + ≥ a + b + c.
4
924. [4] a 4 + b4 + 2 ≥ 4ab.
925. [4] a 4 + 1 > a.
926. [4] a 2 + b2 + 1 ≥ ab + a + b.
a2
927. [4] + b2 + c2 ≥ ab − ac + 2bc.
4
928. [4] a 2 + b2 + c2 + 4 ≥ ab + 3b + 2c.
929. [4] a 2 + b2 + c2 + d 2 ≥ ab + ac + ad.
2
930. [4] a 2 + b2 + c2 + d 2 − ab − bc − cd − d + ≥ 0.
5
931. [4] a 2 + b2 + c2 + d 2 + e2 ≥ a(b + c + d + e).

A 921–933. feladatokban x, y, a, b, c, d, e tetszőleges valós számokat jelent, ahol a


kifejezés értelmezhető.
19. Egyenlőtlenségek 75

a 2 b2 c2 c b a
932. [6] 2
+ 2 + 2 ≥ + + , ahol abc = 0.
b c a b a c
a 4 b4 a b a 2 b2
933. [6] 4
+ 4 + + ≥ 2 + 2 + 2, ahol a > 0, b > 0.
b a b a b a
A számtani és mértani közepek közötti
a1 + a2 + · · · + an √
≥ n a1 · a2 · . . . · an a1 ≥ 0, a2 ≥ 0, . . . , an ≥ 0
n
egyenlőtlenség segítségével igazoljuk a következő egyenlőtlenségeket. Az egyenlőtlen-
ségekben szereplő számok mindegyike pozitív. (934–996. feladatok)
1
934. [3] a) a + ≥ 2,
a
a2 + 2
b) √ ≥ 2,
a2 + 1
a2 + 3
c) √ > 2,
a2 + 2
1
d) a 2 + 2 ≥ 1,
a +1
a+b b+c c+a
e) + + ≥ 6,
c a b
a b c
f) + + ≥ 3.
b c a
1 1 4 16 64
935. [4] + + + ≥ .
a b c d a+b+c+d
936. [3] (a + b)(b + c)(c + a) ≥ 8abc.
937. [3] (a + 1)(b + 1)(a + c)(b + c) ≥ 16abc.
ab bc ca a+b+c
938. [4] + + ≤ .
a+b b+c c+a 2
939. [4] (a 2 + b2 )c + (b2 + c2 )a + (c2 + a 2 )b ≥ 6abc.
940. [4] ab(a + b) + bc(b + c) + ca(c + a) ≥ 6abc.
941. [4] a 2 (1 + b2 ) + b2 (1 + c2 ) + c2 (1 + a 2 ) ≥ 6abc.
942. [4] (a + b + c)(ab + bc + ca) ≥ 9abc.
ab bc ca
943. [4] + + ≥ a + b + c.
c a b
944. [4] abc ≥ (a + b − c)(b + c − a)(c + a − b).
√ √
945. [4] a a 2 + c2 + b b2 + c2 ≤ a 2 + b2 + c2 .
76 Feladatok

946. [3] (a 2 + a + 1)(b2 + b + 1)(c2 + c + 1)(d 2 + d + 1) ≥ 81abcd.


√ √ √
947. [4] ab + bc + ca ≥ a bc + b ca + c ab.
√ √ √
948. [4] a + b + c ≥ ab + bc + ca.
  
[4] a b c
949. + + > 2.
b+c c+a a+b
a b c 3
950. [4] + + ≥ .
b+c c+a a+b 2
a2 b2 c2 a+b+c
951. [4] + + ≥ .
b+c c+a a+b 2
a 2 + b2 b2 + c2 c2 + a 2 a 3 b3 c3
952. [4] a + b + c ≤ + + ≤ + + .
2c 2a 2b bc ca ab
953. [4] (a + b + c)2 ≥ 3(ab + bc + ca).
954. [4] (ab)2 + (bc)2 + (ca)2 ≥ abc(a + b + c).
955. [4] (ab + bc + ca)2 ≥ 3abc(a + b + c).
956. [4] 3(a 2 + b2 + c2 ) ≥ (a + b + c)2 .
957. [4] ab5 + bc5 + ca 5 ≥ abc(a 2b + b2 c + c2 a).
958. [4] a 4 + b4 + c4 ≥ abc(a + b + c).
959. [4] a 4 + b4 + 8 ≥ 8ab.
960. [4] a 6 + b6 + c6 ≥ a 5 b + b5 c + c5 a.
961. [4] a 7 + b7 + c7 ≥ a 2 b2 c2 (a + b + c).
962. [4] a 8 + b8 + c8 ≥ a 2 b2 c2 (ab + bc + ca).
963. [4] a 3 b + b3 c + c3 a ≥ abc(a + b + c).
√ √ √ √
964. [4] (a + b) c + (b + c) a + (c + a) b ≥ 6 abc.
965. [4] (a 2 + b2 )(a 4 + b4 ) ≥ (a 3 + b3 )2 .
b  c  a
966. [4] a + b+ c+ ≥ 8.
ac ba cb
967. [4] (a + b + c)(a 2 + b2 + c2 ) ≥ 9abc.
2 2 2 9
968. [4] + + ≥ .
b+c c+a a+b a+b+c
2 2 2 1 1 1
969. [4] + + ≤ + + .
a+b b+c c+a a b c
19. Egyenlőtlenségek 77

970. [4] ab(a + b) + bc(b + c) + ca(c + a) ≤ 2(a 3 + b3 + c3 ).


971. [4] c2 (a + b) + a 2 (b + c) + b2 (c + a) ≤ 2(a 3 + b3 + c3 ).
972. [4] (a + b + c)(ab + bc + ca) ≤ 3(a 3 + b3 + c3 ).
973. [4] (a + b + c)(a 2 + b2 + c2 ) ≤ 3(a 3 + b3 + c3 ).
974. [4] a 3 + b3 + c3 ≥ a 2 b + b2 c + c2 a.
a 2 b2 c2
975. [4] + + ≥ a + b + c.
b c a
976. [4] a 2 cd + b2 da + c2 ab + d 2 bc ≥ 4abcd.
a 3 + b3 + c3 b3 + c3 + d 3 c3 + d 3 + a 3 d 3 + a 3 + b3
977. [4] + + + ≥ a 2 + b2 + c2 + d 2 .
a+b+c b+c+d c+d +a d +a+b
978. [4] 8(a 3 + b3 + c3 ) ≥ 3(a + b)(b + c)(c + a).
√ √ √
979. [4] 2a + 1 + 2b + 1 + 2c + 1 < 4, ha a + b + c = 1.
√ √ √
980. [4] 4a + 1 + 4b + 1 + 4c + 1 < 5, ha a + b + c = 1.
√ √ √
981. [4] 6a + 1 + 6b + 1 + 6c + 1 < 9, ha a + b + c = 2.
1  1  1
982. [4] 1 + 1+ 1+ ≥ 64, ha a + b + c = 1.
a b c
√ √ √
983. [4] a + b + c < 2, ha a + b + c = 1.
√ √ √ 3 50
984. [4] a + 1 + 2a − 3 + 50 − 3a < 12, ha ≤ a ≤ .
2 3
985. [4] a 2 + b2 + c2 ≥ 14, ha a + 2b + 3c ≥ 14.
1
986. [4] log230 2 + log230 3 + log230 5 > .
3
 √ √
987. [4] (a + c)(b + d) ≥ ab + cd.
988. [3] a + b + c ≥ 3, ha abc = 1.
1 1 1
989. [3] a 2 + b2 + c2 ≥ + + , ha abc = 1.
a b c
990. [4] a 4 + b4 + c4 ≥ a + b + c, ha abc = 1.
√ √ √
991. [4] ab + bc + ca ≥ a + b + c, ha abc = 1.
1 1 1 √ √ √
992. [4] + + ≥ a + b + c, ha abc = 1.
a b c
993. [4] (a + 2b)(b + 2c)(c + 2a) ≥ 27, ha abc = 1.
994. [4] (a + 1)(b + 1)(c + 1) ≥ 8, ha abc = 1.
78 Feladatok

995. [4] a 2 + b2 + c2 + d 2 + ab + bc + cd + ac + bd + ad ≥ 10, ha abcd = 1.


996. [4] (1 + a1 ) · (1 + a2 ) · . . . · (1 + an ) ≥ 2n , ha a1 · a2 · . . . · an = 1.
997. [4] p és q olyan pozitív valós számok, amelyek reciprokainak összege 1.
1 1 1 1 1 1
Igazoljuk, hogy ≤ + < és 1 ≤ + .
3 p(p + 1) q(q + 1) 2 p(p − 1) q(q − 1)
998. [4] Egy egységoldalú négyzetben n db kis négyzetet helyeztünk el úgy, hogy se-
melyik kettőnek nincs közös belső√pontja. Igazoljuk, hogy a négyzetek oldal-
hosszúságának összege legföljebb n.
999. [4] Egy egységnyi élű kockában n db kis kockát helyeztünk el úgy, hogy semelyik
kettőnek nincs közös belső pontja. Igazoljuk, hogy a kockák élének összege
legfeljebb n2/3 .
a 2 + b2 √
1000. [4] Mutassuk meg, hogy ha a > b és ab = 1, akkor ≥ 2 2.
a−b
a 2 − b2 a 2 + b2
1001. [3] Legyen a > 0, b > 0. Mutassuk meg, hogy > .
a−b a+b
1002. [3] Bizonyítsuk be a következő egyenlőtlenségeket.
1 1
a) + > 2,
log2 π logπ 2
1 1
b) + > 2.
log2 π log5 π
1003. [4] Mutassuk meg, hogy log4 5 + log5 6 + log6 7 + log7 8 > 4,4.
1004. [3] Mutassuk meg, hogy sin 40◦ + cos 40◦ > 1.
√ √
1005. [3] Mutassuk meg, hogy sin 40◦ + cos 40◦ > 1.

n+1 n
1006. [4] Mutassuk meg, hogy n! ≤ , n ∈ N.
2
a + bn √
1007. [4] Mutassuk meg, hogy ha a ≥ 0, b ≥ 0, n ∈ N , akkor ≥ a · bn .
n+1

n+1


n+1
[5] 1 n 1
1008. Mutassuk meg, hogy 1 + < 1+ , n ≥ 1, n ∈ N .
n n+1

1 n
1009. [5] Mutassuk meg, hogy 1 + < 4, n ≥ 1, n ∈ N .
n
1010. [5] Mutassuk meg, hogy a pozitív a, b számokra mindig teljesül a
√ √ √
2 a + 3 b ≥ 5 ab
3 5

egyenlőtlenség.
19. Egyenlőtlenségek 79

1011. [5] Mutassuk meg, hogy a pozitíva, b, c számokra mindig teljesül az



a b 3 c
+ + >2
b c a
egyenlőtlenség.
1012. [5] Mutassuk meg, hogy nincsenek olyan pozitív a, b, c számok, melyek egyszerre
kielégítenék az alábbi egyenlőtlenségeket:
1 1 1
a(1 − b) > , b(1 − c) > , c(1 − a) > .
4 4 4
1013. [5] Mutassuk meg, hogy nincsenek olyan pozitív a, b, c számok, melyek egyszerre
kielégítenék az alábbi egyenlőtlenségeket:
1 1 1
a+ < 2, b+ < 2, c+ < 2.
b c a
1014. [5] x > 0 értékekre határozzuk meg a következő függvények legkisebb értékét.
a
a) f (x) = x + , a > 0,
x
x3 + x + 2
b) f (x) = .
x
1
1015. [5] Az f (x) = (1 − x)3 (1 + 3x) függvénynek mennyi a maximuma a − < x < 1
3
intervallumon?
1016. [5] Az f (x) = (1 + x)3 (1 − x) függvénynek mennyi a maximuma a 0 ≤ x ≤ 1
intervallumon?
1017. [6] Az x 4 − 4x 3 + ax 2 − bx + 1 = 0 egyenletnek négy pozitív valós gyöke van.
Határozzuk meg a és b értékét.
1018. [5] Határozzuk meg az alábbi függvények maximumát.
√ √
a) f (x) = x + 7 + 11 − x, ha −7 ≤ x ≤ 11.
√ 
2
b) f (x) = 4 cos2 x + 1 + 4 sin x + 3.
1019. [4] Határozzuk meg azt a legkisebb n természetes számot, amelyre az
(a 2 + b2 + c2 )2 ≤ n(a 4 + b4 + c4 ) egyenlőtlenség igaz minden a, b, c valós számra.
1020. [5] Határozzuk meg k legnagyobb értékét úgy, hogy az
a 4 + b4 + c4 + abc(a + b + c) ≥ k(ab + bc + ca)2 igaz legyen minden a, b, c valós
számra.
1021. [4] Igazoljuk az alábbi egyenlőtlenséget.



1 1 1 1
1+ · 1+ · 1+ · ...· 1 + <2
1·3 2·4 3·5 n · (n + 2)
80 Feladatok

1022. [4] Igazoljuk az alábbi egyenlőtlenséget.





2 2 2 2
1+ · 1+ · 1+ · ...· 1 + <3
1·4 2·5 3·6 n · (n + 3)
1023. [4] Igazoljuk az alábbi egyenlőtlenséget.


1 1 1 1
1 − 2 · 1 − 2 ·...· 1 − 2 >
p1 p2 pn 2
(pk a k-adik prímszámot jelöli.)
1024. [5] Mutassuk meg, hogy nincsenek olyan x, y, z valós számok, amelyekre teljesül
mindhárom egyenlőtlenség.

|x| < |y − z|, |y| < |z − x|, |z| < |x − y|.


[5]
1025. Mutassuk meg, hogy nincsenek olyan x, y, z, t valós számok, amelyekre teljesül
mind a négy egyenlőtlenség.

|x| > |y − z + t|, |y| > |x − z + t|,


|z| > |x − y + t|, |t| > |x − y + z|.
1026. [5] Igazoljuk a következő egyenlőtlenséget.
√n

n! < n+1 (n + 1)!, n = 1, 2, 3, . . .
1027. [5] Igazoljuk a következő egyenlőtlenséget.
√n

n+1
n> n + 1, n = 3, 4, 5, . . .
[4] 1994n
1028. Az tört értéke mely pozitív egész n esetén lesz maximális?
n!
19n + 94n
1029. [4] A tört értéke mely pozitív egész n esetén lesz maximális?
n!
n2
1030. [4] Az tört értéke mely pozitív egész n esetén lesz maximális?
1, 001n
1031. [5] Mutassuk meg, hogy ha a1 , a2 , . . . , an pozitív valós számok, akkor
n
1  ak 
1+ 1 + 2 ≥ (n + 1)2 .
k=1
ak k
1032. [5] Mutassuk meg, hogy minden n pozitív egészre

√ 1 1 1 1
n( n + 1 − 1) ≤ 1 + + + · · · + ≤ 1 + n 1 − √
n
.
2 3 n n
n
1 1 1 1
1033. [6] Legyen Sn = 1 + + + + · · · + . Mutassuk meg, hogy Sp + Sq − Spq ≤ 1,
2 3 4 n
ahol p és q tetszőleges pozitív egészek.
20. Különféle algebrai feladatok 81

20. Különféle algebrai feladatok

1034. [1] Adottak az 1 − x, 2 − x, 3 − x, . . . , 100 − x számok.


a) Számoljuk ki a szorzatukat, ha x = 77.
b) Számoljuk ki az összegüket, ha x = 50,5.
c) Mutassuk meg, hogy szorzatuk x = 0-ra és x = 101-re ugyanaz!
1035. [1] Számoljunk ügyesen!
4372 − 3632
a) =?
5372 − 4632
b) 96 · 104 =?
c) Bontsuk prímtényezőire 899-et.
d) Igazoljuk a 77782 − 22232 = 55 555 555 egyenlőséget.
1036. [2] Alakítsuk a kifejezést minél több egész együtthatós polinom szorzatává.
a) x 4 + x 3 + 2x 2 + x + 1,
b) x 3 + 2x 2 + 2x + 1,
c) x 4 + 2x 3 + 2x 2 + 2x + 1,
d) x 4 + 2x 3 + 3x 2 + 2x + 1,
e) x 4 + x 2 + 1,
f) x 10 + x 8 + x 6 + x 4 + x 2 + 1,
g) x 4 + 4,
h) x 4 − 7x 2 + 1,
i) x 5 + x 4 + 1.
1037. [4] Alakítsuk szorzattá az
x 8 − x 7 y + x 6 y 2 − x 5 y 3 + x 4 y 4 − x 3 y 5 + x 2 y 6 − xy 7 + y 8 kifejezést.
1038. [4] Bizonyítsuk be, hogy minden x racionális számhoz van olyan y racionális szám,
amelyekre 2x 3 + 2y 3 − 3x 2 − 3y 2 + 1 = 0.

Számítsuk ki a következő (teleszkopikus) összegeket, szorzatokat. (1039–1051. felada-


tok)
1039. [2] 1002 − 992 + 982 − 972 + · · · + 22 − 12 .
1 1 1 1
1040. [2] + + + ··· + .
1·2 2·3 3·4 99 · 100

A fejezet feladataiban x, y, z, a, b, c, . . . tetszőleges valós számokat jelent, ahol az


adott kifejezések értelmezhetők.
82 Feladatok

3 5 7 2n + 1
1041. [3] + + + ···+ 2 .
12 · 22 22 · 32 32 · 42 n · (n + 1)2
1042. [3] 1 · 1! + 2 · 2! + 3 · 3! + · · · + n · n!.
1 2 3 n
1043. [3] + + + ··· + .
2! 3! 4! (n + 1)!
3 4 5 n+2
1044. [4] + + + ···+ .
1! + 2! + 3! 2! + 3! + 4! 3! + 4! + 5! n! + (n + 1)! + (n + 2)!
1 1 1 1
1045. [3] √ √ +√ √ +√ √ + ···+ √ √ .
1+ 2 2+ 3 3+ 4 n+ n+1
1 1 1 1
1046. [4] √ √ + √ √ + √ √ + ···+ √ √ .
2 1+1 2 3 2+2 3 4 3+3 4 (n + 1) n + n n + 1
1047. [5] tg α + 2 tg 2α + 4 tg 4α + · · · + 2n tg 2n α.
1 1 1
1048. [5] + + ···+ .
cos α · cos 2α cos 2α · cos 3α cos nα · cos(n + 1)α
1 1 1 1
1049. [5] + + + ··· + .
sin 2α sin 4α sin 8α sin 2n α


1 1 1
1050. [2] 1+ · 1+ · ...· 1 + .
2 3 100


1 1 1
1051. [2] 1 − 2 · 1 − 2 · ...· 1 − .
2 3 1002
1 1 1 1
1052. [4] Mutassuk meg, hogy √ + √ + √ + · · · + √ < 2 minden pozitív
2 1 3 2 4 3 (n + 1) n
n egészre.
√ √ √ √
[3] 2 6 12 n(n + 1) n
1053. Mutassuk meg, hogy + + + ··· + < .
3 5 7 2n + 1 2
1054. [4] Tudjuk, hogy 0 < x1 < 1 és xk+1 = xk − xk2 , k = 1, 2, . . . Mutassuk meg, hogy
x12 + x22 + x32 + · · · + xn2 < 1.
1055. [4] x1 = 0, 001; xn+1 = xn − xn2 ; n = 1, 2, 3, . . . Mutassuk meg, hogy x1001 <
< 0, 0005.
1056. [5] Számoljuk ki a következő tört értékét.
√ √
(4 + 15)3/2 + (4 − 15)3/2
√ √
(6 + 35)3/2 − (6 − 35)3/2
20. Különféle algebrai feladatok 83

1057. [5] Számoljuk ki a következő tört értékét.




1 1 1
24 + 44 + · . . . · 204 +
4 4 4


1 1 1
14 + 34 + · . . . · 194 +
4 4 4
1058. [3] Számoljuk ki a következő kifejezés értékét.

1 · 2 · 4 + 2 · 4 · 8 + 3 · 6 · 12 + · · · + 100 · 200 · 400 1/3


=?
1 · 3 · 9 + 2 · 6 · 18 + 3 · 9 · 27 + · · · + 100 · 300 · 900
1059. [2] Ismertek a következő azonosságok:
0
(x + y)3 = x 3 + y 3 + 3xy(x + y)(x 2 + xy + y 2 ) ,
1
(x + y)5 = x 5 + y 5 + 5xy(x + y)(x 2 + xy + y 2 ) ,
2
(x + y)7 = x 7 + y 7 + 7xy(x + y)(x 2 + xy + y 2 ) .
Vajon igaz-e az ezekkel analóg
3
(x + y)9 = x 9 + y 9 + 9xy(x + y)(x 2 + xy + y 2 )
azonosság?
1060. [4] Igazoljuk az alábbi azonosságokat:
a) (a 2 +b2 +c2 )(x 2 +y 2 +z2 ) = (ax +by +cz)2 +(bz−cy)2 +(cx −az)2 +(ay −bx)2
b) (a + b)(b + c)(c + a) = (a + b + c)(ab + bc + ca) − abc =
= ab(a + b) + bc(b + c) + ca(c + a) + 2abc
c) (a − b)(b − c)(c − a) = a(b2 − c2 ) + b(c2 − a 2 ) + c(a 2 − b2 )
d) a 3 + b3 + c3 = (a + b + c)3 − 3(a + b)(b + c)(c + a)
e) a(b + c)2 + b(c + a)2 + c(a + b)2 − 4abc = (a + b)(b + c)(c + a)
f) (a 2 + b2 + c2 )2 = (ab + bc + ca)2 + (a 2 − bc)2 + (b2 − ca)2 + (c2 − ab)2
g) (a 2 + b2 )(x 2 + y 2 ) = (ax + by)2 + (ay − bx)2
h) (a 2 + b2 + c2 + d 2 )(x 2 + y 2 + z2 + t 2 ) =
= (ax + by + cz + dt)2 + (bx − ay + dz − ct)2 + (cx − dy − az + bt)2 +
+(dx + cy − bz − at)2
i) (a − b)3 + (b − c)3 + (c − a)3 = 3(a − b)(b − c)(c − a)
j) (a + b)3 + (a + c)3 + (a + d)3 + (b + c)3 + (b + d)3 + (c + d)3 =
= 3(a + b + c + d)(a 2 + b2 + c2 + d 2 )
k) (a + b + c)2 + (−a + b + c)2 + (a − b + c)2 + (a + b − c)2 = 4(a 2 + b2 + c2 ) =
= (2a)2 + (2b)2 + (2c)2
l) (a + b + c)3 = (−a + b + c)3 + (a − b + c)3 + (a + b − c)3 + 24abc
m) (a + b + c)5 = (−a + b + c)5 + (a − b + c)5 + (a + b − c)5 + 80abc(a 2 + b2 + c2 )
n) (9a 4 )3 + (1 − 9a 3 )3 + (3a − 9a 4 )3 = 1
84 Feladatok

o) (1 + 6a 3 )3 + (1 − 6a 3 )3 + (−6a 2 )3 = 2
p) [a(a 3 + 2b3 )]3 + [b(2a 3 + b3 )]3 + (3a 2 b2 )3 = (a 6 + 7a 3 b3 + b6 )2
q) (3a 2 +5ab−5b2)3 +(4a 2 −4ab+6b2)3 +(5a 2 −5ab−3b2)3 = (6a 2 −4ab+4b2)3
r) (75b5 −a 5 )5 +(a 5 +25b5 )5 +(a 5 −25b5)5 +(10a 3 b2 )5 +(50ab4)5 = (a 5 +75b5 )5
s) [a(a 2 − 3b2 )]2 + [b(3a 2 − b2 )]2 = (a 2 + b2 )3
1061. [4] Mutassuk meg, hogy ha a + b + c = 0, akkor
a 3 + b3 + c3 a 2 + b2 + c2 a 5 + b5 + c5
a) · =
3 2 5
a 5 + b5 + c5 a 2 + b2 + c2 a 7 + b7 + c7 a 3 + b3 + c3 a 4 + b4 + c4
b) · = =2· ·
5 2 7 3 4
7 7 7 3 3 3 5 5

5 2
a +b +c a +b +c a +b +c
c) · =
7 3 5
1062. [2] Tudjuk, hogy x + ex = y + ey . Következik-e ebből, hogy sin x = sin y?
1063. [2] Legyen a + b + c = 0. Mutassuk meg, hogy ab + bc + ca ≤ 0.
1064. [4] Legyen (a + b + c)2 = 3(a 2 + b2 + c2 ). Igazoljuk, hogy a = b = c.
a b c
1065. [4] Legyen = = . Mutassuk meg, hogy a = b = c.
b c a
1 1 1
1066. [4] Legyen abc = 1 és a + b + c = + + . Mutassuk meg, hogy ekkor az a, b,
a b c
c számok egyike 1-gyel egyenlő.
1 1 1
1067. [4] Legyen abc = 1 és a + b + c > + + . Bizonyítsuk be, hogy ekkor az a, b,
a b c
c számok egyike 1-nél nagyobb, a másik kettő pedig 1-nél kisebb.
1068. [4] Legyen a + b + c > 0, ab + bc + ca > 0, abc > 0. Mutassuk meg, hogy ekkor
az a, b, c számok mindegyike pozitív.
1 1 1
1069. [4] Legyen ab + bc + ca > 0 és + + > 0. Mutassuk meg, hogy ekkor a,
ab bc ca
b, c azonos előjelű számokat jelöl.
1
1070. [3] Mutassuk meg, hogy ha x + y + z − 2(xy + yz + zx) + 4xyz = , akkor az x, y,
2
1
z számok valamelyikének értéke .
2
1 1 1
1071. [4] Mutassuk meg, hogy ha x + = y + = z+ , akkor x = y = z vagy x 2 y 2 z2 = 1.
y z x
1072. [4] Mutassuk meg, hogy ha x + y + z = 0 és xy + yz + zx = 0, akkor x = y = z.
20. Különféle algebrai feladatok 85

1 1 1
1073. [4] Bizonyítsuk be, hogy az x + y + z = 0, + + = 0 egyenletrendszernek
x y z
nincs megoldása a valós számok körében.
1 1 1
1074. [4] Legyen + + = 0. Mutassuk meg, hogy
x y z
y+z z+x x+y
+ + + 3 = 0.
x y z
1075. [4] Tudjuk, hogy
x y z
+ + = 1.
y+z z+x x+y
Mutassuk meg, hogy

x2 y2 z2
+ + = 0.
y+z z+x x+y
1076. [5] Igazoljuk az alábbi azonosságokat:
1 1 1
+ + = 0,
(a − b)(a − c) (b − a)(b − c) (c − a)(c − b)
a b c
+ + = 0,
(a − b)(a − c) (b − a)(b − c) (c − a)(c − b)
a2 b2 c2
+ + = 1,
(a − b)(a − c) (b − a)(b − c) (c − a)(c − b)
a3 b3 c3
+ + = a + b + c.
(a − b)(a − c) (b − a)(b − c) (c − a)(c − b)
(Az azonosságok általánosíthatók 3 helyett több változóra.)
1077. [4] a) Legyen b olyan valós szám, amelyre b3 = b + 1. Mutassuk meg, hogy ekkor
b5 = b4 + 1.
b) Legyen b olyan valós szám, amelyre b5 = b4 +1. Mutassuk meg, hogy ekkor
b3 = b + 1.

1 2 1
1078. [4] Tudjuk, hogy x + = 3. Mivel egyenlő x 3 + 3 ?
x x
1 1
1079. [4] Az x > 0 számra x 2 + 2 = 7 teljesül. Mutassuk meg, hogy x 5 + 5 is egész
x x
szám.
1080. [4] Az a, b, c valós számok kielégítik az a +b+c = 0, a 2 +b2 +c2 = 1 egyenleteket.
Számítsuk ki az a 4 + b4 + c4 értékét.
1081. [4] Az a, b, c egész
 számokra a + b + c = 0 teljesül. Mutassuk meg, hogy
2 a 4 + b4 + c4 négyzetszám.
86 Feladatok

   2
1082. [5] Egy háromszög a, b, c oldalaira 2 a 8 + b8 + c8 = a 4 + b4 + c4 teljesül. Mu-
tassuk meg, hogy a háromszög derékszögű.
1083. [4] Az x 2 +ax +b+1 = 0 egyenlet gyökeinek négyzete pozitív egész szám. Mutassuk
meg, hogy a 2 + b2 összetett szám!
1084. [3] Tudjuk, hogy x + y = a, x 2 + y 2 = b, x 3 + y 3 = c. Mutassuk meg, hogy a 3 −
− 3ab + 2c = 0.
1085. [4] Fejezzük ki a, b, c segítségével xyz-t, ha
x + y + z = a,
x 2 + y 2 + z2 = b,
x 3 + y 3 + z3 = c.
1086. [4] Oldjuk meg a következő egyenletrendszert a valós számok körében.

x + y + z =3 ⎪ ⎬
x 2 + y 2 + z2 =5


x 4 + y 4 + z4 =17
1087. [4] Mutassuk meg, hogy az x 4 − 5x 3 − 4x 2 − 7x + 4 = 0 egyenletnek nincs negatív
gyöke.
1088. [3] Mutassuk meg, hogy nincs olyan a és b valós szám, melyekre
√ 2 √
a + b · 3 = 99 999 + 111 111 · 3
teljesülne.
[3]
1089. Mutassuk meg, hogy nincs olyan m és n pozitív egész szám, melyekre
√ m √ n
5+3· 2 = 3+5· 2
teljesülne.
[3]
1090. Mutassuk meg, hogy nincsenek olyan a, b, c, d egész számok, melyekre
√ 2 √ 2 √
a+b· 2 + c+d · 2 =5+4· 2
teljesülne.
√ 1993
1091. [4] Mutassuk meg, hogy a 26 + 5 számban a tizedesvessző utáni első 1993
jegy mindegyike nulla.
 30 
[5]

1092. Mutassuk meg, hogy 45 + 1975 értéke páratlan szám.
20. Különféle algebrai feladatok 87

√ 19 √ 82
1093. [4] Milyen számjegy áll 15 + 220 + 15 + 220 tizedestört alakjában a
tizedesvessző előtt?
1094. [7] Állapítsuk meg, hogy mely számjegyek állanak közvetlenül a tizedesvesszőtől
√ √ 1980
balra és jobbra a 2+ 3 szám tízes számrendszerben felírt alakjában.

1095. [3] Az (m + 1)x 2 + 2mx + m − 1 = 0 egyenletnek az m valós értékű paraméter mely


értékeire lesz két valós gyöke?
1096. [3] Az x 2 + 5x + c = 0 egyenletnek van egy pozitív és egy negatív gyöke. Lehet-e
c pozitív?
1097. [4] Az ax 2 + bx + c kifejezés minden x-re pozitív. Lehet-e c negatív?
1098. [4] Az ax 2 + bx + c = 0 egyenletnek nincs valós gyöke, és a + b + c < 0. Lehet-e c
pozitív?
1099. [4] Az a, b, c valós számokra (a + b + c)c < 0 teljesül. Mutassuk meg, hogy b2 >
> 4ac.
1100. [4] Vannak-e olyan a, b, c valós szá-
y
mok, melyekre az ábrán látható pa-
rabolák képlete ax 2 + bx + c, cx 2 +
+ ax + b, bx 2 + cx + a?

1101. [4] Vannak-e olyan a, b, c valós szá-


y
mok, melyekre az ábrán látható pa-
rabolák képlete ax 2 + bx + c, cx 2 +
+ ax + b, bx 2 + cx + a?

1102. [2] Mutassuk meg, hogy az ax 2 + bx + c = 0 egyenletnek van racionális gyöke, ha


a, b és c 1-gyel, 2-vel, −3-mal egyenlők valamilyen sorrendben.
1103. [4] Mutassuk meg, hogy az ax 2 + bx + c = 0 egyenletnek nincs racionális gyöke, ha
a, b és c páratlan egész számok.
1104. [5] Az ax 2 + bx + c = 0 egész együtthatós egyenletnek két különböző, 1-nél kisebb
pozitív gyöke van. Határozzuk meg a legkisebb pozitív értékét.
88 Feladatok

1105. [3] Az ax 2 + bx + c kifejezés minden egész x-re egész szám. Mutassuk meg, hogy
2a, a + b, c is egész szám.
1106. [4] Az egész együtthatós ax 3 +bx 2 +cx +d polinom minden egész x-re 5-tel osztható
értéket vesz fel. Mutassuk meg, hogy az együtthatók mindegyike osztható 5-tel.
1107. [4] Tudjuk, hogy 2a + 3b + 6c = 0. Mutassuk meg, hogy az ax 2 + bx + c = 0
egyenletnek van 0 és 1 közé eső gyöke.
1108. [4] Mutassuk meg, hogy a következő egyenleteknek van két különböző valós gyö-
ke. (A paraméterek valós számok.)
a) (x − a)(x − b) + (x − b)(x − c) + (x − c)(x − a) = 0, ahol a > b > c;
b) (x − a)(x − c) + p(x − b)(x − d) = 0, ahol a > b > c > d és p tetszőleges
valós szám, p = −1;
c) a(x − b)(x − c) + b(x − c)(x − a) + c(x − a)(x − b) = 0, ahol a > b > c és
a + b + c = 0;
d) (a + b)(x − a)(x − b) + (b + c)(x − b)(x − c) + (c + a)(x − c)(x − a) = 0, ahol
a 2 < b2 < c2 és a + b + c = 0;
e) (ab + bc + ca)x 2 − (a 2 + b2 + 2c2 )x + c2 = 0, ahol ab + bc + ca = 0;
f) ax 2 + bx + c = 0, ahol a = 0, ac + bc + c2 < 0.
1109. [4] Igazoljuk a következő egyenlőtlenséget.
   
a12 + b12 + a22 + b22 +· · ·+ an2 + bn2 ≥ (a1 + a2 + · · · + an )2 + (b1 + b2 + · · · + bn )2

1110. [5] Igazoljuk a√következő √egyenlőtlenséget.


√ √
a 2 + b2 + b2 + c2 + c2 + a 2 ≥ 2(a + b + c)
1111. [5] Mutassuk meg, hogy ha a, b, c pozitív valós számok, akkor mindig teljesül a
következő egyenlőtlenség.
  √
a 2 + b2 − ab + b2 + c2 − bc ≥ a 2 + c2 + ac
1112. [5] Adva vannak az a, b, c pozitív
√ számok.
 Határozzuk meg, hogy az
f (x) = a 2 + x 2 + (b − x)2 + c2
függvény x mely értékére veszi fel legkisebb értékét.
√ 
1113. [5] Határozzuk meg az f (x) = 2x 2 + 2x + 1 + 2x 2 − 2x + 1 függvény minimu-
mát.

√  17
[5]
1114. Mutassuk meg, hogy 3x + 2x + 1 + 3x − 4x + 2 ≥
2 2 .
3
  √
1115. [5] Mutassuk meg, hogy (a + c)2 + b2 + (a − c)2 + b2 ≥ 2 a 2 + b2 .
20. Különféle algebrai feladatok 89

1116. [5] Az x, y, z pozitív valós számokra teljesülnek a következő egyenletek:



y2 ⎪
2
x + xy + =25 ⎪


3 ⎪

2
y
+ z =9 ⎪
2

3 ⎪

2 2


z + zx + x =16
Számoljuk ki xy + 2yz + 3zx értékét.
1117. [4] Oldjuk meg a következő egyenletrendszert a valós számok halmazán.

2x + x 2 y =y ⎪

2y + y 2 z =z


2z + z2 x =x
1118. [4] Oldjuk meg a
8x(2x 2 − 1)(8x 4 − 8x 2 + 1) = 1
egyenletet a valós számok halmazán.
   
1119. [4] Mutassuk meg, hogy a 1 − a 2 + 1 − b2 + 1 − c2 ≤ 9 − (a + b + c)2 egyen-
lőtlenség teljesül minden olyan, valós számokból álló a, b, c számhármasra,
amelyre értelmezve vannak a benne szereplő kifejezések.
1120. [5] Mutassuk meg, hogy ha 0 ≤ a, b, c ≤ 1, akkor
   √
a(1 − b)(1 − c) + b(1 − a)(1 − c) + c(1 − a)(1 − b) ≤ 1 + abc.
1121. [5] Mutassuk meg, ha x és y 1-nél kisebb pozitív számok, akkor
x−y
< 1.
1 + xy
1122. [5] Mutassuk meg, hogy négy különböző valós számból mindig kiválasztható kettő,
x és y úgy, hogy
x−y
0≤ ≤ 1.
1 + xy
1123. [5] Mutassuk meg, hogy öt különböző valós számból mindig kiválasztható kettő, x
és y úgy, hogy
1 + xy
> 1.
x−y
1124. [5] Mutassuk meg, hogy 13 különböző valós számból mindig kiválasztható kettő,
x és y úgy, hogy  √
x−y 2− 3 √
0< < √ (= 2 − 3).
1 + xy 2+ 3
90 Feladatok

1125. [4] Mutassuk meg, hogy a + b − 2ab < 1, ha 0 < a < 1, 0 < b < 1.
1126. [5] Mutassuk meg, hogy a(1 − b) + b(1 − c) + c(1 − a) < 1, ahol 0 < a < 1,
0 < b < 1, 0 < c < 1.
1127. [5] Mutassuk meg, hogy minden n pozitív egészre
√  √  √  √         
n + 3 n + 4 n + · · · + n n = log2 n + log3 n + log4 n + · · · + logn n .
1128. [4] Mutassuk meg, hogy nincsenek olyan p(x) és q(x) polinomok, melyekre sin x =
p(x)
= teljesülne minden valós x számra.
q(x)
1129. [5] Mutassuk meg, hogy nincsenek olyan p(x) és q(x) polinomok, melyekre 1 +
1 1 1 p(n)
+ + + ···+ = teljesülne minden n pozitív egész számra.
2 3 n q(n)


3k + 1
1130. [4] Számítsuk ki az S = összeg értékét!
k=1
3k

1131. [3] a1 , a2 , a3 , . . . , a1998 , a1999 , a2000 jelöli az 1, 2, 3, . . . , 1998, 1999, 2000 számok
valamilyen sorrendjét. Legfeljebb mekkora lehet az
|a1 − a2 | + |a2 − a3 | + |a3 − a4 | + · · · + |a1999 − a2000 | + |a2000 − a1 |
összeg értéke?
1132. [3] Mutassuk
 meg, hogy ha az   
1 − x + x 2 − x 3 + · · · − x 99 + x 100 · 1 + x + x 2 + x 3 + · · · + x 99 + x 100
szorzatot polinommá alakítjuk, akkor olyan polinomot kapunk, melyben x-nek
csak páros kitevőjű hatványai fordulnak elő.
1133. [4] Mutassuk meg, hogy a
cos 32x + a31 cos 31x + · · · + a2 cos 2x + a1 cos x
kifejezés pozitív és negatív értékeket is felvesz.
1134. [3] A következő módon adott függvényeknek egy sorozata:
1
f0 (x) = , fn (x) = f0 (fn−1 (x)), n = 1, 2, 3, . . . .
1−x
f1994 (1994) =?

21. Függvényegyenletek

Keressük meg az összes olyan, a valós számokon értelmezett folytonos f (x) függvényt,
mely kielégíti a felírt függvényegyenletet. (1135–1178. feladatok)
1135. [3] f (x 1990 ) = x 1991 .
21. Függvényegyenletek 91

1136. [4] 2f (x) + 3f (1 − x) = 4x − 1.


1137. [4] f (x) − f (−x) = x 2 .
 
1138. [4] (x + 1) · f (1 − x) + (x − 1) · f (x + 1) = 2x x 2 − 1 .
1139. [4] f (1 − x) + 2f (1 + x) = x + 3.
 
1140. [4] f (x 2 + x + 1) + 2f x 2 − x + 1 = 3x 2 − x + 6.
1141. [4] f (x − 1) − f (1 − x) = x.

1
1142. [4] 2f (x) − 3f = x 2 , x = 0.
x

[4] 1
1143. x · f + 2f (x) = x − 1, x = 0.
x

[4] 1
1144. f (x) + x + f (1 − x) = 1.
2
1145. [4] 2f (x) + f (1 − x) = x 2 .

[5] x−1
1146. f (x) + f = 1 + x, x = 0, x = 1.
x

1
1147. [4] f + f (1 − x) = x, x = 0, x = 1.
x
1148. [4] x 2 · f (x) + f (1 − x) = 2x − x 4 .
1149. [4] (1 − x) · f (x) + 2x · f (1 − x) = −3x 2 + 4x + 1.

[4] x−1 1
1150. f + f (x) = − x + 1, x = 0, x = 1.
x x
1151. [4] f (x + y) − f (x − y) = 4xy.
1152. [4] f (xy) + x + y = xy + f (x) + f (y).
1153. [4] f (x) · f (y) = f (x + y − xy), f (1) = 0.
1154. [4] f (x) · f (y) = f (x − y).
1155. [4] f (x + 1) ≤ x ≤ f (x) + 1.
1156. [4] f (x) − x ≤ x 2 ≤ f (x − 1) + x.
1157. [4] f (x) ≤ x és f (x + y) ≤ f (x) + f (y).
f (x) + f (y)
1158. [4] f (x + y) = .
1 + f (x) · f (y)
1159. [4] (x + y) · f (xy) = f (x) + f (y).
92 Feladatok

1160. [4] f (x) · f (y) = f (x + xy).


1161. [4] f (x) · f (y) + f (x) + f (y) + 1 = xy.
1162. [4] f (x) · f (y) + x · f (y) = 2y · f (x).
1163. [4] f (x) · f (y) − xy = f (x) + f (y) − 1.
1164. [4] x · f (y) + y · f (x) = (x + y) · f (x) · f (y).
1165. [4] f (x) · f (y) − xy = x · f (x) + y · f (y) − x 2 − y 2 + 1.
1166. [5] f (x + y) + 2f (x − y) = 3f (x) − y.
1167. [5] 3f (x) − 2f (x − y) − f (x + y) = 0, f (0) = 0.
1168. [5] f (x + y) + f (x − y) = 2f (x) · cos y.
1169. [5] f (x − y) = f (x) · f (y) + f (x) · cos y + f (y) · cos x − sin x · sin y.
1170. [5] f (x + y) = f (x) cos y + f (y) cos x.
1171. [6] f (x + y) = f (x) + f (y).
1172. [6] f (x + y) = f (x) · f (y).
1173. [6] f (x · y) = f (x) + f (y), ha x > 0, y > 0.
1174. [6] f (x · y) = f (x) · f (y), ha x > 0, y > 0.
1175. [6] f (x + y) = f (x) + f (y) + 1.
x + y  f (x) + f (y)
1176. [6] f = .
2 2
1177. [7] f (x · y) = x · f (y) + y · f (x), ha x > 0, y > 0.
1178. [6] f (x · y) = f (x) · f (y) − f (x + y) + 1, f (1) = 2.
1179. [5] Van-e olyan f (x) és g(x) függvény, hogy minden x-re és y-ra
f (x) · g(y) = x + y + 1?
1180. [5] Van-e olyan f (x) és g(x) függvény, hogy minden x-re és y-ra
f (x) · g(y) = x 2 y 2 + 1?
1181. [5] Van-e olyan f (x) és g(x) függvény, hogy minden x-re és y-ra
x 2 + xy + y 2 = f (x) + g(y)?
1182. [5] Van-e olyan, minden valós x-re értelmezett h(x) függvény, hogy minden x-re
h(f (x)) + h(g(x)) = g(f (x)), ahol f (x) = x 2 + x + 2 és g(x) = x 2 − x + 2?
1183. [5] Van-e megoldása az f (g(x)) = g(f (x)) egyenletnek, ha f (x) = x 2 + 1 és g(x) =
= 3 − 3x − x 2 ?
21. Függvényegyenletek 93

1184. [5] Adott folytonos f (x) függvény esetén az f (x) = x egyenletnek nincs megoldá-
sa. Van-e megoldása az f (f (x)) = x egyenletnek?
1185. [7] Van-e olyan, minden valós x-re értelmezett f (x) függvény, mely minden x-re
kielégíti az
a) f (f (x)) = x 2 − 2, illetve
b) f (f (x)) = −x 3 függvényegyenletet?
1186. [5] Legyen f (x) = x 2 − 2. Bizonyítsuk be, hogy az f (f (f (x))) = x egyenletnek
nyolc különböző megoldása van.
1187. [4] Az f (x) függvény folytonos, és f (x) = f (2x) teljesül minden valós x-re. Ha-
tározzuk meg az összes ilyen függvényt.
1188. [4] Az f (x, y, z) valós számokon értelmezett folytonos függvény változóinak min-
den értékére kielégíti az f (x, y, z) = 2 · f (y, z, x) függvényegyenletet. Határoz-
zuk meg az összes ilyen függvényt.
1189. [5] Legyenek f (x) és g(x) minden valós x-re értelmezett függvények, amelyekre
teljesül, hogy
f (x − y) = f (x) · g(y) − g(x) · f (y),
g(x − y) = g(x) · g(y) − f (x) · f (y).
Számítsuk ki f (0) és g(0) értékét.
1190. [5] Legyenek f (x) és g(x) minden valós x-re értelmezett függvények, amelyekre
teljesül, hogy
f (x + y) = f (x) · g(y) + g(x) · f (y),
g(x + y) = g(x) · g(y) − f (x) · f (y).
Számítsuk ki f (0) és g(0) értékét.
1191. [7] Legyenek f (x) és g(x) minden valós x-re értelmezett, nem konstans, differen-
ciálható függvények, amelyekre teljesül, hogy
f (x + y) = f (x) · g(y) + g(x) · f (y),
g(x + y) = g(x) · g(y) − f (x) · f (y),

valamint g (0) = 0. Mutassuk meg, hogy (f (x))2 + (g(x))2 = 1 minden valós
x-re.

Melyek azok a pozitív egész számokon értelmezett valós függvények, amelyek kielégítik
a következő függvényegyenleteket? (1192–1197. feladatok)
1192. [5] nf (n)−1 = (n − 1)f (n−1) .
1193. [5] f (n + m) = f (n) + f (m) + nm és f (1) = 1.
1194. [5] f (n + m) = f (n) + f (m) + nm + 1 és f (1) = 1.
1195. [5] f (n · m) = f (n) · f (m), és f (n) > f (m), ha n > m, továbbá f (2) = 2 és f (n)
minden értéke egész szám.
94 Feladatok

1196. [5] f (n + m) + f (n − m) = f (3n), ha n ≥ m, továbbá a függvény a 0 helyen is


értelmezve van.
1197. [5] f (k · m) + f (k · n) − f (k) · f (m · n) ≥ 1, továbbá a függvény a 0 helyen is
értelmezve van.
1198. [5] A pozitív egészeken értelmezett f (n) függvényről a következőket tudjuk:
f (n) = n − 10, ha n > 100; továbbá f (n) = (f (n + 11)), ha n ≤ 100. Ha-
tározzuk meg f (18) értékét.
1199. [5] A pozitív egészeken értelmezett f (n) függvényről a  következőket
 tudjuk:
f (f (n)) = 4n − 3, minden pozitív egész n-re; továbbá f 2k = 2k+1 − 1 minden
nemnegatív egész k-ra. Határozzuk meg f (1985) értékét.
1200. [5] Az f (x) folytonos függvényre f (x) · f (f (x)) = 1 és f (1000) = 999. f (500) =?

Határozzuk meg az összes olyan f (x) polinomot, mely minden valós x-re kielégíti a
következő függvényegyenleteket. (1201–1205. feladatok)
1201. [5] (x − 1) · f (x + 1) = (x + 2) · f (x).
1202. [5] x · f (x − 1) = (x − 2) · f (x).
1203. [5] x · f (x − 1) = (x + 1) · f (x).
1204. [5] x · f (x − 1) = (x − 12) · f (x) és f (12) = 12!.
1
1205. [5] f (x) = [f (x + 1) + f (x − 1)] és f (0) = 0.
2
1206. [5] Az f (n) n-edfokú polinom olyan, hogy f (k) = k/(k + 1) a k = 0, 1, 2, . . . , n
értékekre. Határozzuk meg f (n + 1) értékét.
[5]    
1207. Az f (x) és g(x) polinomok között fennáll az f x 2 − x + 1 = g x 2 + x + 1
egyenlőség minden x valós számra. Mutassuk meg, hogy mindkét polinom azo-
nosan konstans.
1208. [4] Bizonyítsuk be, hogy minden valós függvény előállítható egy páros és egy pá-
ratlan függvény összegeként.
1209. [5] Mutassuk meg, hogy az f (x) = x 2 függvény nem állítható elő két periodikus
függvény összegeként. Mit állíthatunk az f (x) = x 3 függvény esetében?
1210. [7] Van-e olyan, a valós számokon értelmezett folytonos f (x) függvény, melynek
racionális helyen irracionális, irracionális helyen racionális az értéke?
1211. [6] Van-e olyan, a valós számokon értelmezett folytonos f (x) függvény, melyre
f (x) akkor és csak akkor racionális, ha f (x + 1) irracionális?
22. Vektorok a geometriában 95

Mutassuk meg, hogy az alábbi függvényegyenleteket kiegyenlítő függvények periodiku-


sak. (1212–1215. feladatok)
f (x) − 1
1212. [5] f (x + a) = , ahol a ∈ R konstans, f (x) = 1.
f (x) + 1

1213. [5] f (x + 1) + f (x − 1) = 2 · f (x).
1214. [5] f (x + a) · f (x) = b, ahol ab = 0, a, b ∈ R konstansok.
f (x) · cos φ − sin φ π
1215. [5] f (x + a) = , ahol φ = , f (x) · sin φ + cos φ = 0.
f (x) · sin φ + cos φ n

Mutassuk meg, hogy a következő függvények nem periodikusak. (1216–1219. feladatok)


1216. [5] a) f (x) = cos x 2 , illetve
b) f (x) = sin x 2 .

1217. [5] a) f (x) = cos x, illetve

b) f (x) = sin x.
1218. [5] a) f (x) = cos x + cos ax, a ∈ R\Q , illetve
b) f (x) = sin x + sin ax, a ∈ R\Q .

1219. [5] f (x) = cos x · cos 2x.

22. Vektorok a geometriában

1220. [3] Egy szabályos sokszög csúcsai A1 , A2 , . . . , An , köré írt körének középpontja
O, X pedig egy tetszőleges pont. Mutassuk meg, hogy
a) O-ból a csúcsokba mutató vektorok összege nullvektor;
−→
b) X-ből a csúcsokba mutató vektorok összege n · XO.
1221. [3] Egy 1994-oldalú szabályos sokszög belsejében adott az A és a B pont. A-ból
a csúcsokba mutató vektorok összege legyen −→a , B-ből a csúcsokba mutató
−→ −

vektorok összege legyen b . Mikor mondhatjuk, hogy |−→a | < | b |?
1222. [4] Mutassuk meg, hogy az n-oldalú szabályos sokszög csúcsaiba lehet nullától
különböző számokat írni úgy, hogy bármely szabályos sokszög esetén, melynek
csúcsai ezek közül a csúcsok közül valók, a csúcsokban álló számok összege
mindig nulla legyen.
1223. [4] Egy szabályos sokszög csúcsai A1 , A2 , . . . , An , köré írt körének középpontja
O, a kör sugara r, és X egy tetszőleges pont, d = |OX|. Mutassuk meg, hogy
|A1 X|2 + |A2 X|2 + · · · + |An X|2 = n(r 2 + d 2 ).
96 Feladatok

1224. [4] Az ABC szabályos háromszög köré írt körnek egy tetszőleges pontja P . Mu-
tassuk meg, hogy a P A2 +P B 2 +P C 2 összeg értéke P választásától függetlenül
állandó.
1225. [6] Az ABC szabályos háromszög köré írt körnek egy tetszőleges pontja P . Mu-
tassuk meg, hogy a P A4 +P B 4 +P C 4 összeg értéke P választásától függetlenül
állandó.
1226. [6] Egy n oldalú szabályos sokszög csúcsai A1 , A2 , . . . , An ; a sokszög köré írt kör
tetszőleges pontja P . Mutassuk meg, hogy a |P A1 |4 + |P A2 |4 + · · · + |P An |4
összeg értéke a P pont választásától független, állandó érték.
1227. [4] Az ABCD rombusz egyik szöge 60◦ , beírt körének egy tetszőleges pontja P .
Mutassuk meg, hogy a P A2 + P B 2 + P C 2 + P D2 összeg értéke P választásától
függetlenül állandó.
1228. [4] Az a, b, c oldalú ABC háromszög A csúcsát B-re, B csúcsát C-re, C csúcsát
A-ra tükrözzük. Az így kapott három pont egy x, y, z oldalú háromszög csúcsait
alkotja. Mutassuk meg, hogy x 2 + y 2 + z2 = 7 a 2 + b2 + c2 .
1229. [4] Az a, b, c oldalú háromszög súlyvonalainak hossza: sa , sb , sc . Mutassuk meg,
3 2 
hogy sa2 + sb2 + sc2 = a + b2 + c2 .
4
1230. [4] Az ABC háromszög súlypontja S. Mutassuk meg, hogy
 
|AB|2 + |BC|2 + |CA|2 = 3 |SA|2 + |SB|2 + |SC|2 .
1231. [4] Az ABCD tetraéder súlypontja S. Mutassuk meg, hogy

|AB|2 + |AC|2 + |AD|2 + |BC|2 + |BD|2 + |CD|2 =


 
= 4 |SA|2 + |SB|2 + |SC|2 + |SD|2 .
1232. [4] Az ABC háromszög súlypontja S, P tetszőleges pont. Mutassuk meg, hogy
|P A|2 + |P B|2 + |P C|2 = 3|P S|2 + |SA|2 + |SB|2 + |SC|2 .
1233. [4] Adott az ABC háromszög. Mely P pontra lesz a P A2 + P B 2 + P C 2 összeg
értéke minimális, ha
a) P egy adott egyenesen helyezkedik el;
b) P a sík tetszőleges pontja lehet?
1234. [5] Adottak a síkon az A1 , A2 , . . . , An pontok. A sík mely P pontjára lesz mini-
mális a P A21 + P A22 + · · · + P A2n összeg értéke?
1235. [4] Az ABC háromszög súlypontja S, körülírt körének sugara r. Tudjuk, hogy
SA2 + SB 2 + SC 2 = 3r 2 . Határozzuk meg a háromszög szögeit.
22. Vektorok a geometriában 97

1236. [4] Az ABCD konvex négyszög átlóinak felezőpontja M és N. Mutassuk meg,


hogy AB 2 + BC 2 + CD2 + DA2 = AC 2 + BD2 + 4MN 2 .
1237. [4] Az ABCD húrnégyszög AC átlójának felezőpontja a BD átlón van. Mutassuk
meg, hogy AB 2 + BC 2 + CD2 + DA2 = 2BD2 .
1238. [4] Az ABCD húrnégyszög köré írt kör sugara r. Igazoljuk, ha AB 2 + CD2 = 4r 2 ,
akkor a négyszög átlói merőlegesek.
1239. [4] Mutassuk meg, hogy egy négyszög átlói akkor és csak akkor merőlegesek egy-
másra, ha a szemben fekvő oldalak négyzetének összege megegyezik.
1240. [4] Mutassuk meg, hogy egy paralelogramma átlóinak szorzata nagyobb, mint két
szomszédos oldala négyzetének különbsége.
1241. [6] Adott a térben négy pont: A, B, C, D. Mutassuk meg, hogy
AC 2 + BD2 + AD2 + BC 2 ≥ AB 2 + CD2 . Mikor áll fenn az egyenlőség?
1242. [5] Az O középpontú, egységsugarú körön adottak az A1 , A2 , . . . , An pontok úgy,
−−→ −−→ −−→ − →
hogy OA1 + OA2 + · · · + OAn = 0 . Mutassuk meg, hogy tetszőleges P pontra
|P A1 | + |P A2 | + · · · + |P An | ≥ n.
1243. [4] Az ABC háromszög oldalai a, b, c, körülírt körének középpontja O, sugara r,
magasságpontja M, és d = |OM|. Mutassuk meg, hogy
−→ −→ −→ −−→
a) OA + OB + OC = OM;
 
b) d 2 = 9r 2 − a 2 + b2 + c2 .
1244. [4] Az ABC háromszög oldalai a, b, c, körülírt körének sugara r. Mutassuk meg,
hogy a háromszög pontosan akkor hegyesszögű, derékszögű, illetve tompaszö-
gű, ha a 2 + b2 + c2 − 8r 2 pozitív, nulla, illetve negatív.
1245. [4] Az egységsugarú, O középpontú körbe írtuk a tompaszögű ABC háromszöget.
−→ −→ −→
Mutassuk meg, hogy |OA + OB + OC| > 1.
1246. [4] Egy háromszög oldalai a, b, c, körülírt körének középpontja O, sugara r, súly-
1 2 
pontja S. Mutassuk meg, hogy OS 2 = r 2 − a + b2 + c2 .
9
1247. [4] Mutassuk meg, hogy az ABC háromszögben az A és B csúcsból induló súly-
vonalak akkor és csak akkor merőlegesek egymásra, ha a 2 + b2 = 5c2 , ahol a,
b, c a háromszög A, B, ill. C csúcsával szemközti oldalai.
1248. [4] Mutassuk meg, hogy egy háromszög pontosan akkor derékszögű, ha súlyvona-
laira (megfelelően választva azokat) fennáll az sa2 + sb2 = 5sc2 összefüggés.
1249. [4] Az ABC háromszögben az A és B csúcsból induló súlyvonalak merőlege-
sek egymásra. Bizonyítsuk be, hogy a háromszög C csúcsánál levő γ szögre
4
cos γ ≥ .
5
98 Feladatok

1250. [4] Határozzuk meg egy egyenlő szárú háromszög szárszögének cosinusát, ha az
alap végpontjaiból induló súlyvonalak merőlegesek egymásra.
1251. [4] Egy háromszög szögei α, β, γ . Mutassuk meg, hogy
3
a) cos α + cos β + cos γ ≤ ;
2
3
b) cos 2α + cos 2β + cos 2γ ≥ − .
2
1252. [4] Mennyi az alábbi kifejezések minimuma, maximuma?
a) 3 sin x + 4 cos x;
b) 6 sin x · cos y + 2 sin x · sin y + 3 cos x.
1253. [4] Adott nyolc valós szám: a, b, c, d, e, f , g, h. Mutassuk meg, hogy az ac +
+ bd, ae + bf , ag + bh, ce + df , cg + dh, eg + f h számokból legalább az egyik
nemnegatív.
1254. [6] Adott a síkon négy vektor, a, b, c, d, melyekre a + b + c + d = 0. Mutassuk meg,
hogy |a| + |b| + |c| + |d| ≥ |a + d| + |b + d| + |c + d|.
1255. [6] Mutassuk meg, hogy a tetszőlegesen választott a, b, c vektorokra teljesül az
|a| + |b| + |c| + |a + b + c| ≥ |a + b| + |b + c| + |c + a| egyenlőtlenség.
(Hlawka-féle egyenlőtlenség.)
1256. [4] Egy tetszőleges P pontot tükrözzünk először egy A pontra, majd a tükörképet
egy B pontra, az így nyert képet pedig egy C pontra; tovább folytatva újra A-
ra, B-re és végül C-re. Bizonyítsuk be, hogy a hatodik tükrözés visszaviszi a P
pontot az eredeti helyzetbe.
1257. [3] Egy tetszőleges P pontot tükrözünk egy paralelogramma egy csúcsára, majd az
eredményt egy vele szomszédos csúcsra, és így körbe a paralelogramma minden
csúcsára. Bizonyítsuk be, hogy a negyedik tükrözés visszavisz az eredeti pontba.
1258. [3] Egy konvex hatszög másodszomszédos oldalainak felezőpontjait összekötve két
háromszöget kapunk. Igazoljuk, hogy ezek súlypontjai egybeesnek.
1259. [3] Mutassuk meg, hogy egy konvex négyszög szemközti oldalfelező pontjait össze-
kötő szakaszok és az átlók felezőpontját összekötő szakasz egy közös pontban
metszi egymást.
1260. [3] Mutassuk meg, hogy egy tetraéder szemközti éleinek felezőpontját összekötő
szakaszok metszik egymást.
1261. [3] Egy háromszög oldalaira kifelé paralelogrammákat rajzoltunk, s összekötöttük
a szomszédos külső csúcsokat. Mutassuk meg, hogy az így kapott hatszög má-
sodszomszédos oldalaiból szerkeszthető háromszög.
1262. [3] Mutassuk meg, hogy egy háromszög súlyvonalaiból mindig szerkeszthető há-
romszög.
22. Vektorok a geometriában 99

1263. [3] A1 B1 C1 D1 és A2 B2 C2 D2 azonos körüljárású paralelogrammák. Mutassuk meg,


hogy az A1 A2 , B1 B2 , C1 C2 , D1 D2 szakaszok felezőpontjai egy paralelogramma
csúcsai.
1264. [4] Legyenek A1 B1 C1 és A2 B2 C2 azonos körüljárású szabályos háromszögek. Mu-
tassuk meg, hogy az A1 A2 , B1 B2 , C1 C2 szakaszok felezőpontjai egy szabályos
háromszög csúcsai.
1265. [4] Legyenek A1 B1 C1 és A2 B2 C2 azonos körüljárású egyenlő szárú derékszögű há-
romszögek! Mutassuk meg, hogy az A1 A2 , B1 B2 , C1 C2 szakaszok felezőpontjai
egy egyenlő szárú derékszögű háromszög csúcsai.
1266. [4] Az ABCD konvex négyszög AB oldalának felezőpontja E, DC oldalának fele-
zőpontja F . Mutassuk meg, hogy az ED, EC, FA, FB szakaszok felezőpontjai
egy paralelogramma csúcsai.
1267. [4] Mutassuk meg, hogy a háromszög magasságpontja, súlypontja és köré írt köré-
nek középpontja egy egyenesre illeszkedik. (Euler-egyenes)
1268. [3] Az ABCD, AEF G, ADF H , F KLE, BKLC négyszögek paralelogrammák.
Mutassuk meg, hogy AF H G is paralelogramma.
1269. [3] Az OAB háromszög OA és AB oldalát osszuk fel három egyenlő részre, az A-
hoz közelebbi harmadolópontok F és E. Az OE és BF szakaszok a háromszög
−→ −→ −→
belsejében a P pontban metszik egymást. Írjuk fel az OP vektort az OA és OB
vektorok segítségével.
1270. [4] Az ABC háromszög kerületén pozitív körüljárás szerint haladva válasszuk ki
mindegyik oldal első harmadolópontját, ezek: A1 , B1 , C1 . Most az A1 B1 C1 há-
romszögre ismételjük meg ezt az eljárást! Igazoljuk, hogy az így kapott A2 B2 C2
háromszög hasonló az ABC háromszöghöz, amelyből kiindultunk.
1271. [3] Egy paralelogramma oldalaira kifelé négyzeteket írtunk. Bizonyítsuk be, hogy
ezek középpontjai egy négyzet csúcsai.
1272. [3] Egy egyenlő szárú trapéz oldalaira kifelé téglalapokat szerkesztünk; ezeknek a
téglalapoknak a másik oldala egyenlő a trapéz szemközti oldalával. Mutassuk
meg, hogy a téglalapok középpontjai egy négyzet csúcsai.
1273. [5] Konvex négyszög oldalai fölé építsünk négyzeteket az adott oldalhosszal! Mu-
tassuk meg, hogy a szemközti négyzetek középpontját összekötő szakaszok
egyenlő hosszúak és merőlegesek egymásra.
1274. [3] Az ABC háromszög AB és AC oldalaira kifelé szabályos háromszöget rajzo-
lunk, az ABP és ACR háromszögeket. Az AP , AR és BC szakaszok felező-
pontjai M, N és D. Bizonyítsuk be, hogy MND szabályos háromszög.
1275. [3] Az ABCD paralelogramma BC és CD oldalaira kifelé a BCM és CDN sza-
bályos háromszögeket rajzoljuk. Mutassuk meg, hogy az AMN háromszög sza-
bályos.
100 Feladatok

1276. [4] Rajzoljunk egy szakasz mint oldal fölé szabályos háromszöget. Majd osszuk a
szakaszt két tetszőleges részre, és mindegyik rész mint oldal fölé ismét rajzol-
junk szabályos háromszöget, de az elsővel ellentétes félsíkban. Bizonyítsuk be,
hogy a három szabályos háromszög középpontja egy újabb szabályos három-
szög csúcsait alkotja.
1277. [3] Az ABC háromszög AB és AC oldalaira kifelé szerkesztett négyzetek közép-
pontjai O1 ill. O2 , és a BC oldal felezőpontja D. Bizonyítsuk be, hogy O1 D és
O2 D egyenlő hosszú és merőleges egymásra.
1278. [4] Az ABC háromszög AB és AC oldalai fölé kifelé az AC1 C2 B és AB1 B2 C
négyzeteket írjuk. Bizonyítandó, hogy a B2 C2 szakasz felezőpontja a BC átmé-
rőjű körön fekszik.
1279. [4] Rajzoljunk egy háromszög oldalai fölé kifelé négyzeteket. Bizonyítsuk be, hogy
a négyzeteknek a háromszögcsúcsoktól különböző csúcsai olyan hatszöget ha-
tároznak meg, amelynek minden második oldala a háromszög egy-egy súlyvo-
nalának kétszerese.
1280. [5] Az ABC háromszög oldalaira kifelé írunk két egyenlő szárú derékszögű há-
romszöget, a CAN és a CBM háromszögeket, ahol a derékszög A-nál és B-nél
van. Az AM és BN szakaszok metszéspontja legyen H . Bizonyítsuk be, hogy
CH ⊥AB.
1281. [5] Az r sugarú körben a közös pont nélküli AB, CD és EF húrok hossza legyen
r. A BC, DE, FA húrok felezőpontjai P , R, S. Mutassuk meg, hogy P RS
szabályos háromszög. Hogyan lehetne általánosítani a feladatot?
1282. [6] Egy háromszög oldalaira kifelé szabályos háromszögeket írunk. Bizonyítsuk be,
hogy ezek középpontjai egy szabályos háromszög csúcsai.
1283. [7] Legyenek ABC, AA1 A2 , BB1 B2 , CC1 C2 egyező körüljárású szabályos három-
szögek; az A2 B1 , B2 C1 , C2 A1 szakaszok felezőpontjai X, Y , Z. Mutassuk meg,
hogy XY Z szabályos háromszög.
23. Területátalakítások 101

23. Területátalakítások

A következő feladatokban mutassuk meg, hogy a vonalkázott részek területe egyenlő a


jelöletlen részek területével. (1284–1292. feladatok)
1284. [2] A paralelogramma egy belső pontját kö-
töttük össze a csúcsokkal.

1285. [2] Konvex négyszögben oldalfelező pontokat


kötöttünk össze a csúcsokkal.

1286. [2] Konvex négyszög szemközti oldalfelező


pontjait kötöttük össze.

1287. [4] Egy konvex négyszög minden oldalát 8


egyenlő részre osztjuk, majd a szemközti
oldalak megfelelő osztópontjait összeköt-
jük. Az így kapott 64 darab kis négyszö-
get sakktáblaszerűen kiszíneztük.
102 Feladatok

1288. [2] A hatszög szemközti oldalai párhuzamo-


sak és egyenlők.

1289. [2] Az egyenesek párhuzamosak a négyzet


oldalaival, illetve átlóival.

1290. [2] Szabályos csillagötszög.


23. Területátalakítások 103

1291. [2] Szabályos nyolcszög.

1292. [3] Szabályos háromszög belső pontját a csú-


csokkal kötöttük össze, és merőlegeseket
állítottunk az oldalakra.

A következő feladatokban mutassuk meg, hogy a vonalkázott részek területe egyenlő a


pöttyözött részek területével. (1293–1300. feladatok)

1293. [2] Egy paralelogramma átlójának egyik


pontján át párhuzamosakat húztunk az
oldalakkal.

1294. [2] Trapézban az átlókat rajzoltuk meg.


104 Feladatok

1295. [2] Trapézban a párhuzamos oldalak egy-egy


tetszőleges pontját a szemközti csúcsok-
kal kötöttük össze.

1296. [2] Szabályos hatszögben két szomszédos


csúcsot egy-egy oldalfelező ponttal kö-
töttünk össze az ábra szerint.

1297. [2] Téglalapban oldalfelezőpontokat csúcsok-


kal kötöttünk össze az ábra szerint.

1298. [3] A téglalap két szomszédos oldalának egy-


egy tetszőleges pontját a szemközti csú-
csokkal kötöttük össze.

1299. [3] Konvex négyszögben a két szemben le-


vő oldal felezőpontját a szemközti olda-
lak csúcspontjaival kötöttük össze.
23. Területátalakítások 105

1300. [4] Paralelogramma két belső pontját a csú-


csokkal kötöttük össze.

A következő feladatokban mutassuk meg, hogy a vonalkázott részek területe egyenlő a


pöttyözött részek területével. (1301–1311. feladatok)
1301. [3] Konvex négyszögben szomszédos oldal-
felező pontokat kötöttünk össze.

1302. [3] Téglalapot egyik átlója két háromszög-


re bont. Az ezek egyikébe írt kör közép-
pontján át párhuzamosakat húztunk az ol-
dalakkal.

1303. [3] Egy körlemez negyedét határoló kör-


ív harmadolópontjaiból merőlegeseket
bocsátottunk az egyik határoló sugár-
ra, továbbá az egyik harmadolópontot
összekötöttük a kör középpontjával az
ábra szerint.
106 Feladatok

1304. [3] Egy körlemez negyedében két félkörív.

1305. [3] Egy körlemez negyedében félkört rajzoltunk


és berajzoltuk a szögfelezőt.

1306. [3] Az ábrán két négyzetet látunk, me-


lyeknek van egy közös csúcsa.

1307. [3] Paralelogramma csúcsait oldalfelező pon-


tokkal kötöttük össze az ábra szerint.
23. Területátalakítások 107

1308. [4] Konvex négyszög csúcsait oldalfele-


ző pontokkal kötöttük össze az ábra
szerint.

1309. [2] A két négyzet oldalai párhuzamosak egy-


mással.

1310. [3] Egyenlő szárú trapézban átlók és a


csúcsokra illeszkedő magasságvona-
lak az ábra szerint.

1311. [5] Konvex hatszög szemközti oldalai párhuzamosak. (A két ábrán ugyanaz a hat-
szög látható.)
108 Feladatok

A következő feladatokban az egész síkidom területének hányad része a vonalkázott te-


rület? (1312–1328. feladatok)
1312. [2] Négyzetben félkörív és negyedkörívek az
ábra szerint.

1313. [2] Négyzetben negyedkörívek az ábra sze-


rint.

1314. [3] Rombusz oldalainak harmadolópontjait kötöttük össze az ábra szerint.


23. Területátalakítások 109

1315. [3] Szabályos háromszögben oldalak harma-


dolópontjait kötöttük össze az ábra sze-
rint.

1316. [3] Négyzet oldalainak harmadolópontjait


kötöttük össze az ábra szerint.

1317. [3] Négyzetben szakaszfelező pontokat kö-


töttünk össze az ábra szerint.
110 Feladatok

1318. [3] Négyzetben oldalfelező pontokat kötöt-


tünk össze egy-egy csúccsal az ábra sze-
rint.

1319. [3] Egy négyzetbe öt egybevágó négyzetből


álló keresztet írtunk az ábra szerint úgy,
hogy a kereszt négy csúcsa a négyzet egy-
egy oldalfelező pontjába esik.

1320. [3] A téglalap egyik oldala kétszerese a másiknak. Oldalfelező pontokat és csúcso-
kat kötöttünk össze az ábra szerint.
23. Területátalakítások 111

1321. [3] Konvex négyszögben harmadolópontokat


és csúcsokat kötöttünk össze az ábra sze-
rint.

1322. [4] Konvex négyszögben harmadoló-


pontokat kötöttünk össze az ábra
szerint.

1323. [3] Hegyesszögű háromszög oldalfelező pontjaiból merőlegeseket bocsátottunk a


szemközti oldalakra.
112 Feladatok

1324. [3] A téglalap két átellenes csúcsát a szemközti oldalfelező pontokkal kötöttük
össze.

1325. [3] Paralelogramma egy csúcsát a szem-


közti oldalfelező pontokkal kötöttük
össze, és megrajzoltuk a paralelogram-
mának azt az átlóját, amelyik nem il-
leszkedik a kiszemelt csúcsára.

1326. [3] A háromszög egy-egy oldalának felező-,


egyik harmadoló- és negyedelőpontját
kötöttük össze.

1327. [5] Egy háromszög oldalainak harmadolópont-


jait a csúcsokkal kötöttük össze az ábra sze-
rint.
23. Területátalakítások 113

1328. [4] Egy négyzetben oldalfelező pontokat kötöt-


tünk össze a csúcsokkal, ill. berajzoltunk egy
átlót.

1329. [3] Egy háromszög, négyszög, szabályos hatszög oldalait saját hosszukkal, ill. an-
nak kétszeresével meghosszabbítottuk adott körüljárási irányt követve. Hány-
szorosa az így kapott síkidom területe az eredetinek?

1330. [3] Hogyan kell az ADEF G töröttvonalat felvenni, hogy az


ABC háromszöget öt egyenlő területű háromszögre bont- C
suk?
G

F
E
D

A B

1331. [3] Egy négyzet belsejében felveszünk két pontot, s azokat összekötjük a négyzet
csúcsaival. Így a négyzetet kilenc részre osztjuk. Lehetséges-e, hogy e részek
mindegyikének ugyanannyi a területe?
1332. [3] Félkörbe írt téglalapok közül melyiknek legnagyobb a területe?
1333. [3] Az ABCD téglalap belsejében felveszünk egy M pontot. Mutassuk meg, hogy
a téglalap t területére igaz a következő egyenlőtlenség:
t ≤ |AM| · |BM| + |CM| · |DM|.
114 Feladatok

1334. [4] Bizonyítsuk be, hogy a T területű ABCD négyszögre és a belsejében levő P
pontra pontosan akkor áll fenn a 2T = P A2 + P B 2 + P C 2 + P D2 egyenlőség,
ha a négyszög négyzet, és P a négyzet középpontja.
1335. [4] Háromszög egyik oldalán adott egy pont. Szerkesszünk e ponton át olyan egye-
nest, mely felezi a háromszög területét.
1336. [4] Konvex négyszög egyik csúcsán át szerkesszünk olyan egyenest, mely felezi a
négyszög területét.

24. Geometriai konstrukciók

1337. [2] Vághat-e két egyenes egy négyszöget 6 részre?


1338. [2] Az ábrákon látható síkidomokat vágjuk szét 4 db egybevágó részre.
1 2

1 4 4
1 1
1 1 1 1
2 2
2 2 2 6
a) b) c) d)
1339. [2] Hogyan lehet az ábrán látható, 5 négyzet- a) b)
ből álló alakzatokat 3 darabra vágni úgy,
hogy azokat összerakva egy nagy négy-
zetet kapjunk?

1340. [2] Az ábrán látható síkidomot a vonalak mentén vágjuk


két részre úgy, hogy azokból négyzetet lehessen össze-
állítani.
24. Geometriai konstrukciók 115

1341. [2] Az ábrákon látható téglalapok mindegyikét a vonalak mentén vágjuk két részre
úgy, hogy a két részből négyzetet lehessen összeállítani.
a) b)
(9 16-os)
(8 18-as)

1342. [3] Vegyünk három négyzetet úgy, hogy közülük egyet, esetleg kettőt két részre
vágva ezekből egy nagyobb négyzetet tudjunk összerakni.
1343. [2] Egy mennyezetre 12 lámpát függesztenek fel úgy, hogy 6 egyenesen legyenek
és minden egyenesen 4 lámpa helyezkedjék el. Hogyan lehet ezt megvalósítani?
1344. [3] Helyezzünk el 7 pontot és 6 egyenest a síkon úgy, hogy mindegyik egyenesen
3 pont legyen.
1345. [3] Helyezzünk el 8 pontot és 7 egyenest a síkon úgy, hogy mindegyik egyenesen
3 pont legyen.
1346. [3] Helyezzünk el 9 pontot és 9 egyenest a síkon úgy, hogy mindegyik egyenesen
3 pont legyen, és minden pontra 3 egyenes illeszkedjék.
1347. [4] Helyezzünk el 10 pontot és 10 egyenest a síkon úgy, hogy mindegyik egyenesen
3 pont legyen, és minden pontra 3 egyenes illeszkedjék.
1348. [3] Helyezzünk el 10 pontot és 5 egyenest a síkon úgy, hogy mindegyik egyenesen
4 pont legyen.
1349. [3] Az asztalra letettünk néhány egyforma méretű kockát. Ha ezeket elölről, ill.
oldalról nézzük, akkor a következőket látjuk:

Hány kocka lehet az asztalon? Legkevesebb és legfeljebb hány kocka lehet?


[3]
1350. Lehet-e olyan, a kockától különböző testet készíteni, melynek mindegyik oldal-
lapja négyzet?
116 Feladatok

1351. [5] Egy fából készült nem üreges tes-


tet felülről, ill. egyik oldalról néz-
ve az ábrán feltüntetett képet lát-
juk (a testre semmi sincs rajzol-
va). Mit látunk, ha elölről nézzük
a testet?

1352. [4] Adjunk meg 6 pontot a síkon úgy, hogy közülük bármelyiktől három másik
pont legyen 1 egység távolságra.
1353. [7] Megadható-e a síkon 7 pont úgy, hogy közülük bármely három között legyen
kettő, melyek távolsága 1 egység?
1354. [3] Van-e olyan konvex hatszög, melynek minden oldala nagyobb, mint 1 egység,
de egyik átlója sem hosszabb 2 egységnél?
1355. [4] Igaz-e a következő állítás: Ha a síkon 12 pont közül bármelyik kettő távolsága
legfeljebb 3 egység, akkor kiválasztható közülük négy pont úgy, hogy ezek
közül bármely kettő távolsága legfeljebb 2 egység?
1356. [4] Adjunk meg
a) 5, illetve
b) n pontot a síkon úgy, hogy közülük bármely három pont egy tompaszögű
háromszög csúcsait alkossa.
1357. [3] Adjunk meg 6 pontot a síkon úgy, hogy közülük bármely három pont egy
egyenlőszárú háromszög csúcsait alkossa.
1358. [4] 120◦ -os szárszögű egyenlőszárú háromszöget daraboljunk fel 5 egyenlőszárú
háromszögre.
1359. [4] Szabályos háromszöget daraboljunk fel 5 egyenlőszárú háromszögre.
1360. [4] Szabályos háromszöget daraboljunk fel 7 egyenlőszárú háromszögre.
1361. [3] Háromszöget daraboljunk fel 4 egyenlőszárú háromszögre.
1362. [3] Háromszöget daraboljunk fel 6, 7, ill. 8, hozzá hasonló háromszögre.
1363. [4] Van-e olyan háromszög, mely feldarabolható
a) három darab egybevágó, az eredetihez hasonló háromszögre?
b) öt darab egybevágó, az eredetihez hasonló háromszögre?
1364. [4] Felosztható-e az egyenlő szárú derékszögű háromszög hozzá hasonló, páronként
nem egybevágó háromszögekre?
1365. [4] Mutassuk meg, hogy bármely háromszöget négy darabra lehet vágni úgy, hogy
azokból két, az eredetihez hasonló háromszög állítható össze.
24. Geometriai konstrukciók 117

1366. [4] A 2 és 3 befogójú derékszögű háromszöget daraboljuk fel 13 db, az eredetihez


hasonló, egymással egybevágó háromszögre.
1367. [4] Egyenlő szárú háromszögből vágjunk ki 3 db egybevágó háromszöget úgy, hogy
ezek területe (külön-külön) nagyobb legyen, mint az eredeti háromszög terüle-
tének negyede.
1368. [3] Négyzetet daraboljunk fel 6, 7, ill. 8 db négyzetre.
1369. [4] Négyzetet daraboljunk fel hegyesszögű háromszögekre. Mutassuk meg, hogy a
négyzetet nem lehet feldarabolni nyolcnál kevesebb hegyesszögű háromszögre.
1370. [4] Téglalapot daraboljunk fel 5, 6, 7, ill. 8 téglalapra úgy, hogy közülük semelyik
két szomszédos se alkosson együtt téglalapot.
1371. [5] Daraboljunk fel egy négyzetet konvex ötszögekre.
1372. [5] Alkalmasan választott paralelogrammát daraboljunk fel 3 egyenlő szárú három-
szögre.
1373. [5] Fel lehet-e osztani egy négyzetet két egybevágó sokszögre, ahol a sokszög ol-
dalainak száma a) 7; b) 8?
1374. [6] Bontsunk fel egy körlapot egybevágó részekre úgy, hogy legyen olyan rész,
amely nem tartalmazza az origót (még a határán sem).
1375. [5] Azt mondjuk, hogy az A háromszög a B háromszögbe van írva, ha az A csú-
csai a B oldalegyeneseire illeszkednek és a két háromszögnek nincs közös ol-
dalegyenese. Létezik-e három olyan háromszög, A, B és C, melyek ciklikusan
egymásba vannak írva, azaz az A a B-be, B a C-be, és C az A-ba?
1376. [5] Adjunk meg két egymásba írt ötszöget. (Lásd az előző feladatot!)
1377. [5] Adjunk meg három, ciklikusan egymásba írt négyszöget. (Lásd az előző fela-
datokat!)
1378. [6] Adjunk meg két egymásba írt tetraédert. (Egy tetraéder egy másikba van írva,
ha csúcsai illeszkednek a másik lapsíkjaira, egy lapsíkra csak egy csúcs.)
1379. [3] Adjunk meg olyan konvex testet, melyen a lapok, élek, csúcsok száma rendre
6, 10, 6.
1380. [4] Van-e olyan (nem feltétlenül konvex) test, melynek ugyanannyi lapja, éle, csú-
csa van, mint a kockának, de nincs négyszöglapja?
1381. [4] Helyezzünk el a síkon végtelen sok kört úgy, hogy bármely egyenes legfeljebb
két kört metsszen.
1382. [4] Le lehet-e fedni a síkot véges sok parabolatartománnyal?
1383. [5] Pontszerű fényforrás eltakarható-e 4 gömbbel?
118 Feladatok

1384. [5] A síkon adott egy P pont. El lehet-e helyezni a síkon


a) 4 kört, illetve
b) 5 kört úgy, hogy egyik se tartalmazza a P pontot, és bármely P -ből induló
félegyenesnek legyen közös pontja legalább két körrel?
1385. [5] A síkon adott egy P pont. El lehet-e helyezni a síkon
a) 6 kört, illetve
b) 7 kört úgy, hogy egyik sem tartalmazza a P pontot, és bármely P -ből induló
félegyenesnek legyen közös pontja legalább három körrel?
1386. [5] Lehet-e egy egyenessel metszeni egy
a) 1990, illetve egy
b) 1991 oldalú sokszög minden oldalát?
1387. [5] El lehet-e helyezni az asztalon
a) 1990, illetve
b) 1991 egyforma pénzérmét úgy, hogy mindegyik három másikat érintsen?

25. Invariáns tulajdonságok

1388. [3] 24 papírlap közül néhányat 10 részre vágtak, majd az így kapott részek közül
néhányat ismét 10 részre vágtak szét és így tovább. Lehetséges-e, hogy ezt
néhányszor megismételve 1991 papírdarabot kapjunk?
1389. [3] Egy táblára felírtuk az 1, 2, 3, 4, 5, 6, 7, 8, 9, 10 számokat. Egy-egy alkalommal
két számot letörlünk és helyettük azok különbségét írjuk fel. Ezt kilencszer
elvégezve előfordulhat-e, hogy a megmaradó szám a nulla?
1390. [3] A táblán 50 darab 0 és 50 darab 1-es számjegy van. Egy alkalommal két szám-
jegyet törlünk le. Ha ezek egyenlők, akkor 1-est, ha különbözők, akkor egy 0-t
írunk a táblára. Milyen számjegy marad a táblán a kilencvenkilencedik törlés
után?
1391. [3] Béla egy papírlapot tetszőleges módon felosztott hat részre. Az így kapott részek
közül az egyik részt felosztotta 11 részre, majd a kapott részek egyikét ismét
6 részre, és így folytatta, arra sem ügyelve, hogy a 6, illetve 11 részre osztást
váltogatva végezze. Bizonyos számú osztás után megszámolta a kapott részeket,
és azt állította, hogy 1993 részt kapott.
Helyesen számolt-e Béla?
1392. [3] A táblára felírtuk az 1, 2, 3, . . . , 20 számokat. Egy-egy alkalommal letörlünk
két számot, a-t és b-t, s helyettük felírjuk az a + b − 1 számot. Ezt az eljárást
19-szer elvégezve milyen szám maradhat a táblán?
25. Invariáns tulajdonságok 119

1393. [4] A táblára felírtuk az 1, 2, 3, . . . , 20 számokat. Egy-egy alkalommal letörlünk


két számot, a-t és b-t, s helyettük felírjuk az ab + a + b számot. Ezt az eljárást
19-szer elvégezve milyen szám maradhat a táblán?
1394. [3] Egy vázában 75 fehér és 150 fekete babszem lapul. A váza mellett van egy
halom fekete bab. A babszemeket a következőképp szedegetjük a vázából: ta-
lálomra kiveszünk két babszemet; ha van köztük fekete, akkor azt kitesszük a
halomba, a másikat (akár fehér, akár fekete) visszadobjuk a vázába; ha azonban
mindkét babszem fehér, akkor eldobjuk őket, s a halomból egy fekete szemet
dobunk a vázába. Így a vázában levő babszemek száma egyesével csökken. Ha
már csak egy szem bab van a vázában, akkor mit mondhatunk annak színéről?
1395. [3] A Csodakert fáin 25 banán és 30 narancs van. Egy-egy alkalommal két gyümöl-
csöt veszünk le: ha egyformákat vettünk le, akkor egy narancs nő helyettük; ha
különbözőket vettünk le, akkor egy banán nő. Utolsónak milyen gyümölcs ma-
rad?
1396. [3] Kezdetben egy 3 × 3-as táblázat minden mezőjén 0 áll, majd egy-egy lépésben
a tábla valamely 2 × 2-es részén a számok mindegyikét 1-gyel növeljük. Ilyen
lépésekkel megkaphatjuk-e a mellékelt kitöltést?
4 9 5
10 18 12
6 13 7

1397. [3] Két kupacban gyufák vannak. Egy-egy alkalommal valamelyik kupacból elve-
szünk néhány szálat, s a másik kupacba kétszer annyit helyezünk. Elérhető-e,
hogy mindkét kupacban ugyanannyi gyufaszál legyen, ha kezdetben az egyik
kupacban 1, a másikban 2 szál volt?
1398. [4] Egy háromszög csúcsaiba gyufákat helyeztünk. Egy-egy alkalommal bárme-
lyik csúcsból vehetünk el gyufákat, de a másik két csúcs mindegyikébe kétszer
annyit kell helyeznünk. Kezdéskor az egyik csúcsban 1 szál volt, a többiben
egy sem. Elérhető-e, hogy mindegyik csúcsban ugyanannyi gyufa legyen?
1399. [4] Egy négyzet csúcsaiba gyufákat helyeztünk. Egy-egy alkalommal bármelyik
csúcsból vehetünk el gyufákat, de az egyik, ezzel szomszédos csúcsba kétszer
annyi gyufát kell helyezni. Kezdéskor a csúcsokban valamilyen körüljárást te-
kintve a gyufák száma: 1, 0, 0, 0. A fentebb közölt eljárást ismételve elérhető-e,
hogy a csúcsokban levő gyufák száma 1, 9, 8, 9 legyen?
1400. [5] Az asztalon egy kupacban 1001 kavics van. Egy lépésben valamely, 1-nél több
kavicsot tartalmazó kupacból kidobunk egy kavicsot, a maradékot két részre
osztjuk. Elérhető-e ilyen lépésekkel, hogy az asztalon 3–3 kavicsból álló kupa-
cok legyenek?
120 Feladatok

1401. [4] Egy szigeten 13 szürke, 15 barna, 17 zöld kaméleon él. Ha két különböző színű
kaméleon találkozik, akkor annyira megijednek egymástól, hogy mindketten a
harmadik színre változtatják bőrüket. Két azonos színű kaméleon nem ijed meg
egymástól, így találkozáskor nem változtatják meg színüket. Lehetséges-e, hogy
egy idő múlva minden kaméleon ugyanolyan színű legyen? Ha kezdetben 13
szürke, 25 barna és 17 zöld kaméleon él a szigeten, akkor elérhető-e, hogy
mindegyik kaméleon ugyanolyan színű legyen. Milyen szín lehet ez?
1402. [4] Egy táblára 6 db 0, 7 db 1-es és 8 db 2-es számot írtunk fel. Egy-egy alkalommal
két különböző számot letörlünk, s helyettük a harmadikat írjuk fel (pl. ha 0-t
és 2-est törlünk le, akkor 1-est írunk fel). Néhány ilyen törlés-felírás után csak
egy szám marad a táblán. Mi lehet ez a szám?
1403. [4] Egy táblára az 1, 2, 3, . . . , 1988 számokat írtuk fel. Egy-egy alkalommal két
számot letörlünk, s helyettük összegük 13-as osztási maradékát írjuk fel. Né-
hány ilyen törlés-felírás után csak egy szám marad a táblán. Mi lehet ez a szám?
√ √
1404. [5] A táblára a következő számok vannak felírva: 7 + 2, 5 2 − 1, 9. Bármely
lépésben√kiválaszthatunk √ a táblán két számot, mondjuk a-t és b-t; és azokat
(a + b)/ 2-vel és (a−b)/ 2-vel helyettesítjük.
√ √Elérhető-e, hogy valahány lépés
után a táblán valamilyen sorrendben 8 + 2, 5 2, 7 számok álljanak?
1405. [5] A táblára felírtunk néhány, nullától különböző számot, majd egy-egy lépésben
valamely két számot, A-t és B-t letöröljük; s helyettük két új számot írunk fel,
B A
az A + és B − számokat. Néhány ilyen lépés után megkaphatjuk-e újra az
2 2
eredetileg felírt számokat?
1406. [3] Egy négyzet csúcsaiba számokat írtunk. Egy-egy alkalommal két szomszédos
csúcs mindegyikében 1-gyel növeljük az ott levő számokat. Elérhető-e, hogy
mindegyik csúcsban ugyanaz a szám álljon, ha kezdetben
a) az egyik csúcsban 1, a többiben 0 van?
b) két szemközti csúcsban 1, a többiben 0 van?
1407. [4] Egy kocka csúcsaiba számokat írtunk. Egy-egy alkalommal valamelyik él két
végén álló számot 1-gyel növelhetjük. Ezt az eljárást néhányszor megismételve
elérhető-e, hogy minden csúcsban ugyanaz a szám álljon, ha a kezdő állapotban
a) az egyik csúcsban 1-es, a többiben 0 van?
b) az egyik oldallap két szemközti csúcsában 1-es, a többiben 0 van?
1408. [4] Egy asztalt hatan ülnek körbe, s egyikük előtt 6 tányér van. Egy-egy alka-
lommal bárki elvehet az előtte levő tányérok közül kettőt, és azokat valame-
lyik szomszédjának vagy szomszédainak adja át. Ezt néhányszor megismételve
elérhetjük-e, hogy mindenki előtt 1 tányér legyen?
25. Invariáns tulajdonságok 121

1409. [4] Egy asztalon sorban 6 tányér van. Ubul mindkét kezével megragad egy-egy
tányért a hat közül, és mindkettőt egy hellyel jobbra vagy balra helyezi (ha már
van ott tányér, akkor annak tetejére). El tudja-e érni, hogy az összes tányér egy
oszlopba kerüljön?
1410. [4] Egy asztal körül hatan ülnek, és közülük két személy előtt egy-egy tányér van, a
többiek előtt nincs tányér. E két személy között egy ember ül. Egy lépésben va-
lamely két szomszédos személy elé egy-egy újabb tányért helyezünk. Elérhető-e
néhány ilyen lépéssel, hogy mindenki előtt ugyanannyi tányér legyen?
1411. [4] Egy házaspár négy másik házaspárt hív meg vacsorára. Egy kör alakú asztalhoz
felváltva ülnek le a nők és a férfiak. Mindenki előtt egy pohár van, s ezek
egyszer csak elkezdenek vándorolni. Minden percben egy pohár egyet balra,
egy másik pohár egyet jobbra lép. Elérhető-e, hogy egy idő múlva minden pohár
a házigazda előtt legyen?
1412. [4] Egy körvonalon 44 fa helyezkedik el. Mindegyik fán ül egy majom. Egy-egy
perc elteltével valamelyik két majom átugrik a szomszédos fára, az egyik az
óramutató járásával azonos, a másik ellentétes irányba. Lehetséges-e, hogy egy
idő múlva mindegyik majom ugyanazon a fán ül?
1413. [4] Egy tisztáson 14 fa áll körben, és mindegyiken egy-egy mókus ül. Megen-
gedjük, hogy egyszerre két mókus valamelyik szomszédos fára ugorjon át.
Lehetséges-e, hogy egy idő múlva mindannyian ugyanarra a fára kerüljenek?
1414. [6] Van egy N tagú társaság. A társaság minden tagja legfeljebb 19 másik tagot
ismer. Feladatunk az, hogy a társaságot 2 részre bontsuk, a „7-esek” és a „11-
esek” klubjára úgy, hogy a „7-esek” klubjukon belül legfeljebb 7, a „11-esek”
klubjukon belül legfeljebb 11 másik tagot ismerjenek. Egy matematikus a követ-
kező eljárást ajánlja: először osszuk tetszőlegesen két részre a társaságot, majd
„fokozatosan”, lépésenként javítsuk ki az esetleges hibákat: ha valaki egy adott
lépésben a „7-esek” klubjához tartozik és ebben a klubban legalább 8 ismerőse
van, tegyük át a „11-esek” klubjába, ill. fordítva, ha valakinek a „11-esek” klub-
jában legalább 12 ismerőse van, tegyük át a „7-esek” közé. Így minden csere
után megváltozik a klubok összetétele. Igaz-e, hogy ez az eljárás véges számú
lépésben biztosan célra vezet?
1415. [6] Egy 10 × 10-es tábla 9 mezője gyomos. Egy parcella (mező) elgyomosodik, ha
annak legalább két oldalszomszédja gyomos. Lehetséges-e, hogy előbb-utóbb
az egész tábla gyomos lesz?
1416. [6] Egy tornaórán n gyerek áll egy sorban a tanárral szemben. Adott jelre mind-
egyikük 90◦ -kal elfordul: néhányuk balra, mások jobbra. Ezután vezényszóra
mindazok, akik valamelyik szomszédjukkal szemben állnak, hátraarcot csinál-
nak. Bizonyítsuk be, hogy az (n + 1)-edik vezényszóra már senki sem mozdul
meg.
122 Feladatok

1417. [5] Bal oldalon egy sáska, középen egy szöcske, jobb oldalon egy tücsök ül egy
hosszú, egyenes árokban. Időnként valamelyik átugorja egyik szomszédját. Elő-
fordulhat-e, hogy 1999 ugrás után újra a kiinduló sorrendben ülnek, ha végig
csak az árokban (egy egyenes mentén) ugrálnak?
1418. [4] Egy asztalon 6 pohár áll. Egy-egy alkalommal 5 poharat megfordítunk. E mű-
velet ismétlésével elérhetjük-e, hogy mindegyik pohár meg legyen fordítva?
1419. [4] Adott egy 10 elemű sorozat, mely +1 és −1 számokból áll. A sorozat öt ele-
mének megváltoztathatjuk az előjelét. Ezt a változtatást többször megtehetjük.
Elérhető-e bizonyos számú lépés után, hogy olyan sorozatot kapjunk, mely az
eredetitől csak egy helyen különbözik? Mi a válasz akkor, ha egy-egy alkalom-
mal négy elem előjelét változtatjuk meg?
1420. [4] Egy táblára 10 db + és 15 db − jelet írtunk fel. Egy-egy alkalommal két jelet
letörlünk, s helyettük + jelet írunk fel, ha a két jel egyforma volt, különben pedig
− jelet. 24 ilyen törlés-felírás után már csak egy jel marad a táblán. Melyik jel
lehet az?
1421. [5] Az ábrán látható táblázat bármelyik sorában vagy osz- + + − +
lopában, vagy valamelyik átlóval párhuzamosan elhe-
lyezkedő mezőkben ellentétesre változtatjuk az előjele- + + + +
ket. Ezt a műveletet többször megismételve elérhetjük-e, + + + +
hogy mindegyik mezőben + jel legyen? + + + +
1422. [5] Egy 8 × 8-as táblázat mindegyik mezőjébe egy-egy egész számot írtunk. Meg-
engedett a következő művelet: a táblázat bármely 3 × 3-as vagy 4 × 4-es részén
a számok mindegyikéhez 1-et adunk. Ilyen műveletekkel elérhető-e a táblázat
bármilyen kezdeti kitöltése esetén, hogy a benne levő mindegyik szám osztható
legyen 3-mal?
1423. [4] Szabályos 12-szög egyik csúcsában mínusz jel, a többiben plusz jel áll. Egy-egy
alkalommal
a) 6,
b) 5,
c) 4, illetve
d) 3 szomszédos csúcsban változtatjuk meg az előjeleket.
Az eljárást megismételve elérhetjük-e, hogy a mínusz jel az egyik szomszédos
csúcsba költözzék át, s a többi 11 csúcsban plusz jel álljon?
1424. [4] Adott egy négyzet három csúcsa. Új pontot úgy kapunk, hogy a már meglevő
pontot másik (ugyancsak meglevő) pontra tükrözzük. Ezt az eljárást néhányszor
megismételve megkaphatjuk-e a négyzet hiányzó negyedik csúcsát?
25. Invariáns tulajdonságok 123

1425. [4] Adott a síkon négy pont: (0; 0), (1; 1), (3; 0), (2; −1). Új pontot úgy kapunk,
hogy a már meglevő pontot másik (ugyancsak meglevő) pontra tükrözzük. Ezt
az eljárást néhányszor megismételve megkaphatjuk-e a (0; 0), (0; 1), (1; 0), (1; 1)
csúcspontokból álló négyzetet?
1426. [4] Az 1, 9, 9, 1, 0, 9, . . . sorozatot úgy képezzük az ötödik elemétől kezdődő-
en, hogy az azt megelőző négy számot összeadjuk, s az összeg utolsó jegye
lesz ez az elem. Szerepelhet-e a sorozatban valahol egymás után az 1, 2, 3, 4
részsorozat?
1427. [6] Az 1, 0, 1, 0, 1, 0, 3, . . . sorozatot úgy képezzük a hetedik elemétől kezdve,
hogy az azt megelőző hat számot összeadjuk, s az összeg utolsó jegye lesz ez
az elem. Szerepelhet-e ebben a sorozatban valahol egymás után a 0, 1, 0, 1, 0,
1 részsorozat?
1428. [4] Az (a, b) számpárból a (b, a), (a + b, b), (a − b, b) számpárok bármelyikét
származtathatjuk. Ezeket a műveleteket a meglévő számokkal megismételve
eljuthatunk-e a (12, 21) számpárból a
a) (19, 91), illetve a
b) (15, 51) számpárhoz?

[4] a b
1429. Az (a, b) számpárból származtathatjuk az (a + 1, b + 1), , (utóbbit,
2 2
ha a és b is páros) számpárok bármelyikét. Két, már korábban származtatott
(a, b) és (b, c) párból kaphatjuk az (a, c) számpárt is. Ezekkel a műveletekkel
eljuthatunk-e az (5, 19) számpárból az
a) (1, 50), illetve az
b) (1, 100) számpárhoz?
1430. [4] Az 1, 2, 3, . . . , n számok kezdetben ebben a sorrendben állnak. Egy művelet
két szám felcserélését jelenti. Lehet-e 15 cserével újra az eredeti sorrendhez
jutni?
1431. [5] Egy sorban 11 szék áll, melyek az 1, 2, 3, . . . , 11 számokkal ebben a sorrend-
ben vannak megszámozva. Megengedett művelet négy egymás után álló szék
sorrendjének megfordítása (pl. 4, 5, 9, 8-ból 8, 9, 5, 4 lesz). Ilyen műveletek-
kel felcserélhető-e két szomszédos szék úgy, hogy a többi sorrendje változatlan
maradjon? Mi a válasz akkor, ha 11 helyett 12 szék van, vagy 4 helyett 3 szék
sorrendje fordítható meg? Mi a válasz akkor, ha a székek egy asztal körül kör-
ben vannak elhelyezve?
1432. [6] Az ábrán látható 4 × 4-es keretben 15 számozott lapocs- 1 2 3 4
ka van. Az üres helyre mindig oda tudjuk tolni bárme-
5 6 7 8
lyik lapszomszédot. Ilyen tologatással elérhetjük-e, hogy
mindegyik lap visszakerüljön arra a helyre, ahol a kiindu- 9 10 11 12
ló állapotban volt, és csak az 1-es és a 2-es jelű legyen 13 14 15
felcserélve?
124 Feladatok

1433. [7] Képzeljünk el egy 4 × 4 × 4-es kockát, mely 64 egybevágó kis kockából van
kirakva. Ezek egyikét kivesszük, s a többi kockát megszámozzuk 1-től 63-ig, s a
sorszámot ráírjuk a kis kockák mind a hat oldallapjára. A játék elején a kockák
valamilyen sorban vannak. Tegyük fel, hogy az üres helyre mindig oda tudjuk
tolni bármelyik lapszomszédját. A kérdés ezek után, hogy el tudunk-e érni ilyen
tologatásokkal olyan helyzetet, amelyben minden kis kocka ugyanazon a helyen
van, mint a kiinduló állásnál (a belül lévő, sehonnan sem láthatók is!), csak
éppen az 1-es és a 2-es jelű fel van cserélve.
1434. [5] Egyszerű megmutatni, hogy egy háromszög átdarabolható téglalappá; ekkor az
átdarabolás során a részeket elforgatjuk. Felosztható-e a háromszög olyan ré-
szekre, melyeket megfelelően eltolva téglalapot kapunk?
1435. [4] Egy táblára az x 2 +10x +20 polinomot írtuk. Valaki egy-egy alkalommal a poli-
nom egyik tetszőlegesen választott együtthatóját 1-gyel növeli vagy csökkenti.
Egy idő múlva az x 2 + 20x + 10 polinom kerül a táblára. Mutassuk meg, hogy
közben a táblán szerepelt olyan polinom, melynek volt egész gyöke.
1436. [4] Egy konkáv sokszögön a következő műveletet végezzük el. Kiválasztunk két,
nem szomszédos csúcsot, legyenek ezek A és B. Ha a sokszög az AB egyenes
egyik oldalán van, akkor határvonalának azt a részét, amely A és B közé esik,
tükrözzük az AB felezőpontjára. Bizonyítsuk be, hogy elég sokszor ismételve
ezt az eljárást, konvex sokszöget kapunk.
1437. [4] Egy n×m-es táblázat mezőibe számokat írtunk. Egy-egy alkalommal bármelyik
sorban vagy oszlopban megváltoztathatjuk a számok előjelét. Mutassuk meg:
ilyen műveletekkel elérhető, hogy minden sorban és minden oszlopban az ott
álló számok összege ne legyen negatív!
1438. [5] Egy körvonalra felírtunk néhány számot. Ha néhány, egymás után elhelyezkedő
a, b, c, d számra (a − d)(b − c) < 0, akkor b és c helyet cserél. Igazoljuk, hogy
ezt az eljárást csak néhányszor lehet végrehajtani.
1439. [7] Egy szabályos ötszög csúcsaihoz egy-egy egész számot rendelünk úgy, hogy
összegük pozitív legyen. Megengedett a következő művelet. Ha három szom-
szédos csúcs X, Y , Z és a hozzájuk rendelt számok x, y, z és y < 0, akkor az x,
y, z számok helyére ugyanilyen sorrendben az x + y, −y, z + y számokat írjuk.
Ezt a műveletet ismételgetjük addig, amíg csak található negatív y. Döntsük el,
vajon minden esetben véget ér-e az eljárás véges sok lépés után.
1440. [5] A számnégyeseken értelmezzük a következő műveletet:
(a, b, c, d) → (a − b, b − c, c − d, d − a).
Mutassuk meg, hogy ha kezdetben a négy szám között van legalább két külön-
böző, akkor ezt az eljárást egymás után elég sokszor végrehajtva, a négy szám
között lesz olyan, mely nagyobb, mint 1991.
25. Invariáns tulajdonságok 125

1441. [5] A végtelen négyzetrács néhány mezőjét fehérről feketére festettük. Ezt követő-
en percenként minden mező átváltozik arra a színre, amelyikből több van két
oldalszomszédja és a fölötte levő mező színei közt. Bizonyítsuk be, hogy egy
idő múlva minden mező fehér színű lesz.
1442. [5] A végtelen négyzetrács néhány mezőjét fehérről feketére festettük. Ezt követő-
en percenként minden mező átváltozik a másik színre, ha olyan színű a jobb
oldali szomszédja és a fölötte levő mező (különben megőrzi eredeti színét). Bi-
zonyítsuk be, hogy egy idő múlva minden mező fehér színű lesz.
1443. [5] Az erdőben lakó törpék mindegyikének háza vagy fehér, vagy kék színű. Meg-
állapodnak abban, hogy ha találnak maguk közt olyat, akinek háza más színű,
mint barátai többségének háza, akkor az átfesti erre a színre házikóját. Minden
nap keresnek egy-egy ilyen törpét. Mutassuk meg, hogy néhány nap elteltével
már senkinek sem kell átfestenie a házikóját.
1444. [5] A törpék városában járvány tört ki, néhányan megfáztak, és influenzásak lettek.
Ezután a járvány fertőzéssel terjedt: ha egy egészséges törpe meglátogatta beteg
barátait, akkor másnapra ő is beteg lett. A törpék egy napig betegek, utána pe-
dig egy napig immunisak (aznap nem fertőződhetnek meg). Minden egészséges
törpe naponta meglátogatja beteg barátait. Bizonyítsuk be, hogy ha valamelyik
törpe a járvány első napján be volt oltva (vagyis azon a napon immunis volt),
akkor a járvány akár örökké is tarthat, különben biztosan véget ér.
1445. [5] Egy 5 × 5-ös telket 25 darab 1 × 1-es parcellára osztottunk. Van 25 manónk,
akik közül mindegyik legfeljebb három másikat utál (az utálat kölcsönös). Bi-
zonyítsuk be, hogy szétoszthatók a manók a parcellákba úgy, hogy egyetlenegy
manó se utálja a szomszédait. (Két parcella szomszédos, ha van közös oldaluk.)
1446. [5] A parlamentben egy-egy képviselőnek legfeljebb három ellenfele van (az ellen-
ségesség kölcsönös). Mutassuk meg, hogy a képviselők két csoportba oszthatók
úgy, hogy bárkinek a saját csoportjában legfeljebb egy ellenfele van.
1447. [5] Egy 2n fős társaságban mindenkinek legfeljebb n − 1 ellensége van (az el-
lenségesség kölcsönös). Mutassuk meg, hogy a társaság leültethető egy kerek
asztalhoz úgy, hogy az egymás mellett ülők nem ellenségek.
1448. [5] A síkon adott n pont, néhány közülük szakasszal összekötve. Bármely pontból
legfeljebb 11 szakasz indul. Bizonyítsuk be, hogy a pontok 4 színnel színezhe-
tők úgy, hogy az azonos színű pontokat összekötő szakaszok száma nem több
n-nél.
1449. [5] Adott a síkon 2n pont, melyek közül semelyik három sincs egy egyenesen.
A pontok fele kék, fele piros színű. Mutassuk meg, hogy rajzolható e pontok
között n db olyan szakasz, amelyek mindegyike egy piros és egy kék pontot
köt össze, s a szakaszoknak nincs közös pontja.
126 Feladatok

1450. [5] Adott a síkon n db pont, melyek közül semelyik három sincs egy egyenesen.
Mutassuk meg, hogy van olyan, önmagát nem metsző zárt görbevonal, melyre
mindegyik pont illeszkedik.
1451. [5] Egy 3 × 3-as táblázat mezőiben +1 vagy −1 áll. Egy-egy alkalommal min-
den mezőbe az élekkel szomszédos mezőkben álló számok szorzatát írjuk, s
így jutunk a régi táblázatból egy új kitöltéséhez. Igazoljuk, hogy az eljárást
néhányszor megismételve, minden mezőben +1 fog állni.
1452. [3] Egy körre tetszőlegesen 4 db 1-es és 5 db 0 számot írtunk. Egy-egy alkalommal
két szomszédos egyenlő szám közé 0-t, két szomszédos különböző szám közé
1-est írunk, majd letöröljük az előző kilenc számot. Ezt az eljárást néhányszor
megismételve, kaphatunk-e kilenc 0 számot?
1453. [5] Legyen (a1 , a2 , . . . , an ) olyan szám-n-es, ahol az ai számok értéke +1 vagy −1.
Tekintsük a következő műveletet:
(a1 , a2 , . . . , an ) → (a1 a2 , a2 a3 , . . . , an−1 an , an a1 ).
Mutassuk meg, hogy ha n = 4, akkor az eljárást újra és újra megismételve a
kapott számnégyesen, egy idő múlva csupa +1-ből álló számnégyest kapunk.
1454. [5] Tekintsük a következő műveletet:
(a1 , a2 , . . . , an ) → (|a1 − a2 |, |a2 − a3 |, . . . , |an−1 − an |, |an − a1 |),
ahol az ai számok nem negatív egész számok. Mutassuk meg, hogy ha
n = 4, akkor az eljárást újra és újra megismételve a kapott számnégyesen, egy
idő múlva csupa 0-ból álló számnégyest kapunk.

26. Feladatok a sakktáblán

1455. [3] Miért nem lehet egy 8 × 8-as sakktábla bal alsó sarkából a huszárt eljuttatni
a jobb felső sarokba úgy, hogy mire odaér, addig a sakktábla minden mezőjét
pontosan egyszer érinti?
1456. [3] A sakktábla bal alsó sarkát kivágtuk. A jobb felső sarokból indulva be lehet-
e járni ezt a táblát egy huszárral úgy, hogy minden mezőt pontosan egyszer
érintünk?
1457. [3] Egy 8 × 8-as sakktábla egyik mezője hiányzik. Be lehet-e járni ezt a táblát egy
huszárral úgy, hogy minden mezőt pontosan egyszer érintünk, s utolsónak olyan
mezőre érkezünk, mely szomszédos azzal a mezővel, amelyről elindultunk?
1458. [3] A huszár n lépést tett meg a sakktáblán és visszajutott a kiindulási mezőre.
Mutassuk meg, hogy n páros szám.
1459. [3] Egy 8 × 8-as sakktábla bal alsó sarkából el lehet-e jutni egy huszárral a jobb
felső sarokba úgy, hogy közben minden sorba pontosan egyszer lépünk?
26. Feladatok a sakktáblán 127

1460. [4] Bizonyítsuk be, hogy a 4×5-ös sakktáblát bejárhatja a huszár úgy, hogy minden
mezőre pontosan egyszer lép.
1461. [4] Bejárható-e a 4 × n-es sakktábla egy huszárral úgy, hogy az minden mezőre
pontosan egyszer lépjen, és utolsó lépésével éppen visszaérjen a kiindulási me-
zőre?
1462. [4] Bejárható-e a 7 × n-es sakktábla (2, 3) huszárral úgy, hogy minden mezőre
pontosan egyszer lépünk? (A (2, 3) huszár olyan figura, mely L alakban 2-t
vízszintesen, 3-at függőlegesen lép vagy fordítva.)
1463. [4] Egy n × n-es sakktáblán egy bábu léphet a szomszédos mezőre vagy jobb-
ra, vagy felfelé, vagy átlósan balra lefelé. Bejárható-e vele a tábla úgy, hogy
minden mezőre egyszer lép, s útja azon a mezőn ér véget, mely jobb oldali
szomszédja annak a mezőnek, amelyről elindult?
1464. [4] A 8 × 8-as sakktábla bal alsó 3 × 3-as sarkában 9 dámafigura áll. El lehet-e
juttatni ezt a 9 bábut
a) a bal felső, illetve
b) a jobb felső 3 × 3-as sarokba,
ha csak úgy lehet lépni, hogy egy másik figurát vízszintesen, függőlegesen vagy
átlósan átugorva üres mezőre érkezünk?
1465. [4] Legfeljebb hány huszárt helyezhetünk el a 8 × 8-as sakktáblán úgy, hogy egyik
se üsse a másikat?
1466. [4] Legfeljebb hány huszárt helyezhetünk el az 5×5-ös sakktáblán úgy, hogy egyik
se üsse a másikat?
1467. [4] Helyezzünk el minél kevesebb huszárt a 3 × 3-as, 4 × 4-es, . . . , 8 × 8-as,
9 × 9-es, 10 × 10-es sakktáblán úgy, hogy azok ütés alatt tartsák a nem fog-
lalt mezőket.
1468. [4] Legfeljebb hány királyt helyezhetünk el a 8 × 8-as sakktáblán úgy, hogy egyik
se üsse a másikat?
1469. [3] Helyezzünk el a 8 × 8-as sakktáblán minél több királynőt úgy, hogy semelyik
kettő se üsse egymást.
1470. [3] Helyezzünk el a 8 × 8-as sakktáblán minél kevesebb királynőt úgy, hogy azok
a tábla minden mezőjét ütés alatt tartsák.
1471. [4] Legkevesebb hány királyt kell a 8×8-as sakktáblán elhelyezni, ha minden mezőt
ütés alatt akarunk tartani?
1472. [4] Legfeljebb hány királynő helyezhető el a 8 × 8-as sakktáblán úgy, hogy mind-
egyik legfeljebb egy másikat üssön?
128 Feladatok

1473. [5] Helyezzünk el a 8×8-as sakktáblán 32 huszárt úgy, hogy mindegyik két másikat
tartson ütés alatt.
1474. [5] A 3 × 3-as sakktábla két felső sarkában fekete, a két alsó sarokban fehér hu-
szárok állnak. Elérhető-e néhány lépéssel, hogy az azonos színű huszárok a
szemközti sarkokban legyenek?
1475. [5] A 3 × 3-as sakktábla két felső sarkában fekete, a két alsó sarokban fehér huszá-
rok állnak. Elérhető-e néhány lépéssel, hogy a világos huszárok helyet cserél-
jenek a sötétekkel? Mennyi a lépések minimális száma?
1476. [4] A 8 × 8-as sakktábla mezőibe sorfolytonosan beírtuk 1-től 64-ig az egész szá-
mokat. Kiválasztunk 8 számot úgy, hogy minden sorból és minden oszlopból
veszünk. Milyen határok között mozoghat e számok összege?
1477. [4] A 8 × 8-as sakktáblán elhelyeztünk 8 bástyát úgy, hogy nem ütik egymást.
Mutassuk meg, hogy a fekete mezőkön álló bástyák száma páros.
1478. [4] Egy 5 × 5-ös sakktábla minden mezőjén áll egy bogár. Egy-egy perc elteltével
mindegyik bogár átmászik valamelyik oldalszomszédos mezőre. Igaz-e, hogy
minden órában van olyan perc, amikor valamelyik mező üresen áll?
1479. [3] Egy 5 × 5-ös táblázatba lehet-e számokat írni úgy, hogy a számok összege
minden sorban pozitív, minden oszlopban pedig negatív?
1480. [3] Egy 5 × 5-ös táblázatba lehet-e számokat írni úgy, hogy e számok összege
pozitív, ám a táblázat bármely 2 × 2-es részében a számok összege negatív?
1481. [3] Egy 5 × 5-ös táblázatba lehet-e számokat írni úgy, hogy a számok szorzata
mindegyik sorban 20, míg mindegyik oszlopban 30?
1482. [3] Egy 5 × 5-ös táblázat mindegyik sorába beírjuk valamilyen sorrendben az 1, 2,
3, 4, 5 számokat úgy, hogy a kapott kitöltés szimmetrikus a főátlóra. Mutassuk
meg, hogy ekkor a főátlóban ott áll mind az öt szám.
1483. [5] Egy n × n-es táblázat mezőibe egész számokat írtunk úgy, hogy közös oldalú
mezőkbe írt számok között legfeljebb 1 a különbség.
n Mutassuk meg, hogy van
olyan szám, amely a táblázatban legalább -szer szerepel.
2
1484. [4] Egy 8 × 8-as sakktábla mezőit valahogyan megszámoztuk az 1-től 64-ig terjedő
számokkal. Mutassuk meg, hogy van két olyan, oldalban szomszédos mező,
melyen álló számok különbségének abszolútértéke legalább 5.
1485. [5] Egy 8 × 8-as sakktábla mezőit valahogyan megszámoztuk az 1-től 64-ig terjedő
számokkal. Nevezzük két szomszédos mező távolságának a rajtuk álló számok
különbségének abszolútértékét. (Két mező szomszédos, ha van közös csúcsuk.)
Mi az a legnagyobb szám, amelynél nagyobb távolság a sakktábla bármely
számozásakor fellép a szomszédos mezők között?
26. Feladatok a sakktáblán 129

1486. [7] Egy n × n-es táblázat (mátrix) elemei különböző egész számok. Bizonyítsuk be,
hogy van két szomszédos (oszlopban vagy sorban egymás mellett álló) mező,
melyeken álló számok különbségének abszolútértéke legalább n.
1487. [5] Egy 10 × 10-es táblázatba beírunk 10-10 db 0, 1, 2, . . . , 9 számot.
a) Meg lehet-e ezt tenni úgy, hogy minden sorban és oszlopban legfeljebb 4
különböző szám szerepeljen?
b) Bizonyítsuk be, hogy van olyan sor vagy oszlop, amelyben legalább 4 kü-
lönböző szám van.
1488. [5] A végtelen négyzetrács mezőibe be szeretnénk írni a pozitív egész számokat,
mégpedig úgy, hogy az n szám éppen n-szer forduljon elő (azaz 1 darab 1-es,
2 db 2-es stb. szerepeljen); továbbá tetszőleges két, közös oldalú mezőbe került
szám különbségének abszolútértéke kisebb legyen egy előre megadott k-nál.
Melyik az a legkisebb egész k, amelyre a kitöltést el lehet végezni?
1489. [5] Ki lehet-e tölteni egy 1000×1000-es táblázat mezőit az 1-től 200 000-ig terjedő
egészekkel úgy, hogy minden számot pontosan ötször használunk fel, minden
mezőben egy szám áll, és az n-edik sor m-edik mezőjén az n · m szorzatnál
kisebb vagy egyenlő szám áll?
1490. [4] Egy 10 × 20-as táblázat soraiban, oszlopaiban álló számok mindenhol számtani
sorozatot alkotnak. A négy sarokban levő szám összege 30. Mennyi az összes
szám összege?
1491. [4] A négyzetrácsos mezőn kiválasztottunk 100 mezőt. Mutassuk meg, hogy ezek
között van 25 olyan, melyek közül semelyik kettőnek sincs közös pontja.
1492. [4] Egy 29 × 29-es négyzetrácsos papírból kivágtunk 99 olyan 2 × 2-es négyzetet,
amelyek csúcsai rácspontok. Bizonyítsuk be, hogy még egy századikat is ki
tudunk vágni.
1493. [5] Egy 7 × 7-es sakktáblán legfeljebb hány mezőt lehet kijelölni úgy, hogy ne
legyen négy olyan közöttük, amelyek egy, a tábla oldalaival párhuzamos állású
téglalap négy csúcsát alkotnák?
1494. [4] Az ábrán feketével jelzett 15 kör helyén érmék vannak. Az a célunk, hogy vala-
mennyi érme átkerüljön a fehérrel jelzett körök helyére. Egy lépésben bármely
érmével vízszintesen vagy függőlegesen átugorhatunk egy szomszédos érmét,
ha annak túlsó oldalán éppen nincs érme. Legalább hány lépésre van szükség a
cseréhez?
130 Feladatok

1495. [5] Ismert játék a szoliter. A játéktáblán 33 lyuk van,


s kezdetben a középső kivételével mindegyikben
van egy tüske. A játékot egy személy játssza; úgy
lépked, hogy egy tüskével vízszintes vagy függőle-
ges irányban átugrik egy másikat, s ekkor egy üres
lyukba érkezik. Az átugrott tüskét leveszi a tábláról.
A cél az, hogy a játék végén egyetlen tüske marad-
jon a táblán.
Mutassuk meg, hogy ha sikerül elérni a kitűzött
állapotot; akkor az egyedül maradt tüske vagy a tábla közepén áll, vagy
a négy oldal valamelyikének középső mezőjén.

1496. [3] A 8 × 8-as sakktábla bal alsó és jobb felső sarkát kivágtuk. Le lehet-e fedni
ezt a táblát hézagmentesen és átfedés nélkül 1 × 2-es dominókkal? A sakktábla
mely két mezőjének hiánya esetén valósítható meg ez a lefedés?
1497. [4] A 8 × 8-as sakktábla egyik sarokmezője hiányzik. Le lehet-e fedni ezt a táblát
1 × 3-as dominókkal? A sakktábla mely mezőjének hiánya esetén valósítható
meg ez a lefedés?
1498. [4] Egy 10 × 10-es sakktáblát le lehet-e fedni 1 × 4-es dominókkal?
1499. [4] Egy 10 × 10-es sakktáblát le lehet-e fedni alakú dominókkal?
[4]
1500. Lefedhető-e a 8 ×8-as sakktábla 15 db alakú és 1 db alakú dominóval?
[4]
1501. Lefedhető-e a 8 × 8-as sakktábla 15 db alakú és 1 db alakú dominóval?
[4]
1502. Lefedhető-e a 8 × 8-as sakktábla 1 db és néhány , ill. alakú
dominóval?
1503. [4] Lefedhető-e a 6 × 6-os sakktábla 9 db alakú dominóval?
[5]
1504. Egy téglalap lefedhető alakú és alakú dominókkal is. Lefedhető-e 1
db alakú és néhány alakú dominóval is?
1505. [3] Lehet-e téglalapot összerakni a következő öt alakzatból?

1506. [4] Egy 9×14-es táblát le lehet-e fedni 10 db 3×2-es és 11 db 2 × 3-as dominóval?
(Itt a dominók nem forgathatók el, míg a többi feladatban megtehetjük ezt.)
1507. [5] Ki lehet-e rakni egy 101 × 101-es táblát 2 × 2-es és 3 × 3-as dominókkal?
1508. [3] Egy 55 × 39-es táblát lefedhetünk-e 5 × 11-es dominókkal?
1509. [5] A 11 × 12-es sakktábla lefedhető 20 db dominóval, ha 1 × 6-os és 1 × 7-es
dominókból válogatunk (12 db 1 × 7-es és 8 db 1 × 6-os). Le tudjuk-e fedni
ezt a táblát 19 dominóval is, ha most is csak 1 × 6-os és 1 × 7-es dominókat
használhatunk?
26. Feladatok a sakktáblán 131

1510. [5] Hány 6×1-es dominót lehet elhelyezni átfedés nélkül egy 9×10-es sakktáblán?
(A dominókat a tábla oldalaival párhuzamosan helyezhetjük el.)
1511. [6] Bizonyítsuk be, hogy egy n×m-es tábla akkor és csak akkor fedhető le (k×1)-es
dominókkal, ha k|m vagy k|n.
1512. [7] Egy téglalapot le tudunk fedni egyrétűen olyan téglalapokkal, amelyeknek vala-
melyik oldala egész szám. Bizonyítsuk be, hogy az eredeti téglalapnak is egész
az egyik oldala.
1513. [5] 1 × 2-es dominókkal lefedtünk egy
a) 6 × 6-os, illetve egy
b) 4 × 100-as sakktáblát.
Mutassuk meg, hogy van olyan vízszintes vagy függőleges rácsegyenes, melyet
nem keresztez dominó!
1514. [4] A 8 × 8-as sakktábla összes mezőjét egy kivételével átfesthetjük-e fehérre, ha
egy lépésben valamely 2 × 2-es rész mezőinek színét ellentétesre változtatjuk?
(Mit mondhatunk 9 × 9-es sakktábla esetén?)
1515. [4] A 8 × 8-as sakktábla összes mezőjét egy kivételével átfesthetjük-e fehérre, ha
egy lépésben valamely sor és valamely oszlop mezőinek színét ellentétesre vál-
toztatjuk?
1516. [5] A 8 × 8-as sakktáblán egy-egy lépésben felcserélhetek egymással két sort vagy
két oszlopot. Elérhető-e ilyen lépésekkel, hogy a sakktábla egyik fele fehér, a
másik fele fekete legyen?
1517. [5] A 8 × 8-as sakktábla mindegyik mezőjén van egy kocka, melynek valamelyik
oldala fekete, a többi fehér. Elérhető-e mindig, hogy a kockák fekete oldala
legyen felül, ha egy-egy alkalommal valamelyik sorban vagy valamelyik osz-
lopban levő kockákat fordíthatjuk el közös tengelyük mentén? (A kockák lapjai
egybevágók a sakktábla mezőivel.)
1518. [4] Kirakható-e egy 7 × 9 × 11-es tégla 3 × 3 × 1-es téglákból?
1519. [6] Kirakható-e egy 6 × 6 × 6-os kocka 1 × 2 × 4-es téglákból?
1520. [6] Egy téglatest alakú ládát meg tudunk tölteni 1 × 2 × 4-es téglákkal. Mutas-
suk meg: ekkor a láda úgy is kitölthető, hogy az ugyanolyan hosszú téglaélek
párhuzamosak.
1521. [5] A 3 × 3 × 3-as kockát az oldallapokkal párhuzamos síkokkal 27 db egybe-
vágó kockára vágtuk. Levehetjük-e ezeket egymás után úgy, hogy mindegyik
lapszomszédos az előzőleg elvett kockával, s a középső megmarad?
1522. [6] Rajzolható-e a Rubik-kocka minden lapjának minden egyes kis négyzetén egy-
egy átló úgy, hogy egy önmagát nem metsző zárt töröttvonalhoz jussunk?
132 Feladatok

1523. [5] Egy 6 × 6 × 6-os kockába legfeljebb hány 1 × 1 × 4-es tégla helyezhető el? (A
téglák élei párhuzamosak a kocka éleivel.)
1524. [4] Kirakható-e egy 3 × 3 × 3-as kocka 6 db 2 × 2 × 1-es és 3 db 1 × 1 × 1-es
téglából?
1525. [4] Mely k esetén lehet egy k × k × k-as kocka egységkockáit fekete és fehér szí-
nekkel színezni úgy, hogy bármelyik kocka pontosan két ugyanolyan színű koc-
kával legyen szomszédos? (Két kocka szomszédos, ha van közös oldallapjuk.)
1526. [4] Lehet-e bábukat rakni a 8 × 8-as táblára úgy, hogy mind a 30 átlóban páratlan
legyen a bábuk száma?
1527. [4] Legfeljebb hány bábut lehet felrakni a 8×8-as táblára úgy, hogy minden sorban,
minden oszlopban és minden átlóban páros legyen az ott álló bábuk száma?
1528. [4] Egy 17×17-es táblázat mezőibe lehet-e számokat írni úgy, hogy nem mindegyik
nulla, és minden mezőben olyan szám áll, mely az oldalszomszédos mezőkben
álló számok összege?
1529. [4] Egy 20 × 30-as sakktáblán áthaladó egyenes metszheti-e a tábla 50 mezőjét?

27. Skatulyaelv

1530. [1] Egy ládában négyfajta alma van, minden fajtából egyenlő mennyiségű, összesen
100 darab. Hány almát kell kivenni véletlenszerűen, hogy valamelyik fajtából
biztosan legyen 10 alma?
1531. [1] Egy zacskóban 80 cukor van: 20 piros, 20 fekete, 20 zöld, 20 sárga. Egy bekö-
tött szemű gyereknek legalább hány cukrot kell kiemelnie ahhoz, hogy biztosan
legyen közöttük
a) valamelyik színből 4 darab?
b) mindegyik színből 4 darab?
1532. [1] Egy zsákban 11 piros, 8 fehér és 6 fekete golyó van. Hány golyót kell kivenni
véletlenszerűen, hogy biztosan legyen közte
a) fehér vagy fekete?
b) fehér és fekete?
c) két különböző szín?
d) valamelyik színből mind?
e) két színből mindegyik?
f) valamelyik színből legalább három?
27. Skatulyaelv 133

1533. [1] Egy zsákban 20 db fekete és 20 db fehér, egyforma méretű zokni van. Hány
darabot kell találomra kivenni, hogy biztosan legyen közte
a) 1 pár?
b) 1 pár fehér?
c) 2 pár?
d) 2 ugyanolyan színű pár?
e) 2 különböző színű pár?
f) 2 pár fehér?
g) 5 pár?
h) 3 pár fehér és 2 pár fekete?
1534. [1] Egy zsákban 10 fehér, 20 fekete, 30 barna, egyforma méretű zokni van. Hány
darabot kell kivenni véletlenszerűen, hogy biztosan legyen közte
a) 1 pár?
b) 1 pár fehér?
c) 1 pár fekete?
d) 1 pár barna?
e) 2 pár?
f) 5 pár?
g) 3 pár fehér?
h) 3 pár fekete?
i) 2 pár fehér és 1 pár barna?
j) 2 pár fehér, 3 pár fekete, 5 pár barna?
k) k pár?
1535. [1] Egy zsákban 10 pár fekete és 10 pár barna, ugyanolyan méretű kesztyű van.
Hány darabot kell találomra kivenni, hogy biztosan legyen köztük egy pár (azo-
nos színű) kesztyű?
1536. [1] Legalább mekkora létszámú az az osztály, ahol biztosan van két olyan diák,
akinek ugyanannyi foga van?
1537. [2] Legalább hány lakosa van annak az országnak, ahol biztosan van két olyan
lakos, akiknek ugyanolyan a fogazata? (Ugyanazon a helyen hiányoznak, ill.
vannak fogai.)
1538. [1] Igaz-e, hogy egy 37-es létszámú osztályban biztosan van négy diák, akik ugyan-
abban a hónapban ünneplik születésnapjukat?
134 Feladatok

1539. [3] Egy dobozban végtelen sok cédula van. Minden cédulára felírtunk egy termé-
szetes számot. Tudjuk, hogy bármiképpen veszünk is ki a dobozból két cédulát,
a rajtuk levő számok különbsége legfeljebb egymillió. Bizonyítsuk be, hogy
van olyan szám, mely végtelen sok cédulán szerepel. Igaz-e az állítás akkor is,
ha csak azt tudjuk, hogy bárhogyan választva végtelen sok cédulát, mindig lesz
köztük kettő, melyekre felírt számok különbsége legfeljebb egymillió?
1540. [3] Legalább hány, 13 tippes totóoszlop megfelelő kitöltésével biztosítható, hogy
valamelyik oszlopban legyen legalább 5 találatunk?
1541. [3] 8 × 8-as sakktáblán 31 bábut helyeztünk el. Mutassuk meg, hogy a táblának van
olyan alakú része, ahol egy bábu sem áll.
1542. [4] Egy gulyában két falu 65 tehene legel: vörösek, fehérek, feketék és tarkák.
Igazoljuk, hogy ha nincs öt különböző korú, azonos színű tehén a gulyában,
akkor található három azonos színű és egyidős tehén ugyanabból a faluból.
1543. [3] Egységsugarú körlapon 7 pontot helyeztünk el. Igazoljuk, hogy van közöttük
kettő, melyek távolsága nem nagyobb 1-nél.
1544. [4] Egységsugarú körlapon 6 pontot helyeztünk el. Igazoljuk, hogy van közöttük
kettő, melyek távolsága nem nagyobb 1-nél.
1545. [6] Egységsugarú körlapon 8 pontot helyeztünk el. Igazoljuk, hogy van közöttük
kettő, melyek távolsága kisebb 1-nél.
1546. [3] Egy 1 m oldalú szabályos háromszöglapon 10 pontot helyeztünk el. Mutassuk
meg, hogy van közöttük két olyan pont, melyek távolsága kisebb 34 cm-nél.
1547. [3] Egy 20 × 15-ös téglalapban felvettünk 26 db pontot. Mutassuk meg, hogy a
pontok között van kettő, melyek távolsága legfeljebb 5.
1548. [3] Egy 70 × 70-es négyzetben felvettünk 50 db pontot. Mutassuk meg, hogy a
pontok között van kettő, melyek távolsága kisebb 15-nél.
1549. [3] Egy 8×8-as négyzetben felvettünk 260 db pontot. Mutassuk meg, hogy a pontok
között van kettő, melyek távolsága kisebb 1-nél.
1550. [3] Egy 35 × 42-es téglalapban felvettünk 100 db pontot. Mutassuk meg, hogy van
olyan 6 × 5-ös téglalap, melybe legalább 2 pont került.
1551. [3] Egy 80 × 90-es téglalapban felvettünk 365 db pontot. Mutassuk meg, hogy van
olyan 5 × 8-as téglalap, melybe legalább 3 pont került.
1552. [3] Adott egy négyzet, melynek átlója 2 egység. Melyik az a legkisebb d szám,
amelyre igaz, hogy bármiképpen veszünk is fel öt pontot a négyzet belsejében
vagy kerületén, mindig kiválasztható közülük két olyan pont, amelyek távolsága
legfeljebb d?
27. Skatulyaelv 135

1553. [3] Adott egy 2 egység oldalú négyzet. El lehet-e helyezni a négyzet belsejében
vagy határán 9 pontot úgy, hogy bármely két pont távolsága legalább 1 legyen?
Mi a válasz 10 pont esetén?
1554. [4] Egy 3 × 4-es téglalapban felvettünk 6 db pontot.
√ Mutassuk meg, hogy a pontok
között van kettő, melyek távolsága legfeljebb 5.

1555. [5] A 6 3 és 18 egység befogójú derékszögű háromszög belsejében adott 865 pont.
Mutassuk meg, hogy a pontok közül kiválasztható 3, amelyeket lefed egy 1
egység átmérőjű zárt félkörlemez.
1556. [5] Egy egységnyi átfogójú derékszögű háromszögben, amelynek egyik szöge 30◦ ,
adott 25 pont. Bizonyítsuk be, hogy van a pontok között 3, amelyek lefedhetők
egy 5/17 átmérőjű körrel.
[6]

1557. Egy derékszögű háromszög befogói 1 és 3 egységnyi hosszúak. A háromszög
belsejében adott √
20 pont. Bizonyítsuk be, hogy kiválasztható közülük 3, amely
lefedhető egy 1/ 3 átmérőjű zárt félkörlemezzel.
1558. [4] Egy szabályos háromszög alakú, 1 m oldalhosszúságú céltáblát 7 találat ért. Bizo-
nyítsuk be, hogy ezek között van két olyan, amelyek távolsága kisebb 40 cm-nél.
1559. [4] Egy 8 cm oldalú négyzetben adott 33 pont. Mutassuk meg, hogy ezek között
van 3 olyan, melyek által meghatározott háromszög területe legfeljebb 2 cm2 .
1560. [4] Egységnégyzetben adott 51 pont. Igazoljuk, hogy van köztük 3 olyan, melyek
1
egy egység sugarú körben vannak.
7
1561. [4] Egy négyzet alakú, 1 m2 -es céltáblát 49 találat ért. Bizonyítsuk be, hogy van
4 olyan találati pont, melyek közül bármely kettőnek a távolsága kisebb, mint
36 cm.
1562. [6] Az 1 egység oldalú négyzetbe elhelyeztünk
a) 101; b) 59
pontot úgy, hogy nincs három, amely egy egyenesbe esik. Mutassuk meg, hogy
ezen pontok közül kiválasztható három úgy, hogy az általuk alkotott háromszög
1
területe legfeljebb területegység.
100
1563. [4] Egységnyi élű kocka belsejében felvettünk 400 különböző pontot. Mutassuk
4
meg, hogy van köztük 4 olyan, melyek benne vannak egy sugarú gömbben.
23
1564. [4] Egy 10 egység sugarú körben felvettünk 450 pontot. Igazoljuk, hogy ezek között
van kettő, melyek távolsága kisebb, mint 1 egység.
1565. [5] Egy 16 egység sugarú körlemezen elhelyeztünk 650 pontot. Rendelkezésünkre
áll egy körgyűrű, melynek külső sugara 3 egység, belső sugara 2 egység. Mutas-
suk meg, hogy ez elhelyezhető úgy, hogy a pontokból legalább 10-et lefedjen.
136 Feladatok

1566. [4] Egy kör belsejében tetszőlegesen kijelölünk 17 pontot. Bizonyítsuk be, hogy
mindig található közülük kettő, amelyek távolsága kisebb a kör sugarának 2/3
részénél.
1567. [4] Egy egység sugarú kör kerületét egy szabályos n-szög és egy szabályos (n + 1)-
szög csúcsai ívekre darabolják. Bizonyítsuk be, hogy van olyan ív, melynek
π
hossza legfeljebb .
n(n + 1)
1568. [5] Az n-oldalú szabályos sokszög csúcsai A1 , A2 , A3 , . . . , An , és adott a belsejé-
ben egy O pont.

1
Mutassuk meg, hogy van olyan i, j , melyekre 1 − π ≤ Ai OAj  ≤ π.
n
1569. [4] Az egységnyi élű kocka minden pontját kiszínezzük 3 szín valamelyikével.
Igaz-e, hogy mindig van két egyszínű pont, melyek távolsága legalább 1,4?
1570. [5] Egy szabályos háromszöget lefedtünk 5 egybevágó szabályos háromszöggel. Bi-
zonyítsuk be, hogy megfelelő elhelyezéssel ezek közül már négy háromszöggel
is lefedhető az eredeti háromszög.
1571. [4] 9 egyenes mindegyike metszi egy adott négyzet két szemközti oldalát, s a négy-
zet területét 2:3 arányban osztja. Mutassuk meg, hogy van közöttük 3 egyenes,
mely egy ponton megy át.
1572. [4] Egység sugarú körvonalon elhelyeztünk 4 pontot.
√ Mutassuk meg, hogy van
közöttük kettő, melyek távolsága nem nagyobb 2-nél.
1573. [4] Egység sugarú gömb felületén elhelyeztünk 5 pontot.
√ Mutassuk meg, hogy van
közöttük kettő, melyek távolsága nem nagyobb 2-nél.
1574. [4] Egység oldalú négyzet belsejébe köröket írtunk, melyek sugarainak összege
0,51. Bizonyítsuk be, hogy van olyan, a négyzet valamelyik oldalával párhu-
zamos egyenes, mely legalább két kört metsz.
1575. [4] Egység oldalú négyzet belsejébe köröket írtunk, melyek kerületének összege
10 egység. Bizonyítsuk be, hogy van olyan egyenes, amely legalább négy kört
metsz.
1576. [5] Egy 3 egység sugarú gömb belsejében elhelyeztünk néhány kisebb gömböt
(ezek metszhetik is egymást), melyek sugarainak összege 25 egység. Mutassuk
meg, hogy van olyan sík, amely legalább kilenc kisebb gömböt metsz.
1577. [5] Egy egységnyi oldalhosszúságú négyzet belsejébe egy 1000 egységnél hosz-
szabb, önmagát nem metsző töröttvonalat rajzoltunk (a töröttvonal egymáshoz
végpontjaikkal csatlakozó egyenesszakaszokból áll). Bizonyítsuk be, hogy van
olyan, a négyzet valamely oldalával párhuzamos egyenes, mely a töröttvonalat
legalább 501 pontban metszi.
27. Skatulyaelv 137

1578. [5] Adott a síkban egy egyenes, egy n cm sugarú kör (n pozitív egész szám) és
a körben 4n db 1 cm-es szakasz. Bizonyítsuk be, hogy húzható az adott egye-
nessel párhuzamosan vagy rá merőlegesen olyan húr, amelynek legalább két
szakasszal van közös pontja.
1579. [4] Megjelöltük a 8×8-as sakktábla mezőinek középpontjait Feloszthatjuk-e a sakk-
táblát 13 egyenessel részekre úgy, hogy egyik részben se legyen két megjelölt
pont?
1580. [4] A síkon adott 25 pont úgy, hogy közülük bárhogyan választva 3 pontot, azok
közt van kettő, melyek távolsága kisebb, mint egy egység. Mutassuk meg, hogy
van olyan 1 egység sugarú kör, mely lefed e pontokból legalább 13-at.
1581. [5] Adott a síkon 1000 pont: P1 , P2 , P3 , . . . , P1000 és egy 1 egység sugarú kör.

1000
Mutassuk meg, hogy van a körnek olyan P pontja, melyre |P Pi | ≥ 1000.
i=1
[3]
1582. A sík minden pontját kékre vagy pirosra színeztük. Igazoljuk, hogy van két
azonos színű pont, melyek távolsága 1 egység.
1583. [3] A sík minden pontját kékre vagy pirosra színeztük. Mutassuk meg, hogy van
két különböző színű pont, melyek távolsága 1 egység.
1584. [7] A sík minden pontját három szín valamelyikével színeztük. Mutassuk meg,
hogy van két azonos színű pont, melyek távolsága 1 egység.
1585. [4] A sík minden pontját kékre vagy pirosra színeztük. Mutassuk meg, hogy van
olyan szabályos háromszög, melynek csúcsai azonos színűek.
1586. [3] Egy 100-oldalú szabályos sokszög csúcsaiból 76 piros, 24 kék. Mutassuk meg,
hogy van olyan négyzet, melynek csúcsai e 100 csúcs közül valók, és a csúcsok
azonos színűek.
1587. [4] Egy szabályos tízszög csúcsai közül 5-öt pirosra festettünk, a többit kékre. Mu-
tassuk meg, hogy mindig van olyan háromszög, amelyiknek minden csúcsa pi-
ros; és amelyikhez található vele egybevágó, csupa kék csúcsú háromszög.
1588. [5] Egy szabályos 13-szög csúcsait két színnel színeztük. Bizonyítsuk be, hogy van
három egyszínű csúcs, melyek egy egyenlő szárú háromszög csúcsait alkotják.
1589. [5] Egy szabályos 13-szög csúcsait két színnel színeztük. Mutassuk meg, hogy van
két egybevágó háromszög, melynek csúcsai a 13 pontból valók, és az egymás-
nak megfelelő csúcsok azonos színűek.
1590. [5] A négyzetrács rácspontjait 6 színnel színeztük. Bizonyítsuk be, hogy van olyan
téglalap, melynek csúcsai azonos színűek, s oldalai rácsegyenesekre illeszked-
nek.
138 Feladatok

1591. [4] Négyzetrácson kijelöltünk öt rácspontot és azokat páronként összekötöttük. Mu-


tassuk meg, hogy e szakaszok valamelyike a végpontokon kívül is tartalmaz
további rácspontot.
1592. [4] Egy konvex poliédernek legalább 9 csúcsa van, a csúcsok koordinátái egész
számok. Mutassuk meg, hogy található a poliéderben olyan, a csúcsoktól kü-
lönböző pont, amelynek koordinátái szintén egész számok.
1593. [4] Mutassuk meg, hogy ha a síkbeli négyzetrácson adott 9 rácspont úgy, hogy
semelyik három nem esik egy egyenesre, akkor az általuk meghatározott há-
romszögek között van olyan, amelyiknek a súlypontja is rácspont.
1594. [4] A sík minden pontját pirosra vagy kékre színeztük. Bizonyítsuk be, hogy létezik
olyan háromszög, amelynek csúcsai és súlypontja azonos színűek.
1595. [3] Mutassuk meg, hogy öt, 10-nél nagyobb prímszám közül mindig kiválasztható
kettő, melyek különbsége osztható 10-zel.
1596. [3] Bizonyítsuk be, hogy hét négyzetszám között mindig van kettő, melyek különb-
sége osztható 10-zel.
1597. [3] Mutassuk meg, hogy három négyzetszám között mindig van kettő, melyek kü-
lönbsége osztható 3-mal.
1598. [3] Igazoljuk, hogy három négyzetszám között mindig van kettő, melyek különb-
sége osztható 4-gyel.
1599. [3] Mutassuk meg, hogy hét egész szám között mindig van kettő, melyek különb-
sége osztható 6-tal.
1600. [3] Mutassuk meg, hogy hét egész szám között mindig van kettő, melyek összege
vagy különbsége osztható 11-gyel.
1601. [3] Bizonyítsuk be, hogy bármely n természetes számhoz végtelen sok 2-nek pozi-
tív egész kitevős hatványa tartozik úgy, hogy közülük bármely kettő különbsége
osztható n-nel.
1602. [4] Mutassuk meg, hogy 1991-nek van olyan többszöröse, melynek minden szám-
jegye 1.
1603. [4] Mutassuk meg, hogy n egész szám között mindig van néhány szám, melyek
összege osztható n-nel.
1604. [3] Igazoljuk, hogy 3 egész szám között mindig van kettő, melyek összege osztható
2-vel.
1605. [4] Mutassuk meg, hogy 5 egész szám között mindig van három, melyek összege
osztható 3-mal.
1606. [5] Mutassuk meg, hogy 7 egész szám között mindig van négy, melyek összege
osztható 4-gyel.
27. Skatulyaelv 139

1607. [6] Adottak az a1 , a2 , . . . , a37 , b1 , b2 , . . . , b37 , c1 , c2 , . . . , c37 egész számok.


Mutassuk meg, hogy van olyan k, l, m index, melyekre 3 | ak + al + am ,
3 | bk + bl + bm , 3 | ck + cl + cm .
1608. [4] Legfeljebb hány különböző prímszámot lehet megadni úgy, hogy közülük bár-
mely három összege prímszám legyen?
1609. [3] Mutassuk meg, hogy a 2 pozitív egész kitevőjű hatványainak utolsó két jegyéből
álló sorozat periodikus.
1610. [4] Bizonyítsuk be, hogy a Fibonacci-sorozat elemeinek utolsó három jegyéből álló
sorozat periodikus.
1611. [4] Legyen a0 pozitív egész, s an az an−1 szám jegyei köbének összege, ahol n =
= 1, 2, . . . . Mutassuk meg, hogy az így kapott sorozat periodikus.
1612. [5] Igazoljuk, hogy a 3-nak van olyan pozitív egész kitevőjű hatványa mely a 0001
számjegyekre végződik.
1613. [6] Mutassuk meg, hogy bármely n páratlan számhoz van olyan k egész szám, hogy
n | 2k − 1.
1614. [6] Mutassuk meg, hogy ha (m, n) = 1, akkor van olyan k, (m, n, k ∈ N ) hogy
n | mk − 1.
1615. [3] Kiválasztunk az 1, 2, 3, . . . , 100 számokból 27 számot. Mutassuk meg, hogy
ezek közt van kettő, melyek nem relatív prímek.
1616. [4] Legyen p = 3 prímszám n-edik hatványa 20-jegyű szám! Mutassuk meg, hogy
ebben a számban van olyan jegy, mely legalább háromszor fordul elő.
1617. [7] Mutassuk meg, hogy ha a és b relatív prím egészek, akkor a 2 + b2 minden
pozitív osztója előáll két négyzetszám összegeként.
1618. [6] Mutassuk meg, hogy ha A
√és n relatív
√ prímek, akkor vannak olyan x és y egész
számok, melyekre |x| ≤ n, |y| ≤ n és n | Ax − y.
1619. [4] Mutassuk meg, hogy a sík 50 egyenese közül kiválasztható 8 úgy, hogy azok
vagy páronként metszik egymást, vagy páronként nem metszik egymást (pár-
huzamosak).
1620. [6] Igazoljuk, hogy az egyenesen kijelölt n2 +1 intervallum közül kiválasztható n+1
intervallum úgy, hogy azok között vagy egyik sem tartalmaz valamely másikat,
vagy bármely két intervallum egyike tartalmazza a másikat.
1621. [4] Mutassuk meg, hogy a π, 2π, 3π, . . . , 11π számok közt van kettő, melyek
végtelen sok tizedesjegyben megegyeznek egymással.
1622. [2] Mutassuk meg, hogy π tizedesjegyei közt van három egymást követő számjegy,
melyek így együtt végtelen sokszor fordulnak elő.
140 Feladatok

1623. [4] Mutassuk meg, hogy egy konvex poliédernek mindig van két azonos oldalszámú
lapja.
1624. [4] Mutassuk meg, hogy véges, egyszerű gráfban van két azonos fokszámú csúcs.
1625. [4] Bizonyítsuk be, hogy egy 6 tagú társaságnak mindig van vagy három olyan
tagja, akik kölcsönösen ismerik egymást, vagy három olyan, akik kölcsönösen
nem ismerik egymást.
1626. [5] Mutassuk meg, hogy 6 irracionális szám között mindig van három olyan, hogy
közülük bármely kettő összege irracionális.
1627. [5] Mutassuk meg, hogy bármely 5 irracionális szám között mindig van három
olyan, hogy közülük bármely kettő összege irracionális.
1628. [5] 17 tudós mindegyike levelezik a többivel angol, német vagy francia nyelven.
Mutassuk meg, hogy van közöttük három, akik egymás közt ugyanazt a nyelvet
használják.
1629. [5] 66 színész közül bármely kettő játszott egymással vagy közös színházi előadá-
son, vagy szinkronban, vagy filmen, vagy tv-játékban, de mindenki mindenkivel
pontosan csak egyféle produkcióban. Bizonyítsuk be, hogy van köztük három
olyan színész, akik ugyanabban a produkcióban vettek részt.
1630. [5] Egy sakkmester 11 hetes túrára készül. A túrán mindegyik nap lejátszik legalább
egy partit, de 12-nél többet nem játszik le egy hét alatt. Mutassuk meg, hogy
lesznek olyan, egymást követő napok, melyekben összesen 21 partit játszik le.
1631. [5] Egy sorba leírtunk 1000 természetes számot. Mindegyik természetes szám alá
írjuk le, hogy hányszor fordult elő a sorban. Az így kapott sor alá ugyanilyen
módon készítsünk újabb sort úgy, hogy csak az előző sort vegyük figyelembe.
Igaz-e, hogy ezt az eljárást ismételve egy idő után csupa egyforma sort fogunk
kapni?
1632. [4] Egy 2n × 2n-es sakktáblára 3n bábut tettünk fel. Mutassuk meg, hogy el lehet
hagyni n sort és n oszlopot úgy, hogy a megmaradó n×n-es sakktáblán egyetlen
bábu se maradjon.
1633. [4] Egy 5 × 5-ös táblázat mezőibe 1 és −1 számokat írunk tetszőlegesen, és ki-
számítjuk soronként is, oszloponként is az ott álló számok összegét. Mutassuk
meg, hogy a kapott összegek között lesz két azonos.
1634. [4] Egy n × n-es táblázat mezőibe tetszőlegesen beírjuk az 1, 2, 3 számokat, és ki-
számítjuk soronként, oszloponként és a két átlóban az ott levő számok összegét.
Igazoljuk, hogy a kapott összegek között lesz két azonos.
1635. [5] Egy 100×100-as táblázat mezőibe egészeket írunk úgy, hogy két oldalszomszé-
dos mezőben álló szám különbségének abszolút értéke legfeljebb 20. Mutassuk
meg, hogy legalább három mezőben ugyanaz a szám áll.
28. Matematikai játékok 141

28. Matematikai játékok

1636. [3] Egy téglalap alakú asztalra két játékos felváltva helyez 1 Ft-os érméket (az
érmék nem fedhetik egymást). Az a győztes, aki utoljára tud tenni. A kezdő
nyerhet. Hogyan?
1637. [3] Egy szabályos 2n oldalú sokszögben ketten felváltva átlókat húznak be úgy,
hogy azok a sokszög belsejében nem metszhetik egymást. Az veszít, aki nem
tud lépni. A kezdő nyerhet. Hogyan? Mi a válasz, ha az oldalak száma 2n + 1?
1638. [4] Egy szabályos 2n oldalú sokszög csúcsait felváltva pirosra és kékre színeztük.
Két játékos felváltva átlókat rajzol a sokszögbe úgy, hogy azok azonos színű
csúcsokat kötnek össze és a sokszög belsejében nem metszhetik egymást. Az a
vesztes, aki nem tud lépni. Melyik játékosnak van nyerő stratégiája?
1639. [4] Egy szabályos n oldalú sokszög csúcsait valamilyen körüljárást követve meg-
számoztuk az 1, 2, 3, . . . , n számokkal. Két játékos felváltva átlókat rajzol a
sokszögbe úgy, hogy azok azonos párosságú csúcsokat kötnek össze és a sok-
szög belsejében nem metszhetik egymást. Az a vesztes, aki nem tud lépni. Me-
lyik játékosnak van nyerő stratégiája?
1640. [3] Sorban egymás után felállítottunk n db kuglibábut. Két játékos felváltva felborít
egy bábut vagy két szomszédos bábut. Az veszít, aki nem tud lépni. A kezdő
nyerhet. Hogyan?
1641. [3] Körben felállítottunk n db kuglibábut. Két játékos felváltva felborít egy bábut
vagy két szomszédos bábut. Az veszít, aki nem tud lépni. Melyik játékosnak
van nyerő stratégiája?
1642. [3] Két játékos az előttük levő 2 piros, 2 fehér, 2 fekete, 2 sárga korongból felváltva
helyez egy-egy korongot egy kocka csúcsaira. A kezdő játékos akkor nyer, ha
a kockának lesz olyan éle, melynek két végén azonos színű korongok vannak;
ha nincs ilyen él, akkor a második játékos nyer. Kinek van nyerő stratégiája?
1643. [3] Két játékos felváltva színezi egy kocka 3–3 élét pirossal, illetve feketével. Az a
győztes, aki a kocka valamely lapjának mind a négy élét saját színével színezte
ki. Melyik játékosnak van nyerő stratégiája?
1644. [3] 60 és 70 kavics van egy-egy kupacban. Két játékos felváltva vesz valamelyik
kupacból legalább egyet, de abból az összeset is elveheti. Az győz, aki utolsó-
ként vesz. Melyik játékosnak van nyerő stratégiája?
1645. [4] Három kupacban kavicsok vannak: 1, 65 és 117 darab. A játék ugyanaz, mint
az előző feladatban. Melyik játékosnak van nyerő stratégiája?
142 Feladatok

1646. [4] Az asztalon két kupacban kavicsok vannak. Ketten felváltva vesznek a kavi-
csokból: vagy valamelyik kupacból 1 kavicsot, vagy mindkét kupacból 1–1 ka-
vicsot. Az nyeri a játékot, aki az utolsó kavicsot veszi fel. Melyik játékosnak
van nyerő stratégiája?
1647. [4] Két játékos felváltva helyez egy-egy huszárt (lovat) a sakktáblára. Csak olyan
mezőre lehet tenni a bábut, amely nem áll ütés alatt. Az a játékos veszít, aki
nem tud lépni. Melyik játékosnak van nyerő stratégiája?
1648. [4] Két játékos felváltva helyez egy-egy királyt a 9 × 9-es sakktáblára. Csak olyan
mezőre lehet tenni a bábut, amely nem áll ütés alatt. Az a játékos veszít, aki
nem tud lépni. Melyik játékosnak van nyerő stratégiája?
1649. [4] Két játékos felváltva helyez egy-egy 1 × 2-es dominót a 10 × 10-es sakktáblára.
A dominókat úgy kell letenni, hogy két szomszédos üres mezőt fedjenek le. Az
a játékos veszít, aki nem tud lépni. Melyik játékosnak van nyerő stratégiája?
1650. [4] Két játékos felváltva festi feketére egy egységkockákból épített téglatest va-
lamely, egységkockákból álló oszlopát. (Az oszlop lehet függőleges vagy víz-
szintes helyzetű is.) Csak olyan oszlop választható, melynek még nincs befestett
kockája. Az a játékos veszít, aki nem tud lépni. Melyik játékosnak van nyerő
stratégiája, ha a téglatest méretei:
a) 4 × 4 × 4;
b) 4 × 4 × 3;
c) 4 × 3 × 3?
1651. [4] Két játékos felváltva ír egy-egy számot egy 9×9-es táblázatba. A kezdő játékos
0-t, a második 1-est ír. A játék végén megszámolják, hogy hány sorban és
oszlopban van többségben a 0 szám, ill. az 1-es. Az a játékos nyer, aki több
sorban és oszlopban szerez többséget. Melyik játékosnak van nyerő stratégiája?
1652. [3] Négyzet alakban leraktunk az asztalra
a) 5 × 5, illetve
b) 6 × 6 kavicsot.
Két játékos felváltva vesz belőlük egy darabot vagy két „oldalszomszédosat”.
Az győz, aki utolsóként vesz. Melyik játékosnak van nyerő stratégiája?
1653. [4] Két kupacban cukorkák vannak, mindegyikben 9 darab. Két játékos felváltva a
következőt teszi: valamelyik kupacból 1 cukorkát áttesz a másikba és az egyik
kupacból kivesz magának két cukorkát. Az a játékos veszít, aki nem tud lépni.
Melyik játékosnak van nyerő stratégiája?
1654. [5] Két kupacban kavicsok vannak, mindegyikben 9–9 darab. Ketten felváltva vesz-
nek a kavicsokból: vagy az egyik kupacból 1 vagy 2 kavicsot, vagy mindkét ku-
pacból 1–1 kavicsot. Az nyeri a játékot, aki az utolsó kavicsot veszi fel. Melyik
játékosnak van nyerő stratégiája?
28. Matematikai játékok 143

1655. [4] Ketten felváltva futókat helyeznek a 8 × 8-as sakktáblára. Az a játékos veszít,
aki nem tud úgy futót helyezni a táblára, hogy az ne álljon ütés alatt. Melyik
játékosnak van nyerő stratégiája?
1656. [4] A 10 × 10-es tábla mezőit ketten felváltva színezik. Egy lépésben a játékos
kiszínezheti a tábla egy 1 × 1-es, egy 1 × 2-es vagy egy 1 × 3-as részét, ha
annak egyik mezője sincs még kiszínezve. Az a játékos nyer, aki a tábla utolsó
mezőjét színezi ki. Melyik játékosnak van nyerő stratégiája?
1657. [3] Egy 8 × 8-as sakktábla bal alsó sarkából indul egy bábu. Két játékos felváltva
lépteti vagy vízszintesen jobbra néhány mezőt, vagy felfelé valamennyi mezőt.
Az a játékos nyer, aki a jobb felső sarokba lép. Melyik játékosnak van nyerő
stratégiája?
1658. [4] (Mérgezett csoki) Van egy n × k-s csokitáblánk, melynek bal felső mezője mér-
gezett. Két játékos felváltva tör a táblából úgy, hogy valamelyik mezőt kivá-
lasztja, s azt meg az egész tőle jobbra és lefelé eső szeletet letöri. Az veszít, aki
kénytelen elvenni a mérgezett mezőt. Mutassuk meg, hogy a kezdő megnyerheti
a játékot, ha a tábla
a) n × n-es, illetve
b) 2 × n-es.
1659. [4] Egy n × m-es csokoládétáblából ketten felváltva „rácsegyenesek” mentén tör-
delnek le darabokat. Az nyer, aki először tud 1 × 1-es darabot letörni. Kinek
van nyerő stratégiája?
1660. [4] Egy 10×5-ös csokoládétáblát, ill. annak a tördelések során kapott részeit ketten
felváltva tördelik „rácsegyenesek” mentén. Az nyer, aki először tud 1 × 1-es
darabot letörni. Kinek van nyerő stratégiája?
1661. [4] Egy 10×5-ös csokoládétáblát, ill. annak a tördelések során kapott részeit ketten
felváltva tördelik „rácsegyenesek” mentén. Az veszít, aki először kénytelen 1 ×
× 1-es darabot letörni. Kinek van nyerő stratégiája?
1662. [5] Adott a síkon
a) 5, illetve
b) 6 pont, melyek közül semelyik három sincs egy egyenesen.
Ketten felváltva kötnek össze két, még össze nem kötött pontot szakasszal, és
saját színükkel színezik azt. Az a játékos veszít, akinek hamarabb lesz saját
háromszöge, azaz három olyan pont, melyek közti három szakasz mindegyikét
ő színezte. Melyik játékosnak van nyerő stratégiája?
1663. [4] Adott a síkon 6 pont, melyek közül semelyik 3 nincs egy egyenesen. Ketten
felváltva kötnek össze két, még össze nem kötött pontot szakasszal. Az a játékos
veszít, aki olyan háromszöget hoz létre, melynek minden csúcsa az adott 6 pont
közül való. Igazoljuk, hogy a kezdő megnyerheti a játékot, ha ügyesen játszik.
144 Feladatok

1664. [4] Két játékos a négyzetrács rácspontjaiból felváltva jelöl meg egy-egy pontot. Ar-
ra kell törekedniük, hogy a megjelölt pontok együtt egy konvex sokszög csúcsai
legyenek. Aki már nem tud úgy választani rácspontot, hogy ez teljesüljön, el-
veszti a játékot. Melyik játékosnak van nyerő stratégiája?
1665. [3] Két játékos felváltva kiválasztja a táblán levő szám valamely, nullától külön-
böző számjegyét, s azt levonja a számból. A régi számot letörli, s felírja a kü-
lönbséget. Az a játékos nyer, aki különbségként nullát kap. Melyik játékosnak
van nyerő stratégiája, ha kezdetben 1991 volt a táblán?
1666. [3] Az asztalon 27 db gyufaszál van, s ketten felváltva vesznek 1, 2 vagy 3 szálat.
Az a játékos nyer, aki utolsóként vesz. Melyik játékosnak van nyerő stratégiája?
1667. [4] Az asztalon 25 db gyufaszál van, s ketten felváltva vesznek 1, 2 vagy 3 szálat,
egészen addig, míg a gyufák el nem fogynak. Az a játékos nyer, aki utoljára
vesz el egyszerre két szál gyufát. Melyik játékosnak van nyerő stratégiája?
1668. [3] Az asztalon 27 db gyufaszál van, s ketten felváltva vesznek 2 vagy 3 szálat,
utolsó lépésként 1 is elvehető. Az a játékos nyer, aki utolsóként vesz. Melyik
játékosnak van nyerő stratégiája?
1669. [3] Az asztalon 40 db gyufaszál van, s ketten felváltva vesznek 2, 3, 4 vagy 5 szálat.
Az a játékos nyer, aki utolsóként vesz. Melyik játékosnak van nyerő stratégiája?
1670. [3] Az asztalon 40 db gyufaszál van, s ketten felváltva vesznek 2, 3, 4 vagy 5
szálat. Az a játékos veszít, aki utolsóként vesz. Melyik játékosnak van nyerő
stratégiája?
1671. [4] Az asztalon 1991 db gyufaszál van, s ketten felváltva vesznek pn szálat, ahol
p prím, n nemnegatív egész (pl. 8, 25, 7, 1 stb. szálat). Az nyer, aki utolsóként
vesz. A kezdő nyerhet. Hogyan?
1672. [3] Egy kupacban 101 db kavics van. Ketten felváltva vesznek a kavicsokból 1, 2,
3, 4 vagy 5 darabot. A játék véget ér, amikor a kupacból elfogynak a kavicsok;
ekkor a kezdő játékos a db, a második b db kavicsot gyűjtött magának. A kezdő
játékos akkor nyer, ha a és b relatív prímek, különben a második játékos a
győztes. Melyik játékosnak van nyerő stratégiája?
1673. [4] Ketten felváltva vesznek az asztalon levő n db gyufaszálból 2k db-ot (1, 2,
4, . . . ). Az a játékos nyer, aki utolsóként vesz. Melyik játékosnak van nyerő
stratégiája?
1674. [4] Az asztalon n db gyufaszál van, s ketten felváltva vesznek. k db-ot akkor ve-
hetnek el, ha k relatív prím a kupacban levő gyufaszálak számához. Az nyer,
aki utolsóként vesz. Kinek van nyerő stratégiája?
1675. [5] Ketten felváltva húznak páratlan sok golyóból tetszés szerinti számú golyót
úgy, hogy minden húzáskor legfeljebb az eredeti golyószám felét veszik el.
A játékot az nyeri, akinek a golyók elfogytával páros sok golyója van. Van-e
nyerő stratégiája valamelyik játékosnak?
28. Matematikai játékok 145

1676. [6] Két játékos felváltva vesz n db kavicsból. Az első nem veheti el kezdéskor az
összeset. A soron következő játékos legalább egyet, legfeljebb az előző által
elvettek számának kétszeresét veheti el. Az nyer, aki az utolsó kavicsot veszi
el. Melyik játékosnak van nyerő stratégiája?
1677. [5] Ketten felváltva húznak páratlan sok golyóból 1, 2 vagy 3 golyót. A játékot
az nyeri, akinek a golyók elfogytával páros sok golyója van. Kinek van nyerő
stratégiája?
1678. [3] Két játékos felváltva osztja az előttük levő kupacok valamelyikét két részre. Az
győz, aki utolsónak lép. Melyik játékosnak van nyerő stratégiája, ha kezdéskor
az asztalon egy kupac volt 1991 db kaviccsal?
1679. [5] Két játékos felváltva osztja az előttük levő kupacok mindegyikét — melyben
legalább két kavics van — két részre. Kezdetben az asztalon
a) egy kupac volt 31 kaviccsal.
b) egy kupac volt 100 kaviccsal. Az győz, aki utolsónak lép. Kinek van nyerő
stratégiája?
1680. [4] Két játékos játszik. Az első mond egy egész számot, mely 1-nél nagyobb és
10-nél kisebb. A második játékos megszorozza ezt a számot egy, az előbbi
feltételeknek eleget tevő számmal. Ezt a szorzatot most az előző szorozza egy,
a megadott számkörből való számmal és így tovább. A játékot az nyeri, aki
szorzatával először lépi túl az 1000-et. Melyik játékosnak van nyerő stratégiája?
1681. [4] Ketten felváltva mondanak számokat. A kezdő elsőként 1-et mond. A soron
következő játékos az előzőleg elhangzott számnál legalább 1-gyel nagyobbat
mond, de nagyobb számot nem mondhat, mint az előzőleg elhangzott szám és
e szám jegyeinek összege. Az nyer, aki kimondja a 100-at. Kinek van nyerő
stratégiája?
1682. [4] Egy 8 × 8-as sakktábla bal alsó sarkából indul egy bábu. Két játékos felváltva
tolja egy-egy mezővel jobbra, felfelé vagy jobbra átlósan felfelé. Az nyer, aki
a jobb felső mezőre lép. A kezdő játékos nyerhet. Hogyan?
1683. [5] A sötét király az n × m-es sakktábla bal felső sarkán áll. Ketten a következő
játékot játsszák: felváltva lépnek a királlyal, de mindig csak olyan (csúcsban
vagy oldalban szomszédos) mezőre, ahol addig még nem járt. Az veszít, aki
nem tud lépni. Kinek van nyerő stratégiája?
1684. [4] A 8 × 8-as sakktábla bal fölső sarkában egy bábu áll, amely vízszintesen jobbra
léphet legfeljebb 4 mezőt, vagy függőlegesen lefelé legfeljebb 3 mezőt. András
és Balázs felváltva lépnek a bábuval. Kinek van nyerő stratégiája, ha
a) az nyer,
b) az veszít,
aki a tábla jobb alsó mezőjére lép?
146 Feladatok

1685. [4] Egy 8 × 8-as sakktábla bal alsó sarkában áll a királynő. Ezután ketten felváltva
lépnek a királynővel. Lépni a sakk szabályai szerint lehet, és minden lépésnek
közelebb kell vinnie a jobb felső sarokhoz. Az a játékos veszít, aki a jobb felső
sarokba kénytelen tolni a bábut. Melyik játékosnak van nyerő stratégiája?
1686. [5] Mutassuk meg, hogy véges sok lépésben győzelemmel véget érő kétszemélyes
játékban mindig van nyerő stratégiája valamelyik játékosnak.
1687. [4] Egy táblára m darab 1-est és n darab 2-est írtunk fel. Két játékos felváltva
letöröl a tábláról két számot és felír helyettük egy számot: ha két azonos számot
töröl le, akkor egy 2-est; ha különbözőket, akkor egy 1-est ír helyettük a táblára.
Néhány ilyen lépés után a táblán csak egy szám marad. Ha ez a szám 1-es, akkor
a kezdő játékos; ha pedig 2-es, akkor a második játékos nyer. Melyik játékosnak
van nyerő stratégiája?
1688. [5] Két játékos felváltva ír a táblára egy adott n számnál nem nagyobb pozitív
egészeket. Az veszít, aki olyan számot ír, mely egy már korábban felírt számnak
osztója. Mutassuk meg, hogy a kezdő játékosnak mindig van nyerő stratégiája.
Mi ez a stratégia, ha n = 10?
1689. [5] Két játékos felváltva írja a táblára egy adott n szám osztóit. Az veszít, aki olyan
számot ír, mely egy már korábban felírt számnak osztója. Mutassuk meg, hogy
a kezdő játékosnak mindig van nyerő stratégiája. Mi ez a stratégia, ha n = 72?
1690. [4] A táblára két számot írtunk, a 25 és 36 számokat. Ketten felváltva írnak fel egy-
egy újabb számot. Olyan számot lehet felírni, amely a táblán levő valamely két
szám különbsége, és még nem szerepel a táblán. Az a játékos veszít, aki nem
tud lépni. Melyik játékosnak van nyerő stratégiája?
1691. [3] A táblára ez van felírva: . . . x 3 + . . . x 2 + . . . x + · · · = 0. Ketten felváltva ír-
ják a kipontozott helyekre az együtthatókat: valamelyik még üres helyre egy
szabadon választott valós számot.
A második játékos célja, hogy a kapott egyenletnek legyen egész gyöke. Elér-
heti-e biztosan ezt a második játékos?
1692. [4] A táblára ez van felírva: x 3 + . . . x 2 + . . . x + · · · = 0. Ketten felváltva írják a ki-
pontozott helyekre az együtthatókat: valamelyik még üres helyre egy szabadon
választott egész számot.
Az első játékos célja, hogy a kapott egyenletnek mindhárom gyöke egész szám
legyen. Elérheti-e biztosan ezt az első játékos?
1693. [4] A táblára ez van felírva: x 3 + . . . x 2 + . . . x + · · · = 0. Ketten felváltva írják a ki-
pontozott helyekre az együtthatókat: valamelyik még üres helyre egy szabadon
választott valós számot.
Az első játékos célja, hogy a kapott egyenletnek pontosan egy valós gyöke
legyen. Elérheti-e biztosan ezt az első játékos?
28. Matematikai játékok 147

1694. [3] A táblára ez van felírva: . . . x 2 + . . . x + · · · = 0. Az első játékos választ három


valós számot, a második játékos pedig beírja ezeket (általa eldöntött sorrendben)
a kipontozott helyekre együtthatóknak.
Az első játékos célja, hogy a kapott egyenletnek két különböző racionális gyöke
legyen. Elérheti-e biztosan ezt az első játékos?
1695. [3] Két játékos felváltva ír az egyenletrendszerben a csillagok helyére szabadon vá-
lasztott számokat együtthatónak. Mutassuk meg, hogy a kezdő játékos elérheti
azt, hogy az egyenletrendszernek legyen nullától
⎫ különböző gyöke.
x + y + z = 0 ⎪

x + y + z = 0


x + y + z = 0
1696. [5] Bizonyítsuk be, hogy a mérgezett csoki játékban mindig a kezdőnek van nyerő
stratégiája. (Lásd az 1658. feladatot.)
1697. [5] A dupla sakk ugyanaz a játék, mint a hagyományos sakk, csak egy-egy alka-
lommal két lépést léphet a játékos. Mutassuk meg, hogy a kezdőnek van nem
vesztő stratégiája.
1698. [5] Mutassuk meg, hogy az amőba játékban a kezdőnek van nem vesztő stratégi-
ája. (Az amőba játékot ketten játszák egy négyzetrácsos papíron. A játékosok
felváltva írják egy általuk választott üres négyzetbe a jelüket: egyikük O-t, má-
sikuk X-et. Az a játékos nyer, akinek hamarabb lesz jeleiből öt egymás mellett
vízszintesen, függőlegesen vagy átlós irányban.)
1699. [5] 5 × 5-ös táblázat mezőibe a kezdő játékos mindig csak 1 jelet, a második pedig
2 jelet tesz. Az nyer, akinek először van öt saját jele vízszintesen vagy függő-
legesen. A második nyerhet. Hogyan?
1700. [5] Egy 10 × 10-es táblázat mezőibe két játékos felváltva írhat egy keresztet vagy
egy karikát, minden alkalommal amelyiket jónak látja. Az nyer, aki eléri, hogy
sorban egymás mellett három azonos jel álljon. Kinek van nyerő stratégiája?
1701. [5] Egy 11 × 11-es sakktábla középső mezőjében áll egy bábu. A bábut két játé-
kos felváltva eltolhatja egy másik mezőre, ahol még nem állt eddig a bábu; s
mindig nagyobb távolságra kell tolni a bábut, mint előző alkalommal (két mező
közti távolságon a köztük levő vízszintes és függőleges szakaszokból álló leg-
rövidebb útvonal hosszát értjük). Az veszít, aki nem tud lépni. Kinek van nyerő
stratégiája?
1702. [5] n × n-es sakktábla bal alsó sarkában áll egy bábu, melyet ketten felváltva lép-
tetnek. Lépni csak olyan oldalszomszédos mezőre lehet, melyen még nem járt
a bábu. Az veszít, aki nem tud lépni. Bizonyítsuk be, hogy ha n páros, akkor
a kezdő nyerhet; ha n páratlan, akkor a második. Mi a helyzet, ha a bábu a
sarokmezővel oldalszomszédos mezőről indul?
148 Feladatok

1703. [5] Két játékos közül az egyik a sakktábla valamelyik mezőjére egy huszárt helyez.
Ezután a játékosok felváltva lépnek a sakkjáték szabálya szerint; de tilos olyan
mezőre lépni, ahol már korábban is járt a huszár. Az veszít, aki már nem tud
lépni. Kinek van nyerő stratégiája, ha a sakktábla mérete
a) 8 × 8
b) m × n, ahol m ≥ n ≥ 3?
1704. [5] Egy 5 × 5-ös táblán az ábra szerint fehér
és fekete bábuk állnak. A kezdő játékos
fehérrel, a második feketével lép. Lépni
csak oldalszomszédos üres mezőre lehet.
Az nyer, aki utolsóként lép. Kinek van
nyerő stratégiája?

1705. [3] Két játékos felváltva foglal le egész számokat. A kezdő akkor nyer, ha számai
közt lesz három egymás utáni, különben a második győz. Kinek van nyerő
stratégiája?
1706. [3] Két játékos felváltva írja egy n-jegyű szám számjegyeit. A kezdő játékos nyer,
ha a kapott szám osztható 9-cel, különben a második a győztes. Melyik játé-
kosnak van nyerő stratégiája?
1707. [5] Két játékos felváltva írja egy 2n-jegyű szám számjegyeit. E jegyek csak a 6,
7, 8 és 9 lehetnek. A kezdő játékos akkor nyer, ha a kapott szám nem osztható
9-cel, különben a második a győztes. Melyik játékosnak van nyerő stratégiája?
1708. [5] Két játékos felváltva írja egy 2n-jegyű szám számjegyeit. E jegyek csak az 1, 2,
3, 4 és 5 lehetnek. A második nyer, ha a kapott szám osztható 9-cel, különben
a kezdő a győztes. Melyik játékosnak van nyerő stratégiája?
1709. [5] Két játékos a következő játékot játssza. Egy papírra felírják a pozitív egész
számokat 1-től 100-ig egymás mellé, majd a számok közti 99 helyre felváltva
írják az összeadás, a kivonás vagy a szorzás jelét. A játék végén kiszámolják az
így kialakult számítás eredményét, és ha az páros, akkor a kezdő nyer. Kinek
van nyerő stratégiája? Kinek van nyerő stratégiája akkor, ha a kezdő páratlan
végeredmény esetén nyeri meg a játékot?
1710. [4] Egy asztalra sorban 7 korongot helyeztünk el. A korongok egyik oldala fehér,
a másik fekete, és az asztalon levő korongok egy része fehér oldalával, a többi
pedig fekete oldalával van felfelé. Ketten játszanak. A soron következő mindig
kiválaszt egy fehér oldalával felfelé álló korongot; s azt, és az összes tőle jobbra
levőt átfordítja a másik oldalára. Az veszít, aki nem tud lépni, azaz mindegyik
korongnak a fekete oldala van felül.
A kezdőnek vagy a másodiknak van-e nyerő stratégiája, és mi a nyerő stratégia?
29. Különféle kombinatorikai feladatok 149

1711. [5] Egy 2n + 1 fokú lépcső alsó n fokának mindegyikén egy-egy kő van. András
és Béla felváltva rakosgatják a köveket a következő szabályok szerint: And-
rás bármely követ áthelyezheti felfelé az első szabad lépcsőfokra; Béla pedig
bármelyik követ eggyel lejjebb teheti, ha az a lépcsőfok üres. Meg tudja-e Bé-
la akadályozni, hogy András a legfelső fokra rakjon követ, ha András kezdi a
játékot?
1712. [4] Az asztalon fekszik kilenc kártya, rajtuk az 1, 2, . . . , 9 számok, mindegyik
kártyán egy szám van. Ezek közül két játékos felváltva választ magának egy-egy
kártyát. Az a játékos nyer, akinek elsőként lesz három olyan kártyája, melyeken
a számok összege 15.
Melyik játékosnak van nyerő stratégiája?
1713. [5] Az 1, 2, . . . , 101 számokból ketten felváltva vesznek el 9 számot. 11 lépés után
két szám marad. A kezdő nyer, ha különbségük abszolút értéke 55. A kezdő
megnyerheti-e biztosan a játékot?
1714. [5] Az 1, 2, . . . , 27 számokból ketten felváltva vesznek egy-egy számot addig, míg
2 db szám marad. Ha összegük osztható 5-tel, akkor a kezdő nyert. Kinek van
nyerő stratégiája?
1715. [5] A 0, 1, 2, . . . , 1024 számokból ketten felváltva vesznek: az első alkalommal
512 db számot, a második alkalommal 256 db számot, a harmadik alkalommal
128 db számot vesznek, . . . . A tizedik elvétel után 2 db szám marad. A kezdő
nyer, ha ezek különbségének abszolút értéke legalább m. Mely m-nél előnyös
ez a játék a kezdő számára?

29. Különféle kombinatorikai feladatok

1716. [3] Hét ember találkozott, egyesek kezet is fogtak egymással. Lehetséges-e, hogy
mindenki pontosan háromszor fogott kezet?
1717. [3] Egy tíztagú társaságban mindenkitől megkérdezték, hogy hány ismerőse van a
jelenlevők között. A válaszok: 6, 3, 4, 5, 7, 2, 3, 5, 6, 8. Mutassuk meg, hogy
valaki tévedett. (Az ismeretségek mindig kölcsönösek.)
1718. [3] Felvehető-e a síkon 5 szakasz úgy, hogy mindegyik három másikat messen?
1719. [3] Négyzetrácsos papíron ki lehet-e jelölni 25 mezőt úgy, hogy mindegyik páratlan
sok másikkal legyen oldalszomszédos?
1720. [3] Lehet-e konvex testet építeni 9 db háromszöglapból?
1721. [3] Lehet-e egy szabályos hétszög oldalait és átlóit hat színnel színezni úgy, hogy
minden csúcsból induljon mindenféle színű él?
150 Feladatok

1722. [3] Miért nem lehet beírni egy szabályos 45-szög csúcsaiba a 0, 1, 2, . . . , 9 szám-
jegyeket úgy, hogy bármely, különböző számjegyekből álló pár előforduljon
valamely oldal két végén?
1723. [3] Lehet-e egész számokat írni a körökbe
úgy, hogy az öt vonal mindegyikén a
négy szám összege páratlan legyen?
1724. [3] Helyezzük el az ábra kis köreibe az 1,
2, 3, 4, 5, 6, 8, 9, 10, 12 számokat úgy,
hogy az egy egyenesen elhelyezkedő
4-4 szám összege ugyanannyi legyen.
1725. [5] Lehet-e az ábra kis köreibe beírni az
1, 2, 3, 4, 5, 6, 7, 8, 9, 10 számokat
úgy, hogy az egy egyenesen elhelyez-
kedő 4-4 szám összege ugyanannyi le-
gyen?
1726. [3] Elhelyezhetők-e a szabályos nyolcszög csúcsaiba az 1, 2, . . . , 8 számok úgy,
hogy bármely három szomszédos csúcsban levő szám összege 13-nál nagyobb
legyen?
1727. [3] Felírhatjuk-e egy kocka éleire az 1, 2, . . . , 12 számokat úgy, hogy az egy-egy
csúcsba befutó három élen levő szám összege ugyanannyi legyen?
1728. [3] Lehet-e különböző számokat írni egy tetraéder 4 lapjára úgy, hogy ha kiszá-
moljuk mindegyik csúcsnál a csúcsra illeszkedő lapokon álló számok összegét,
akkor mindig ugyanazt az eredményt kapjuk?
1729. [3] Lehet-e különböző számokat írni egy tetraéder 6 élére úgy, hogy ha kiszámoljuk
mindegyik csúcsnál a csúcsra illeszkedő éleken álló számok összegét, akkor
mindig ugyanazt az eredményt kapjuk?
1730. [3] Egy kör alakú asztalnál 77-en ülnek; mindenki gondol egy egész számra, majd
mindenki felírja egy cédulára két szomszédja számának összegét. Bizonyítsuk
be, hogy nem állhat minden cédulán 1991.
1731. [3] Mutassuk meg, hogy ha az 1, 2, 3, . . . , 9 számokat három csoportba osztjuk,
akkor valamelyikben a számok szorzata legalább 72.
1732. [3] Mutassuk meg, hogy az 1, 2, 3, . . . , 30 számokat nem lehet tíz csoportba
osztani úgy, hogy minden csoportban három szám legyen, s a három szám közül
a legnagyobb egyenlő legyen a másik kettő összegével.
1733. [3] Szét lehet-e osztani az 1, 2, 3, . . . , 21 számokat néhány csoportba úgy, hogy
minden csoportban a legnagyobb szám egyenlő legyen a többi összegével?
29. Különféle kombinatorikai feladatok 151

1734. [3] Van-e öt olyan egész szám, melyekből képezve az összes kéttagú összeget,
eredményül tíz egymást követő számot kapunk?
1735. [4] 11 érme közül 5 hamis. A hamis érme 1 grammal különbözik a valóditól. Egy
érmét kiválasztva, arról egy kétkarú mérleg és mérősúlyok segítségével egy
méréssel döntsük el, hogy az érme valódi-e vagy sem.
1736. [4] Egy 25 × 25-ös táblázat mezőibe a +1 és −1 számokat írjuk. Kiszámoljuk
minden sorban és minden oszlopban a számok
a) összegét, illetve
b) szorzatát.
Lehet-e az így kapott 50 szám összege nulla?
1737. [4] Adottak az x1 , x2 , . . . , xn számok, melyek értékei +1 vagy −1, továbbá x1 x2 +
+ x2 x3 + · · · + xn−1 xn + xn x1 = 0. Mutassuk meg, hogy 4 | n.
1738. [4] Egy 101 csúcsú poliéder éleit megjelöltük a +1 és −1 számokkal. Mutassuk
meg, hogy van a poliédernek olyan csúcsa, amelybe befutó élekre írt számok
szorzata +1.
1739. [4] Igaz-e, hogy bármiképpen is írjunk egy 100 csúcsú konvex poliéder csúcsaira
+1 és −1 számokat, lesz a testnek olyan csúcsa, amelybe befutó élek másik
végén álló számok szorzata +1?
1740. [4] Bizonyítsuk be, hogy egy 100 csúcsú konvex poliéder élei megszámozhatók a
+1 és −1 számokkal úgy, hogy minden egyes csúcsba befutó élekre írt számok
szorzata −1.
1741. [4] Egy kocka csúcsaiba a +1 és −1 számokat írtuk, majd minden lapra a négy
csúcsban levő szám szorzatát. Lehet-e nulla a csúcsokon és a lapokon levő
számok összege?
1742. [4] Le lehet-e fedni egy
a) 4 × 100-as, illetve egy
b) 5 × n-es táblát +1 −1 dominókkal úgy, hogy minden sorban és minden
oszlopban +1 legyen az ott álló számok szorzata?
1743. [4] Egy kör kerületére 50 számot írtak fel, ezek mindegyike +1 vagy −1. Meg akar-
juk tudni a számok szorzatát. Legkevesebb hány kérdésre van szükség ehhez,
ha egyszerre három szomszédos szám szorzatát tudakolhatjuk meg?
1744. [3] Fel lehet-e írni egy kör kerületére 10 különböző számot úgy, hogy mindegyik
szám két szomszédjának számtani közepe legyen?
1745. [3] Egy végtelen négyzetrács mezőibe írjunk egészeket úgy, hogy mindegyik szám
négy oldalszomszédjának számtani közepe legyen. Mutassuk meg, hogy ez csak
úgy lehet, ha ugyanazt a számot írtuk mindegyik mezőbe.
152 Feladatok

1746. [4] Fel lehet-e írni egy kör kerületére az 1, 2, 3, . . . , 10 számokat úgy, hogy
bármely két szomszédos szám különbsége 3, 4 vagy 5 legyen?
1747. [4] Fel lehet-e írni egy kör kerületére az 1, 2, 3, . . . , 13 számokat úgy, hogy
bármely két szomszédos szám különbsége 3, 4 vagy 5 legyen?
1748. [4] Lehet-e egymás után írni 25 számot úgy, hogy bármely három egymás után
következő szám összege pozitív legyen, de a 25 szám összege negatív? Fel
lehet-e írni 25 számot egy kör kerületére úgy, hogy bármely három egymás
után következő szám összege pozitív legyen, de a 25 szám összege negatív?
1749. [4] Lehet-e egymás után írni 50 számot úgy, hogy bármely 17 egymás után követ-
kező szám összege pozitív legyen, és bármely 10 egymás után következő szám
összege negatív?
1750. [6] Adjunk meg minél több egész számból álló sorozatot úgy, hogy a sorozat bár-
mely hét, egymás mellett levő számának összege negatív legyen; és a sorozat
bármely öt, egymás mellett levő számának összege pozitív legyen. Legfeljebb
hány eleme lehet ennek a sorozatnak?
1751. [4] Mutassuk meg, hogy nem lehet pozitív az a11 a22 a33 , a12 a23 a31 , a13 a21 a32 ,
−a13 a22 a31 , −a12 a21 a33 , −a11 a23 a32 számok mindegyike.
1752. [4] Bizonyítsuk be, hogy egy 13 oldalú szabályos sokszög csúcsaiból nem tudunk
kiválasztani ötöt úgy, hogy közülük bármely két csúcs távolsága különböző
legyen.
1753. [4] Mutassuk meg, hogy az 1, 2, 3, . . . , 20 számok közül nem lehet kiválaszta-
ni tizet úgy, hogy a belőlük képzett kéttagú összegek értékei különbözzenek
egymástól.
1754. [4] 100 fős katonai alakulatnál a napi ügyeletet mindig hárman látják el. Meg lehet-
e szervezni az ügyeletet egymást követő napokon át úgy, hogy bármely két fő
csak egyszer ügyeljen együtt?
1755. [4] Egy országban a repülőjáratok néhány város között úgy vannak szervezve, hogy
bármely városból legfeljebb 3 másikba indul járat, és bárhonnan bármely másik
városba el lehet jutni legfeljebb egy átszállással. Legfeljebb hány városban van
repülőtér?
1756. [4] Rajzoljunk egy négyzetrácsos papírra olyan (konvex vagy konkáv) sokszöget,
amelynek oldalai rácsegyenesek. Mutassuk meg, hogy egy ilyen sokszög olda-
lainak száma páros.
1757. [3] Egy kocka éle 3 cm. Hat síkkal a kockát fel lehet vágni 27 kis kockára, amelyek
mindegyikének 1 cm az élhossza: két-két, a kocka lapjaival párhuzamos sík
kell. Azonban megtehetjük, hogy minden egyes vágás után a kapott részeket
elmozgatjuk. Ily módon feldarabolható-e hatnál kevesebb vágással a kocka 27
kis kockára?
29. Különféle kombinatorikai feladatok 153

1758. [3] Egy 4 egység élhosszúságú kockát szét akarunk vágni 64 db 1 élhosszúságú
kockára. Ezt megtehetjük egyszerűen 9 vágással, ha a szétvágással keletkező
darabokat nem mozgatjuk el egymástól. Hány vágásra csökkenthető ez le, ha
az egyes vágások után kapott darabokat alkalmas módon átrendezhetjük?
1759. [4] Írjuk fel egy papírszalagra harmincszor egymás után a 123-at:
123 123 123 . . . 123. Legfeljebb hány részre vágható szét a szalag úgy, hogy
minden darabkán más-más számot kapjunk?
1760. [3] Valaki úgy megy fel a lépcsőn, hogy egy-egy lépésével vagy 1, vagy 2 lépcső-
fokot lép át. Hányféleképpen juthat fel a 10. lépcsőfokra?
1761. [3] Hányféleképpen lehet egy 2 × 10-es téglalapot 2 × 1-es dominókkal kirakni?
1762. [4] Hány olyan nyolc számból álló, csak 0-t vagy 1-et tartalmazó sorozat van,
amelyben nem fordul elő két szomszédos 1-es?
1763. [4] Szétoszthatunk-e 10 000 kavicsot 100 halomba úgy, hogy mindegyik különbö-
ző számú kavicsot tartalmazzon, de bármelyik halmot osztjuk is két részre, a
keletkezett 101 halomra ez már nem teljesül?
1764. [4] Egy 10 résztvevős pingpong-bajnokságon mindenki egyszer játszott mindenki-
vel. Az egyes versenyzők győzelmeinek és vereségeinek számát jelölje rendre
x1 , x2 , . . . , x10 , illetve y1 , y2 , . . . , y10 . Bizonyítsuk be, hogy x12 + x22 + · · · +
+ x102
= y12 + y22 + · · · + y102
.
1765. [5] Arthur király kerekasztala mellett k különböző család 13 lovagja ül, ahol
1 < k < 13. Valamennyien vagy egy arany-, vagy egy ezüstserleget tartanak
a kezükben; az aranyserlegek száma éppen k. Arthur király parancsot adott a
lovagoknak, hogy a következő percben valamennyien adják át a serlegüket a
jobbra ülő szomszédjuknak; majd ismételjék meg ezt a műveletet még tizen-
kétszer. Bizonyítsuk be, hogy valamelyik percben volt két, azonos családból
származó lovag, akik aranyserleget tartottak a kezükben.
1766. [5] Arthur király ötven lovagja a kerek asztal mellett ül. Előttük egy-egy serleg,
tele vörös vagy fehér borral. Éjfélkor mindenki átadja valakinek a serlegét,
mégpedig mindazok, akiknél vörös bor van, a jobb szomszédjuknak, a többiek
pedig a bal oldali másodszomszédjuknak. Bizonyítsuk be, hogy így lesz olyan
lovag, akinek nem jut serleg. (Az asztalon vörös bor is, fehér bor is volt.)
1767. [5] Egy n × n-es táblázat mezőibe nemnegatív egész számokat írtunk úgy, hogy
ha valamely sor és oszlop kereszteződésében nulla áll, akkor ebben a sorban
és oszlopban a számok összege legalább n. Mutassuk meg, hogy a táblázatban
lévő számok összege legalább n2 /2.
154 Feladatok

1768. [5] 0, 01, 0110, 01101001, 0110100110010110, . . . A sorozat következő elemét


úgy kapjuk, hogy az utolsó 0–1 sorozathoz hozzáillesztjük a sorozat komple-
menterét (melyet az eredetiből úgy nyerünk, hogy a 0-t és az 1-est felcseréljük).
A sorozat 12. eleme egy 2048 számjegyből álló 0–1 sorozat. Ebben a sorozatban
milyen számjegy áll az 1996. helyen?
1769. [5] Van 1000 kártyánk, ezeket 000-tól 999-ig megszámozták. Van továbbá 100
dobozunk, ezek száma 00-tól 99-ig tart. Egy kártyát csak olyan dobozba szabad
betenni, amelynek számát úgy kapjuk, hogy a kártyáról egy jegyet letörlünk.
Mutassuk meg, hogy a kártyákat már 50 dobozba is berakhatjuk, de kevesebbe
nem!
1770. [5] Egy asztalon 100 kavicshalom van, bennük rendre 1, 2, . . . , 100 darab kavics.
Egy lépésben akárhány kiszemelt halmot csökkenthetünk; feltéve, hogy minde-
gyikből ugyanannyi kavicsot veszünk el.
Legkevesebb hány lépésben tudjuk valamennyi kavicsot elvenni az asztalról?
1771. [4] A kilenctagú (1, 2, 3, 4, 5, 6, 7, 8, 9) számsorozatot állítsuk elő minél keve-
sebb, olyan 9 tagú számsorozat „összegeként”, amelyek mindegyikében csak
kétféle szám szerepel (például: (0, 2, 2, 0, 0, 2, 2, 0, 0) egy ilyen sorozat). A
9 tagú sorozatok „összegét” úgy értelmezzük, hogy az azonos helyen álló szá-
mokat adjuk össze (például: (1, 1, 0, 0, 1, 0, 0, 0, 1) + (0, 2, 2, 0, 0, 2, 2, 0, 0) =
= (1, 3, 2, 0, 1, 2, 2, 0, 1)).
1772. [3] Az 1, 2, 3, . . . , 100 számok közül tetszőlegesen kiválasztottunk 25 számot.
Ezekhez lehet-e mindig további 25 számot választani úgy, hogy a kiválasztott
50 szám összege ugyanannyi legyen, mint a megmaradó 50 szám összege?
1773. [3] Az asztalra egy sorban kiraktunk 100 darab mérősúlyt úgy, hogy bármely két
szomszédos súly között 1 gramm az eltérés. Bizonyítsuk be, hogy a mérősúlyok
szétoszthatók két, 50–50 darabból álló csoportra úgy, hogy a két csoport azonos
tömegű.
1774. [5] 2k érme között nincs három különböző súlyú. Egy kétkarú mérlegen súlyok
használata nélkül k méréssel meg kell mondanunk, hogy van-e köztük két kü-
lönböző súlyú; és ha van, kell mutatni egy könnyebbet és egy nehezebbet. Bi-
zonyítsuk be, hogy ez lehetséges és mutassuk meg, hogyan.
1775. [6] 100 darab érme között nincs három különböző súlyú. Egy kétkarú mérlegen
súlyok használata nélkül 7 méréssel meg kell mondanunk, hogy van-e köztük
két különböző súlyú, és ha van, kell mutatni egy könnyebbet és egy nehezebbet.
Bizonyítsuk be, hogy ez lehetséges és mutassuk meg, hogyan.
1776. [5] Van 6 egyenlő átmérőjű golyónk, melyek közül 2 piros, 2 fehér, 2 pedig kék. Az
egyszínű golyók egyike aranyból, a másika ezüstből készült. Legkevesebb hány
mérés kell ahhoz, hogy egy kétkarú mérleg segítségével ki tudjuk választani a
három aranygolyót? (Az ugyanolyan anyagból készült golyók egyenlő súlyúak.)
29. Különféle kombinatorikai feladatok 155

1777. [4] Tudjuk, hogy 11, külsőre egyforma golyó közül kettő radioaktív. Egy méréssel
meg tudjuk állapítani, hogy a golyók egy csoportjában van-e radioaktív vagy
nincs. Mutassuk meg, hogy hét méréssel ki lehet választani a két radioaktív
golyót.
1778. [6] Van egy kétkarú mérlegünk, továbbá 101 darab mérősúly, amelyek együttes
tömege 200 gramm. Valamennyi mérősúly tömegének grammokban kifejezett
mérőszáma egész szám. A súlyokat csökkenő sorrendben rakjuk be a serpenyők-
be úgy, hogy ha a mérleg egyensúlyban van, akkor a bal oldaliba; ha pedig nem,
akkor a könnyebbikbe tesszük a következőt.
Bizonyítsuk be, hogy miután valamennyi súlyt föltettük, a mérleg egyensúlyban
lesz.
1779. [5] 100 kavicsot 50 kupacba raktunk, mindegyikbe legalább egyet. Bizonyítsuk
be, hogy ha egyetlen kupacban sincs 50-nél több kavics, akkor a kupacok két
csoportba rendezhetők úgy, hogy a két kupacban ugyanannyi kavics van.
1780. [5] Adott 35 pozitív egész szám, amelyek összege 100, és egyikük sem nagyobb
50-nél. Bizonyítsuk be, hogy van köztük néhány olyan, amelyek összege 50.
1781. [5] Van néhány arany és néhány ezüst érménk. Az érmék közül bármely kettő
súlya különböző. Valaki sorba rendezte őket súlyuk szerint, de tévedésből a
legnehezebbet nem az első, hanem az utolsó helyre rakta. (Így az 1. helyre a 2.
legnehezebb, a 2. helyre a 3. legnehezebb stb. került.) Át akarjuk rendezni őket a
helyes nehézségi sorrendbe, de többlet helyünk nincs, ezért egy lépésben mindig
csak két (nem feltétlenül szomszédos) érmét tudunk felcserélni. Igazoljuk, hogy
ha érméinket ilyen feltétel mellett rendeztük át, valamelyik lépésben egy arany
és egy ezüst érmét is fel kellett cserélnünk.
1782. [4] Az 1 − 2 − 3 − 4 − · · · − 1999 kifejezésben zárójelpárok alkalmas elhelyezé-
sével jelölhetjük ki a műveletek sorrendjét. Bizonyítsuk be, hogy vannak olyan
zárójelezések, amelyek ugyanazt az eredményt adják.
1783. [4] Mutassuk meg, hogy egy 2n oldalú konvex sokszögnek mindig van olyan átlója,
amely egyik oldallal sem párhuzamos!
1784. [4] Egy 12 × 12-es sakktábla bizonyos mezői feketék, a többiek pedig fehérek.
A fekete mezők fehérre, a fehérek pedig feketére festhetők, de csak úgy, ha
egy lépésben egy teljes sor vagy egy teljes oszlop valamennyi mezőjének meg-
változtatjuk a színét. Elérhető-e mindig, hogy a sakktábla valamennyi mezője
fekete legyen?
1785. [5] Egy 8×8-as tábla mezőiben egész számok állnak. Egy lépésben valamely 3×3-
as vagy 4 × 4-es részen mindegyik számot 1-gyel növeljük. Elérhető-e mindig,
hogy minden szám 10-zel osztható legyen?
156 Feladatok

1786. [5] Egy iskolában a tanulók 10 fős csapatokat szerveztek. Egy diák több csapatnak
is tagja lehet, vagy akár egyiknek sem. A csapatok száma 500. Bizonyítsuk be,
hogy a diákokat el lehet helyezni két teremben úgy, hogy minden csapatnak
mindkét teremben legyen tagja.
1787. [6] Legyen n 2-nél nagyobb páros szám. Egy n × n-es sakktábla mezőit kiszíneztük
n2
színnel úgy, hogy minden színű mezőből pontosan kettő van. Bizonyítsuk
2
be, hogy el lehet helyezni n bástyát csupa különböző színű mezőre úgy, hogy
semelyik kettő se üsse egymást.
1788. [6] Egy 3n + 1 tagú társaság bármely két tagja vagy teniszezni, vagy sakkozni,
vagy pingpongozni szokott egymással. Mindegyiküknek n tenisz-, n sakk- és
n pingpongpartnere van. Bizonyítsuk be, hogy van a társaságban három olyan
ember, akik egymás között mind a három játékot játsszák.
1789. [6] Bizonyítsuk be, hogy 1-től 1986-ig a természetes számok kiszínezhetők pirossal
és kékkel úgy, hogy ne forduljon elő 18 tagú számtani sorozat, melynek minden
tagja ugyanolyan színű.
1790. [6] Bizonyítsuk be, hogy egy 1000 oldalú szabályos sokszög átlóit és oldalait szí-
nezhetjük úgy piros és kék színnel (mindegyik szakaszt csak az egyik színnel),
hogy a sokszögnek ne legyen 20 olyan csúcsa, amelyek közötti szakaszok mind-
egyike ugyanolyan színű.
1791. [6] Egy 500 tagú vállalat dolgozói 2n nyelv közül beszélnek néhányat, minden
dolgozó legalább n nyelvet beszél. Bizonyítsuk be, hogy ekkor kiválasztható
14 nyelv úgy, hogy minden dolgozó a 14 nyelv közül legalább egyen beszél.
1792. [7] Legyen adva egy X alaphalmaz n-elemű részhalmazainak egy F rendszere.
Mutassuk meg, hogy ha |F| < 2n−1 , akkor X-nek van olyan, két színnel való
színezése, hogy egyik F-beli n-es sem lesz egyszínű.
29. Különféle kombinatorikai feladatok 157

30. Konstrukciók

1793. [5] Az ötágú csillagon megjelölt 10 köröcs-


ke közül minél többre helyezzünk ko-
rongot. Korongot a következő módon
lehet felrakni: valamelyik üres köröcs-
kébe teszünk egyet, majd valamelyik
szomszédos köröcskét átugorva (mind-
egy, hogy ott van korong vagy nincs)
és egy üres köröcskére érkezve, a ko-
rong ott marad. Ugrani csak valamelyik
egyenes vonal mentén lehet.

1794. [3] Egy kereskedőnek volt egy 40 kg-os mérősúlya, ami egyszer leesett és négy
darabra tört szét. Amikor a darabokat megmérte, kiderült, hogy mindegyik darab
egész számú kg-ot nyom, és a négy darabbal minden egész kg súlyt le lehet
mérni egy kétkarú mérlegen 1 kg-tól 40 kg-ig. Hány kg-os darabokra tört szét
a mérősúly?
1795. [3] Van 100 db, páronként különböző súlyú golyónk és egy kétkarú mérlegünk.
Válasszuk ki minél kevesebb mérlegeléssel a legkönnyebb golyót.
1796. [3] Van 100 db, páronként különböző súlyú golyónk és egy kétkarú mérlegünk.
Válasszuk ki minél kevesebb mérlegeléssel a legkönnyebb és a legnehezebb
golyót.
1797. [3] Van 100 db, páronként különböző súlyú golyónk és egy kétkarú mérlegünk.
Válasszuk ki minél kevesebb mérlegeléssel a két legnehezebb golyót.
1798. [3] 9 érme közül egy hamis, s ez könnyebb, mint a többi (a többi egyenlő súlyú).
Egy kétkarú mérlegen súlyok felhasználása nélkül két mérlegeléssel keressük
ki közülük a hamis érmét.
1799. [5] 12 pénzérme közül egy hamis. Tudjuk, hogy a hamis pénz súlya különbözik
a többitől, de nem tudjuk, hogy könnyebb-e náluk vagy nehezebb. Az igazi
pénzdarabok mind egyenlő súlyúak. Súlyok nélkül kétserpenyős mérlegen há-
rom méréssel meg kell állapítanunk, hogy melyik a hamis pénz, és könnyebb-e
vagy nehezebb, mint a többi.
1800. [3] Egy repülőtéren a várakozó 128 utas között van egy terrorista, aki ruházatában
fegyvert rejteget, s persze nem tudjuk, hogy melyik ez az utas. Van egy készü-
lék, amely jelzi, ha a vizsgálati teremben levők között valaki fegyvert rejteget.
158 Feladatok

Csoportokba osztva kívánják megvizsgálni az utasokat. A lehető leggyorsabban,


legkevesebb vizsgálattal szeretnék megtalálni a terroristát. Hogyan csinálják?
1801. [3] 10 láda pénz között az egyik ládában csupa 11 grammos érme van, a többiben
10 grammosak az érmék. Egykarú mérlegen mérősúlyokkal egyetlen méréssel
állapítsuk meg, hogy melyik a nehezebb érméket tartalmazó láda.
1802. [4] Két rabló úgy szokott megosztozni a zsákmányon, hogy az egyik kétfelé osztja
azt, és a másik azt a részt veszi el, amelyiket akarja. Ez így igazságos, mert
mindkettőnek megvan a lehetősége arra, hogy megszerezze a zsákmány felét.
Hogyan osztozkodjon három rabló, ha azt szeretnék biztosítani, hogy bármelyi-
kük megkapja a zsákmány harmadát, bármit is csinál a másik kettő?
1803. [4] A folyón átfektetett kábelt — amelynek 49, egymástól elszigetelt áramkört kell
átvezetnie — a szokásostól eltérően egyszínű szigetelésű drótokból sodorták.
Így aztán nem lehet tudni, hogy a két parton látható drótvégződések közül me-
lyik melyikkel tartozik össze (azaz, melyik ugyanannak a vezetéknek a két vé-
ge). Az azonosítást úgy kell elvégezni, hogy mindegyik drótvégződésre cédulát
kötünk, és az összetartozókra mindkét parton ugyanazt a számot írjuk. Ehhez
rendelkezésre áll a part mentén futó villanyvezeték, egy fázisceruza, valamint
egy csónak a folyón való átkelésre. Szervezzük meg a munkát úgy, hogy minél
kevesebbszer kelljen átkelni a folyón.
1804. [4] Egy versenyen 18 csapat mérkőzik egymással. Szervezzük meg a fordulókat
úgy, hogy egy-egy fordulóban minden csapatnak legyen ellenfele, s a verseny
során minden csapat mindegyikkel pontosan egy mérkőzést játsszék.
1805. [5] Mutassuk meg, hogy megszervezhető a sportélet egy 3n + 1 tagú társaságban
úgy, hogy bármely két tag vagy teniszezik, vagy sakkozik, vagy pingpongozik
egymással; és mindegyiküknek n tenisz-, n sakk- és n pingpongpartnere legyen.
1806. [3] Egy 4 × 4-es táblázat mezői közül kiválasztottunk hatot, s mindegyikbe raj-
zoltunk egy-egy csillagot. Mutassuk meg, hogy van két olyan sor és két olyan
oszlop, melyek együtt tartalmazzák mind a hat csillagot! Helyezzünk el a táblá-
zatban 7 csillagot úgy, hogy ne lehessen kiválasztani két olyan sort és két olyan
oszlopot, amelyek együtt tartalmaznák mind a 7 csillagot.
1807. [4] Adél, Bori, Csilla és Dóri egy sötét, szűk alagúton szeretne átjutni. Van egy
12 percig égő lámpásuk. Adél 1, Bori 2, Csilla 4 és Dóri 5 perc alatt képes
megtenni a távot. A sötétben félnek, ezért az alagútban lámpás nélkül nem
mehetnek és a szűk alagútban egyszerre legfeljebb ketten férnek el. Átjuthatnak-
e mindannyian a szűk alagúton?
1808. [4] Egy szigeten felszállásra készen áll néhány repülőgép. Mindegyik tartályában
annyi üzemanyag van, amennyi egy világ körüli út felére elegendő. Repülés
közben az egyik repülőgép tartályából át lehet szivattyúzni az üzemanyagot a
másik gép tartályába. A speciális üzemanyag egyetlen ellátó telepe a szigeten
31. Teljes indukció 159

van. Az üzemanyag felvételével kapcsolatban feltételezzük, hogy a gépek sem


a földön, sem a levegőben nem vesztenek időt. Hány egyforma repülőgépre van
szükség a szigeten ahhoz, hogy biztosítsák egy gép világ körüli útját leszállás
nélkül? Valamennyi gép sebessége egyforma, üzemanyagfelhasználása azonos,
mindnek vissza kell térnie a szigetre.
1809. [4] Egy kutató két teherhordó segítségével át szeretne vágni a sivatagon a legköze-
lebbi, 380 km-re levő oázisba. Napi 60 km-t tudnak megtenni, és mind a kuta-
tó, mind a teherhordók legfeljebb négy napra elegendő élelmet és vizet tudnak
magukkal vinni. A kutatónak biztosítania kell, hogy a teherhordók visszatérhes-
senek oda, ahonnan elindultak. A sivatagban élelmiszert nem tárolhatnak.
Átjuthat-e a kutató ezekkel a feltételekkel az oázisba, ha nem számít arra, hogy
onnan elébe jönnek?
1810. [4] Két várost egy autóút köt össze. A városokban olyan autók vannak, amelyek
éppen annyi benzint képesek tankolni, amennyi a két város közti út felének
megtételéhez elegendő. Segédautók felhasználásával az egyik városból egy A
autót akarunk eljuttatni a másik városba. A segédautók az út bizonyos pontjain
benzint adhatnak át egymásnak, és A-nak; de csak annyit, hogy kiindulási állo-
másukra visszatérhessenek. Mennyi a szükséges segédautók minimális száma?
1811. [4] 10 cowboy párbajt vív a következő szabályok szerint:
— Mindenki egy lövést ad le, s ez a lövés halálos.
— Mindenki a hozzá legközelebbit lövi le; ha több ilyen is van, akkor közülük
valamelyiket.
— Mindenki ugyanabban a pillanatban adja le lövését.
Lehetséges-e, hogy ennek a vérengzésnek csak két áldozata van?

31. Teljes indukció

1812. [3] Hol a hiba a következő bizonyításban?


Állítás: Bármely n pozitív egészre a n−1 = 1, ahol a > 0 tetszőleges szám.
Bizonyítás: Ha n = 1, akkor a n−1 = a 1−1 = a 0 = 1.
Ha feltesszük, hogy a tétel igaz az 1, 2, . . . , n esetre, akkor azt kapjuk, hogy
a n−1 · a n−1 1 · 1
a (n+1)−1 = a n = = = 1;
a n−2 1
tehát a tétel (n + 1) esetére is igaz.
1813. [3] Hol a hiba a következő bizonyításban?
1 1 1 3 1
Állítás: + + ··· + = − .
1·2 2·3 (n − 1) · n 2 n
160 Feladatok

Bizonyítás: Indukciót alkalmazunk n-re. n = 1 esetén 3/2 − 1/n = 1/(1 · 2); ha


feltesszük, hogy a tétel igaz n-re, akkor
1 1 1 3 1 1
+ ···+ + = − + =
1·2 (n − 1) · n n · (n + 1) 2 n n(n + 1)

3 1 1 1 3 1
= − + − = − .
2 n n n+1 2 n+1
1814. [3] Hol a hiba a következő bizonyításban?
Állítás: Minden lánynak ugyanolyan színű a szeme.
Bizonyítás. Teljes indukcióval bizonyítunk. Legyen a lányok száma: n.
n = 1-re az állítás nyilvánvalóan (semmitmondóan) igaz. Marad az n-ről (n +
+ 1)-re való átmenet. Hogy konkrét esetről beszéljünk, 3-ról 4-re lépünk át; az
általános esetet az olvasóra bízzuk.
Legyen a négy lány Anna, Bea, Cecília és Dorottya, vagy röviden A, B, C és
D. Állítólagosan (n = 3) A, B és C szeme ugyanolyan színű. A B, C és D
szeme szintén ugyanolyan színű (n = 3). Következésképp mind a négy lány,
A, B, C és D szeme ugyanolyan színű; hogy teljesen világos legyen, nézzük a
diagramot:
  
A, B, 
C, D
Ez bizonyítja az állítást n + 1 = 4 esetén; a 4-ről 5-re való átlépés nyilván nem
sokkal nehezebb.
1815. [3] Igazoljuk a következő oszthatóságokat.
a) 4 | 7n + 3n+1 , n = 1, 2, . . .
b) 9 | 7n + 3n − 1, n = 1, 2, . . .
c) 7 | 5 · 9n−1 + 24n−3 , n = 1, 2, . . .
d) 17 | 7 · 52n−1 + 23n+1 , n = 1, 2, . . .
e) 19 | 5 · 23n−2 + 33n−1 , n = 1, 2, . . .
f) 17 | 62n + 19n − 2n+1 , n = 1, 2, . . .
g) 9 | n3 + (n + 1)3 + (n + 2)3 , n = 1, 2, . . .
h) 7 | 32n+1 + 2n+2 , n = 1, 2, . . .
i) 133 | 11n+2 + 122n+1 , n = 0, 1, . . .
j) 16 | 32n+2 + 8n − 9, n = 1, 2, . . .
k) 19 | 52n+1 · 2n+2 + 3n+2 · 22n+1 , n = 1, 2, . . .
l) 5 | 174n+1 + 3 · 92n , n = 1, 2, . . .
m) 4 | 3 · 172n + 25n , n = 1, 2, . . .
n) 3 | 5 · 17n + 52n , n = 1, 2, . . .
o) 23 | 27n+3 + 32n+1 · 54n+1 , n = 1, 2, . . .
n
p) 3n+1 | 23 + 1, n = 0, 1, . . .
31. Teljes indukció 161

1816. [3] Mutassuk meg, hogy (1 + 2 + 3 + · · · + n)2 = 13 + 23 + 33 + · · · + n3 .


1817. [3] Mutassuk meg, hogy
1 · 2! 2 · 3! n · (n + 1)! (n + 2)!
+ 2 + ···+ = − 2.
2 2 2n 2n
1 · 3! 2 · 4! n · (n + 2)!
1818. [4] + 2 + ··· + =?
3 3 3n
1819. [3] a) Tudjuk, hogy a1 = 4, an+1 = 3an − 2, n = 1, 2, 3, . . . Mutassuk meg, hogy
an = 3n + 1.
b) Tudjuk, hogy a1 = 2, a2 = 8, an+2 = 4an+1 − 3an , n = 1, 2, 3, . . . Mutassuk
meg, hogy an = 3n − 1.
c) Tudjuk, hogy a1 = 1, a2 = 5, an+2 = 5an+1 − 6an , n = 1, 2, 3, . . . Mutassuk
meg, hogy an = 3n − 2n .
d) Tudjuk, hogy a1 = 1, a2 = 9, an+2 = 9an+1 − 20an , n = 1, 2, 3, . . . Mutassuk
meg, hogy an = 5n − 4n .
e) Tudjuk, hogy a1 = 3, a2 = 15, an+2 = 5an+1 − 4an , n = 1, 2, 3, . . . Mutassuk
meg, hogy an = 4n − 1.
f) Tudjuk, hogy a1 = 29, a2 = 85, an+2 = 5an+1 −6an , n = 1, 2, 3, . . . Mutassuk
meg, hogy an = 2n + 3n+2 .
g) Tudjuk, hogy a1 = 3, a2 = 6, an+2 = 3an+1 − 2an − 1, n = 1, 2, 3, . . .
Mutassuk meg, hogy an = 2n + n.
1 1 an · an+1
1820. [4] Tudjuk, hogy a1 = , a2 = , an+2 = , n = 1, 2, 3, . . . Mutassuk
2 3 3an − 2an+1
1
meg, hogy an = n−1 .
2 +1
2
an−1 +2
1821. [5] Tudjuk, hogy a0 = a1 = 1, an = , n = 2, 3, 4, . . . Mutassuk meg, hogy
an−2
an értéke egész szám minden n-re.

Igazoljuk a Fibonacci-sorozat alábbi tulajdonságait.


(f1 = f2 = 1, fn+2 = fn+1 + fn , n = 1, 2, 3, . . . )
1822. [4] f1 + f2 + · · · + fn = fn+2 − 1, n = 1, 2, 3, . . .
1823. [4] f1 + f3 + · · · + f2n−1 = f2n , n = 1, 2, 3, . . .
1824. [4] f2 + f4 + · · · + f2n = f2n+1 − 1, n = 1, 2, 3, . . .
1825. [4] f1 + 2f2 + 3f3 + · · · + nfn = (n − 1)fn+2 − fn+1 + 2, n = 1, 2, 3, . . .
1826. [4] f1 − f2 + f3 − f4 + · · · + (−1)n+1 fn = (−1)n+1 fn−1 + 1, n = 2, 3, 4, . . .
1827. [4] f12 + f22 + · · · + fn2 = fn fn+1 , n = 1, 2, 3, . . .
162 Feladatok

1828. [4] f1 f2 + f2 f3 + · · · + f2n−1 f2n = f2n


2
, n = 1, 2, 3, . . .
1829. [4] f1 f2 + f2 f3 + · · · + f2n f2n+1 = f2n+1
2
− 1, n = 1, 2, 3, . . .
1830. [4] a) 3 | f4k , k = 1, 2, 3, . . .
b) 4 | f6k , k = 1, 2, 3, . . .
c) 5 | f5k , k = 1, 2, 3, . . .
d) 7 | f8k , k = 1, 2, 3, . . .
e) 9 | f12k , k = 1, 2, 3, . . .
1831. [4] fn2 = fn+1 fn−1 + (−1)n+1 , n = 2, 3, 4, . . .
1832. [4] f2n−1 = fn fn+1 − fn−2 fn−1 , n = 3, 4, 5, . . .
1833. [4] (−1)n = fn+1 fn+2 − fn fn+3 , n = 1, 2, 3, . . .
1834. [4] fn2 + fn+1
2
= f2n+1 , n = 1, 2, 3, . . .
1835. [4] fn+1
2
− fn−1
2
= f2n , n = 2, 3, 4, . . .
1836. [4] (fn fn+3 )2 + (2fn+1 fn+2 )2 = f2n+3
2
, n = 1, 2, 3, . . .
1837. [5] fn+h fn+k − fn fn+k+h = (−1)n fk fh , h = 1, 2, 3, . . . , k = 1, 2, 3, . . . , n =
= 1, 2, 3, . . .
1838. [5] f3n = fn+1
3
+ fn3 − fn−1
3
, n = 2, 3, 4, . . .
1839. [5] fm+n = fm−1 fn + fm fn+1 , m = 2, 3, 4, . . . , n = 1, 2, 3, . . .
1840. [5] fk | fk+2n + fk−2n , n = 1, 2, 3, . . . , k > 2n
1841. [6] Igazoljuk, hogy ha m | n, akkor fm | fn .
1 1 1 1 1
1842. [4] Mutassuk meg, hogy + + + ···+ + < 1.
f1 f3 f2 f4 f3 f5 f1999 f2001 f2000 f2002
1843. [4] fn4 − fn−2 · fn−1 · fn+1 · fn+2 = 1, n = 3, 4, 5, . . .
1844. [4] fn+1
2
= 4fn · fn−1 + fn−2
2
, n = 3, 4, 5, . . .
1845. [5] fn2 | fn−1
k
− fkn−1 , n = 2, 3, 4, . . . , k = 1, 2, 3, . . .
1846. [5] fn2 | (−1)k+1 fn−2
k
− fkn−2 , n = 3, 4, 5, . . . , k = 1, 2, 3, . . .
1847. [4] Mutassuk meg, hogy minden természetes szám előállítható a Fibonacci-sorozat
páronként különböző elemeinek összegeként.
1848. [5] Bizonyítsuk be, hogy ha a az n!-nál nem nagyobb pozitív egész szám, akkor a
előállítható az n! szám legfeljebb n db, páronként különböző osztójának össze-
geként.
31. Teljes indukció 163

1849. [3] Mutassuk meg, hogy egy négyzet feldarabolható n db négyzetre, ahol n ≥ 6.
1850. [5] Mutassuk meg, hogy van olyan N, hogy minden n ≥ N esetén egy kockát fel
lehet darabolni n darab kisebb kockára.
(Bizonyítható, hogy már N = 48 esetén is igaz az állítás.)
1851. [3] Mutassuk meg, hogy egy háromszög feldarabolható n db, hozzá hasonló há-
romszögre, ahol n ≥ 6.
1852. [4] Igazoljuk, hogy bármely háromszög feldarabolható n darab (n ≥ 4) egyenlő
szárú háromszögre!
1853. [3] Néhány egyenes a síkot tartományokra bontja. Mutassuk meg, hogy ezek a
részek két színnel kiszínezhetők úgy, hogy az oldalszomszédos tartományok
különböző színűek legyenek.
1854. [5] Igazoljuk, hogy p | np − n, ahol p prímszám, n ∈ N (kis Fermat-tétel).
1855. [4] Legyen n 3-mal osztható természetes szám! Hagyjunk el az n − 1, n − 2, n − 3,
. . . , 3, 2, 1 sorozatból minden harmadik számot! A megmaradók közül az első
kettőt lássuk el pozitív előjellel, a következő kettőt negatív előjellel, az utánuk
következő kettőt ismét pozitív előjellel és így tovább. Bizonyítsuk be, hogy az
így kapott számok összege n-nel egyenlő.
1856. [4] Sorban egymás után írtunk 1992 db számot, ezek mindegyike +1 vagy −1. A
következő műveletet végezzük: két szomszédos szám alá szorzatukat írjuk. Ezt
az eljárást a kapott 1991 db számon is elvégezzük, majd újra és újra ismételjük
ezt. Bizonyítsuk be, hogy ha az első sorban volt −1, akkor a kapott számhá-
romszögben legalább 1992 db −1 van.
1857. [4] Mutassuk meg, hogy 2n | (n + 1)(n + 2) . . . (2n), n ∈ N .
1858. [4] Mutassuk meg, hogy 2n+1 − 1 egész szám közül mindig kiválasztható 2n db
szám úgy, hogy összegük osztható legyen 2n -nel (n ∈ N ).
1859. [4] Bizonyítsuk be, hogy minden n pozitív egészre van n-jegyű, 1 és 2 jegyekből
álló szám, mely többszöröse 2n -nek.
1860. [5] Igazoljuk, hogy ha n 2-nél nagyobb egész szám, akkor van olyan x és y páratlan
egész, melyekre 2n = 7x 2 + y 2 .
1861. [4] Az 1, 2, 3, . . . , 2n számokból kiválasztjuk az a1 , a2 , . . . , an+1 számokat.
Mutassuk meg, hogy ezek között van két olyan szám, amelyek közül egyik
osztója a másiknak.
[5]

1862. Mutassuk meg, hogy p < 4n , ahol p az n-nél nem nagyobb prímszámok
p≤n p≤n
szorzatát jelöli.
164 Feladatok

1863. [5] Mutassuk meg, hogy minden k egész szám végtelen sokféle módon felírható
k = ±12 ± 22 ± 32 ± · · · ± n2 alakban, ahol n valamely természetes szám; a ±
jelek közül pedig a megfelelőt kell választani.
1864. [4] Az a0 , a1 , . . . , a100 pozitív egészekből álló sorozatra a1 > a0 ,
a2 = 3a1 − 2a0 , a3 = 3a2 − 2a1 , . . . , a100 = 3a99 − 2a98 . Mutassuk meg, hogy
a100 ≥ 2100 .
1 1 1
1865. [5] a1 = 1, a2 = a1 + , a3 = a2 + , . . . , an+1 = an + . Bizonyítsuk be, hogy
a1 a2 an
a100 > 14.
1 1 1
1866. [5] a1 = 1, a2 = a1 + , a3 = a2 + 2 , . . . , an+1 = an + 2 . Mutassuk meg, hogy
a12 a2 an+1
a9000 > 30.
1 2 n √
1867. [5] a1 = 1, a2 = 1 + , a3 = 1 + , . . . , an+1 = 1 + . Igazoljuk, hogy n ≤
√ a 1 a 2 a n
≤ an ≤ n + 1.
1868. [5] 1 < a1 < 2, an+1 = an 2 − 2an + 2, n = 1, 2, 3, . . . Mutassuk meg, hogy 1 <
< an < 2, n = 2, 3, 4, . . . .
1869. [6] A pozitív a1 , a2 , a3 , . . . számokra teljesül az an2 ≤ an − an+1 , n = 1, 2, 3, . . .
1
egyenlőtlenség. Mutassuk meg, hogy an < .
n

[5] 22n 2n 22n 22n


1870. Mutassuk meg, hogy ≤ < √ (ill. < √ , s ez az élesebb
2n n 2n 2n + 1
egyenlőtlenség könnyebben igazolható).

22n 2n
1871. [5] Mutassuk meg, hogy < , n > 1.
n+1 n

22n 2n 22n
1872. [5] Mutassuk meg, hogy √ < <√ , n > 1.
4n n 3n + 1
3n (3n)!
1873. [3] Mutassuk meg, hogy < , n > 1.
2(n + 2) (2n!)2
1 1 3 5 2n − 1 1
1874. [4] Mutassuk meg, hogy √ < · · · . . . · <√ , n > 1.
2 n 2 4 6 2n 3n + 1
1 1 1 √
1875. [4] Igazoljuk, hogy 1 + √ + √ + · · · + √ > 2( n + 1 − 1).
2 3 n
1876. [3] Melyik ! szám a nagyobb: !
2 3
·· ··
2 · n db 2-es 3· n − 1 db 3-as
22 vagy 33 ?
32. Kombinatorika a geometriában 165

1877. [5] Melyik ! szám a nagyobb: !


3 4
·· ··
3· n db 3-as 4· n − 1 db 4-es
33 vagy 44 ?
[6]
1878. Legyenek a1 , a2 , . . . , an tetszőleges pozitív számok, amelyek nem mind egyen-
lők. Bizonyítsuk be, hogy
√ a1 + a2 + · · · + an
n
a1 a2 . . . an < .
n
1879. [4] Adott egy kör s annak egy átmérője. Az átmérő mindegyik végére az 1 szá-
mot írtuk. Első lépésben megfelezzük mindegyik félkörívet, s a felezőpontokra
az ívek végén levő számok összegét írjuk. Második lépésben megfelezzük a
negyed körívek mindegyikét, s a felezőpontokra az ívek végén levő számok
összegét írjuk. Az eljárást hasonlóan folytatjuk tovább. Ha az n-edik lépés után
összeadjuk a kör kerületén levő számokat, mit kapunk végeredményül?
1880. [4] Adott a síkon n pont, s közülük néhány pontpárt szakasszal kötünk össze.
Megtehető-e ez úgy, hogy ne legyen 3 pont, melyből ugyanannyi szakasz indul?
1881. [5] 1001 kavicsot gyűjtöttünk egy kupacba. A kupacot tetszés szerint két részre
osztjuk, és összeszorozzuk a két részben levő kavicsok számát. Ezután valame-
lyik részre — ha legalább két kavicsot tartalmaz — megismételjük az előbbi
műveletet; azaz újabb két részre osztjuk, és kiszámítjuk a két új rész elemszá-
mának szorzatát. Ezt így folytatjuk egészen addig, amíg minden részben csak
egy kavics marad. Milyen határok között mozog az így kapható 1000 darab
szorzat összege?
1882. [6] Adottak az x1 , x2 , . . . páronként különböző valós számok, amelyeket András
ismer, de Tamás nem. Tamás bármely xi , xj párra kérdést tehet fel; s András
megmondja, hogy xi < xj igaz-e. Mutassuk meg, hogy ha Tamás okosan,
ügyesen tesz fel 2k − 1 kérdést, akkor legalább k · 2k−1 egyenlőtlenséget fog
ismerni a számok közötti egyenlőtlenségekből.
1883. [6] n (≥ 4) idős hölgy mindegyike tud egy pletykát, azonban csak telefonon tudnak
beszélni egymással. Mutassuk meg: 2n−4 hívással megoldható, hogy mindenki
ismerje mindegyik pletykát.

32. Kombinatorika a geometriában

1884. [3] Egy kör kerületén 2 millió pont van úgy, hogy semelyik kettő sincs egy átmérőn.
Bizonyítsuk be, hogy akkor van olyan átmérő, melynek mindkét oldalán 1 millió
pont van.
166 Feladatok

1885. [3] Adott a síkon 2000 pont. Mutassuk meg, hogy


a) van olyan egyenes, melynek mindkét oldalán 1000 pont van.
b) van olyan kör, melynek belsejében 1000 pont van.
1886. [3] Adott a síkon 4000 pont, melyek közül semelyik három sem esik egy egyenes-
be. Bizonyítsuk be, hogy szerkeszthető 1000, egymást nem metsző négyszög,
melyeknek az adott pontok a csúcsai.
1887. [3] Mutassuk meg, hogy egy korlátos síkidom két egyenlő területű részre osztható
a) adott irányú;
b) adott ponton átmenő
egyenessel. Igaz-e ez a kerületre is?
1888. [3] Mutassuk meg, hogy egy korlátos, konvex alakzathoz mindig található olyan
egyenes, mely felezi annak kerületét és területét is.
1889. [4] Mutassuk meg, hogy egy korlátos síkidomot szét lehet vágni két egymásra me-
rőleges vágással négy egyenlő területű részre.
1890. [5] Az asztalon fekszik két palacsinta. Mutassuk meg, hogy van olyan vágás (olyan
egyenes), mely felezi mindkét palacsinta területét.
1891. [5] Bizonyítsuk be, hogy a síkban minden korlátos, konvex alakzat köré lehet négy-
zetet írni úgy, hogy a négyzet oldalai érintik az alakzatot.
1892. [6] Bizonyítsuk be, hogy a síkban minden korlátos, konvex alakzat köré lehet olyan
hatszöget írni, melynek szomszédos oldalai egymással 120◦ -os szöget zárnak
be, és a hatszög oldalai érintik az alakzatot.
1893. [4] Egy konvex görbének P középpontja, ha a görbe bármely P -n átmenő húrja
egyenlő hosszú. Lehet-e egy zárt görbének három középpontja? (Lehet-e egy
zárt görbének két középpontja?)
1894. [3] Mutassuk meg, hogy egy ellipszis belsejében tetszőlegesen felvett P ponton át
húzható olyan húr, amelyet P felez.
1895. [4] Szabályos nyolcszöget feldaraboltunk véges sok paralelogrammára. Mutassuk
meg, hogy a paralelogrammák között van két téglalap.
1896. [4] Mutassuk meg, hogy nincs olyan 13 oldalú konvex sokszög, amelyet fel lehetne
darabolni véges sok paralelogrammára.
1897. [4] Adott a síkon 6 pont, közülük semelyik három sem esik egy egyenesre. Bizo-
nyítsuk be, hogy kiválasztható közülük három olyan pont, amelyek által meg-
határozott háromszögnek van egy, legalább 120◦ -os szöge.
1898. [4] Adott a síkon 6 pont, közülük semelyik három sem esik egy egyenesre. Bi-
zonyítsuk be, hogy kiválasztható közülük két olyan (nem feltétlenül diszjunkt)
ponthármas, amelyek által meghatározott két háromszögben különbözők a leg-
kisebb szögek.
32. Kombinatorika a geometriában 167

1899. [5] Igazoljuk, hogy hat, egy síkban levő pont által alkotott húsz háromszög között
mindig van három, nem egybevágó háromszög. Mi mondható öt pont esetén?
1900. [4] Adott a síkon 5 pont, közülük semelyik három sem esik egy egyenesre. Bizo-
nyítsuk be, hogy kiválasztható közülük három olyan pont, amelyek egy tompa-
szögű háromszög csúcsait alkotják.
1901. [5] Adott a síkon 5 pont, közülük semelyik három sem esik egy egyenesre. Bi-
zonyítsuk be, hogy kiválasztható közülük négy olyan pont, amelyek konvex
négyszög csúcsait alkotják.
1902. [5] Adott a síkon 5000 pont, melyek közül semelyik három sincs egy egyenesen.
Mutassuk meg, hogy van 1000 olyan konvex négyszög, melyek csúcsai ezek
közül a pontok közül valók, és a négyszögeknek nincs közös pontjuk.
1903. [6] Adott a síkon n pont (n > 4), közülük

semelyik három nem esik egy egyenesbe.
1 n
Bizonyítsuk be, hogy legalább olyan konvex négyszög van, amelyeknek
5 4
csúcspontjai az adott pontok közül valók.
1904. [6] Adott a síkon 9 pont, közülük semelyik három sem esik egy egyenesre. Bizo-
nyítsuk be, hogy kiválasztható közülük öt olyan pont, amelyek konvex ötszög
csúcsait alkotják.
1905. [6] Adott a síkon n pont, melyek közül semelyik három nem esik egy egyenesre.
Háromszögeket jelölünk ki úgy, hogy csúcsaikat az adott pontok közül választ-
juk,
és

semelyik kettőnek nincs közös oldala.

Bizonyítsuk be, hogy legalább
1 n 1 n
háromszöget kijelölhetünk így, de -nél többet nem jelölhetünk ki.
9 2 3 2
1906. [4] Könnyen látható, hogy n = 1, 2, 3 mellett n kör legfeljebb 2n számú részre
bontja fel a síkot. Legfeljebb hány részre lehet felosztani a síkot 4 kör megraj-
zolásával?
1907. [4] Lehet-e szabályos ötszög és tízszög alakú lapokból parkettát készíteni?
1908. [4] Hány szabályos test létezik? (Most nevezzük szabályos testnek az olyan testet,
amely egybevágó szabályos sokszöglapokból áll, és minden csúcsában ugyan-
annyi lap találkozik.)
1909. [4] Adott a síkon 1000 pont, melyek közül semelyik három nincs egy egyenesen.
Egymást nem metsző szakaszokkal egymás után összekötjük ezeket a pontokat.
Ezt a műveletet mindaddig folytatjuk, míg végül már nem találunk olyan két
pontot, melyeket összeköthetnénk az eddigieket nem metsző szakasszal. Bizo-
nyítsuk be, hogy az összekötő szakaszok száma nem függ a pontok összeköté-
sének sorrendjétől.
1910. [5] Legfeljebb hány átlót lehet berajzolni egy konvex sokszögbe úgy, hogy közülük
bármely kettőnek legyen közös pontja (belső vagy végpontja)?
168 Feladatok

1911. [5] Egy n oldalú sokszög szögei között legfeljebb hány hegyesszög lehet?
1912. [4] Felbontható-e a tér 2000 darab diszjunkt, egybevágó részre?
1913. [4] Mutassuk meg, hogy a síkon adott végtelen sok pont között végtelen sok kü-
lönböző távolság lép fel.
1914. [5] A síkon adva van n pont úgy, hogy bármely három által meghatározott három-
szög területe legfeljebb 1 egység. Mutassuk meg, hogy a pontok lefedhetők egy
4 egység területű háromszöggel.
1915. [5] Egy szabályos tízszög csúcsai közül ötöt pirosra festettünk, a többit kékre. Mu-
tassuk meg, hogy van két olyan egybevágó háromszög, hogy az egyik csúcsai
kékek, a másiké pirosak.
1916. [4] Egy 13 oldalú konvex sokszög átlói is meghatároznak konvex sokszögeket.
Legfeljebb hány oldala van egy ilyen sokszögnek?
1917. [4] Adott a síkon n (n > 3) pont, feketék és pirosak; és bármely 4 pont közül a
pirosak és a feketék egy egyenessel szétválaszthatók. Mutassuk meg, hogy van
olyan egyenes, mely az összes piros pontot elválasztja az összes feketétől.
1918. [4] Miért nem lehet 10 egység sugarú körbe 450 pontot elhelyezni úgy, hogy bár-
melyik két pont távolsága legalább 1 egység legyen?
1919. [5] Egy 20 × 25-ös téglalapban elhelyeztünk 120 db egységnégyzetet. Mutassuk
meg, hogy a téglalapban elhelyezhető egy egységnyi átmérőjű kör, melynek
egyik négyzettel sincs közös pontja.
1920. [5] Egy R sugarú kerek asztalon n db r sugarú pénzérme fekszik úgy, hogy egyik
sem fedi — még részben sem — valamely másikat; és az asztalra újabb érme
már
nem helyezhető

el, mert nem találunk számára helyet. Mutassuk meg, hogy
1 R √ R
−1 < n < .
2 r r
1921. [4] Egy test valamennyi oldallapja háromszög, és minden csúcs megjelölhető az 1,
2, 3 szám valamelyikével úgy, hogy minden él különböző számmal jelölt két
csúcsot köt össze. Bizonyítsa be, hogy ekkor a test minden oldallapja befesthető
két adott szín egyikével úgy, hogy bármely két, közös él mentén csatlakozó lap
különböző színű legyen.
1922. [4] Rajzoljunk egy papírra tetszés szerint köröket, amelyek különbözőképpen metsz-
hetik egymást. Ily módon a síkot tartományokra bontottuk. Mutassuk meg, hogy
kiszínezhetjük ezt a térképet két színnel úgy, hogy bármely két, közös határvo-
nallal rendelkező tartomány különböző színű.
32. Kombinatorika a geometriában 169

1923. [5] Rajzoljunk egy papírra tetszés szerint köröket, és minden körben húzzunk meg
egy-egy húrt. Ily módon a síkot tartományokra bontottuk. Mutassuk meg, hogy
ha bármely két húrnak legfeljebb egy közös pontja van; akkor kiszínezhetjük ezt
a térképet három színnel úgy, hogy bármely két, közös határvonallal rendelkező
tartomány különböző színű.
1924. [5] Mutassuk meg, hogy a négyzetrácson nem lehet úgy kiválasztani rácspontokat,
hogy azok egy szabályos n-szög (n ≥ 5 és n = 6) csúcsai legyenek.
1925. [5] Mutassuk meg, hogy a négyzetrácson nem lehet úgy kiválasztani rácspontokat,
hogy azok egy szabályos
a) háromszög,
b) hatszög
csúcsai legyenek.
1926. [4] Mutassuk meg, hogy egy konvex, középpontosan szimmetrikus zárt görbén ta-
lálható 4 olyan pont, amelyek egy négyzet csúcsai.
1927. [6] Mutassuk meg, hogy bármely zárt térgörbének van négy olyan pontja, amelyek
egy síkban vannak. Igaz-e ez az állítás öt pontra is?
1928. [5] Mutassuk meg, hogy ha egy sík pontjai 4 színnel vannak kiszínezve, akkor van
olyan egyenes a síkban, melyen előfordul 3 különböző színű pont.
1929. [6] Mutassuk meg, hogy ha a tér pontjai 5 színnel vannak kiszínezve, akkor van
olyan sík, melyen előfordul 4 különböző színű pont.
1930. [3] Kovácséknál egy este sok vendég fordult meg. Nem volt olyan, aki távozott
és újra visszatért. Bármely kettő találkozott egymással a lakásban. Igaz-e, hogy
volt olyan időpont, amikor mindnyájan egyszerre Kovácséknál voltak?
1931. [6] [Helly tétele.] Mutassuk meg, hogy ha a sík n darab konvex halmaza közül
bármely háromnak van közös pontja, akkor az összesnek is van.
1932. [4] Mutassuk meg, hogy a sík bármely n pontja között a minimális távolság 3n-nél
kevesebbszer fordul elő.
1933. [5] Mutassuk meg, hogy a sík bármely n pontja között a maximális távolság leg-
feljebb n-szer léphet fel.
1934. [6] Mutassuk meg, hogy ha egy ponthalmazban minden távolság egész, és a pontok
nincsenek mind egy egyenesen; akkor a ponthalmazban csak véges számú pont
lehet.
1935. [5] [A Sylvester feladat.] Meg lehet-e adni a síkon n pontot úgy, hogy azok ne
legyenek mind egy egyenesen, és közülük bármely kettőn áthaladó egyenes
tartalmazzon még egy további pontot is a megadottak közül?
170 Feladatok

1936. [7] Ha a sík n pontja között nincs három, mely egy egyenesre esne, és megraj-
zoljuk minden ponthármas köré a rájuk illeszkedő kört, akkor legfeljebb hány
egységsugarú kört rajzoltunk?
Jelölje az egységsugarú

körök maximális számát m. Mutassuk meg, hogy
2 n
c·n ≤m≤
3/2
, ahol c egy pozitív konstanst jelöl.
3 2
1937. [5] [Izoperimetrikus probléma.] Adott kerületű síkidomok közül melyiknek legna-
gyobb a területe?

33. Gráfok

1938. [4] Néhány csapat körmérkőzést játszik (mindenki mindenkivel egy mérkőzést).
Minden csapatnak van győzelme, s döntetlen nem volt. Mutassuk meg, hogy
van három olyan csapat: A, B és C, hogy A legyőzte B-t, B legyőzte C-t és C
legyőzte A-t.
1939. [4] 8 csapat körmérkőzést játszott, döntetlen nem volt. Bizonyítsuk be, hogy van
köztük négy olyan A, B, C és D csapat, hogy A legyőzte B-t, C-t, D-t; B
legyőzte C-t és D-t; végül C legyőzte D-t.
1940. [4] Két csapat teniszmérkőzést vívott úgy, hogy az egyik csapat minden játékosa
egy mérkőzést játszott a másik csapat minden játékosával. A végén megállapí-
tották, hogy mindenkinek volt veresége is, győzelme is. Igazoljuk, hogy akkor
van négy olyan játékos, akik „körbe verték” egymást.
1941. [4] Bizonyítsuk be, hogy ha egy társaságban mindenki mindenkivel egy pingpong
mérkőzést játszott, akkor van a társaságnak olyan A tagja, hogy a társaság bár-
mely további B tagjára a következő két állítás közül legalább az egyik igaz:
(a) A legyőzte B-t.
(b) Van a társaságnak olyan C tagja, hogy A legyőzte C-t és C legyőzte B-t.
1942. [4] Egy labdarúgó-bajnokságon 18 csapat vesz részt. Bizonyítsuk be, hogy a nyol-
cadik forduló után még biztosan van három olyan csapat, melyek közül seme-
lyik kettő nem játszott egymással.
1943. [5] Bizonyítandó, hogy egy pingpong körmérkőzés résztvevői sorbaállíthatók úgy,
hogy mindenki mögött olyasvalaki álljon, akit legyőzött.
1944. [5] Egy ismerkedési est résztvevői úgy álltak sorba, hogy az első és az utolsó ember
kivételével mindenkiről tudjuk, hogy az előtte álló emberek közül ugyanannyit
ismer, mint a mögötte állók közül. Bizonyítsuk be, hogy a két szélsőnek ugyan-
annyi ismerőse van.
33. Gráfok 171

1945. [5] Lehet-e úgy ismeretségeket szervezni n személy között, hogy ne legyen 3 fő,
akinek ugyanannyi ismerőse van?
1946. [4] Egy üdülő bármely 3 lakója között van kettő, akik nem ismerik egymást, de
bármely hét között van legalább kettő, akik ismerik egymást. Az üdülés befe-
jeztével mindenki megajándékozza minden ismerősét egy-egy ajándéktárggyal.
Bizonyítsuk be, hogy n nyaraló esetén legfeljebb 6n tárgy kerül ajándékozásra.
1947. [4] Egy iskolában 100 diák tanul. Közülük bárhogyan választva 4-et, ezek közül
legalább egy ismeri a többi hármat. Bizonyítsuk be, hogy van olyan diák, aki
ismeri a többi 99 diákot. Legalább hány olyan tanuló van, aki ismeri az összes
többit?
1948. [4] Egy iskola 90 végzős diákja közül mindegyiknek legalább 10 barátja van ebben
a társaságban. Bizonyítsuk be, hogy bármelyik meghívhat vendégségbe három
másikat úgy, hogy a négy résztvevő mindegyikének legalább két barátja is jelen
legyen.
1949. [4] Egy társaságban bárkinek van ismerőse; és tudjuk, hogy ha két embernek azonos
számú ismerőse van, akkor nincs közös ismerősük. Mutassuk meg, hogy van,
aki csak egy embert ismer a társaságból.
1950. [4] Egy n tagú (n > 3) társaságról tudjuk, hogy ha mindannyian együtt vannak,
akkor bárki is megy ki a szobából, a többieknek ugyanannyi ismerőse marad
a szobában. Mutassuk meg, hogy a társaságban vagy senki sem ismer senkit,
vagy mindenki ismer mindenkit.
1951. [4] Egy baráti összejövetelen, ahol legalább 3 házaspár volt jelen; bármely 3 há-
zaspárból vagy az asszonyok, vagy a férjek ismerik egymást. Elhelyezhetők-e
a párok két teremben úgy, hogy az első teremben bármely két asszony, a másik
teremben bármely két férj ismerje egymást?
1952. [4] Egy rendezvényen 2n személy vesz részt. Mindenki legalább n másikat ismer,
s az ismeretségek kölcsönösek. Mutassuk meg, hogy kiválasztható közülük 4
személy, akik úgy körbeállíthatók, hogy mindenki ismerje a szomszédjait.
1953. [4] Egy 9 tagú társaságban bármely 3 ember között van két olyan, akik ismerik egy-
mást. Bizonyítsuk be, hogy ekkor van közöttük 5 olyan, hogy mindegyiküknek
legalább 4 ismerőse van.
1954. [5] Egy 9 tagú társaságban bármely 3 ember között van két olyan, akik ismerik
egymást. Bizonyítsuk be, hogy ekkor van közöttük 4 olyan, akik páronként
ismerik egymást.
1955. [5] Egy 11 fős társaságból bárhogyan is választunk ki kettőt, a többiek közül pon-
tosan egy ismeri mindkettőjüket. Mutassuk meg, hogy van köztük olyan, aki
mindenki mást ismer.
172 Feladatok

1956. [6] Egy társaságban mindenki legalább k másikat ismer. Mutassuk meg, hogy a
társaság tagjai közül legalább (k +1)-et le tudunk ültetni egy kör alakú asztalhoz
úgy, hogy mindenki ismerje a szomszédait.
1957. [4] Egy ország minden városát vagy hajó-, vagy repülőút köti össze. Bizonyítsuk
be, hogy vagy hajóval, vagy repülővel bejárható az egész ország.
1958. [5] Legfeljebb hány kézfogásra kerülhetett sor egy 20 tagú társaságban, ha tudjuk,
hogy akárhogyan is választunk ki a társaságból három embert, azok között biz-
tosan akad kettő, akik nem fogtak kezet?
1959. [4] Barna úrhoz és feleségéhez három házaspár érkezett vendégségbe. Az örömteli
találkozás alkalmából egyesek kezet is fogtak egymással. (Senki sem fogott
kezet a saját feleségével.) Később Barna úr megkérdezte mindenkitől, hogy
hány jelenlévővel fogott kezet. Kérdésére, amelyet hétszer tett fel, hiszen rajta
kívül heten voltak a szobában, csupa különböző választ kapott. Hány vendéggel
fogott kezet Barnáné?
1960. [4] A Kovács házaspárhoz négy házaspár érkezett vendégségbe. A bemutatkozások
után Kovács házigazda megállapította, hogy rajta kívül a többi kilenc jelenlevő
mindegyike más-más számú személlyel ismerkedett össze.
Hány ember mutatkozott be Kovácsnénak?

34. Halmazrendszerek

1961. [3] Mutassuk meg, hogy egy n elemű halmaz részhalmazainak száma 2n .
1962. [4] A1 , A2 , . . . , Am ⊂ H, |H| = n, és Ai ∩ Aj = ∅.
Mutassuk meg, hogy m ≤ 2n−1 . (|H| jelöli a H halmaz elemeinek számát.)
1963. [5] Legyenek A1 , A2 , . . . , Am egy n-elemű X halmaz részhalmazai. Tegyük fel,
hogy közülük bármely kettőnek van közös eleme, és m < 2n−1 . Bizonyítsuk be,
hogy ekkor X-nek létezik olyan részhalmaza, amely nem azonos egyik Ai -vel
sem, de bármelyikkel van közös eleme.
1964. [7] S = {1, 2, . . . , n}; A1 , A2 , . . . , Am ⊂ S, és Ai ∩ Aj tartalmaz két szomszédos
elemet minden i, j indexpárra. Mutassuk meg, hogy m ≤ 2n−2 .
1965. [7] A1 , A2 , . . . , Am ⊂ H, |H| = n, és Ai ∩ Aj = ∅, valamint Ai ∪ Aj = H.
Mutassuk meg, hogy m ≤ 2n−2 .
1966. [4] A1 , A2 , . . . , Am ⊂ H, |H| = n, és Ai ∪ Aj = H, ha i = j .
Mutassuk meg, hogy m ≤ n.
1967. [4] A1 , A2 , . . . , Am ⊂ H, |H| = n, és egyik Ai -t sem tartalmazza a többi részhal-
maz uniója. Mutassuk meg, hogy m ≤ n.
34. Halmazrendszerek 173

1968. [5] Legfeljebb hány, nem üres részhalmaz választható ki egy 100 elemű halmazból
úgy, hogy bármely két kiválasztott részhalmaz vagy diszjunkt legyen, vagy az
egyik tartalmazza a másikat?
1969. [6] [Ray-Chaudhuri–Wilson tétel, 1975] A1 , A2 , . . . , Am ⊂ H, |H| = n, és
k+1

n
|Ai ∩ Aj | ≤ k, ha i = j . Mutassuk meg, hogy m ≤ .
i=0
i

1970. [2] A1 , A2 , . . . , Ak halmazok végesek.


1
Mutassuk meg, hogy |A1 ∩ A2 ∩ . . . ∩ Ak | ≤ (|A1 | + |A2 | + · · · + |Ak |).
k
1971. [4] A1 , A2 , . . . , Ak halmazok olyanok, hogy bármely kettő metszete egy elemű, s
bármely három halmaz közös része üres.
k(k − 1)
Mutassuk meg, hogy |A1 ∪ A2 ∪ . . . ∪ Ak | ≥ .
2
1972. [5] A1 , A2 , . . . , Am ⊂ H, |H| = n és ezekből a halmazokból bármely kettő met-
szete nem üres, s bármely három halmaz metszete " üres. Jelölje#m lehetséges

1 + 8n + 1
legnagyobb értékét M. Mutassuk meg, hogy M = .
2

1973. [4] Mutassuk meg, hogy egy 9 elemű halmaz bármely négy 7 elemű részhalmazá-
nak közös része nem üres.
1974. [4] A1 , A2 , . . . , An+1 ⊂ H, |H| = n. Mutassuk meg, hogy kiválasztható közülük
néhány úgy, hogy ezek uniója megegyezik a megmaradt halmazokból néhány-
nak az uniójával.
1975. [5] A1 , A2 , . . . , An+1 ⊂ H, |H| = n (n ≥ 5), |Ai | = 3, i = 1, 2, . . . , n. Mutassuk
meg, hogy kiválasztható közülük két halmaz, melyeknek pontosan egy közös
eleme van.
1976. [5] Adott 1978 halmaz, melyek mindegyike 40 elemű, és bármely két halmaznak
van közös eleme. Mutassuk meg, hogy van olyan elem, amely mindegyik hal-
mazban benne van.
1977. [5] Adott 1985 halmaz, melyek mindegyike 45 elemű, és bármely kettő uniója 89
elemű. Hány elemből állhat mind az 1985 halmaz uniója?
1978. [6] Legyenek H1 , H2 , . . . , H50 a |H| véges halmaz olyan részhalmazai, amelyekre
1
|Hi | > |H|, minden 1 ≤ i ≤ 50-re. Bizonyítsuk be, hogy létezik a H-nak
2
öt olyan h1 , h2 , . . . , h5 eleme, hogy valamennyi Hi részhalmaz tartalmaz ezek
közül legalább egyet.
174 Feladatok

1979. [6] Legyenek H1 , H2 , . . . , H1066 a |H| véges halmaz olyan részhalmazai, amelyek-
1
re |Hi | > |H|, minden 1 ≤ i ≤ 1066-ra. Bizonyítsuk be, hogy létezik a H-nak
2
tíz olyan h1 , h2 , . . . , h10 eleme, hogy valamennyi Hi részhalmaz tartalmaz ezek
közül legalább egyet.
1980. [5] Egy bizottság 40-szer ülésezett. Mindegyik ülésen 10 fő volt jelen. A bizottság
bármely két tagja legfeljebb egy ülésen vett együtt részt. Bizonyítsuk be, hogy
a bizottság legalább 64 tagból állt.
1981. [5] Egy osztályban bármely két gyerek jár közös szakkörbe, de mindenki legfel-
jebb két szakkörnek a tagja. Bizonyítsuk be, hogy van olyan szakkör, ahová az
osztálynak legalább a kétharmad része jár.
1982. [5] Egy nemzetközi konferencián 9 tudós vesz részt. Egyikük sem beszél háromnál
több nyelvet; és közülük bármely három között van kettő, akik beszélnek kö-
zös nyelvet. Bizonyítsuk be, hogy van olyan nyelv, amelyen legalább hárman
beszélnek.
1983. [5] Egy bank páncélszekrényén több különböző zár található. Kulcsaikat úgy osz-
tották szét a bank négy pénztárosa között, hogy a páncélszekrény kinyitásához
legalább hármójuknak jelen kell lenni (de mind a négynek nem szükséges), hogy
a náluk levő kulcsokkal ki lehessen nyitni az összes zárat. (Egy zárhoz többük-
nél is lehet kulcs, és egy embernél többféle kulcs is lehet.) Legkevesebb hány
zár van a páncélszekrényen?
1984. [5] Egy nemzetközi bizottság 5 tagból, 5 állam 1–1 képviselőjéből áll. Azokat az
iratokat, amelyeken a bizottság dolgozik, páncélszekrényben őrzik. Hány zárjá-
nak kell lennie a szekrénynek, és hány kulccsal kell ellátni a bizottság minden
egyes tagját, hogy az iratokhoz csak akkor lehessen hozzáférni, ha a bizottság-
nak legalább három, tetszés szerinti tagja együtt van?
1985. [5] Tíz rabló egy többzáras ládában őrzi a kincsét. Minden rablónak bizonyos zá-
rakhoz van kulcsa. A kulcsok úgy vannak elosztva, hogy semelyik három rabló
se tudja a nála levő kulcsokkal kinyitni a ládát, de bármely négy közülük már
hozzá tud férni a kincshez. Legalább hány zár szükséges a fenti feltételek telje-
süléséhez?
1986. [5] A H véges halmaznak kijelöltük néhány valódi részhalmazát. Tudjuk, hogy
a) a halmaz bármely két különböző eleme pontosan egy kijelölt részhalmazban
van benne;
b) bármely kijelölt R részhalmazhoz és abban nem lévő p elemhez pontosan
m darab, p-t tartalmazó és R-t metsző kijelölt részhalmaz, és pontosan n
darab, p-t tartalmazó és R-t nem metsző kijelölt részhalmaz van.
Fejezzük ki m és n segítségével H elemeinek és kijelölt részhalmazainak szá-
mát.
34. Halmazrendszerek 175

1987. [5] Igazoljuk, hogy elég sok (egy bizonyos számúnál több) különböző halmaz közül
mindig kiválasztható 2000 db halmaz úgy, hogy semelyikük se álljon elő másik
kettő egyesítéseként.
1988. [5] Mutassuk meg, hogy egy n-elemű halmaz részhalmazaiból készíthető párok
3n + 1
közül olyan van, melyben a két részhalmaz metszi egymást.
2
1989. [5] Mutassuk
meg, hogy ha az n-elemű H halmaz összes részhalmazát tekintjük,
akkor |Ai ∩ Aj | = n · 4n−1 . (Az összegzésben i = j is megengedett.)
i,j

[5]
1990. Az A halmaz minden X részhalmazához hozzárendeljük az f (X) részhalmazt
úgy, hogy ha X ⊆ Y , akkor f (X) ⊆ f (Y ). Mutassuk meg, hogy van A-nak
olyan T részhalmaza, amelyre f (T ) = T .
1991. [5] Jelölje Tn egy n elemű halmaz különböző partícióinak (részhalmazokra bontá-
sainak) számát. Bizonyítsuk be, hogy Tn ≤ n!.
1992. [5] Egy adott n természetes számra legyen Mn = {1, 2, . . . , n}. Az Mn valamely
T részhalmaza „kövér”, ha a T egyetlen eleme sem kisebb, mint T elemeinek
száma, és van olyan eleme, amely megegyezik a T halmaz elemeinek számával.
Az Mn „kövér” részhalmazainak számát jelölje f (n). f (11) =?
1993. [7] [Sperner-tétel, 1928] A1 , A2 , . . . , Am ⊂ H, |H| = n és ezekből a halmazokból
egyik
sem
tartalmazza részként valamely másikat. Mutassuk meg, hogy m ≤
n
≤ n .
[2]
1994. [7] [Erdős–Ko–Radó tétel, 1961] Ha A1 , A2 , A3 , . . . , Am egy n elemű H halmaz
olyan k-elemű részhalmazai

(k < n/2), hogy bármely kettőnek van közös ele-
n−1
me, akkor m ≤ .
k−1
1995. [7] Ha A1 , A2 , . . . , Am ⊂ H; B1 , B2 , . . . , Bm ⊂ H, |H| = n, |Ai | = k, |Bi | = l,
i = 1, 2, . . . + l ≤ n, és Ai ∩ Bj = ∅ pontosan abban az esetben, ha i = j ;
, m, k

k+l
akkor m ≤ .
l
1996. [5] Bizonyítsuk be, hogy az első 10 000 természetes szám halmazának kijelölhe-
tő 16 részhalmaza úgy, hogy a 10 000 egész szám bármelyike előáll alkalmas
kijelölt 8 részhalmaz metszeteként.
1997. [7] Nevezzük független metszőrendszernek az n-elemű H halmaz A1 , A2 , . . . , Am
részhalmazait, ha
(1) ∀x ∈ H előáll néhány Ai metszeteként;
(2) Az A1 , A2 , . . . , Am rendszerből bármely Ai -t elhagyva (1) nem teljesül.
176 Feladatok

Mutassuk meg, hogy A1 , A2 , . . . , Am pontosan akkor független metszőrendszer,


ha A1 , A2 , . . . , Am is független metszőrendszer.
1998. [7] Mutassuk meg, hogy ha A1 , A2 , . . . , Am az n-elemű H halmazon független met-
szőrendszer, akkor c1 · log2 n ≤ m ≤ c2 · n2 ; és ezek a korlátok nagyságrend-
jükben pontosak. (c1 és c2 pozitív konstansokat jelölnek.)
1999. [7] A1 , A2 , . . . , Am ⊂ H, |H| = n, |Ai | = 3; i = 1, 2, 3, . . . , m és |Ai ∩ Aj | ≤ 1,
ha i = j . Mutassuk
√ meg, hogy van olyan M ⊂ H, hogy minden i-re Ai ⊂ M
és |M| ≥ [ 2n].
2000. [7] Legyenek n és k adott természetes számok. Legyen A véges halmaz. Tegyük
n(n + 1)
fel, hogy |A| ≤ . Legyenek Ai (i = 1, 2, . . . , n + 1) olyan halmazok,
k+1
n+1
melyekre |Ai | = n (1 ≤ i ≤ n + 1), |Ai ∩ Aj | ≤ k (i = j ), A = ∪ Ai .
i=1
Határozzuk meg A elemeinek számát.
Megoldások, útmutatások

1. Fejtörők

1. A busz eleje balra néz, mert az ajtók a túlsó oldalon vannak. (Ez a hazai viszo-
nyokra értendő, pl. Angliában ez másképp van.)
2.

3. A csúcsokat tükrözzük a háromszög középpontjára.


4.

5. A vadász az Északi–sarkon indult el a nyomok után (esetleg a Déli–sark köze-


lében), mert máshonnan indulva a megadott útvonalon nem érkezhetne vissza a
kiindulási helyre. Ha az Északi–sarkon lakik a medve, akkor az fehér színű.
6. A négy hajó mindegyike nem lehet egy síkban (például a tenger felszínén). A
négy hajó egy szabályos tetraéder csúcsaiban helyezkedik el, így a negyedik hajó
tengeralattjáró, esetleg léghajó.
178 Megoldások, útmutatások

7. 323 935 = 5 · 17 · 37 · 103. A kapitány 37 éves és a kisfia 5 éves (esetleg 1 éves).


8.

9. 101 − 102 = 1
10. 1975 + 146 = 210 + 1911.
A feladat igazi, „szép” megoldása: 1995 + 146 = 210 + 1117.
11. 72 = 49. (A 6-ost megfordítva, 9-esként helyeztük el.)
12. A 9-es és a 8-as kártyákat cseréltük meg, és a 9-es kártyát megfordítva, 6-osként
tettük le, így az összeg mindkét oszlopban 18.
13. (Deriválás.)

14. Mindegyik oszlopban az első két számból a nagyobbat írtuk a harmadik helyre.

12 15 30 17 26 22 30
24 20 25 20 13 28 18
24 20 30 20 26 22 30
15. Az ábrán az 1, 2, 3, 4, 5 számokat látjuk „visszatükrözött” alakjuk-
kal együtt, így a sorozat következő eleme:

16. a) 23 (a prímszámok sorozata).


b) 5040 (1, 1 · 2 = 2, 1 · 2 · 3 = 6, 1 · 2 · 3 · 4 = 24, . . . )
c) 89 (ez a Fibonacci–sorozat, a sorozat mindegyik eleme az őt megelőző két
szám összege).
1. Fejtörők 179

d) 127 (a sorozat következő elemét úgy kapjuk, hogy az utolsó számhoz hozzá-
adjuk a szám jegyeinek összegét).
e) 49 (a négyzetszámok sorozata).
f) 8 (7 · 7 = 49, 4 · 9 = 36, 3 · 6 = 18, 1 · 8 = 8).
17. A 8-at írtuk fel 2-es, 3-as, 4-es, 5-ös és 6-os számrendszerben (1000, 22, 20, 13,
12), így a sorozat következő eleme a 8 hetes számrendszerben felírt alakja: 11.
18. A 111, 213, 141, 516, 171, 819, 202, 122, . . . sorozat valójában a 11, 12, 13,
14, 15, 16, 17, 18, 19, 20, 21, 22, . . . sorozat, csak a számjegyek tagolása,
csoportosítása másképp történt.
19. A sorozat első eleme 1db 1-es, ezért a második elem: 11; itt 2 db 1-est írtunk le,
így a következő elem: 21; és így tovább. A felírt sorozat soron következő eleme:
1113213211.
20. 42 = 16 → 4 16 → 461, 52 = 25 → 5 25 → 552, 62 = 36 → 6 36 → 663,
72 = 49 → 7 49 → 794, 82 = 64 → 8 64 → 846.
A következő elem: 92 = 81 → 9 81 → 918.
21. A 0, 1, 2, 3, 4, 5, 6, . . . sorozat elemeinek első betűit felsorolva kapjuk a meg-
adott sorozatot.
22. A sakktábla alapsorában álló bábuk (bástya, huszár, futó, vezér, király, . . . ) ne-
vének első betűit adtuk meg. A hiányzó betűk: F , H , B.
23. A válasz: mennyország. (A szavakat egészítsük ki sorszámmal: elsősegély, máso-
dik otthon, harmadik világ, negyedik birodalom, ötödik hadoszlop, hatodik érzék,
hetedik mennyország.)
c
24. A táblázatban bárhol választunk négy egymás melletti a b számot, igaz
d
rájuk az ab = cd + 1 összefüggés.
25. 79479479.
26. 41312432.
27. 12132003, 30023121, 23121300 vagy 31213200.
180 Megoldások, útmutatások

28. 29.

7 15 2 1 2 3

3 8 13 5 4 7

14 1 9 8 6 9

30.

1 5 4
6 2 9
7 8 3

31.
20 20 20
9 11 8 12 9 11
6 3 8 5 3 9 5 4 7
4 2 1 7 4 1 2 7 2 3 1 6

32.
1. Fejtörők 181

33.
2
1 2
1 1

{1 1

{1 {1
{2 {1
{2
34. 3 2 2 4 3
74 64 46 35 45
+ 985 + 987 + 987 + 987 + 978
1062 1053 1035 1026 1026
35. 627 219 341 154 317 216 215 318 235
+ 354 + 438 + 586 + 782 + 628 + 738 + 748 + 654 + 746
981 657 927 936 945 954 963 972 981
384 482 182 391 269 192 162 654 546
+ 192 + 157 + 367 + 435 + 514 + 546 + 783 + 129 + 381
576 639 549 826 783 738 945 783 927
439 168 571 295 129 348 735 234
+ 128 + 327 + 293 + 173 + 357 + 219 + 186 + 657
567 495 864 468 486 567 921 891
36. 495
+ 459
954
37. 6174 4950
+ 1467 + 4590
7641 9540
38. 123456789 + 864197532 = 987654321.
39. A törtkifejezésben 10 különböző betű van, tehát mind a 10 számjegyet használ-
nunk kell, így a 0-t is. A 0 nem kerülhet a nevezőbe, csak a számlálóban állhat.
Ezért a számlálóban levő szorzat értéke 0, emiatt a tört értéke is 0 lesz.
40. a) 9567 b) 9486 9376 c) 6054 7894
+ 1085 + 1076 és + 1086 1720 és 1038
10652 10562 10462 + 9734 + 2054
17508 10986
182 Megoldások, útmutatások

d) 932 e) 26811 f) 29786


+ 9338 + 4708 850
10270 31519 + 850
31486
g) 627 h) 79 · 19 i) 103 · 273
627 79 206
746 711 721
+ 746 1501 309
2726 28119
41. a) 97 · 11 19 · 53 b) 99 · 91 c) 12 · 79
97 és 95 891 84
97 57 99 108
1067 1007 9009 948
42. a) 775 · 33 b) 325 · 777
2325 2275
2325 2275
25575 2275
252525
43. a) 286 · 379 b) 339 · 268 c) 285 · 39
858 678 855
2002 2034 2565
2574 2712 11115
108394 90852
44. (2 : 3) : (4 : 5 : 6) = 5.
1 6
45. 1 : 2 : 3 : 4 : 5 : (6 : 7 : 8 : 9 : 10) = : = 7.
2 · 3 · 4 · 5 7 · 8 · 9 · 10
46. 1 + 2 + 3 − 4 + 5 + 6 + 78 + 9 = 100, 1 + 2 + 34 − 5 + 67 − 8 + 9 = 100,
1 + 23 − 4 + 56 + 7 + 8 + 9 = 100, 1 + 23 − 4 + 5 + 6 + 78 − 9 = 100,
12 + 3 + 4 + 5 − 6 − 7 + 89 = 100, 12 + 3 − 4 + 5 + 67 + 8 + 9 = 100,
123 + 4 − 5 + 67 − 89 = 100, 123 − 4 − 5 − 6 − 7 + 8 − 9 = 100,
123 + 45 − 67 + 8 − 9 = 100, 123 − 45 − 67 + 89 = 100.
47. 98 − 76 + 54 + 3 + 21 = 100, 98 + 7 − 6 + 5 − 4 + 3 − 2 − 1 = 100,
98 − 7 + 6 − 5 + 4 + 3 + 2 − 1 = 100, 98 − 7 − 6 − 5 − 4 + 3 + 21 = 100,
9 + 8 + 76 + 5 + 4 − 3 + 2 − 1 = 100, 9 − 8 + 76 + 54 − 32 + 1 = 100.
48. 9 + 8 + 7 + 65 + 4 + 3 + 2 + 1 = 99, 9 + 8 + 7 + 6 + 5 + 43 + 21 = 99.
49. {1, 14, 15}, {2, 10, 12}, {3, 6, 9}, {4, 7, 11}, {5, 8, 13}.
50. {1, 4, 10, 13}, {2, 3, 11, 12}, {5, 6, 7, 8, 9}.
    
51. 10, 11, 12, . . . , 18, 19 40, 41, 42, . . . , 48, 49 és 20, 21, 22, . . . , 39 .
1. Fejtörők 183

52. 51, 52, 53, . . . , 100.


53. 51, 52, 53, . . . , 100 vagy 1, 3, 5, 7, . . . , 99.
54. 123121321.
55. A legnagyobb hányadost a 100, 200, . . . , 900 számok adják, a legkisebb hánya-
dost pedig a 199 szám adja.
56. 599 . . . 999 (221 db 9-es).
57. 558.
58. 9831.
59. 720 = 6 · 5 · 4 · 3 · 2 · 1 = 10 · 9 · 8.
60. 1, 7, 9, 10, 8, 11, 2, 5, 3, 4, 6, 12.
61. 4163 vagy 6314.
62.

vagy vagy

63. A négyszög egy oldalán legfeljebb két


metszéspont lehet, ezért a metszéspon-
tok száma legfeljebb 8.
184 Megoldások, útmutatások

64.

vagy

65.

66.

67.

68. Egy 200-jegyű szám számjegyösszege legfeljebb 200 · 9 = 1800 lehet. Vidor
tehát egy 1800-nál nem nagyobb számot tudott meg Tudortól, s Vidor ennek a
számnak a számjegyösszegét súgta meg Morgónak. Az 1800-nál nem nagyobb
1. Fejtörők 185

számok közül a 999-nek van a legnagyobb számjegyösszege. Tehát Morgó 27-


nél nem nagyobb számot tudott meg Vidortól, ám ezek közül csak a 19-nek a
számjegyösszege kétjegyű.
Így Vidor által Morgónak súgott szám a 19, Morgó 1+9 = 10-et súgott Hapcinak,
Hapci pedig 1 + 0 = 1-et súgott Kukának.
69. Az öntögetés befejeztével mindegyik pohárban 1-1 dl folyadék lesz. Ezért ameny-
nyi bor hiányzik a boros pohárban az 1 dl-ből, annak a helyén víz van, tehát a
boros pohárból hiányzó bor mennyisége egyenlő a vizes pohárból hiányzó víz
mennyiségével. Ez azt jelenti, hogy ugyanannyi víz van a boros pohárban, mint
amennyi bor van a vizes pohárban.
70. Nem. Készítsük el a családfát.
71. Okoskodjunk „visszafelé”. A harmadik átmenetel előtt 12 krajcárja volt. Előzőleg
fizetett 24 krajcárt, tehát a második átmenetel után 36, előtte 18 krajcárja volt.
Ezért az első átmenetel után 18 + 24 = 42 krajcárja volt, s kezdetben 21 krajcár
volt a zsebében.
72. Okoskodjunk „visszafelé”. A játékosoknak az ötödik játszma után (128, 128,
128, 128, 128) Ft-ja volt, a negyedik játszma után (64, 64, 64, 64, 384) Ft-ja,
a harmadik után (32, 32, 32, 352, 192) Ft-ja, a második után (16, 16, 336, 176,
96) Ft-ja, az első után (8, 328, 168, 88, 48) Ft-ja volt, így eredetileg (324, 164,
84, 44, 24) Ft-juk volt.
73. Elérhető, hogy 9 szék legyen foglalt (10 nyilván nem lehet). Bemegy az első, leül
az első székre. Bemegy a második, leül a harmadik székre. Bejön a következő,
leül a második székre, és kimegy a harmadik széken ülő személy. Valaki ismét
bemegy, leül a negyedik székre. Jön a következő, leül a harmadik székre, és
kimegy az, aki a negyedik széken ül. Ekkor már az első három szék mindegyikén
ülnek. Az eljárást így folytatjuk.
74. Válasszuk a P F feliratú ládát. Ha például pirosat húzunk, akkor az a láda P P
(P F nem lehet, hisz hamis a felirat, és F F nyilván szóba sem jöhet). Ekkor a
P P ládikóban P F nem lehet, mert akkor az F F ládában csak F F lehetne, de
ekkor ez a felirat nem lenne hamis. Tehát a P P ládikó valójában F F , az F F
láda pedig P F . Hasonlóan tárgyalható az az eset, amikor a P F feliratú ládából
fehér színű golyót húzunk.
75. Az állítások mindegyike mást-mást állít, ezért nem lehet közöttük két igaz állítás:
vagy egy állítás igaz, vagy egy sem. Ha nincs igaz állítás, akkor 5 állítás lenne
hamis, de ekkor — ellentétben feltevésünkkel —, az ötödik állítás igaz lenne.
Egy lehetőség marad: az állítások között 1 igaz van, azaz 4 állítás hamis, s
valóban, a negyedik állítás igaz.
76. Az első két állítás hamis, a következő három igaz.
186 Megoldások, útmutatások

77. Az A-n és a C-n levő állítások ellentétesek, így egyikük igaz. Ezért a B-n levő
állítás csak hamis lehet. B-ben van az arany.
78. Ha az arany a C ládikóban van, akkor mindhárom állítás igaz; ha a B ládikóban
lenne az arany, akkor mindhárom állítás hamis lenne. Az A ládikóban van az
arany, s ekkor az első két állítás igaz, a harmadik hamis.
79. Csak az A és az 1 feliratú kártyákat kell megfordítani.
80. a) A igaz, B hazug; b) A hazug, B hazug;
c) A hazug, B igaz, C hazug; d) C igaz.
81. A kérdés: „Te melyik utat szoktad Mekkába vezető útként mutatni?”. Az igaz
erre a kérdésre a helyes utat mutatja, és a hazug (mivel a rossz utat szokta mu-
tatni, és ezt ebben a válaszban hamisan kell mondania) is a helyes utat mutatja.
(Kérdezheti azt is, hogy: „A társad melyik utat mutatná Mekkába vezető útként?”
Erre mindegyik a rossz utat mutatja.)
82. Egy jó kérdés: „Ebben a városban lakik Ön?”. Ha a hazugok városában van
és igazmondót kérdez, az Nem-mel válaszol, és Nem választ kap akkor is, ha
hazugot kérdez. Az igazmondók városában erre a kérdésre Igen választ kap.
Tehát ha a kérdésre Igen-nel válaszolnak, akkor az igazmondók városában, ha
Nem-mel válaszolnak, akkor a hazugok városában van a turista.
83. B igazat mond, A és C hazudik.
84. Tegyük fel az elején, hogy az (A) állítás igaz (később kiderül, hogy ez a felté-
telezés hamis). Következésképpen a (B) állítás is igaz volna, tehát legalább két
állítás: (A) és (B) igaz volna. Ez ellenkezik a (B) állításával, hogy az (A), (B),
(C), (D), (E) állítások közül legfeljebb egy igaz. Ellentmondásba ütköztünk, ezért
(A) nem lehet igaz, csak hamis.
Ha (A)-t hamisnak fogadjuk el, akkor (B)-t is annak kell elfogadnunk. Követ-
kezésképpen pedig (C)-nek is hamisnak kell lennie. A (B) állítás hamis voltából
következik, hogy az állítások közül legalább kettő igaz. Mivel (A), (B), (C) ha-
misak, ezért (D), (E) igaz.

2. Páros vagy páratlan?

85. Nem lehet. Öt páratlan szám összege páratlan, míg a 100 páros.
86. Nem lehet. Ha a szorzatuk páratlan, akkor mind a négy szám páratlan, s ekkor
összegük páros.
87. Ha a három szám összege 1994, akkor van köztük páros szám, ezért szorzatuk
páros lesz, így a szorzat nem végződhet 1-re.
3. Párbaállítás 187

88. Nem lehet, mert az 1, 2, 3, . . . , 1994 számok összege páratlan, és ez az összeg


nem állítható elő két páratlan szám összegeként.
89. 3, 3, 1, 1, 1 vagy 9, 1, 1, −1, −1.
90. Lehet, pl. 9, 1, 1, 1, 1, −1, −1, −1, −1.
91. Nem. Ha a tíz szám szorzata 10, akkor közülük az egyik páros, s a többi páratlan.
Azonban kilenc páratlan és egy páros szám összege páratlan.
92. Lehet, pl. 4, 2, 1, 1, −1, −1, 1, 1.
93. Nem, mert ha a szorzat 9, akkor mind a kilenc szám páratlan, így összegük
páratlan.
94. Nem lehet, hasonló okok miatt, mint a 91. feladatban.
95. Lehet, pl. 4, 2, −1, −1, −1, −1, −1, −1.
96. Nem lehet. Tekintsük ugyanis az (a1 − 1), (a3 − 3), (a5 − 5), (a7 − 7) és (a9 − 9)
számokat! E számok egyike páros szám.
98. Legyen a vízszintes irányú vágások száma a, a függőleges irányú vágásoké pedig
b. a + b = 13, így a és b közül az egyik páratlan. Ha pl. a páratlan, akkor a
vízszintes csíkok száma a +1, tehát páros, s a részek párosságát már a függőleges
vágások nem változtatják meg.
99. Nem lehet, mert a hat páratlan szám összege mindig páros.
100. Nem lehet, mert az összeg mindig páros.
101. Ha a törteket közös nevezőre hozzuk, a számláló páros (100 db páratlan szám
összege), a nevező páratlan (100 db páratlan szám szorzata), így a tört értéke
nem lehet 1. 100 különböző egész szám reciprokának összege már lehet 1. Ez
megmutatható pl. teljes indukcióval, vagy felhasználhatjuk az 1040. feladat ered-
ményét.
103. Az egyik tényező páros.

3. Párbaállítás

104. a) 1 + 99 = 3 + 97 = · · · = 49 + 51, a végeredmény 2500.


5151
b) = 101.
51
188 Megoldások, útmutatások

105. a) 1 + (2 − 3 − 4 + 5) + (6 − 7 − 8 + 9) + · · · + (1990 − 1991 − 1992 + 1993) =


= 1 + 0 + 0 + · · · + 0 = 1.
b) Felhasználva az
(n + 1)2 + (n + 4)2 + (n + 6)2 + (n + 7)2 = (n + 2)2 + (n + 3)2 + (n + 5)2 + (n + 8)2
azonosságot, az a) feladathoz hasonlóan megmutatható, hogy itt is 1 az összeg
legkisebb pozitív értéke.
106. Az (a, 999 − a) párokban a két szám jegyeinek összege 27. A végeredmény:
499 · 27 + 27 + 1 = 13 501.
108. Ugyanannyi. A 4 és 5 lyukú lyukasztásokat párba lehet állítani.
109. A térrészekhez rendre hozzárendelhetjük a kocka egy-egy lapját, élét, csúcsát, s
még ott maradt a kocka belseje. A térrészek száma: 6 + 12 + 8 + 1 = 27.
110. Hasonló az előzőhöz.
111. Számoljuk meg a lapokhoz illeszkedő részeket! Minden laphoz a tetraéder 6 da-
rabkája csatlakozik, minden darabka csak egy lapra illeszkedik, ezért a darabo-
lással kapott részek száma 4 · 6 = 24.
112. A mérkőzések száma 1991, ugyanis minden mérkőzéshez tartozik egy kieső já-
tékos, s 1991 kieső játékos van.
113. A vágások száma mindig 59, ugyanis minden vágás 1-gyel növeli a részek szá-
mát, s a darabolás befejeztével 60 db kis négyzetünk van.
114. Állítsuk párba a szám osztóit: osztó–társosztó. Csak négyzetszámnál van, hogy
pontosan egy osztónak önmaga a társa. (Az n szám a osztójához b a társosztó,
ha a · b = n.)
Megjegyzés. Bebizonyítható, hogy ha n prímtényezős alakja n = p1α1 p2α2 . . . pkαk ,
akkor az n szám pozitív osztóinak száma d(n) = (α1 + 1)(α2 + 1) . . . (αk + 1).
115. Használjuk az előző feladat állítását.
116. Az a sorszámú ajtó marad nyitva, mely számnak páratlan számú osztója van. A
114. feladat szerint ezek a négyzetszámok.
117. A végeredmény: n50 . Párosítsuk az osztókat a társosztókkal.
118. 24 · 32 · 53 = 18 000.
119. Figyelembe véve, hogy az n tökéletes szám pozitív osztóinak összege 2n, a ke-
resett összeget n-nel szorozva, 2n-et kapunk. Tehát a kérdéses összeg értéke 2.
120. Ha a nevezőt n-nel szorozzuk, megkapjuk az osztók összegét (azaz a számlálót).
121–122. Indirekt úton bizonyítsunk, s használjuk az osztó–társosztó párosítást!
123. Párosítsuk az osztókat és a társosztókat!
3. Párbaállítás 189

124. Ha d páratlan osztója egy páros számnak, akkor 2d egy páros osztó. Tehát a
páros osztók összege legalább kétszerese a páratlan osztók összegének.
125–126. Használjuk fel, hogy ha (n, k) = 1, akkor (n, n − k) = 1.
127. Vegyük figyelembe, hogy ha (m, n) = 1, akkor (n − m, n) = 1. Ennek alapján
elég megmutatni, hogy azoknak az m számoknak a száma, amelyekre m < 2k
és (m, 4k) = 1, páros. Ehhez pedig mutassuk meg, hogy (m, 4k) = 1 akkor és
csak akkor, ha (m, 2k) = 1! Ám az olyan m < 2k számok száma, amelyekre
(m, 2k) = 1 — könnyen látható — mindig páros.
128. A lehetőségek száma ugyanannyi, mint ahányféle módon választhatunk öt szám-
jegyet ismétlődéssel az 1, 2, 3, . . . , 9 számjegyek közül.
 
n
129. , ugyanis a metszéspontok és 4–4 csúcs között kölcsönösen egyértelmű meg-
4
feleltetés létesíthető. (A metszéspontok egybe is eshetnek.)
 
n
130. , ugyanis a metszéspontok és 6–6 csúcs között kölcsönösen egyértelmű meg-
6
feleltetés létesíthető. (A háromszögek ponttá is zsugorodhatnak.)
132. Két vízszintes és két függőleges rácsegyenes kölcsönösen egyértelműen megha-
tároz egy téglalapot.    
7 11
A rácsegyenesek ilyen kiválasztásainak száma: · .
2 2
133. 216.
134. a) Nem lehet, mert a 6. lépésben páros számon állunk.
b) Lépéseink során 4-et kell jobbra, 3-at balra lépni,
 ezek sorrendje tetszőleges.
7
A 7 lépésből azokat, melyeket jobbra lépünk, -féleképp tehetjük meg.
4
135. A bástyának, míg céljához nem ér, 14 lépést kell megtennie; ebből 7 előre,
 7
14
fölfelé visz, ezek váltakozása tetszőleges lehet. A lehetőségek száma: .
7
136. a + b | a 3 + b3 , így pl. 1991 | 13 + 19903 .
137. A végeredmény: 1, ugyanis sin 0◦ = cos 90◦ , sin 1◦ = cos 89◦ , sin 2◦ = cos 88◦
stb.
138. sin2 1◦ + sin2 2◦ + sin2 3◦ + · · · + sin2 89◦ + sin2 90◦ = 45, 5.
Használjuk fel, hogy sin α = cos(90◦ − α) és sin2 x + cos2 x = 1.
Így sin2 1◦ + sin2 89◦ = sin2 1◦ + cos2 1◦ = 1,
sin2 2◦ + sin2 88◦ = sin2 2◦ + cos2 2◦ = 1, . . .
190 Megoldások, útmutatások

139. lg tg 45◦ = 0, így a bal oldalon 0 áll.


A jobb oldal másképp írva: lg( tg 1◦ · tg 2◦ · tg 3◦ ·. . .· tg 89◦ ), s a tangensek szorzata
1 (hiszen tg 1◦ · tg 89◦ = 1, tg 2◦ · tg 88◦ = 1 stb.), így ennek logaritmusa is 0.
140. Az első sorozatból tetszőleges számot választva, annak valamely számjegye után
beszúrunk egy nullát. Ez a megfeleltetés (a számjegy és a mögé beszúrt nul-
la között) kölcsönösen egyértelmű az első sorozatbeli számok számjegyei és a
második sorozatbeli számok 0 számjegyei között.
141. Legyen pl. n = 20, és a 20-at ábrázoljuk egy 20 egység hosszú mérőrúddal, rajta
a beosztások 0, 1, 2, . . . , 19, 20. A 20 bármely felbontásához egyértelműen
rendelhető a mérőrúd valamely feldarabolása (a rudat csak beosztás helyén lehet
eltörni). Pl. a 20 = 2 + 5 + 11 + 2 felbontáshoz a rúdnak az a feldarabolása, amikor
a rudat a 2, 7, 18 pontokban törjük el.
Annyi felbontás van, ahányféleképpen feldarabolhatjuk a rudat. Törni az 1, 2,
3, . . . , 19 pontokban lehet, azaz 19 helyen. Egy-egy helyen vagy törünk, vagy
nem; ezért a darabolás lehetőségeinek száma: 2 · 2 · 2 · . . . · 2 = 219 .
Az általános esetben a válasz: az n természetes számot 2n−1 módon bonthatjuk
fel pozitív egészek összegére.
142. Ahhoz, hogy a nagy téglatestet 330 db egységkockára vághassuk szét, 5 víz-
szintes, 10 függőleges és 4 elölről-hátra vágást kell végeznünk, azaz összesen
5 + 10 + 4 = 19 vágást.
Amikor a testátló az egyik egységkockából a másikba halad, a 19 sík közül egyet
mindig átdöf. Pontosan egyet és nem többet, mert 11, 6 és 5 relatív prímek.
(Ellenkező esetben fejtegetésünk nem lenne érvényes.)
Ezért a megkárosított kockák száma 20 lesz.
143. Egy-egy érdekes sorszámhoz párként választunk egy másik sorszámot úgy, hogy
az első három számjegy helyébe azokat a számjegyeket írjuk, amelyek ezeket a
jegyeket 9-re egészítik ki.
144. 0 ≤ n ≤ 999, ha n-hez 1000-et adunk, a szám jegyeinek összege 1-gyel nő. Az
összegben n és n + 1000 közül pontosan az egyik szerepel. Továbbá n és 999 −
− n közül pontosan az egyik szerepel az összegben. Így az összegben a 0, 1,
2, . . . , 999 számok fele szerepel, míg a megmaradó számokat 1000-rel növelve
megkapjuk az összeg 999-nél nagyobb tagjait, tehát az összeg értéke 0 + 1 + 2 +
+ · · · + 999 + 500 · 1000 = 999 500.
145. A táblára írt számokból újabbakat készítünk. Mindegyik számból n db újabb szá-
mot nyerünk úgy, hogy annak egy-egy jegyét kiválasztjuk, és az „ellenkezőjére”
változtatjuk (1 ↔ 2). Így nem kaphatunk olyan számot, mely már kezdetben is
fel volt írva; sőt olyat sem kaphatunk, melyet egy másik, a táblán levő számból
készítettünk. Ennek oka a feladatbeli feltétel: bármely két szám legalább három
helyen különbözik.
4. Miért nem négyzetszám? 191

Ha a táblán kezdetben m db szám szerepelt, akkor most, az újabb számok felírása


után a táblán levő számok száma: m(n + 1); és ez legfeljebb annyi, mint a csupa
1-esekből és 2-esekből álló n-jegyű számok száma: 2n . Azaz m(n + 1) ≤ 2n ,
2n
m≤ .
n+1

4. Miért nem négyzetszám?

Egy szám nem lehet négyzetszám, ha


(i) utolsó számjegye 2, 3, 7, 8;
(ii) osztható 3-mal, de 9-cel nem;
(iii) 3-mal osztva 2 maradékot ad;
(iv) 4-gyel osztva 2 vagy 3 a maradék;
(v) két szomszédos négyzetszám között van.
146. (i) miatt nem lehet.
147. (i) miatt nem lehet. Más indoklás: a szám osztható 7-tel, de nem osztható 49-cel.
148. (i) miatt nem lehet.
149. (ii) miatt nem lehet.
150. (i), ill. (v) miatt sem lehet.
151. (ii), ill. (v) miatt sem lehet. Más indoklás: a szám osztható 5-tel, de nem osztható
25-tel.
152. (ii), ill. (v) miatt sem lehet.
153. abab = 1010a +101b = 101·(10a +b), azonban 101 prím és 10a +b nem osztható
101-gyel.
155. abcabc = 1001 · abc = 7 · 11 · 13 · abc. Ezért, ha abcabc négyzetszám, akkor abc
osztható lenne 7 · 11 · 13 = 1001-gyel, de ez nem lehet.
156. abc + bca + cab = 111 · (a + b + c) = 3 · 37 · (a + b + c), s ez nem lehet négyzetszám,
mert a + b + c nem osztható 37-tel.
157. (ii) miatt nem lehet.
158. (iii) miatt nem lehet.
160. (iii) miatt nem lehet.
161. (ii) miatt nem lehet.
192 Megoldások, útmutatások

162. Ha ez a szám négyzetszám lenne, nem végződhetne 06-ra vagy 66-ra mert akkor
4-gyel osztva 2 maradékot adna; ezenkívül 60-ra sem végződhet. Tehát 00-ra
végződik. Ezt a számot 100-zal osztva ismét négyzetszámot kell kapnunk. A
kapott számra az előző okoskodást ismételhetjük. (A 600 db-os megkötésnek
a bizonyítás szempontjából nem volt szerepe.) Eszerint a feladatban megjelölt
számok nem lehetnek négyzetszámok.
163. (ii) miatt nem lehet.
164. 1111, 5555, 9999 jegyekre nem végződhet, mert ekkor 4-gyel osztva 3 maradékot
kapnánk. 6666-ra sem végződhet, mert ekkor 4-gyel osztva 2 a maradék. 4444-re
pedig azért nem, mert ha ezt 4-gyel osztjuk, nem kapunk négyzetszámot, hiszen
a hányados 11-re végződik, ami 4-gyel osztva 3 maradékot ad.
Megjegyzés. 3 egyforma jegyre végződhet négyzetszám, pl.: 382 = 1444 vagy
44622 = 19909444.
166. (iv) miatt nem lehet.
167. 444 . . . 444 = 4 · 111 . . . 111, s 111 . . . 111 nem lehet négyzetszám, mert 4-gyel
osztva 3 a maradék. Vizsgálható a 16-os osztási maradék is. Lehet indokolni a
következőképpen is: a szám szorzattá alakítva 4 · 11 · 1010101 . . . 01 alakú, s
ebből látható, hogy a szám osztható 11-gyel, de 112 -nel nem.
168. 144 . . . 444 = 4 · 36111 . . . 111, s 36111 . . . 111 nem lehet négyzetszám, mert
4-gyel osztva 3 a maradék. Vizsgálható itt is a 16-os osztási maradék.
169. (i), (iii), (iv) ill. (v) miatt sem lehet.
170. n2 < n(n + 1) < (n + 1)2 , vagy indokolhatunk úgy is, hogy n és n + 1 relatív
prímek, így mindkettőnek négyzetszámnak kell lennie, de ez nem lehet.
171. (n − 1)n(n + 1) = n · (n2 − 1); n és n2 − 1 relatív prímek, ezért mindegyiknek
négyzetszámnak kell lennie, de n2 − 1 sohasem négyzetszám.
172. n(n + 1)(n + 2)(n + 3) = (n2 + 3n + 1)2 − 1.
173. (iv) miatt nem lehet.
174. (iii) miatt nem lehet.
175. (iv) miatt nem lehet.
176. (n − 2)2 + (n − 1)2 + n2 + (n + 1)2 + (n + 2)2 = 5(n2 + 2), s ez nem négyzetszám,
mert n2 + 2 nem osztható 5-tel, hiszen négyzetszám 3-ra, 8-ra nem végződhet.
177. (iii) miatt nem lehet.
178. (iv) miatt nem lehet.
179. Vizsgáljuk az összeg 4-es osztási maradékát.
5. Négyzetszámok 193

180. ab = n2 és ha ab + a = m2 lenne, akkor ab(ab + a) = a 2 b(b + 1) is négyzetszám,


de b(b + 1) nem négyzetszám (170. feladat), így ab(ab + a) sem, végül ab + a
sem az.
182. Ha x ≤ y, akkor y 2 < y 2 + x ≤ y 2 + y < (y + 1)2 .
184. (n2 + n)2 < n4 + 2n3 + 2n2 + 2n + 1 < (n2 + n + 1)2 .
Vagy: n4 + 2n3 + 2n2 + 2n + 1 = (n + 1)2 (n2 + 1), s ez a szorzat csak akkor lehetne
négyzetszám, ha n2 + 1 is négyzetszám lenne.
 2  2
186. 1157995 < 11571990 +341990 < 1157995 + 2 . A megadott intervallumban egy
négyzetszám van, de az páros.
187. Páros n esetén az összeg utolsó jegye vagy 3, vagy 7. Páratlan n esetén pedig
3-mal osztva 2 maradékot ad.

5. Négyzetszámok

188. Lehet, pl. 4 és 16.


189. Nincs, ugyanis a szomszédos négyzetszámok különbsége egyre nagyobb lesz.
190. (2n − 1)2 + (2n + 1)2 − 2 = 2 · (2n)2 .
191. n ≥ 1 esetén n2 + n + 41 < (n + 6)2 , tehát, ha n2 + n + 41 négyzetszám, akkor csak
(n + k)2 alakú lehet, ahol 0 ≤ k < 6. Az egyetlen megoldás: n = 40.
192. A keresett számban a prímtényezők csak páros hatvánnyal szerepelhetnek. A pá-
ros számok közül így b) és e) kiesik, mivel 4-gyel nem oszthatók; a) osztható
8-cal, de 16-tal nem; d) osztható 3-mal, de 9-cel nem; c) 3-mal osztva 2 mara-
dékot ad. Tehát az utolsó szám a négyzetszám.
2. megoldás. Az a), c), d) számok 9-cel osztva 8, 5, ill. 3 maradékot adnak. b)
és e) utolsó jegye miatt nem lehet négyzetszám. Az utolsó szám 50 123 246-nak
a négyzete.
193. Ha a szám jegyeinek összege 1991, akkor ez a szám 3-mal osztva 2-t ad mara-
dékul, míg négyzetszámnál ez a maradék nem léphet fel.

194. 1991 = 44,6206230 . . . ,
4462 = 198 916,
44622 = 19 909 444,
44632 = 19 918 369.
198. Legyen a sorozat differenciája d, egyik eleme m2 . Ekkor
(m + d)2 = m2 + 2md + d 2 = m2 + (2m + d) · d szintén eleme a sorozatnak.
199. Sok lehetőség van, pl.: 10k + 3; 4k + 2; 3k + 2; 9k + 3 stb.
194 Megoldások, útmutatások

200. 2, 23 , 25 , 27 , 29 , . . .
201. a) Négyzetszám 4-gyel osztva 0 vagy 1 maradékot ad. Ezt felhasználva az egyik
lehetséges válasz: a 4k + 3 alakú számok.
b) Köbszám 9-cel osztva 0, 1 vagy 8 maradékot ad. Ennek ismeretében az egyik
lehetséges válasz: a 9k + 3 alakú számok.
202. 10 = 32 + 12 , 100 = 82 + 62 , 1000 = 302 + 102 , 10000 = 802 + 602 , 100000 =
= 3002 + 1002 , . . .
203. Ha a · b = m2 , b · c = n2 , akkor (ab) · (bc) = (n · m)2 , (ac) · b2 = (n · m)2 , mivel
n · m
2
b2 = 0, így ac = . Ha az ac egész szám egy törtszám négyzete, akkor
b
az a törtszám egész szám. (Miért?)
204. a) Nem, mert 1987 4-gyel osztva 3 maradékot ad.
b) Tegyük fel, hogy az egyenlet megoldható, és egy megoldása x = a és y = b.
Mivel 1986 osztható 3-mal, így a 2 + b2 is. Tekintettel arra, hogy egy 3-mal
nem osztható négyzetszám 3-mal osztva mindig 1 maradékot ad, ez csak úgy
lehet, ha a és b is osztható 3-mal. Ekkor 9 | a 2 + b2 , de 9  | 1986, ezzel
ellentmondásra jutottunk.
c) Az előző gondolatmenetet követve: x = 3a, y = 3b, az egyenletből így az
a 2 + b2 = 221 egyenletet nyerjük. Ennek megoldásai a = 10, b = 11, és a = 5,
b = 14. Innen x = 30, y = 33 és x = 15, y = 42 (ill. x = 33, y = 30 és x = 42,
y = 15).
d) (x −y)(x +y) = 1989 = 32 ·13·17, így pl. egy megoldás x −y = 9, x +y = 221,
azaz x = 115, y = 106.
e) Négyzetszám 4-gyel osztva 0 vagy 1 maradékot adhat, ezért két négyzetszám
különbsége 4-gyel osztva nem ad 2 maradékot.
205. a) (a 2 + b2 )(c2 + d 2 ) = (ac + bd)2 + (ad − bc)2 .
b) (a 2 + 2b2 )(c2 + 2d 2 ) = (ac + 2bd)2 + 2(bc − ad)2 .
206. a) Ha x = a 2 + b2 , akkor 4x = (2a)2 + (2b)2 . Ha 4x = a 2 + b2 , akkor vagy a
és b is páros, vagy a és b is páratlan. Ez utóbbi nem lehet, mert két páratlan
szám négyzetének összege nem osztható 4-gyel. Tehát a = 2c, b = 2d, így az
egyenletből helyettesítés után nyerjük az x = c2 + d 2 alakot.
Megjegyzés. Az állítás következik a feladat erősebb, b) állításából.
b) Ha x = a 2 + b2 , akkor 2x = (a + b)2 + (a − b)2 . Fordítva, ha 2x = a 2 + b2 ,
 
a+b 2
akkor vagy a és b is páros, vagy a és b is páratlan. Ekkor az x = +
2
 2
a−b
+ felbontás x-et is két négyzetszám összegeként állítja elő. (A
2
második tag lehet 0.)
5. Négyzetszámok 195

207. Az állítás egyik része következik a 205. b) állításból. Ha 3x = a 2 + 2b2 , akkor


3 | a 2 + 2b2 , így 3 | a 2 + 2b2 − 3b2 = a 2 − b2 = (a − b)(a + b), tehát vagy 3 | a − b
(ekkor 3 | a + 2b) vagy 3 | a + b (ekkor 3 | a − 2b).
3 | a − b esetén az
   
a + 2b 2 a−b 2
x= +2 ,
3 3
3 | a + b esetén pedig az
   
a − 2b 2 a+b 2
x= +2 ,
3 3
előállítást nyerjük, ahol a tagok egész számok.
208. a) (a 2 + 5b2 )(c2 + 5d 2 ) = (ac − 5bd)2 + 5(ad + bc)2 .
b) Van. 84 = 4 · 21 = 6 · 14. E számok mindegyike a 2 + 5b2 alakú:
84 = 22 + 5 · 42 ,
4 = 22 + 5 · 02 ,
6 = 12 + 5 · 12 ,
14 = 32 + 5 · 12 és
21 = 42 + 5 · 12 .
Ezek mindegyike alapszám, kivéve természetesen a 84-et.
c) (Hasonló a prímszámok végtelenségének Euklidesz-féle bizonyításához.) Te-
gyük fel, hogy csak véges sok alapszám van: x1 , x2 , . . . , xn .
Az 1 + 5(x1 x2 . . . xn )2 szintén M-hez tartozik, nem osztható egyik xi -vel sem,
tehát újabb alapszámot találtunk.
209. Előbb megmutatjuk, hogy két a 2 + 3b2 alakú szám szorzata is ilyen alakú:
(x 2 + 3y 2 )(z2 + 3t 2 ) = (xz + 3yt)2 + 3(xt − yz)2 .
Ha az a két szám, melyek összege a 2 + 3b2 alakú, A és B, akkor a feladat
állításának igazolásához elegendő belátni, hogy A2 − AB + B 2 is a 2 + 3b2 alakú,
hiszen A3 + B 3 = (A + B)(A2 − AB + B 2 ).
   
A+B 2 A−B 2
Ha A és B azonos párosságú, akkor A − AB + B =
2 2
+3 ;
2 2
ha A és B különböző párosságú, és pl. A páros, akkor
 2  2
A A
A − AB + B =
2 2
−B +3
2 2
egy-egy megfelelő felírás.
210. Legyen n = 3x 2 + 32y 2 , akkor 96n = 96 · 3x 2 + 96 · 32y 2 = 3(32y)2 + 32(3x)2 ,
196 Megoldások, útmutatások

tehát 96n is érdekes.


97n = n + 96n = [3x 2 + 32y 2 ] + [3(32y)2 + 32(3x)2 ] =
= 3[x 2 + (32y)2 ] + 32[y 2 + (3x)2 ] =
= 3[x 2 + 64xy + (32y)2 ] + 32[y 2 − 6xy + (3x)2 ] =
= 3(x + 32y)2 + 32(y − 3x)2 ,
tehát 97n is érdekes szám.
Általánosabban megmutatjuk, hogy ha n = ax 2 + by 2 , akkor (ab + 1)n is előáll
ilyen alakban (a, b, x, y egész számok).
Ha n = ax 2 + by 2 , akkor
(ab + 1)n = (ab + 1)(ax 2 + by 2 ) = a 2 bx 2 + ax 2 + ab2 y 2 + by 2 =
= a 2 bx 2 + 2abxy + ax 2 + ab2 y 2 − 2abxy + by 2
= b(a 2x 2 + 2axy + y 2 ) + a(b2 y 2 − 2bxy + x 2 ) =
= a(by − x)2 + b(ax + y)2 .
(Speciálisan, ha a = 3 és b = 32, akkor ab + 1 = 97.)
211. Az egyenletet rendezve kapjuk, hogy z2 = (x − z)(y − z). Ha a d egész szám
osztója (x − z)-nek és (y − z)-nek is, akkor z2 osztható d 2 -tel, tehát z osztható
d-vel. De ekkor x = (x − z) + z és y = (y − z) + z alapján d az x, y és z számok
mindegyikének osztója, így a feladat feltétele szerint csak 1 lehet. Tehát (x − z)-
nek és (y − z)-nek nincs 1-nél nagyobb közös osztója, azaz relatív prímek. Ezért
és mert a szorzatuk négyzetszám, mindkettő külön-külön négyzetszám; vagyis
alkalmas k és l egészekre x − z = k 2 , y − z = l 2 , és az átrendezett egyenlet
alapján z = kl. Innen
x + y = (x − z) + (y − z) + 2z = k 2 + l 2 + 2kl = (k + l)2 .

212. A feltételben szereplő egyenletből kapjuk:


(3x + 3y + 1)(x − y) = x 2 és (2x + 2y + 1)(x − y) = y 2 .
Ezek szorzata (3x + 3y + 1)(2x + 2y + 1)(x − y)2 = (xy)2 . Ha xy = 0, akkor az
utóbbi egyenlőségből kapjuk, hogy x = 0 és y = 0; s ekkor igaz a feladat állítása.
Ha xy = 0, akkor az egyenlőségből kapjuk, hogy (3x + 3y + 1)(2x + 2y + 1) is
négyzetszám.
Elegendő belátni, hogy (3x + 3y + 1) és (2x + 2y + 1) relatív prímek, hiszen
ekkor mindkettő négyzetszám. Nos, ha d | 3x + 3y + 1 és d | 2x + 2y + 1, akkor
d | (3x + 3y + 1) − (2x + 2y + 1) = x + y, de ekkor d | (2x + 2y + 1) − 2(x + y) = 1.
213. Legyen N = a1 a2 . . . a16 , az ai -k között tehát csak a 2, 3, 5, 6, 7, 8 jegyek
fordulhatnak elő. Képezzük a következő 16-tagú sorozatot: a1 , a1 · a2 ,
a1 · a2 · a3 , . . . , a1 · a2 · a3 · . . . · a16 .
Ezek prímtényezős felbontása: 22α1 +β1 · 32α2 +β2 · 52α3 +β3 · 72α4 +β4 alakú, ahol αi
nemnegatív egész, βi pedig 0 vagy 1.
6. Két szomszédos egész szám szorzata 197

A 16-tagú sorozat elemeihez hozzárendelhető egy 0-kból és 1-esekből álló


(β1 , β2 , β3 , β4 ) négyes. A különböző ilyen számnégyesek száma éppen 16. Ezért
vagy szerepel közöttük a (0, 0, 0, 0), s akkor ezt olyan a1 · a2 · a3 · . . . · ak szor-
zathoz rendeltük, amely négyzetszám; ellenkező esetben van két azonos szám-
négyes. Ekkor annak a két számnak (szorzatnak) a hányadosa lesz négyzetszám,
amelyekhez ez a két megegyező számnégyes tartozik.
A feladat állítása 15-jegyű számra nem igaz; erre példa 232523272325232.
214. 12 egymást követő szám összege többszöröse 3-nak, míg 12 egymást követő
négyzetszám összege 3-mal osztva 2 maradékot ad.
215. a) Ha n > 2, akkor n2 > n2 − n + 2 > (n − 1)2 . Válasz: n = 2.
b) n3 − n + 2 3-mal osztva 2 maradékot ad, így nem lehet négyzetszám.
c) Ha n > 2, akkor (n2 )2 > n4 − n + 2 > (n2 − 1)2 . Válasz: n = 2.
d) n5 − n + 2 5-tel osztva 2 maradékot ad, így nem lehet négyzetszám.
n n
 1  1
216–218. Használjuk az 111 . . . 11 = · 999 . . . 99 = (10n − 1) összefüggést. Példá-
9 9
nak lássuk a 215. d) feladat megoldását.
44 . . . 488 . . . 89 =
4 8 4 · 102n − 4 · 10n + 8 · 10n − 80 + 81
= (10n − 1)10n + (10n−1 − 1)10 + 9 = =
9 9 9
 
4 · 102n + 4 · 10n + 1 2 · 10n + 1 2
= = .
9 3

6. Két szomszédos egész szám szorzata

Két szomszédos egész szám szorzata


(i) mindig páros;
(ii) 0, 2 vagy 6 számjegyre végződik;
(iii) 3-mal osztva 0 vagy 2 maradékot ad.
220. (i) miatt nem lehet.
221. (iii) miatt nem lehet.
222. (ii) vagy (iii) miatt sem lehet.
223. (ii) miatt nem lehet.
224. (i) miatt nem lehet.
198 Megoldások, útmutatások

1 2
225. Ha k páratlan, akkor (k − 1) két szomszédos egész szorzata.
4
1 4 1  2 2 
(n + 2n3 + 3n2 + 2n) = n +n+1 −1
4 4
227. Vegyük figyelembe, hogy a k(k+1) és a (k+1)(k+2) (k egész) számok között nincs
olyan egész szám, amely előállítható két szomszédos egész szám szorzataként.
Ezt felhasználva, az állítást először bizonyítsuk be n ≥ 0 egészekre, ezután n <
< −6 számokra. Végül tekintsük a többi megvizsgálandó esetet.
228. n(n + 1) < n2 + 2n + 12 < (n + 1)(n + 2), ha n > 10; és
(n + 1)(n + 2) < n2 + 2n + 12 < n(n + 1), ha n < −12.
Így megoldásokat csak olyan páros n-ekre kell keresnünk, amelyekre
−12 ≤ n ≤ 10. A megoldás: −12, −4, −2, 0, 2, 10.
229. (ii), ill. (iii) miatt semmilyen n sem lehet.
230. Mivel n2 + n + 1 páratlan szám ((i) miatt), ezért nem lehet páros osztója.
231. (ii) miatt az egyenletnek nincs megoldása.
232. Jelöljük a felírt számot a-val és feltételezzük, hogy a = k(k + 1) alakban írható.
Mivel a 3-mal osztva 2 maradékot ad, k = 3m + 1, ahol m egész. k(k + 1) =
= (3m + 1)(3m + 2) = 9(m2 + m) + 2, vagyis k(k +1) 9-cel osztva 2 maradékot ad.
Az adott számjegyekkel felírt szám pedig 9-cel osztva nem adhat 2-t maradékul.
233. Ha n két szomszédos egész szám szorzata, vagyis n = k(k + 1), akkor 9n + 2 =
= (3k + 1)(3k + 2) is ilyen. Fordítva, legyen 9n + 2 két szomszédos egész szám
szorzata: 9n + 2 = k(k + 1). Mivel 9n + 2 nem osztható 3-mal, így k = 3m + 1, azaz
9n + 2 = (3m + 1)(3m + 2), s ilyenkor 9n + 2 = 9[m(m + 1)] + 2, tehát n = m(m + 1).
234. (iii) miatt nem lehet: vizsgáljuk meg az n = 3m, n = 3m − 1 és n = 3m + 1
eseteket!
235. Hasonló a 216–218. feladatok megoldásához.

7. Diofantoszi egyenletek

236–242. feladatokban az egyenlőségjel egyik oldalán páros, másik oldalán páratlan


szám áll.
243. A bal oldalon levő kifejezés vagy páratlan, vagy 4-gyel osztható szám.
244. A bal oldal osztható 3-mal, a jobb oldal nem.
246–248. Az egyenlőségjel bal és jobb oldalán lévő számoknak különböző a 3-mal
való osztási maradéka.
7. Diofantoszi egyenletek 199

249. Az egyenlőségjel bal és jobb oldalán lévő számok 4-gyel való osztási maradéka
különböző.
250–253. Az egyenlőségjel bal és jobb oldalán lévő számok 8-cal való osztási mara-
déka különböző.
254–255. Az egyenlőségjel bal és jobb oldalán lévő számok 9-cel való osztási mara-
déka különböző.
257. Az egyenlőségjel bal és jobb oldalán lévő számok 7-tel való osztási maradéka
különböző.
259. Az egyenlőségjel bal és jobb oldalán lévő számok 9-cel való osztási maradéka
különböző.
260. Az egyenlőségjel bal és jobb oldalán lévő számok 16-tal való osztási maradéka
különböző.
261. Az egyenlőségjel bal és jobb oldalán lévő számok 7-tel való osztási maradéka
különböző.
263. Tekintsük az egyenlet mindkét oldalán szereplő számok 9-cel való osztási mara-
dékát!
264. 7 | 12n − 5n , 7  | 11n .
265. Átalakítás után az (x − 1)2 + (y − 1)2 = −1 egyenletet nyerjük.
266. Átalakítás után az (x 2 − y)2 + 2y 2 = −2 egyenletet kapjuk.
267. Átalakítás után az (a − b)2 = b2 (4b − 1) egyenletet kapjuk. Az egyenlet jobb
oldalán azért nem állhat négyzetszám, mert 4b − 1 4-gyel osztva 3 maradékot ad,
így ez a tényező nem négyzetszám.
1 1 1 1 1 1 3
268. + + ≤ + + = < 1, hiszen a és b egyike sem lehet 1.
a 2 ab b2 4 4 4 4
270–283. feladatoknál olyan alakra hozzuk az egyenletet, amelynek egyik oldalán egy
szorzat, a másikon pedig egy szám áll. Ezt a számot bontsuk fel prímtényezőkre.
270–275. feladatoknál az (a + 1)(b + 1) = ab + a + b + 1 azonossággal rokon szorzattá
alakítást célszerű elvégezni.
276. a(2a 2 + b) = 7; innen vagy a = 1, 2a 2 + b = 7 és így a = 1, b = 5, vagy a = 7,
2a 2 + b = 1 és így b = −97. A 7 = (−1) · (−7) = (−7) · (−1) szorzatok vizsgálata
adja az a = −1, b = −9, ill. az a = −7, b = −99 megoldást.
277. (a − 5)(b + 3) = −18.
278. a 2 − b2 = (a − b)(a + b) = 100, a − b = 2, a + b = 50 vagy a − b = 50, a + b = 2.
279. (a − 2b)(a + 2b) = 116.
200 Megoldások, útmutatások

280. (2a − 3b)(a + 4b) = 28.


281. 243 = 35 .
282. (a + b + 1)(a − b − 1) = 12.
283. a 2 = b3 +1 = (b + 1)(b  − b + 1) = (b + 1)(b + b − 2b − 2 + 3) =
2 2

3
= (b + 1)2 · b − 2 + .
b+1
284. Átalakítva az egyenletet az (a − b)2 + (b − c)2 + (c − a)2 = 0 egyenletet kapjuk.
285. Átalakítva az egyenletet az (a − 1)2 + (b − 1)2 + (a − b)2 = 2 egyenletet kapjuk.
286. Átalakítva az egyenletet az (a −1)3 +(b+2)3 = 1 egyenletet kapjuk. A bal oldalon
szereplő köbszámok közül az egyik 1, a másik 0.
287. Vizsgáljuk a kifejezések párosságát!
288. n! 0-ra végződik, ha n ≥ 5. Ha az egyenletben szereplő n értéke legalább 5,
akkor a bal oldalon álló összeg utolsó számjegye 3, így ez a szám nem lehet
négyzetszám. Ezért elegendő az n < 5 eseteket vizsgálni.
289. 145.
290. Bontsuk −16-ot három egész szám szorzatára minden lehetséges módon, s vá-
lasszuk ki azt a számhármast, mely kielégíti a másik két egyenletet.
291–293. Ha egy — nullára redukált és rendezett — algebrai egyenletnek van egész
gyöke, akkor ez a gyök osztója a polinom konstans tagjának.
294. Tegyük fel, hogy a ≤ b ≤ c. Ekkor abc = a + b + c ≤ c + c + c = 3c, tehát ab ≤ 3.
További rövid elemzés adja a megoldást, az 1, 2, 3 számhármast.
295–296. Osszuk az egyenlet mindkét oldalát abc-vel, és tegyük fel, hogy a ≥ b ≥ c.
297. Itt is tekintsük az a ≥ b ≥ c feltevést.
298–300. A 270–275. feladatok megoldásához hasonló.
301. Ha b2 − 4ac = 1994, akkor b páros; ám ekkor b2 és 4ac is osztható 4-gyel, így
b2 − 4ac is, míg 1994 nem osztható 4-gyel, ezért a b2 − 4ac = 1994 egyenlet
nem oldható meg az egész számok körében.
Ha n = 2k + 1, akkor n2 = 4k 2 + 4k + 1, tehát páratlan szám négyzete 4-gyel
osztva 1-et ad maradékul. Ha b2 − 4ac = 1995, akkor b páratlan, b2 − 4ac 4-
gyel osztva 1-et ad maradékul, míg 1995 4-gyel osztva 3 maradékot ad. Ezért a
b2 − 4ac = 1995 egyenlet nem oldható meg az egész számok körében.
Az x 2 + 46x + 30 = 0 egyenlet diszkriminánsa 1996.
7. Diofantoszi egyenletek 201

302. Ha a + b + c ≤ 11, akkor 28a + 30b + 31c ≤ 11 · 31 = 341.


Ha a + b + c ≥ 13, akkor 28a + 30b + 31c ≥ 13 · 28 = 364, egyenlőség csak akkor,
ha a = 13, b = c = 0; minden más esetben 28a + 30b + 31c ≥ 366.
Az egyenletnek megoldása (gondoljunk arra, hogy egy év 365 napból áll): x = 1
(a február 28 napos), y = 4 (április, június, szeptember, november), z = 7 (a
többi hét hónap). Más megoldás: x = 2, y = 1, z = 9.
303. Használjuk a következő becslést: k[x] ≤ [kx] ≤ k[x] + k − 1. Ezzel a feladatbeli
összegre (jelöljük ezt S-sel) a 63m ≤ S ≤ 63m + 57 egyenlőtlenségeket nyerjük,
ahol m = [x]. Ugyanakkor 12345 = 63k + 60.
304. Szorozzuk meg az egyenlet mindkét oldalát 4-gyel, ekkor az egyenlet a követke-
ző alakba írható: (2x +3y)2 −17y 2 = 488. Most már könnyű megmutatni, hogy az
egyenlet nem oldható meg az egész számok körében. Vizsgáljuk a 17-es osztási
maradékokat! Az egyenlet jobb oldala 17-tel osztva 12 maradékot ad, ám a bal
oldalon ilyen maradék nem léphet fel.
305. Az (a + b + c)3 − (a 3 + b3 + c3 ) = 3(a + b)(b + c)(c + a) azonosságot használva
(a + b)(b + c)(c + a) = 114, és a feladat első egyenlete másképp (a + b) + (b + c) +
+ (c + a) = 14. Mivel 114 = 1 · 2 · 3 · 19, így csak a + b = 19, b + c = −3, c + a = −2
lehet megoldás. Az egyenletrendszer megoldása: a = 10, b = 9, c = −12 (ill.
ezeknek a számoknak más sorrendjei).
306. A második egyenlet 3-szorosából kivonva az első négyzetét:
(a − b)2 + (b − c)2 + (c − a)2 = 14.
307. Dolgozzunk komplex számokkal! (m2 + n2 )2 = |m + in|4 = |(m + in)2 |2 = |m2 +
+ 2mni + i 2 n2 |2 = |(m2 − n2 ) + i(2mn)|2 = (m2 − n2 )2 + (2mn)2 .
Ha (m, n) = 1, m > n, valamint az m és n számok különböző párosságúak, akkor
(m2 − n2 , 2mn, m2 + n2 ) = 1, s ezek a számhármasok megoldásai az egyenletnek.
308. Dolgozzunk komplex számokkal! (m2 + n2 )3 = |m + in|6 = |(m + in)3 |2 = |m3 +
+ 3m2 in + 3mi 2 n2 + i 3 n3 |2 = |(m3 − 3mn2 ) + i(3m2 n − n3 )|2 = (m3 − 3mn2 )2 +
+ (3m2 n − n3 )2 .
Ha (m, n) = 1, valamint az m és n számok különböző párosságúak, akkor (m3 −
− 3mn2 , 3m2 n − n3 , m2 + n2 ) = 1, s ezek a számhármasok megoldásai az egyen-
letnek.
309. Dolgozzunk komplex számokkal!
(m2 + n2 )4 = |m + in|8 = |(m + in)4 |2 =
= |m4 + 4m3 in + 6m2 i 2 n2 + 4mi 3 n3 + i 4 n4 |2 =
= |(m4 − 6m2 n2 + n4 ) + i(4m3 n − 4mn3 )|2 =
= (m4 − 6m2 n2 + n4 )2 + (4m3 n − 4mn3 )2 .
Ha (m, n) = 1, valamint az m és n számok különböző párosságúak, akkor
(m4 − 6m2 n2 + n4 , 4m3 n − 4mn3 , m2 + n2 ) = 1, s ezek a számhármasok megol-
dásai az egyenletnek.
202 Megoldások, útmutatások

310. A végtelen leszállás (descente infinie) módszerével bizonyítunk. Tegyük fel,


hogy a1 , b1 megoldás. Ekkor a1 nyilván páros, de b1 is páros lesz: a1 = 2a2 ,
b1 = 2b2 . Ha a12 = 2b12 , azaz (2a2 )2 = 2(2b2 )2 , akkor a22 = 2b22 , tehát a2 , b2 is
megoldása az egyenletnek.
Így az egyenletnek végtelen sok ai , bi megoldásához jutunk, melyekre
|a1 | > |a2 | > · · · > |ak | > . . . (ugyanígy bi -kre is). Ez azonban nem lehetséges,
hiszen a pozitív egészeknek nincs végtelen sok tagból álló, szigorúan csökkenő
sorozata.
Ellentmondásra jutottunk, az ellentmondás oka a kiinduló feltevés (az, hogy az
egyenletnek létezik megoldása), tehát a kiinduló feltevés hamis.
(Reductio ad absurdum.)
311. Alkalmazzuk a végtelen leszállás módszerét.
312. Alkalmazzuk a végtelen leszállás módszerét. Használjuk fel, hogy ha 3 | a 2 + b2 ,
akkor 3 | a és 3 | b.
313. Alkalmazzuk a végtelen leszállás módszerét. Használjuk fel, hogy ha 7 | a 2 + b2 ,
akkor 7 | a és 7 | b.
314. Alkalmazzuk a végtelen leszállás módszerét. Használjuk fel, hogy ha 3 | a 2 + b2 ,
akkor 3 | a és 3 | b.
315. Alkalmazzuk a végtelen leszállás módszerét. Lássuk be, hogy a is, b is, c is
osztható 3-mal.
316. Alkalmazzuk a végtelen leszállás módszerét. Lássuk be, hogy a is, b is, c is
osztható 2-vel.
317. Alkalmazzuk itt is a végtelen leszállás módszerét. Tegyük fel, hogy az egyen-
letnek vannak nullától különböző megoldásai. Tudjuk, hogy négyzetszám 4-gyel
osztva 0 vagy 1 maradékot ad. Innen következtethetünk arra, hogy ha a1 , b1 , c1
megoldás, akkor mindegyik szám páros: a1 = 2a2 , b1 = 2b2 , c1 = 2c2 . Ekkor
a22 + b22 + c22 = 4a22 b22 . Ebből következik, hogy a2 , b2 , c2 mindegyike páros: a2 =
= 2a3 , b2 = 2b3 , c2 = 2c3 . Egyszerűsítés után az egyenlet: a32 + b32 + c32 = 16a32 b32 .
Láthatjuk, hogy a3 , b3 , c3 mindegyike páros. Ezt a gondolatmenetet újra és újra
megismételve kapjuk, hogy az |a1 | > |a2 | > · · · > |ak | > . . . sorozat mindegyik
eleme páros, ami nem lehet. Ellentmondásra jutottunk, az ellentmondás oka a
hibás feltevés. Tehát az egyenlet nem oldható meg a nullától különböző egészek
körében.
318. Alkalmazzuk a végtelen leszállás módszerét. A gondolatmenet hasonló, mint az
előző feladat megoldásában.
319. Alkalmazzuk a végtelen leszállás módszerét. A gondolatmenet hasonló, mint az
előző két feladat megoldásában.
8. Prímszámok 203

320. Alkalmazzuk most is a végtelen leszállás módszerét. Azt látjuk be, hogy az a 4 +
+b4 = c2 egyenlet nem oldható meg a pozitív egészek körében. Legyen (a, b, c) =
= 1. Ismert, hogy ekkor a 2 = 2uv, b2 = u2 − v2 , u > v > 1, (u, v) = 1 és v páros;
hiszen u nem lehet páros, mert ekkor u2 = 4A, v2 = 4B + 1, így b2 = u2 − v2 =
= 4A − (4B + 1) = 4C + 3 lenne. Így aztán a 2 = 2uv miatt és mert v páros; u = x 2 ,
v = 2y 2 . A b2 = u2 − v2 összefüggésből b2 = x 4 − 4y 4 , azaz b2 + 4y 4 = x 4 . Innen
y 2 = u1 v1 és x 2 = u21 + v12 , ahol u1 = a12 , v1 = b12 ; tehát x 2 = a14 + b14 , a1 < a
és b1 < b, mert a 2 = 2uv = 4x 2 y 2 = 4u1 v1 (u21 + v12 ) = 4a12 b12 (a14 + b14 ) > a12 ,
így a1 < a; b2 = u2 − v2 = x 4 − 4y 4 = (x 2 + 2y 2 )(x 2 − 2y 2 ) ≥ x 2 + 2y 2 >
> x 2 = a14 + b14 > b14 ≥ b12 , innen b1 < b. Tehát, ha az eredeti egyenletnek volt
a, b, c megoldása a pozitív egészek körében, akkor van ezeknél kisebb pozitív
egészekből álló a1 , b1 , c1 megoldása is.
321. Az egyenlet átalakítás után a (2a + 1)2 + (2b + 1)2 + (2c + 1)2 = 7 alakot ölti.
Ha ennek az egyenletnek van megoldása a racionális számok körében, akkor az
x 2 + y 2 + z2 = 7t 2 egyenlet megoldható az egész számok körében. Páratlan szám
négyzete 8-cal osztva 1 maradékot ad; emiatt t nem lehet páratlan, hiszen ekkor
az egyenlőség két oldalán a 8-as maradékok különbözők lesznek. Mivel t páros,
a 4-es maradékot figyelve x, y, z mindegyike páros lesz: x = 2x1 , y = 2y1 ,
z = 2z1 és t = 2t1 .
Az egyenletnek x1 , y1 , z1 , t1 is megoldása. Alkalmazhatjuk a végtelen leszállás
gondolatát.
322. Ha létezik racionális számokból álló megoldás, akkor az x 2 + xy + y 2 = 2z2
egyenletnek van megoldása az egész számok körében. Ha x, y valamelyike vagy
mindkettő páratlan, akkor a bal oldalon páratlan szám áll. Tehát x = 2x1 , y = 2y1 .
Ekkor a bal oldalon álló szám osztható 4-gyel, így z is páros, z = 2z1 . Az egyen-
letnek x1 , y1 , z1 is megoldása. Alkalmazhatjuk a végtelen leszállás gondolatát.

8. Prímszámok

323. Páratlan.
324. 1 és 2.
325. −3, −1, 1.
326. 5 és 2.
327. A két összeadandó egyike páros, ez csak 2 lehet. A másik összeadandó 1989, ez
pedig összetett, osztható 9-cel.
328. Mind a három prím nem lehet páratlan, így a legkisebb összeadandó a 2.
204 Megoldások, útmutatások

329. Három páratlan prímszám összege páratlan, emiatt az összeg 1234 csak úgy lehet,
ha a három prím egyike a 2. Ekkor a három prímszám szorzata páros, így az nem
lehet 87654321. Tehát a kívánt módon nem lehet három prímszámot megadni.
330. Legyen a két prím p és q, és q = p + 100; az egymás után írásukkal keletkező
prím r.
Ha p 3-mal osztva 1 maradékot ad, akkor q 3-mal osztva 2 maradékot ad, és
ekkor az r szám osztható 3-mal, nem lehet prím (> 100).
Ha p 3-mal osztva 2 maradékot ad, akkor q osztható 3-mal.
Ha p 3-mal osztva 0 maradékot ad, akkor csak p = 3 lehetséges. Ekkor q = 103
(ami prím).
Az egymás után írásukkal keletkező szám a 3103 (3103 = 29 · 107) és 1033 (ez
prímszám).
331. Ha mind a hat prím páratlan, akkor az összeg páros szám, nem prím. A hat szám:
2, 3, 5, 7, 11, 13.
332. Két páratlan szám között levő egészek száma páratlan.
333. n egymást követő egész szám összege osztható n-nel, ha n páratlan.
Megjegyzés. Ha n páratlan prím, akkor n egymást követő egész szám összege
lehet prímszám. Pl. n = 7 esetén: −2, −1, 0, 1, 2, 3, 4.
334. Az összeg páros szám.
335. Nem. Ha lehetne, akkor mind a 3 sorban ugyanannyi lenne a számok összege,
tehát a kilenc szám összege osztható lenne 3-mal, s ez nem teljesül. Más indoklás:
Abban a sorban, amelyikben a 2 áll, a számok összege páros, míg a többi sorban
páratlan.
336. y osztható 2-vel, y = 2; x osztható 3-mal, x = 3. Így z = 11.
337. p értéke csak 2 lehet. Ha q páratlan, akkor 2q +1 osztható 3-mal. Tehát p = q = 2,
r = 5.
338. 2q 2 = p2 − 1 = (p − 1)(p + 1). p = 2 nem megoldás; ha p páratlan, akkor
(p − 1)(p + 1) osztható 4-gyel is, tehát q = 2. Ezért p = 3.
340. 4, 8, 32 évesek.
341. Mivel a keresett szám jegyeinek szorzata kisebb, mint 93 és egy prím köbe, a
szorzat lehetséges értékei: 23 , 33 , 53 , 73 .
342. 2n − 1, 2n , 2n + 1 három egymást követő egész közül az egyik osztható 3-mal.
343. p − 1, p, p + 1 három egymást követő egész közül az egyik osztható 3-mal, s ez
a szám nem a p, tehát 3 | (p − 1)(p + 1) = p2 − 1. Továbbá, p − 1 és p + 1 két
szomszédos páros szám, az egyik osztható 4-gyel, tehát
2 · 4 = 8 | (p − 1)(p + 1) = p2 − 1.
8. Prímszámok 205

344. A számok egyike osztható 3-mal.


345. a) Ha p = 3, akkor vagy p + 5, vagy p + 10 osztható 3-mal.
b) Hasonló az a) ponthoz.
c) 8p − 1, 8p és 8p + 1 közül az egyik osztható 3-mal. Ha p = 3, akkor 3  | 8p,
tehát vagy 8p − 1, vagy 8p + 1 osztható 3-mal.
d) Hasonló az c) ponthoz.
e) Ha p = 3, akkor p2 + 2 mindig osztható 3-mal.
f) Hasonló az e) ponthoz.
346–347. p = 3-ra igaz. Ha p = 3, akkor p2 −1 osztható 3-mal (lásd a 343. feladatot!),
s ekkor p2 + 8 és 8p2 + 1 is osztható 3-mal.
348. p és q nem lehet egyszerre páros, ill. páratlan. Ha p = 2 és q páratlan, akkor
p2 + q 2 − q osztható 2-vel, így q = 2; tehát p2 + q 2 − q = p2 + 2 = (p2 − 1) + 3.
Ha p = 3, akkor ez a szám osztható 3-mal. A megoldás tehát q = 2, p = 3.
350. p és q közül az egyik egyenlő 2-vel. Ha p = q = 2, akkor pq − 1 és pq + 1
prímszámok. Tegyük fel, hogy p = 2 és q páratlan! Akkor ha q = 3, 2q − 1 és
2q + 1 közül az egyik osztható 3-mal.
351. 4p + 1 = (2k + 1)2 -ből p = k(k + 1) következik. Ez csak akkor prímszám, ha k = 1.
p+q p+q
355. p + q = 2 · , és összetett szám, hiszen két szomszédos (páratlan) prím
2 2
között van.
356. a) n4 + 2n3 + 2n2 + 2n + 1 = (n + 1)2 (n2 + 1),
b) 6n + 3n + 2n+1 + 2 = (3n + 2)(2n + 1),
c) 32n+1 − 22n+1 − 6n = (3n − 2n )(3n+1 + 2n+1 ),
d) 8n2 + 10n + 3 = (2n + 1)(4n + 3),
e) 25n4 + 9n2 + 1 = (5n2 + 1)2 − n2 ,
f) 4n3 + 6n2 + 4n + 1 = (n + 1)4 − n4 ,
g) |n4 + n3 − 2n2 − 3n − 3| = |(n2 − 3)|(n2 + n + 1).
357. Adjunk e prímek szorzatához 1-et.
359.
101 29 47 103 79 37 271 379 73
5 59 113 7 73 139 43 241 439
71 89 113 109 67 43 409 103 211

71 167 29 571 1051 181 823 1093 643


47 89 131 211 601 991 673 853 1033
149 11 107 1021 151 631 1063 613 883
206 Megoldások, útmutatások

307 607 97 569 59 449 829 1879 409


127 337 547 239 359 479 619 1039 1459
577 67 367 269 659 149 1669 199 1249

360. Ha f (x) = p, akkor f (x + p) osztható p-vel.


361. Következik az előzőből; vagy mutassuk meg, hogy ha a sorozat d differenciája
nem osztható p-vel, akkor a sorozat p egymást követő eleme közt van olyan,
amely osztható p-vel.
362. 7, 37, 67, 97, 127, 157.
363. 41, 47, 53, 59, 65, 71, 77, 83, 89.
364. Például: 11, 17, 23, 29, 35, 41, 47, . . . . Hiszen ha e számok valamelyike előállna
két prím összegeként, akkor az egyik összeadandónak 2-nek kell lennie, s a másik
így 6k + 3 alakú összetett szám lesz.
365. Nem, pl. 320 páros, így utolsó jegyét kell változtatni, viszont a 320 és 330 közötti
számok mind összetettek.
366. c | ab, ezért c = xz, a = xy, b = zt, s ezek miatt d = yt. Ekkor
a 2000 + b2000 + c2000 + d 2000 = (xy)2000 + (zt)2000 + (xz)2000 + (yt)2000 =
= (x 2000 + t 2000 )(y 2000 + z2000 ).

367. Ha p összetett, p = mn, m > 1, n > 1, akkor p = (m − 1)(n − 1) + 1 + (m − 1) +


+ (n − 1) = a + b + c + d, s ekkor valóban ab = cd.
Ha p = a + b + c + d, és ab = cd, akkor pb = ab + b2 + cb + db = cd + b2 +
+ cb + db = (c + b)(d + b). A p szám összetett, hiszen ha p prím volna, akkor az
előző egyenlőségből p | c + b vagy p | d + b lenne, ami nem teljesülhet, hiszen
p = a + b + c + d, ezért p > c + b, p > d + b.
368. Az, hogy a prímszámok száma végtelen, már 2000 éve ismert. Euklidesz bizonyí-
tásának eleganciája megragadja a figyelmes olvasót, s hasonló élményeket nyújt
a többi bizonyítás is. (A bizonyítások többnyire vázlatosan vannak leírva.)
1. bizonyítás. (Euklidesz) Tegyük fel, hogy az állítás hamis: a prímszámok száma
véges; a p1 , p2 , . . . , pn prímeken kívül nincs más prímszám. Vizsgáljuk az N =
= p1 · p2 · . . . · pn + 1 számot. Ez nyilván nem osztható az eddig felsorolt prímek
egyikével sem, tehát vagy prímszám, vagy olyan összetett szám, melynek csak az
előzőktől különböző prímosztói vannak. Ezzel újabb prímszám létezését mutattuk
meg, ami ellentmond a feltevésnek.
8. Prímszámok 207

2. bizonyítás. (Euler) Tegyük fel ismét, hogy hamis az állítás; a 2, 3, . . . , p


prímeken kívül nincs több prím. Ekkor az
1 1 1
1 + + + + ··· =
 2 3 4     
1 1 1 1 1 1 1 1 1
= 1+ + 2 + 3 +. . . · 1+ + 2 + 3 +. . . · . . . · 1+ + 2 + 3 +. . . =
2 2 2 3 3 3 p p p
1 1 1
= · · ...·
1 − 12 1 − 13 1 − p1
azonosságban a bal oldalon álló kifejezés értéke a végtelenhez tart, míg a jobb
oldal véges. Ellentmondásra jutottunk, tehát a feltevés hibás.
3. bizonyítás. Tegyük fel, hogy a 2, 3, 5, . . . , p prímeken kívül nincs több prím.
Ekkor az
1 1 1
1 + 2 + 2 + 2 + ··· =
2 3 4   
1 1 1 1 1 1
= 1 + 2 + 4 + 6 + ... · 1 + 2 + 4 + 6 + ... · ...
2 2 2 3 3 3
 
1 1 1 1 1 1
...· 1 + 2 + 4 + 6 + ... = · ·...·
p p p 1 − 212 1 − 312 1 − p12
π2
azonosságban a bal oldalon irracionális szám áll (= , lásd pl.: Skljarszkij–
6
Csencov–Jaglom: Aritmetika és algebra (Tankönyvkiadó, Budapest, 1967) c.
könyvben a 233. feladatot!), míg a jobb oldalon racionális szám. Az ellentmondás
oka a hibás feltevés.
4. bizonyítás. Ismét indirekt úton bizonyítunk. Ha k db prímszám van, akkor a
belőlük képezhető p1α1 · p2α2 · . . . · pkαk alakú természetes számok száma, ahol a
kitevők nem nagyobbak n-nél: (n + 1)k . E számok között persze szerepelnie kell
az 1, 2, 3, . . . , 2n számok mindegyikének, tehát (n + 1)k > 2n lenne, de ez az
egyenlőtlenség nem teljesül elegendően nagy n-re.
5. bizonyítás. Tegyük fel, hogy csak k db különböző prímszám van: p1 , p2 , . . . ,
pk . Legyen n = p1 · p2 · . . . · pk . Ekkor az egyedüli, n-hez relatív prím szám az
1, tehát φ(n) = 1, azaz φ(n) = (p1 − 1) · (p2 − 1) · . . . · (pk − 1) = 1, de ez az
egyenlőség nem teljesülhet.
(Az 1, 2, . . . , n−1számok közül az n-hez relatív prím számok számát az Euler-
 1
féle φ(n) = n · 1− függvény adja meg.)
p|n
p
6. bizonyítás. Ha tudunk olyan végtelen sorozatot konstruálni, melynek elemei
páronként relatív prímek; abból már következik, hogy végtelen sok prím van.
1. konstrukció: Legyenek a és b egymáshoz relatív prím természetes számok. A
sorozat a következő: a0 = a, an+1 = a0 · a1 · . . . · an + b, n = 0, 1, 2, . . .
Lássuk be, hogy a sorozat elemei páronként relatív prímek. Legyen d egy közös
208 Megoldások, útmutatások

prímosztója ai -nek és aj -nek, ahol i < j .


Az aj = a0 · a1 · . . . · ai · . . . · aj −1 + b előállítás miatt d | b. Mivel ai = a0 · a1 ·
· . . . · ai−1 + b és d | ai , d | b, így d | a0 · a1 · . . . · ai−1 , tehát d osztója a sorozat
valamely, ai -t megelőző elemének is. Ezt folytatva kapjuk, hogy d osztója a-nak
is, b-nek is; ami ellentmond a kezdeti feltevésnek.
n
2. konstrukció: Nézzük a Fermat-féle számok sorozatát: Fn = 22 + 1; n = 0,
n
1, 2, . . . A sorozat elemei páronként relatív prímek; ugyanis, ha d | 22 + 1 és
m m  
m 2 n−m n
d | 22 + 1, m < n, akkor a 22 + 1 | 22 − 1 = 22 − 1 oszthatóság és az
2n
 2n   2n 
előbbi oszthatóság miatt d | 2 − 1, így d | 2 + 1 − 2 − 1 = 2, s mert d
páratlan, ezért d = 1.
7. bizonyítás. Felhasználjuk a Fibonacci-sorozatra vonatkozó Lucas tételt (D. E.
Knuth: A számítógép-programozás művészete 1. (Műszaki Könyvkiadó, Buda-
pest, 1987) c. könyv 101–102. oldalak). Jelölje fn a Fibonacci-sorozat n-edik
elemét, akkor (fm , fn ) = f(m,n) . Ha csak véges sok prím van: p1 , p2 , . . . , pk ,
akkor Lucas tétele szerint az fp1 , fp2 , . . . , fpk elemek páronként relatív prímek,
s mivel a felsoroltakon kívül más prím nincs, így mindegyik fpi -nek csak egy
prímosztója lehet. Ennek mond ellent, hogy f19 = 4181 = 113 · 37.
Megjegyzés. A prímek számának végtelenségére egyéb bizonyítások is ismertek.

9. Oszthatósági feladatok

369–370. Használjuk az oszthatósági szabályokat!


371. a) 119944. b) 519948.
372. Mindkettő hamis, pl. 24, illetve 8 · 9.
373. a) A szám osztható 5-tel.
b) A szám osztható 3-mal.
c) A szám osztható 3-mal.
d–e) A szám a 3 + b3 alakú.
f) A szám osztható 9-cel.
g) A szám osztható 11-gyel.
h) 111 . . . 121 . . . 111 = 111 . . . 110 . . . 000 + 11 . . . 111 = 11 . . . 111 · 100 . . . 001.
i) Vizsgáljuk a szám tízes számrendszerben felírt alakját.
j) A szám osztható 3-mal.
374. 347 777 743 = 333 333 300 + 11 111 110 + 3 333 333, azaz a szám osztható
1 111 111-gyel.
9. Oszthatósági feladatok 209

375. 49 + 610 + 320 = (29 + 310 )2 .


376. 210 + 512 = (25 + 56 )2 − 26 · 56 = (25 + 56 − 23 · 53 ) · (25 + 56 + 23 · 53 ) =
= 14 657 · 16 657.
Megjegyzés. 14 657 és 16 657 prímszámok.
377. Vegyük észre, hogy 53 + 96 = 83 + 66 = 109 + 40 = 149. Vezessük be a következő
jelöléseket: A = 53, B = 83, C = 109, x = 149. Ekkor
53 · 83 · 109 + 40 · 66 · 96 = ABC + (x − A)(x − B)(x − C) =
= x(x 2 − [A + B + C]x + [AB + BC + CA]).
Azaz a megadott szám osztható 149-cel (s ez a szám nagyobb 149-nél), tehát
összetett szám.
378. Legyen x = 991. Ekkor
989 · 1001 · 1007 + 320 = (x − 2)(x + 10)(x + 16) + 320 = x 3 + 24x 2 + 108x =
= x(x + 6)(x + 18) = 991 · 997 · 1009.

379. Legyen x = 512, y = 675, z = 720. Mivel 2z2 = 3xy, így


x 3 + y 3 + z3 = x 3 + y 3 − z3 + 3xyz = (x + y − z)(x 2 + y 2 + z2 − xy + xz + yz),
ezért x 3 + y 3 + z3 = 5123 + 6753 + 7203 osztható x + y − z = 467-tel. Pontosabban:
5123 + 6753 + 7203 = 229 · 467 · 7621.
380. 77a = 34a + 43a = 43b + 43a = 43(a + b). Ezért 43(a + b) osztható 77-tel. Mivel
(43, 77) = 1, emiatt a + b osztható 77-tel, és 77 összetett szám, így az a + b szám
összetett szám.
381. Az m = n + 2, m = n − 2, m = n2 értékek bármelyikét választhatjuk.
382. Ha n páros, akkor a szám 3-mal osztható; ha n = 4k + 1 alakú, akkor 13-mal; ha
n = 4k + 3, akkor 5-tel osztható a szám.
383. A nemnegatív egészek körében 0, a pozitív egészek körében 2520 a válasz.
384. Jelölje a középső számot n, ekkor a kilenc szám összege 9n.
385. A számok összege páratlan.
386. A szorzat osztható 10-zel.
387. A szorzat osztható 5-tel.
388. A négy szám között nincs 5-tel osztható. Így a lehetséges megoldások: 1, 2, 3, 4
vagy 6, 7, 8, 9 vagy 11, 12, 13, 14 stb. Az első négy szám szorzata kicsi, a har-
madik számnégyes szorzata nagy, csak a 6, 7, 8, 9 számok lehetnek megfelelők,
és csakugyan meg is felelnek.
389. 195, 285, 375.
210 Megoldások, útmutatások

390. A keresett szám osztható 5-tel és 9-cel. A válasz: 8 888 888 880.
391. Határozzuk meg, hogy mivel kell megszorozni ehhez az 1991-es számot. A szor-
zó utolsó jegye nyilván 2. A szorzást felírva, visszafelé haladva meg tudjuk ál-
lapítani a szorzó számjegyeit. A megoldás: 18 141 992.
392. 111 = 3 · 37.
393. Ha a törtek mindegyike egész, akkor n + 1 és n + 8 is osztható 3-mal, de ez nem
lehet egyszerre (n + 8) − (n + 1) = 7 miatt.
394. a) 1 + 2 + · · · + 10 = 55, s ez páratlan.
b) Az egyik szorzat osztható 7-tel, a másik nem.
395. Az egyik szorzat osztható 5-tel, a másik nem.
396. Ha a számok között van 7-tel osztható, akkor az egyik szorzat osztható lesz 7-tel,
a másik nem. Ha a számok között nincs 7-tel osztható, akkor a hat szám szorzata
nem lehet négyzetszám (mivel 7-tel osztva 6 maradékot ad), noha ez szükséges
lenne ahhoz, hogy a két csoportban a számok szorzata egyenlő legyen.
397. 654 312.
398. 102 000 564.
399. 123654 vagy 321654.
400. 381 654 729.
401. 123 475 869.
402. Azt a legkisebb pozitív egész A számot keressük, amelyre a B = 999A =
= 1000A−A számban nincs 9-es számjegy. A B szám mindkét alakjából látható,
hogy A utolsó jegye nem lehet 1, hiszen ekkor B utolsó jegye volna 9. Nem lehet
0 sem A utolsó jegye, hiszen ekkor B tizedrésze is megfelelő volna. A B szám
második alakjából látható, hogy emiatt A utolsó előtti jegye sem lehet 0, hiszen
ekkor az 1000A − A kivonást szokás szerint számjegyenként végezve A többi
jegyétől függetlenül B utolsó előtti jegyére 9-est kapunk. Ámde ekkor A százas
helyiértékű jegye sem lehet 0, mert akkor B-ben a százas helyiértékű számjegy
volna 9-es. Ha tehát A-ban minden számjegyet a lehető legkisebbnek választunk,
akkor a 112-es számot kapjuk, mellette B = 111888 nem tartalmaz 9-est, tehát
ez a keresett szám.
403. 7 639 128. (Ilyen tulajdonságú 8-jegyű szám nem létezik.)
404. Mivel a szám 9-re végződik, a páros számjegyeket és az 5-öt elhagyhatjuk. A
keresett számban csak az 1, 3, 7, 9 számjegyek szerepelhetnek. A szám abc9
alakú, és 9 | a + b + c. Ezért a + b + c értéke 9, 18 vagy 27. Az ilyen tulajdonságú
számhármasok a 333, 117, 171, 711, 999. Az elvárt feltételeknek csak a 3339,
7119 és 9999 felelnek meg.
9. Oszthatósági feladatok 211

405. A keresett szám: 8 757 193 191.


406. m = abcde = 45abcde. Ha m-nek van páros számjegye, akkor a szorzat osztható
10-zel, ezért e = 0, ám ebből m = 0 adódik. Tehát minden számjegy páratlan.
e = 5, m osztható 25-tel, d = 7.
m osztható 9-cel, így a + b + c + 12 osztható 9-cel, a + b + c = 15. Továbbá
45 · 5 · 7 · abc < 100 000, ezért abc ≤ 63. Ezeket kielégítő megoldások a páratlan
számjegyek körében: 1, 7, 7, ill. 1, 5, 9.
A feladat megoldását közöttük találjuk: 77 175.
407. Ha a 7-esek száma a, a 3-asok száma b, akkor a számjegyek összege 7a + 3b.
Ez akkor osztható 7-tel és 3-mal, ha 3 | a és 7 | b. A legkisebb számot keres-
ve először az a = 3, b = 7 esetet kell vizsgálnunk. A szóba jöhető legkisebb
számok: 3 333 333 777, 3 333 337 773, 3 333 377 733, . . . Közülük az első kettő
nem osztható 7-tel.
A keresett szám: 3 333 377 733.
408. 7 | 1001, 7 | 222 222 és 7 | 119, ezért 7 | 11 900 000 + 222 222 = 12 122 222.
(Egy másik szám: 22 122 212.)
409. Legyen a keresett szám 100B + 28. Nyilván 100B is osztható 28-cal, azaz B
osztható 7-tel; továbbá B számjegyeinek összege 28 − (2 + 8) = 18 lévén, B
osztható 9-cel is, tehát 63 többszöröse: 63, 126, 189, . . . Látható, hogy a legki-
sebb olyan szám, amelyben a számjegyek összege 18, a 189. A keresett szám:
189 · 100 + 28 = 18 928.
410. A számjegyek összege 56, így az 56 előtt álló számjegyek összege 45. A
9 999 956 szám nem osztható 56-tal, így a számban 56 előtt legalább 6 számjegy
áll. Egy 56-tal osztható szám 8-cal is osztható, ezért az 56 előtt álló számjegy
páros. Az utóbbiak alapján felírható legkisebb szám 19 999 856 nem osztható 56-
tal. Ezért az első jegy legalább 2. Így felírhatjuk a 28 999 856 számot, ám ez sem
osztható 56-tal. A következőként felírható 29 899 856 szám lesz a keresett szám.
411. Ilyen számok: 166667 és 333334.
412. Vizsgáljunk egyszerűbb eseteket: 5-jegyű számok helyett 1-, 2-, 3-, 4-jegyű szá-
mokat.
Két 1-jegyű számot könnyű megadni: 1 · 1 = 1, 1 · 3 = 3, . . .
Keressünk két 2-jegyű számot! 11 · 11 = 121, 11 · 13 = 143, 13 · 13 = 169,
. . . , 13 · 15 = 195. Az utolsóként felírt eset megoldást jelent. Ezt az eredményt
használjuk fel a 3-jegyű számok kereséséhez: 133 · 135 = 17 955. Biztatóan
alakul, látszik milyen alakú számokat keressünk:
1333 · 1335 = 1 779 555, 13333 · 13335 = 177 795 555.
Az utolsóként felírt szorzás megoldást jelent a feladat kérdésére.
413. 1944 = 23 · 35 .
212 Megoldások, útmutatások

414. Legfeljebb 99 ilyen szám van. Például 10001, 10002, . . . , 10099. Ezek minde-
gyike 100 · 100 és 100 · 101 között van.
100 egymást követő felbonthatatlan szám nincsen, mert 100 egymást követő
szám között van 100-zal osztható szám.
415. Legyen a keresett közös osztó d. Ez a szám mind a 49 számnak osztója, így
osztója összegüknek, 999 = 33 · 37 = 27 · 37-nek. A számok mindegyike ≥ d, így
999
49d ≤ 999, azaz d ≤ < 21. A 999 legnagyobb osztója, mely kisebb 21-nél:
49
9, azaz a közös osztó legfeljebb 9 lehet. Van olyan eset, amikor 9 a legnagyobb
közös osztó; ha a 49 szám közül 48 szám mindegyike 9 és a 49. szám az 567.
A számok közös osztójának lehetséges legnagyobb értéke: 9.
416. Legyen a keresett közös osztó d. Ez a szám mind a 10 számnak osztója, így
osztója összegüknek, 1001 = 7 · 11 · 13-nak. A számok mindegyike ≥ d, így
1001
10d ≤ 1001, azaz d ≤ < 101. Az 1001 legnagyobb osztója, mely kisebb
10
101-nél: 91, azaz a közös osztó legfeljebb 91 lehet. Van olyan eset, amikor 91 a
legnagyobb közös osztó; ha a 10 szám közül 9 szám mindegyike 91 és a tizedik
szám a 182.
A számok közös osztójának lehetséges legnagyobb értéke: 91.
417. Az összeg 0-ra végződik.
418. 1110 − 1 = (11 − 1)(119 + 118 + 117 + · · · + 11 + 1), a szorzat mindkét tényezője
osztható 10-zel.
419. 1993 − 199 = 199(1992 − 1) = 199(199 − 1)(199 + 1) = 199 · 198 · 200.
420–430. feladatokban használjuk az
a − b | a n − bn , a + b | a 2n − b2n , a + b | a 2n+1 + b2n+1
oszthatósági szabályokat. Például:
 35  35
13 = 22 + 32 | 22 + 32 = 270 + 370 ;
18 | 1719 + 119 és 18 | 1917 − 117 , tehát 18 | (1719 + 1) + (1917 − 1), azaz
18 | 1719 + 1917 ;
31974 + 51974 = (31974 + 21974 ) + (51974 − 21974 ), és
32 + 22 | 31974 + 21974 , 53 − 23 | 51974 − 21974 , tehát mindegyik osztható 13-mal
(32 + 22 = 13, 53 − 23 = 9 · 13), így összegük is.
431–435. feladatokat teljes indukcióval igazoljuk.
436. 2002 · 2003 · 2004 · . . . · 4002 =
= (4003 − 2001) · (4003 − 2000) · (4003 − 1999) · . . . · (4003 − 1) =
= 4003 · A − 1 · 2 · 3 · . . . · 2000 · 2001, ahol A valamely egész szám.
440. Legyen x = 1992, ekkor 19921993 −1992 = x(x 1992 −1), 19922 +1993 = x 2 +x +1.
Mivel x 1992 − 1 osztható x 3 − 1 = (x − 1)(x 2 + x + 1)-gyel, ezért x(x 1992 − 1)
osztható (x 2 + x + 1)-gyel.
9. Oszthatósági feladatok 213

441–442. feladatok megoldásánál használjuk a 420–430. feladatoknál alkalmazott ösz-


szefüggéseket!
443. Legyen d = (2m − 1, 2n + 1).
d | 2m − 1 | (2m )n − 1 = 2mn − 1 és d | 2n + 1 | (2n )m + 1 = 2mn + 1.
Ezekből d | (2mn + 1) − (2mn − 1) = 2, azonban d páratlan, d = 1.
444. Lásd a prímszámok számának végtelenségére adott 6. bizonyítást (368. feladat)!
2n − 2
446. Legyen k = , akkor 2n − 1 = kn + 1.
n
22 −1 − 2 2kn+1 − 2 2(2kn − 1)
n
(2n )k − 1
= = = 2 · .
2n − 1 2n − 1 2n − 1 2n − 1
4(2p−1 + 1)(2p−1 − 1)
447. n − 1 = , akkor 4 | n − 1 és a kis-Fermat tétel miatt
3
p | n − 1, azaz 2p | n − 1. De akkor 22p − 1 | 2n−1 − 1, így 22p − 1 | 2n − 2. A
feladat feltételéből n | 22p − 1, ezért n | 2n − 2.
449. a) (n + 3)(n + 4) < n2 + 8n + 15 < (n + 4)(n + 4).
b) n2 + 3n + 5 = (n + 7)(n − 4) + 33. Ahhoz, hogy 121 | n2 + 3n + 5 legyen,
(n + 7)(n − 4)-nek 121A + 88 alakúnak kell lennie. Tehát (n + 7) vagy (n − 4)
osztható 11-gyel. De az első tag két tényezője egyszerre osztható vagy nem
osztható 11-gyel, így (n + 7)(n − 4) osztható 121-gyel.
450. x 5 + 3x 4 y − 5x 3 y 2 − 15x 2 y 3 + 4xy 4 + 12y 5 = (x − 2y)(x − y)(x + y)(x + 2y)(x + 3y),
s a 33 nem lehet öt különböző egész szorzata.
451. 19 + 81 | 1989 + 8189 , tehát a szám utolsó két jegye: 00.
452. Nem, mert pl. 1899 = 9 · 211 nem osztható 27-tel.
453. Figyeljük az a, b, c számok 3-as osztási maradékát.
454. A szám (1072 + 1063 + · · · + 109 + 1) · 111 111 111 alakban írható és mind a két
tényező osztható 9-cel.
455. 1980 = 99·20. A szám 20-szal és 99-cel is osztható, mert 100 = 99+1 és 19+20+
+21+· · ·+80 = 99·31. Ugyanis, ha a felírt számot 100-as alapú számrendszerben
felírt számnak tekintjük, akkor osztható 99-cel, ha a szám jegyeinek összege
osztható 99-cel. (Természetesen bizonyíthatunk úgy is, hogy megmutatjuk a 9-
cel és 11-gyel való oszthatóságot.)
457. Közülük végtelen sok osztható 3-mal. Igaz az is, hogy közöttük végtelen sok
37-tel osztható van, mert 555 = 5 · 111 = 5 · 3 · 37, tehát ha a felírt számok
valamelyikében az 5-ösök száma 3-mal osztható, akkor a szám osztható 37-tel.
458. Hasonló az előző feladathoz. Ezek között is végtelen sok 3-mal és végtelen sok
13-mal osztható van, mert 149 149 = 1001 · 149 = 13 · 11 · 7 · 149.
214 Megoldások, útmutatások

459. Mivel 333 333 = 333·1001 = (3·3·37)·(7·11·13) osztható 7-tel, azért 3 333 337 =
= 10 · 333 333 + 7 is osztható 7-tel. Általában a sorozatnak azok a tagjai, melyek
alakja 333 333 333
 333 . . . 333
 333 7, szintén oszthatók 7-tel.
6 db 6 db 6 db

460. Tekintsük a következő számokat: a1 = 3, a2 = 33, . . . , a32 = 33 . . . 3 (32 db


3-as). Mivel 31-gyel való osztáskor összesen 31 különböző maradék lehetséges,
és nekünk 32 db számunk van; akad közöttük kettő, ai és aj (i < j ), melyek
különbsége osztható 31-gyel. Ámde, ha 31 osztója az aj − ai = aj −i · 10j −i
számnak, akkor 31 osztója aj −i -nek is, hiszen 31 prímszám, és a szorzat második
tényezője, 10j −i nem osztható 31-gyel. Ez pedig azt jelenti, hogy a j −i = k darab
hármasból álló szám oszthatók 31-gyel. Általában a sorozatnak azok a tagjai,
melyek alakja 33 .
. . 33 33
.
. . 33 . . . 33
.
. . 33 31, szintén oszthatók 31-gyel.
k db k db k db

462. abcabc = abc000 + abc = 1000 · abc + abc = 1001 · abc = 7 · 11 · 13 · abc.
463. bca = 10 · abc − 999 · a = 10 · abc − 27 · 37 · a.
464. Jelöljön egy 1989-jegyű számot A, a szám utolsó jegyét a.
Az „elcsúsztatott” szám: 10 · A − (101989 − 1) · a, s tudjuk, hogy 27 | A és
 663
27 | 103 − 1 | 103 − 1 = 101989 − 1.
465. bcdea = 10abcde + a − 100000a = 10abcde − 99999a. Mivel abcde is, 99999
is osztható 41-gyel, ezért bcdea is osztható 41-gyel.
466. A számot jelölje A, utolsó jegyét a. A számjegy áthelyezésével kapott szám
legyen B. Ekkor 10 · B − 999 999 · a = A, azaz
10 · B = 999 999 · a + A = 999 · 1001 · a + A = 999 · 7 · 11 · 13 · a + A.
467. Hasonló az előző négy feladathoz.
468. a − 6b = (5a − 4b) − 2(2a + b).
469. 4(100a + b) = 399a + (a + 4b) és 7 | 399.
470. 5(10a + b) = 49a + 7b + (a − 2b).
471. 4(3a + 4b) = 11(a + b) + (a + 5b).
n + 11 n − 9 + 20 20
473. a) = =1+ ,
n−9 n−9 n−9
3n + 5 3(n + 3) − 4 4
b) = =3− ,
n+3 n+3 n+3
n2 + 1 n2 − 1 2 2
c) = + = (n − 1) + ,
n+1 n+1 n+1 n+1
d) A c) feladat megoldásához hasonló.
10. Különféle számelméleti feladatok 215

474. Ha (n + 11)-nek és (n − 9)-nek van közös osztója, akkor az osztója (n + 11) −


− (n − 9) = 20-nak is. Tehát, ha egyszerűsíthető a tört egy egész számmal, akkor
az a szám csak a 20 osztói közül kerülhet ki.
475. b) Ha (12n + 1)-nek és (30n + 2)-nek van közös osztója, akkor az osztója 5(12n +
+ 1) − 2(30n + 2) = 1-nek is.
c) 3(14n + 3) − 2(21n + 4) = 1.
476. b) Mivel 7(8n + 3) − 8(7n + 1) = 13, a közös osztó csak 13 lehet.
478. A legkisebb ilyen szám: 153 846.
479. A legkisebb ilyen szám: 105 263 157 894 736 842.
480. 1 034 482 758 620 689 655 172 413 793.
A szám számjegyei az utolsó jegytől kezdve egymás után meghatározhatók. Írjuk
fel a megadott szorzást. A 3-ra végződő számot 3-mal szorozva a kapott szorzat
utolsó jegye 9, ezért a feladatban leírt feltétel miatt a szorzandó utolsó előtti
jegye is 9 . . .
A felírt 28-jegyű szám a legkisebb a megadott tulajdonságúak között. Az összes
többit úgy kaphatjuk meg, hogy ezt az „alapszámot” tetszőlegesen sokszor egy-
más után írjuk.
481. Hasonló az előző feladatokhoz.
482. Ha az n szám elé az a számjegyet írjuk, akkor a feladat követelménye: a · 10k +
+ A = 58 · n, azaz a · 10k = 57 · n = 3 · 19 · n, s ez nem lehet, mert a bal oldal nem
osztható 19-cel, tehát nincs ilyen szám. A második kérdésre már igen a válasz:
57 · 125 = 7125.

10. Különféle számelméleti feladatok

483. Három egymást követő egész szám összege és szorzata is osztható 3-mal.
484. (n − 1)3 < (n − 1)n(n + 1) = n(n2 − 1) = n3 − n < n3 .
485. Ha egy páros szám hatványszám, akkor a törzstényezős felbontásában 2-nek leg-
alább 2 a kitevője, így osztható 4-gyel. Azonban négy egymást követő egész
szám közül az egyik páros szám nem osztható 4-gyel.
486. a) Van közöttük három páros szám és egy 3-mal osztható páratlan szám.
b) A számok között van olyan, mely nem osztható a 2, 3, 5 számok egyikével
sem.
487. Tekintsük azt a számot, amelyik osztható 12-vel.
216 Megoldások, útmutatások

488. 18, 19, 20, 21, 22 és 9, 10, 11, 12, 13, 14, 15, 16 megoldások. Ezekből kapjuk
a −17, −16, . . . , 17, 18, 19, 20, 21, 22 és a −8, −7, . . . , 8, 9, . . . , 15, 16
megoldásokat.
490. A szám mindig osztható 11 111-gyel, hiszen 105 = 9·11 111+1, ezért a felírt szám
11 111-gyel osztva annyi maradékot ad, mint az egymás után helyezett ötjegyű
számok összege.
491. Nem, ugyanis az (a + 1) + (a + 2) + · · · + (a + k) = 2100 , azaz a k(2a + k +
+ 1) = 2101 egyenlőség azt jelentené, hogy 2101 -nek van 1-nél nagyobb páratlan
osztója. (Negatív számokat is megengedve van megoldás.)
492. Nem. A végződés csak 2222, 4444, 6666, 8888 lehetne, azonban az ilyen számok
egyike sem osztható 16-tal.
Megjegyzés. 239 = 549 755 813 888.
493. 210 = 1024 > 1000 = 103 , ezért 2100 = (210 )10 > (103 )10 = 1030 .
213 = 8192 < 10 000 = 104 , ezért 2100 = 291 · 29 = (213 )7 · 29 < (104 )7 · 512 <
< 1028 · 600 = 6 · 1030 .
Ezek alapján 1030 < 2100 < 6 · 1030 . Ez azt jelenti, hogy a 2100 szám tízes
számrendszerben felírt alakja 31 jegyű.
494. Ha a 21996 szám m-jegyű, akkor 10m−1 < 21996 < 10m . Ha az 51996 szám n-
jegyű, akkor 10n−1 < 51996 < 10n . A két számban összesen m + n számjegy van.
Szorozzuk össze a két egyenlőtlenséget! 10m−1 · 10n−1 < 21996 · 51996 < 10m · 10n .
10m+n−2 < 101996 < 10m+n , azaz m + n − 2 < 1996 < m + n, m + n = 1997.
A két számban összesen 1997 számjegy van.
495. 25, 76.
496. 376, 625.
497. Mivel 1 + 2 + · · · + 13 = 91 és az 1, . . . , 13 számok közül bármelyiket helyettesí-
tenénk egy nála több, mint 1-gyel nagyobb számmal, az összeg 92-nél nagyobb
lenne. Így a keresett számok: 1, . . . , 12, 14.
498. 76 543 210.
499. 16-tal, ill. 17-tel osztható.
500. Ha egy szám nem osztható a-val, akkor persze 2a-val sem osztható. Tehát a
hamis állítás nem vonatkozhatott a 2, 3, 4, 5, 6 számokkal való oszthatóságra. Ha
hamis a 10-zel való oszthatóság, akkor a 2-vel és az 5-tel való oszthatóság állítása
közül legalább az egyik hamis. Hasonlóan, ha hamis a 12-vel való oszthatóság,
akkor a 4-gyel és a 3-mal való oszthatóság egyike is biztosan hamis.
A két egymás után elhangzó hamis állítás egyike a 8-ra vonatkozott, ugyanis a
többi, 2 és 13 közötti páros számot már kizártuk. A másik hamis állítás vagy a
9-re, vagy a 7-re vonatkozik. Ha a téves állítás 8-ra és 9-re vonatkozott, akkor a
10. Különféle számelméleti feladatok 217

felírt szám osztható a 2, 3, 4, 5, 6, 7, 10, 11, 12, 13 számokkal. Ezek legkisebb


közös többszöröse: 3 · 4 · 5 · 7 · 11 · 13 = 60 060, ami nagyobb 50 000-nél, ezért
nem ez a megoldás.
Egy lehetőség marad: a felírt szám 50 000-nél kisebb, 7-tel és 8-cal nem osztható,
de osztható a 2, 3, 4, 5, 6, 9, 10, 11, 12, 13 számokkal. E számok legkisebb közös
többszöröse: 4 · 5 · 9 · 11 · 13 = 25 740. Mivel 2 · 25 740 > 50 000, ezért a felírt
szám csak a 25 740 lehet.
501. A sorozat elemeinek szorzata 8, így a 0 nem eleme a sorozatnak. Ha a sorozat el-
b 1 1 a
ső két eleme a és b, akkor felírhatók a sorozat elemei: a, b, , , , , a, b, . . .
a a b b
Tehát a sorozat első hat eleme periodikusan ismétlődik. Egy-egy periódusban a
hat elem szorzata 1.
b 1 b2
Az első 40 elem szorzata: a ·b · · = = 8, az első 80 elem szorzata: a ·b = 8.
a a a
3
Ezt a két egyenletet összeszorozva: b = 64, azaz b = 4, a = 2.
502. Vegyük észre, hogy a sorozat 7-tel osztható tagjainak indexe osztható 8-cal; a
sorozat 3-mal osztható tagjainak indexe pedig osztható 4-gyel. Ezért a sorozat
7-tel osztható tagjai 3-mal is oszthatók.
503. Az állítással ellentétben tegyük fel, hogy mindkét sorozat periodikus; akkor van
közös periódushossz. Mivel an = bn (1 + cn ), így azt kapjuk, hogy az an sorozat
is periodikus.
504. an+1 − 1 = an (an − 1) = an an−1 (an−1 − 1) = · · · = an an−1 . . . a1 (a1 − 1) =
= an an−1 . . . a1 .
505. abcd − dcba = 999(a − d) + 90(b − c) = 1008, azaz 111(a − d) + 10(b − c) = 112,
tehát a − d = 1, 10(b − c) = 1, s ez utóbbinak nincs megoldása.
506. ab = 10a + b > 9a ≥ a · b.
507. A szép számok: 6, 8, 10, 14, 15, 21, 22, 26, 27, 33, . . .
 3  3
508. 1050 < 10150 + 5 · 1050 + 1 < 1050 + 1 . Indokolható azzal is, hogy köbszám
9-cel osztva nem adhat 7 maradékot.
511. A számok összege egy számmal osztva annyi maradékot ad, mint az összeadan-
dók osztási maradékainak összege, 3-mmal való osztás esetén ez pedig mege-
gyezik a számjegyek összegével. Azt kapjuk, hogy ez az összeg 3-mal osztva 0
maradékot ad, ezért az összeg nem lehet 100.
512. Ha egy számból levonjuk jegyei összegét, 9-cel osztható számot kapunk. Ezért az
első elvétel után 9-cel osztható számok követik egymást. Legutoljára 9 aranyból
vettek. Visszafelé követve, az aranyak száma ebben a sorrendben rendre: 9, 18,
27, 36, 45, 54, 63, 72, 81, (vigyázat! a 90 nem szerepelhet) 99, 108, 117, 126,
135. Innen leolvasható, hogy a főnök ötödik a névsorban, s neki 27 arany jutott
a zsákmányból.
218 Megoldások, útmutatások

513. Az 1, 2, 3, . . . , 16 számokból ki kell hagyni négyet. Figyeljünk arra, hogy mind-


két szorzatban minden prímnek ugyanolyan kitevővel kell szerepelnie! 11 és 13
elhagyása egyértelmű, továbbá a 2 és az 5 prímekkel osztható számokból is el
kell hagyni. Mivel a megmaradó 12 szám összege páros, ezért a 11 és 13 szá-
mok mellé vagy két páratlan, vagy két páros számot kell választani. Páros számot
mindenképp kell választani, ezért a másik két szám csak a 10 és a 4 lehet (a fel-
adat kikötése miatt a 4 helyett a 16 nem hagyható el). A megmaradó 12 szám
összege 98, tehát egy-egy padon az életkorok összege 49 év. A 16 mellé a 2,
6, 14 számokból kell kettőt választani (így teljesülhet az, hogy a 2-es tényezők
száma mindkét szorzatban ugyanannyi, és a hat szám összege páratlan). Ezeket a
lehetőségeket megvizsgálva két megoldást találunk: (16, 15, 7, 6, 3, 2), (16, 14,
9, 5, 3, 2).
514. A legkisebb szám az, amellyel bármely két szám összege osztható, és ennek
a számnak többszöröse a többi szám. Ezen szám ismeretében akkor található
ki mind az öt szám, ha 90-ig csak öt olyan szám van, mely osztható ezzel a
számmal. Három ilyen szám van: 16, 17, 18. Ezek közül akkor egyértelmű a
választás, ha a szám páratlan, azaz 17. A nyerő számok: 17, 34, 51, 68, 85.
515. Haladjunk sorban a kérdésekkel és a lehetséges válaszokkal!
1. „Nagyobb, mint 500?” Ha igennel válaszol: 501–1300 közötti szám (valójában
— hiszen a válasz hamis — 13–500 közti szám).
2. „Négyzetszám?” Ha igennel válaszol: 16, 25, 36, 49, 64, 81, 100, 121, 144,
169, 196, 225, 256, 289, 324, 361, 400, 441, 484, 529, 576, 625, 676, 729, 784,
841, 900, 961, 1024, 1089, 1156, 1225, 1296 (valójában az összes többi szám).
3. „Köbszám?” Ha igen a válasz: 27, 64, 125, 216, 343, 512, 729, 1000.
4. „Az utolsó előtti számjegye 1-es?” Az erre adott válasz után kitalálható a
házszám. Tekintsük át, hogy az eddigi kérdésekre kapható válaszok után milyen
információink vannak! (A kérdésekre kapható igen és nem válaszokat I és N
jelöli.)
I I I : 729.
NI I : 64.
I NI : 512, 1000.
NNI : 27, 125, 216, 343.
I I N: 529, 576, 625, 676, . . . (túl sok megoldás).
NI N: 16, 25, 36, 49, . . . (túl sok megoldás).
I NN: túl sok megoldás.
NNN: túl sok megoldás.
Ezek alapján biztos, hogy Horváth az I NI válaszsort kapta kérdéseire, mert csak
ez ad kétesélyes lehetőséget (amelyben az egyik szám utolsó előtti jegye 1-es).
Mivel azonban Kovács hazudott, ennek ellentéte: NI I adja a helyes válaszokat,
eszerint a házszám 64.
10. Különféle számelméleti feladatok 219

516. 1664 = 128 · 13 = 27 · 13. A gyerekek között van 13 vagy 26 éves. Ha az egyik
gyerek 26 éves, akkor lehetne még 16, ill. 32 éves korú gyermeke az anyának
(apának), de ekkor a szorzat nem lehet megfelelő. Az egyik gyerek 13 éves, két
testvére van; egyik 8, másik 16 éves. 3 gyerek van a családban.
517. Anna nem tudta meghatározni a gyerekek életkorát. Ez azt jelenti, hogy szor-
zatukat többféleképpen is fel lehet bontani két 10-nél kisebb természetes szám
szorzatára. Ez a következő lehetőségeket jelenti. (Egymás alá írtuk az egyforma
szorzatokat adó párokat.)
1, 4 1, 6 1, 8 1, 9 2, 6 2, 8 2, 9 3, 8 4, 9
2, 2 2, 3 2, 4 3, 3 3, 4 4, 4 3, 6 4, 6 6, 6

Bori tudhatja, hogy az életkorokat a felsorolt számpárok valamelyike adja meg.


Tudjuk, hogy a korkülönbség ismerete egyértelművé teszi a választást. A 18
számpár között csak a (6, 2), (8, 2), (9, 1) párok azok, melyek különbsége egyszer
fordul elő. Ezek közül kell választani. Mivel az életkorok hányadosának ismere-
tében nem egyértelmű a választás, ezért 6 és 2, ill. 9 és 1 évesek nem lehetnek
a gyerekek, mert itt a hányados ismeretében ki lehet választani a számpárt. A
gyerekek csak 8 és 2 évesek lehetnek.
518. A 2450-et szorzatokra bontjuk. A három személy életkorára több lehetőség kí-
nálkozik, mindegyikhez más összeg tartozik.
Mivel a sekrestyés ismeri a saját életkorát, meg kellene találnia a helyes választ.
Ez mégsem sikerül, amiből arra következtetünk, hogy több lehetőség áll előttünk.
Megvizsgáljuk, hogy a három életkor összege milyen kombinációkban fordul elő;
és azt tapasztaljuk, hogy csak a 64-es szám jelenik meg kétszer: 49 + 10 + 5 = 64,
50 + 7 + 7 = 64. Tehát a sekrestyés 32 éves.
Amikor a pap megjegyzi, hogy az egyik személy idősebb nála, akkor e kijelentés
alapján a sekrestyés választani tud a két lehetőség közül.
Tehát a pap 49 éves.
519. Három olyan számot keresünk, melyek szorzata 225, összege egy ismeretlen
szám; melyet az összeíró tudott, s még mindig kérdeznie kellett. A 225-nek csak
két olyan felbontása van, melynél a 3–3 szám összege ugyanaz: 1, 15, 15, és 3,
3, 25. Ez utóbbi adja a megoldást.
520. Soroljuk fel azokból a pozitív egészekből álló számhármasokat, melyeknél a
három szám szorzata 36. Ismerjük még a három szám összegét is, ám ebből nem
állapítható meg egyértelműen, hogy melyik számhármasról van szó. Azaz, a felírt
számhármasok közül válasszuk ki azokat, melyek összege másik számhármasnál
is ugyanaz. Szerencsére csak az 1, 6, 6 és a 2, 2, 9 számhármas ilyen. Tehát a
három gyerek életkorát ezek egyike adja meg. A választást a két idősebb gyerekre
történő utalás segíti. Az életkorok 1, 6, 6 év. A beszélgetés a hónap 13. napján
(1+6+6=13) történt.
220 Megoldások, útmutatások

521. András a különbség ismeretében akkor tudná megmondani Tamás számát, ha


gondolt száma a és 0 < a ≤ 1995. Tehát a > 1995. Ha Tamás gondolt száma
t, és t ≤ 3990, akkor tudná, hogy a = t + 1995. Ezért t > 3990. Ezt követően
András már tudja Tamás számát, ami csak akkor lehet, ha 1995 < a ≤ 5985, s
ekkor t = a + 1995. Ha 1-gyel nagyobb számra gondoltak volna, akkor András
még nem tudná Tamás számát; ami úgy lehet, ha András száma 5985, Tamás
száma 7980.
522. András első kijelentése után tudjuk, hogy a nála levő szám nem 1. Tamás első
válasza után tudható, hogy a nála levő szám nem 1 vagy 2. András második
válasza után tudjuk, hogy a nála levő szám nem 1, 2 vagy 3. Tamás második
válasza után tudható, hogy a nála levő szám nem 1, 2, 3 vagy 4. . . . Tamás
tizedik válasza után tudjuk, hogy a nála levő szám nem 1, 2, . . . , 19 vagy 20.
Andrásnál vagy a 20 van és Tamásnál a 21, vagy a 21 van nála és Tamásnál a
22.
523. A szorzat ismeretében A nem tudta meghatározni a gondolt számokat, tehát a
p = a · b szorzat nem lehet két prím szorzata. B az s = a + b összeg ismeretében
már tudta, hogy p nem lehet két prím szorzata, tehát s nem állítható elő két prím
összegeként. Ezért s nem lehet páros (hiszen a Goldbach-sejtés szerint a páros
számok előállnak két prím összegeként), s a páratlanok közül sem lehet olyan,
amely egy prímszámnál kettővel nagyobb. Így s lehetséges értékei:
M = {11,17,23,27,29,35,37,41,47,51,53,57,59,65,67,71,77,79,83,87,89,93,95,97}.
Ha s = 11, akkor 11 = 2 + 9 = 3 + 8 = 4 + 7 = 5 + 6, így p-nek a 18, 24, 28 és 30
számok egyikének kell lennie. Azonban a 18, 24 és 28 egyértelműen bontható
két olyan tényező szorzatára, melyek összege M-ben van: 18 = 2 · 9 = 3 · 6,
2 + 9 = 11 ∈ M, és 3 + 6 = 9 ∈ / M; 24 = 3 · 8 = 4 · 6, 3 + 8 = 11 ∈ M, és
4 + 6 = 10 ∈ / M; 28 = 4 · 7 = 2 · 14, 4 + 7 = 11 ∈ M, és 2 + 14 = 16 ∈ / M. Ezért
a legvégén B nem tudna választani. Ha pedig s = 11 és p = 30 lenne, akkor
p = 2 · 15 = 5 · 6 alapján A nem tudna dönteni, mert 2 + 15 = 17 és 5 + 6 = 11
is a lehetséges s értékek között van. (A azért tudja a két szám összegét kitalálni,
mert a p szorzat csak egyféleképp írható úgy a · b alakba, hogy a + b nem áll
elő két prím összegeként.) Tehát s = 11 esetén nem tudnák kitalálni a gondolt
számokat.
Vegyük a következő lehetséges értéket: s = 17. Ekkor 17 = 2 + 15 = 3 + 14 =
= · · · = 8 + 9 felbontások alapján p értéke 30, 42, 52, 60, 66, 70, ill. 72 lehet.
Ha p = 30, akkor — mint előbb láttuk — A nem tudna dönteni, hiszen s-re két
lehetséges érték is szóba jön.
Ugyanígy kétesélyes a p = 42 lehetőség is, mert a 42 = 3 · 14 = 2 · 21 felbontá-
sokból kapható 3 + 14 = 17, 2 + 21 = 23 számok mindegyike eleme M-nek.
Ha s = 17 és p = 52, akkor az 52 = 2 · 26 = 4 · 13 felbontásokból kapható
2 + 26 = 28, 4 + 13 = 17 számokból csak a 17 eleme M-nek; A kitalálhatja s
értékét, ill. a két gondolt számot: a 4 és 13 számokat.
10. Különféle számelméleti feladatok 221

A megmaradó lehetőségeket elemezve sem találunk további megoldásokat; így a


gondolt számok: 4 és 13.
Megjegyzés. A megoldásban kihasználtuk az s < 100 megkötést. Számítógéppel
az s < 2 000 000 feltétel mellett is vizsgálták a feladatot, s akkor is csak a 4
és 13 adódott egyedüli megoldásnak. Valószínűnek látszik, hogy ez az egyetlen
megoldás akkor is, ha s nagyságára nem teszünk semmilyen korlátozást.
524. x = 4,75.
525. a) 4,
b) 6 és 25.
526. Csak a p4 (p prím) alakú számoknak van öt osztója.
527. a) 48,
b) 60.
528. 60, 72, 84, 90, 96.
529. Ha az N szám prímtényezős alakja N = p1α1 · p2α2 · . . . · pkαk , akkor N osztóinak
száma d(N) = (α1 + 1) · (α2 + 1) · . . . · (αk + 1).
Egy szám osztóinak száma pontosan akkor páratlan, ha az a szám négyzetszám.
Az N szám nem négyzetszám, mert prímtényezős felbontásában az 5 kitevője
páratlan szám (= 3). Ezért az osztók száma nem lehet 25 323.
Mivel a prímtényezős felbontásban 53 szerepel, ezért az osztók száma
d(N) = . . . · (3 + 1) · . . . osztható 4-gyel, míg 25 322 nem osztható 4-gyel, ezért
az osztók száma nem lehet 25 322.
530. 23 · 34 .
531. Nincs.
532. 215 · 320 · 524 .
533–538. feladatokhoz:
 Legendre
  tétele
 szerint n! prímtényezős felbontásában a p
n n n
prím kitevője: + + +...
p p2 p3
539. 333. (1000−{2-vel oszthatók száma}−{3-mal oszthatók száma}+{6-tal oszthatók
száma}).
540. 1000 − n2 − n3 − n5 + n6 + n10 + n15 − n30 , ahol nk az 1000-et meg nem haladó,
k-val osztható pozitív egészek száma („szita-formula”).
541. Az előző két feladat megoldásához hasonlóan járjunk el.
542. Amivel osztottunk, az osztója 539 − 327 = 212-nek.
543. 17.
222 Megoldások, útmutatások

544. A keresett számból vonjunk ki 1-et. Ez a szám már maradék nélkül osztható a 2,
3, 4, 5, 6 számokkal.
545. A keresett számhoz adjunk 1-et.
546. A keresett szám legyen x. 2x 5-tel osztva 1, 7-tel osztva 1, 9-cel osztva is 1
maradékot ad, tehát 2x − 1 osztható 5-tel, 7-tel, 9-cel, azaz 315-tel. Ezért 2x −
− 1 = 315, 2x = 316, x = 158.
549. n = 8, n = 10, n ≥ 12. (Ha 2 | n, akkor n = 4 + (n − 4), ha 2  | n, akkor
n = 9 + (n − 9).)
550. n ≥ 5, n = 6.
Ha 2  | n, akkor n = 2 + (n − 2),
ha n = 4k, akkor n = (2k − 1) + (2k + 1),
ha n = 4k + 2, akkor n = (2k − 1) + (2k + 3).
551. n = 10, n ≥ 12. n ≥ 12 esetben a következőképpen láthatjuk be:
ha n = 4k + 2, akkor n = (2k − 1) + 2 + (2k + 1),
ha n = 4k + 3 és 3 | k, akkor n = (2k − 1) + 3 + (2k + 1),
ha n = 4k + 3 és 3  | k, akkor n = (2k − 3) + 3 + (2k + 3),
ha n = 4k + 4, akkor n = (2k − 1) + 4 + (2k + 1),
ha n = 4k + 5 és 5 | k, akkor n = (2k − 1) + 5 + (2k + 1),
ha n = 4k + 5 és 5  | k, akkor n = (2k − 5) + 5 + (2k + 5).
Lehetne másképp is, pl. a 6k, 6k + 2, 6k + 4, 12k + 1, 12k + 3, 12k + 5, 12k + 7,
12k + 9, 12k + 11 alakú számokat állítanánk elő a kívánt alakban.
552. Hívjuk a racionális számok egy 13 elemű halmazát jó-nak, ha teljesül rá a fel-
adat feltétele, tehát közülük bármely 12 szám két hatos csoportba osztható úgy,
hogy az egyes csoportokban levő számok összege megegyezik. Szorozzuk meg
a 13 racionális számot a nevezők egy közös többszörösével, ekkor olyan egész
számokat kapunk, melyek jó halmazt alkotnak. Válasszuk ki a számok közül
valamelyiket, és vonjuk ki ezt a számot mindegyikből. Az így kapott 13 szám
jó halmazt alkot, és közte van a 0 szám. Ha a számok között van páros, akkor
mindegyik szám páros. (Miért? Gondoljuk végig!) Vegyük mindegyik számnak
a felét. Az így kapott 13 szám mindegyike egész szám és jó halmazt alkotnak. A
számok között van a nulla, tehát mindegyik páros, így az előző eljárást megismé-
telhetjük. Mivel ezt az eljárást a számokon végtelen sokszor megismételhetjük,
ez csak úgy lehet, ha a számok mindegyike 0. Ez azt jelenti, hogy a feladatban
szereplő 13 szám egyenlő.
553. Hasonló a következő feladathoz.
554. A számokból képezhető szorzatok száma 211 − 1. Vizsgáljuk a szorzatokat asze-
rint, hogy prímtényezős alakjukban a prímek kitevője páros vagy sem. A prí-
mek között csak az első tíz prím szerepel, és ezért 210 a különböző lehetőségek
száma arra, hogy egy-egy számban e tíz prím kitevője páros vagy páratlan. A
11. Számok reciprokainak összege 223

lehetőségek száma kevesebb a szorzatok számánál, ezért van két azonos visel-
kedésű szorzat. A két szorzat mindegyikéből hagyjuk el a közös tényezőket (ez
nem változtat a két szorzatban a prímek kitevőjének azonos paritásán), és az így
megmaradt két szorzatot összeszorozva négyzetszámot kapunk.
   
48 48
556. A 48 számból kéttényezős szorzatot lehet készíteni. Mivel > 210 ,
2 2
így van két olyan szorzat, melynek prímtényezős alakja azonos kitevő-paritású.
Legyen ez a két szorzat ab és cd. Ha a, b, c, d különböző számok, akkor ez a
négy szám jó, azaz szorzatuk négyzetszám. Ha nem különbözők, például b = d,
akkor ac négyzetszám.
  A maradék 46 számra az eddigi okoskodás megismétel-
46
hető, mert > 210 , ezért ezek között is lesz vagy négy különböző szám,
2
vagy csak kettő, x és y, melyek szorzata négyzetszám. Ez utóbbi esetben a, b,
x, y a megfelelő négy szám.
557. A feladat állítása már 1537 számra is igaz. Vegyünk közülük 513 db számot.
Ezek között van kettő – (a1 , b1 ) –, melyek szorzata négyzetszám. (Az 513 szám
között van kettő, melyek prímtényezős felbontásában a prímkitevők párossága
azonos, ezért szorzatuk négyzetszám. 26-ig 9 prímszám van, ezért a pozitív egész
kitevőjű hatványaik szorzatából álló számokban a prímkitevők párossága szerint
29 = 512 különböző lehetőség van.) Az 513 számból az (a1 , b1 ) számpárt ki-
vesszük, helyettük a többiek közül hozzáveszünk kettőt. E között az 513 szám
között is lesz kettő – (a2 , b2 ) –, melyek szorzata négyzetszám. Ily módon az 1537
szám közül kiválaszthatunk 513 számpárt, melyekben a két szám szorzata négy-
zetszám, xi2 = ai · bi . Az xi2 számokat aszerint tekintsük, hogy benne egy-egy
prím (páros) kitevője 4-gyel osztva 0 vagy 2 maradékot ad! Így 29 = 512 eset
lehetséges, tehát van e között az 513 négyzetszám között kettő, mely azonosan
viselkedik ebből a szempontból, ezek szorzata negyedik hatvány lesz. Megtalál-
tuk azt a négy számot, melyek szorzata negyedik hatvány.

11. Számok reciprokainak összege

560. Legyen 2t ≤ n < 2t+1 . Vegyük a nevezők legkisebb közös többszörösét, s hozzuk
a törteket erre a legkisebb közös nevezőre! Mindegyik tört számlálója páros lesz,
1
kivéve azt a törtet, melyet az t törtből kapunk. Így az összeg számlálója páratlan
2
lesz (hiszen páros számokat és egy páratlan számot adunk össze), nevezője páros,
tehát a hányados nem lehet egész szám.
n
Második megoldás. Tekintsünk egy olyan p prímet, melyre < p ≤ n (ilyen
2
prím Csebisev tétele miatt létezik)! A törteket a legkisebb közös nevezőre hozva
a nevező p többszöröse lesz, míg a számláló nem.
224 Megoldások, útmutatások

1 1 1 1 1
561. < 2 = = − , ahol n > 1.
n2 n − n n(n − 1) n − 1 n     
n
1  n
1 1 1 1 1
Ennek alapján < 1+ − = 1+ 1− + − +
k2 k−1 k 2 2 3
  
k=1 k=2
1 1 1 1 1
+ − + ···+ − = 2− .
3 4 n−1 n n
562. Hasonló az 560. feladat első megoldásához.
563. Hasonló az 560. feladat első megoldásához, csak itt a 2-hatványok helyett 3-
hatványokat figyelünk.
564. Hasonló az 560. feladat első megoldásához. Két lehetőség is kínálkozik: csak
az egyik tört nevezője osztható 27 -nel, ill. csak az egyik tört nevezője osztható
34 -nel.
565. Hasonló az előző feladat megoldásához. A nevezők közül csak az egyik osztható
53 -nel.
566. A hatványszámok
 reciprokainak
  összege kisebb2-nél, hiszen:
1 1 1 1 1 1 1 1
1+ 2 + 3 + 4 + . . . + 2 + 3 + 4 + . . . +· · · = 1+ + +· · · = 2.
2 2 2 3 3 3 1·2 2·3
567. Levezethető az előző feladat állításából.
  
∞  ∞
1 ∞
1 ∞
1 3
568. = = 2 · = 3.
α=0 β=0
2 ·3
α β
α=0
2 α
β=0
3 β 2
 n
1 1
569. Lássuk be teljes indukcióval, hogy =2− .
a
i=1 i
a1 a2 . . . an
    
1 1 1 n
570. Az összeg értéke: 1 + 1+ · ...· 1 + −1= .
2 3 n+1 2
572. Az összeg értéke: 88.
573. a) Nem. Az összeg értéke mindig páratlan.
b) Nem. Lásd az 560. feladatot!
574. Állítsuk párba az összeadandókat, elsőt az utolsóval, másodikat az utolsó előttivel
stb. Egy-egy kéttagú összegben a közös nevezőre hozás után a számláló mindig
p, a nevező pedig p-vel nem osztható szám. Ez a tulajdonsága meglesz a törtek
összeadása után kapott törtnek is.
576. Előbb lássuk be az
1 1 1 1 1 1 1 1 1
1 − + − + − ··· − + = + + ···+
2 3 4 5 2n 2n + 1 n + 1 n + 2 2n + 1
12. Számok és számjegyek 225

összefüggést. Ez igazolható teljes indukcióval is, de most lássunk egy másik


bizonyítást.
1 1 1 1 1 1
1 + + + ···+ = 1 + + + ··· +
2 3 2n 2 3 2n
1 1 1 1 1 1 1
1 + + + ···+ = 2 · + 2 · + 2 · + ···+ 2 ·
2 3 n 2 4 6 2n
A két egyenlőség különbsége adja a kívánt összefüggést.
Ezeknek az összefüggéseknek a segítségével a feladatbeli összeg másképp írható:
     
1 1 1 1 1 1 1 1
+ ···+ = + + + + ···+ + .
660 1319 660 1319 661 1318 989 990
Mivel 1979 prímszám és a zárójelekben a közös nevezőre hozás után mindenhol
1979 a számláló, a nevező pedig nem osztható 1979-cel, így igaz az állítás.
 
1 1 1 1 (k + 1) − (k − 1) 1 1 1
579. 3 < 3 = = · = − ,
k k − k (k − 1)k(k + 1)  2 (k − 1)k(k + 1) 2 (k − 1)k k(k + 1)
1 1 1 1 1 1 1
így a feladatbeli összeg < · − + − + − +···+
2 2 · 3 3 · 4 3 · 4 4 · 5 4 · 5 5 ·6
1 1 1 1 1
+ − < · = .
(n − 1) · n n · (n + 1) 2 2 · 3 12

12. Számok és számjegyek

580–585. Használjuk fel, hogy n ≡ S(n) (mod 9).


586. Nem igaz. Legyen n = 99, m = 100, ekkor f (99) = f (18) + 1 = f (9) + 2 = 2,
f (100) = f (1) + 1 = 1, és így f (99) > f (100).
587. Nem lehet. Lásd az 580. feladat állítását.
588. Tegyük fel, hogy létezik két ilyen szám, a és b. Legyen a < b. Ekkor a | b és
a | b − a, továbbá 9 | b − a. Mivel a és 9 relatív prímek, így 9a | b − a, de ez
nem lehet, mert 9a > b − a; 10a > b, hiszen 10a már nyolcjegyű szám.
589. Nyilván A, B és C is osztható 9-cel. A < 1994 · 9 ≤ 18 000, B < 37, C < 18,
tehát C = 9.
590. Hasonló az előző feladathoz.
593. n = 69 999, n + 1 = 70 000. (n = 159 999, n + 1 = 160 000.)
594. n = 48 999, n + 1 = 49 000. (n = 98899 . . . 99 (a 8-as után 16 db 9-es), n + 1 =
= 989000 . . . 00.)
595. n + 1 = 8999 . . . 9900 . . . 00 (13 db 9-es, 14 db 0).
226 Megoldások, útmutatások

596. a) Nincs. b) Nincs.


597. Legyen 10a < 21994 < 10a+1 , valamint 10b < 51994 < 10b+1 .
Ekkor 10a+b < 21994 · 51994 = 101994 < 10a+b+2 , tehát a + b = 1993.
Mivel 21994 (a + 1)-jegyű, 51994 (b + 1)-jegyű, így a keresett érték 1995.
Megjegyzés. Lásd még a 494. feladat megoldását.
598. Az előző feladat megoldását követve kapjuk, hogy:
k(21090701 ) + k(51090701 ) = 1090702. Mivel a két szám összege páros, különbségük
is páros.
599. Nem. Legyen a két hatvány 2n és 2k , n > k. 9 | 2n − 2k = 2k (2n−k − 1), azaz
2n
9 | 2n−k − 1; de ez nem lehet, hiszen 2n−k = k < 10, ezért 2n−k − 1 < 9.
2
600. A mind a tíz számjegyet tartalmazó tízjegyű szám osztható 9-cel. A feladatban
leírt átalakítások során kapott számok 9-es maradéka mindig ugyanaz, és így nem
kaphatunk 9-cel osztható számot.
601. Az n számnak a p + 1, p + 2, …. , p + n számok valamelyike többszöröse. Legyen
p = 1234567890 · 10k , ahol 10k > n, s ekkor az előbbi többszörös megfelelő
tulajdonságú.
603. A sorozat első eleme a1 , differenciája d. Ekkor a sorozatnak két megfelelő eleme
lesz a1 + 10n d és a1 + 10n+1 d, ha n elegendően nagy.
604. A polinom értékének pozitívnak kell lennie, ezért n > 11. P (n) ≤ n, ezért n < 13.
Így n értéke csak 12 lehet, s ez ki is elégíti a megadott egyenlőséget.
605. n = 222 . . . 22111 . . . 11 (k db 2, 2k − 2k db 1).
606. Ilyenek például az olyan, 9-cel osztható számok, melyek nem tartalmaznak 3, 6,
9 számjegyeket. (Megfelelő számok azok is, melyekben van 0 számjegy.)
608. Ilyen szám pl. n = 111 . . . 11995125 (a szám elején 94 db 1-es). S(n) = 125.
609. Válasszuk a 18 szám közül azt az n számot, mely osztható 18-cal. Ekkor S(n)
értéke csak 9 vagy 18 lehet, így S(n) | n.
614. Lássuk be, hogy a sorozat korlátos.
615. Ha k n + 1-jegyű, úgy bármely m > n esetén t = 10m − 1 jó szám.
617. n = 111 . . . 11000 . . . 00 (10 db 1-es, 10 db 0).
618. Van, pl.: 10 111 111 111. (Erre az is igaz, hogy S(n) = 10.)
619. Ha S(n) = k és S(n2 ) = k 2 , k(n) = t, akkor az n1 = 10t+1 · n + 1 számra S(n1 ) =
= k + 1, S(n21 ) = (k + 1)2 .
13. Racionális és irracionális számok 227

629. n = 1995 · S(n). Tehát n osztható 3-mal, ekkor S(n) is osztható 3-mal, azaz n
osztható 9-cel, így S(n) osztható 9-cel. Ezért n = 1995 · 9k, azaz n az 1995 · 9,
1995 · 18, . . . számok közül kerül ki. n = 1995 · 18 = 35 910 lesz a keresett szám.
630. Legyenek a számok: a = 5 555 554 445, b = 5 554 445 555, c = 4 445 555 555.
Ekkor S(a + b) = S(11 110 000 000) = 4, S(b + c) = S(10 000 001 110) = 4,
S(c + a) = S(10 001 110 000) = 4, S(a + b + c) = S(15 555 555 555) = 51 > 50.
Hogyan lehet ezekre a számokra gondolni?
S(2(a + b + c)) = S((a + b) + (b + c) + (c + a)) ≤ S(a + b) + S(b + c) + S(c +
+ a) ≤ 12. Ez azt jelenti, hogy az n = 2(a + b + c) szám felében a számjegyek
összege lényegesen nagyobb, mint az n szám esetén. Ez lehetséges akkor, ha az
n szám sok egyesből áll, míg a szám felében sok 5-ös számjegy van. Legyen n =
= 31 111 111 110, ekkor S(n) = 12, S(n/2) = 51. Az n számot bontsuk fel három
n/2-nél kisebb szám összegére úgy, hogy ezekben a számokban a számjegyek
összege 4 legyen: n = 11 110 000 000 + 10 000 001 110 + 10 001 110 000, továbbá
a + b = 11 110 000 000, b + c = 10 000 001 110, c + a = 10 001 110 000. Innen
megkapjuk a fent megadott példákat.
631. Vannak ilyen számok, pl.:
x = 999 999 999 999 891, y = 999 999 999 999 900, z = 1 000 000 000 000 008.
Ekkor x + S(x) = y + S(y) = z + S(z) = 1 000 000 000 000 017.
Teljes indukcióval igazolható, hogy tetszőleges k pozitív egészhez léteznek olyan
x1 , x2 , . . . , xk pozitív egészek, amelyekre x1 + S(x1 ) = x2 + S(x2 ) = · · · = xk +
+ S(xk ). Ez a k = 3 esetben megválaszolja a feladat kérdését.

13. Racionális és irracionális számok

632. A bal oldalon irracionális, a jobb oldalon racionális szám áll.


√ √
633. Lehet. Pl.: 2 − 2, 2 + 2.
634. Nem lehet. Ha x, y irracionális számok, s x +y és x −y is racionális lenne, akkor
(x + y) + (x − y) = 2x is racionális lenne, noha 2x irracionális.
635. Nincs. Legyenek x, y, z irracionális számok, s tegyük fel, hogy p = x + y, q =
= y + z, r = z + x racionális számok. Ekkor 2y = p + q − r racionális számot
jelöl, ami nem lehet, hiszen y irracionális.

636. Van. a = 2, b = log√2 3.
p
637. Ha log2 5 = , ahol p és q egész számok, akkor 2p = 5q lenne.
q
638. A tg 2x-re ismert összefüggés alapján az indirekt feltevés azt adja, hogy tg 10◦ ,
tg 20◦ racionális, majd az addíciós tételből tg (10◦ + 20◦ ) = tg 30◦ racionális lesz,
holott tg 30◦ irracionális.
228 Megoldások, útmutatások

639. sin 3x = 3 sin x − 4 sin3 x miatt az indirekt


√ feltevés szerint sin 15◦ értéke racio-
2 √

nális, de sin 15◦ = sin(45◦ − 30◦ ) = 3 − 1 , amely irracionális.


4
p
640. logn (n + 1) = (p, q egész) feltevésből np = (n + 1)q , de ez nem lehet, mert az
q
egyenlőségjel egyik oldalán páros, a másikon páratlan szám áll.
641. Mutassuk meg, hogy a tizedes tört nem válik periodikussá.
642. Mutassuk meg, hogy a tizedes tört nem válik periodikussá. Dirichlet tétele sze-
rint, ha a és b relatív prímek, akkor az ak +b, k = 0, 1, 2, . . . sorozatban végtelen
sok prím van. Ha a tizedes tört periodikussá válik és a periódus hossza t, akkor
a 102t k + 1 alakú prímek miatt a periódus csak nullákból állna. (Igazolható Cse-
bisev tételének felhasználásával is, mely tétel azt állítja, hogy n és 2n (n ≥ 2)
között mindig van prím.)
p
643. Tegyük fel, hogy az összeg racionális, értéke . Ekkor az összeget q!-sal szo-
q
rozva egész számot kell kapnunk. Azonban az összegben az első q összeadandó
egész szám lesz a szorzás után, míg a többi összege 0 és 1 közötti szám.
644. Az előbbi megoldásához hasonló, itt (q!)2 -nel szorzunk.
645. A szám bináris számrendszerbeli felírásában a „tizedesvessző” után a számjegyek
felírásában nincs periodikusság.
p
646. Tegyük fel, hogy az összeg értéke racionális szám, = . A feltételből következik,
q
ai
hogy van olyan k, melyre i ≥ k esetén > 3, és ai > 3ai−1 .
a1 · a2 · . . . · ai−1 · q
Ekkor
p · a1 · a2 · . . . · ak−1 =
 
a1 · a2 · . . . · ak−1 · q a1 · a2 · . . . · ak−1 · q a1 · a2 · . . . · ak−1 · q
= + + ···+ +
a1 a2 ak−1
 
a1 · a2 · . . . · ak−1 · q a1 · a2 · . . . · ak−1 · q
+ + + ... .
ak ak+1
A bal oldal értéke egész szám, a jobb oldalon az első zárójelben levő összeg
értéke is egész, így a második zárójelben álló összegnek is egésznek kell lennie.
1 1 1
Azonban annak tagjai rendre kisebbek az , , , . . . számoknál, melyek
3 9 27
1
összege .
2
p r q s
647. Ha lg n = , lg(n + 1) = , azaz 10 = n p , 10 = (n + 1) r , akkor nqr = (n + 1)ps ,
q s
de ez nem lehet, hiszen az egyik oldalon páros, a másikon páratlan szám áll.
13. Racionális és irracionális számok 229

648. Ha a sorozat két eleme racionális, akkor a sorozat differenciája is racionális, s


ekkor a sorozat minden eleme racionális.
649. Az előző feladat következménye.
√ √  √ √  √ √
650. x − y = x+ y x − y azonosság miatt x − y is racionális, s mivel
√ √ √
x + y racionális, így ezek összege 2 x is racionális.
√ √ √ √
651. Mutassuk meg, hogy z racionális. x + y + z = w, w racionális. Egysze-
 2 2
√ w + z − x − y + 4w2 z − 4xy
rű számítások után z = , s ha a nevező nem
4w(w2 + z − x − y)
nulla, akkor a tört értéke racionális.
652. Mindig irracionális szám lesz a kifejezés értéke.

654. Legyen ai = pi , i = 1, 2, 3, . . . , ahol pi az i-edik prímet jelöli. Tekintsük a
következő táblázatot:
a1 x1 , a1 x2 , a1 x3 , . . . a1 xn
a2 x1 , a2 x2 , a2 x3 , . . . a2 xn
a3 x1 , a3 x2 , a3 x3 , . . . a3 xn
...
...
Ha itt mindegyik sorban van racionális szám, akkor van két sor, melyben va-
lamely k-ra ai xk és aj xk racionális. Ebből következik, hogy ezek hányadosa is
racionális, ami könnyen látható, hogy nem teljesül.
655. Ha mindegyik xi racionális szám, akkor bármely a irracionális szám megfelelő.
Ha pl. x1 irracionális, akkor a1 = x1 , a2 = 2x1 , . . . , an+1 = (n + 1)x1 számok
valamelyike jó, hiszen különben lenne olyan i, j , melyekre valamely k-ra ai + xk
és aj + xk racionális, de ekkor különbségük is racionális lenne.
656. Indirekt feltevéssel élve, vegyük a három pontból két-két pont szakaszfelező me-
rőlegesét! Ezek egyenletében az együtthatók racionális számok, így a két egyen-
let közös megoldása is racionális. Azonban ez a megoldás a kör középpontjának
két koordinátája, s ezek irracionális számok.
657. Legyenek
√ P (a,
√ b) és Q(c, d) két pont a körön, melyek koordinátái racionálisak.
u = 2, v = 3 a kör középpontjának két koordinátája. Felírhatjuk, hogy (a −
−u)2 +(b −v)2 = (c −u)2 +(d −v)2 , ebből 2u(c −a)+2v(d −b) = c2 +d 2 −a 2 −b2 .
Ezt az egyenletet a fentiekben megadott számok nem elégítik ki.
2x ny (n + 1)z
658. A háromszög területe + + , s ez racionális, ha x, y, z racionális.
2 2 2
1
Ha a területet a Heron-képlettel számoljuk, akkor a t = 12n2 + 12n − 9 irra-
4
cionális értéket kapjuk. (A gyökjel alatt 4a + 3 alakú szám áll, amely nem lehet
négyzetszám.)
230 Megoldások, útmutatások

659. Legyenek a négyzet csúcsai A, B, C, D és a kör egy pontja P , melyet A és D


között veszünk fel. Az AP ívhez tartozó kerületi szög α, a kör sugara r. Ekkor
AP = 2r sin α, CP = 2r cos α, √ √
◦ 2 2
BP = 2r sin(α + 45 ) = 2r(sin α + cos α) = (AP + CP ). Ha AP és CP
2 2
racionális, akkor BP irracionális.
660. A négyszöget a koordináta tengelyekkel párhuzamos oldalú téglalapba foglalva,
majd a téglalap területéből elhagyva a „felesleges” háromszögek területét, meg-
kapjuk a négyszög területét, s ez racionális szám lesz,√ha a csúcsok koordinátái
e · 2e · sin 45◦ 2
racionális számok. Másrészt: t = = e2 (e a négyszög kisebb
2 2
2
átlója), s ez irracionális, mert e racionális.
661. Igen, van. Legyen D az AB oldal felezőpontja, s válasszuk P -nek az AB szakasz
egy pontját. Ha a P D = x távolság
 racionális, akkor AP és P B is racionálisak,
√ 3 1
továbbá CP = CD2 + DP 2 = x 2 + = (8x)2 + 48 alapján CP is racio-
4 8
1
nális lesz, ha 8x = 1, azaz x = értéket választjuk.
8
Megjegyzés. A hasonló kérdés négyzetre, vagyis hogy van-e olyan P pont az
egységnyi oldalú ABCD négyzet síkjában, amelyre P A, P B, P C, P D minde-
gyike racionális, mindmáig megoldatlan probléma. A sejtés az, hogy nincs.
662. Legyen (m, n) a kiválaszott rácspont. Tekintsük a négyzetnek azt az (x, y) csú-
csát, amelyre m−x, n−y mindegyike páratlan. E két pont távolságának négyzete
(m − x)2 + (n − y)2 , amely 4-gyel osztva 2 maradékot ad, így nem lehet négyzet-
szám. Emiatt a két pont távolsága irracionális.
663. Van ilyen négyszög.√Válasszunk tetszőlegesen egy egységnyi területű trapézt,
3
melynek alapjai 1 és 2. Tegyük fel, hogy van olyan pont, amelyre mind a négy
2
háromszög területe racionális. A trapéz magassága √3
. A trapéz alapjaira
1+ 2
β
illeszkedő háromszögek magasságai α és √ 3
, ahol α és β racionálisak. Mivel
2
β 2 √
α+ √ √
3
3
= 3
; innen adódik, hogy 2 gyöke lenne az αx 2 +(β+α−2)x+β = 0
2 1+ 2
egyenletnek, ami nem lehetséges.
1 1
664. Lássuk be az + = 1 feltétel szükségességét. Legyen N > 1 egész szám. Az
α β
N
[α · k] ≤ N feltételnek eleget tevő k-k száma + λ1 , ahol |λ1 | ≤ 1; ugyanígy
α
N N
β-ra, s ezek együttes száma + λ1 + + λ2 = N. Az egyenlőséget N-nel osztva
α β
14. Egész együtthatós polinomok 231

1 1
és az N → ∞ határátmenetet elvégezve + = 1.
α β
a 1 1 a
Ha α racionális, α = , akkor + = 1 miatt β = , s ekkor fennáll az
b α β a−b
[α · b] = [β · (a − b)], tehát α nem lehet racionális.
1 1
Lássuk még be, hogy ha α > 0 és β > 0 irracionális számokra + = 1
α β
teljesül, akkor bármely N pozitív egész szám szerepel az [α · k], k = 1, 2, . . . ,
vagy a [β · k], k = 1, 2, . . . sorozat valamelyikében, de csak az egyikben.

14. Egész együtthatós polinomok

665. Ha x 4 + x 3 + ax 2 + bx + c = (x − 1)f (x) + 1, úgy az x = 1 helyettesítés az


1 + 1 + a + b + c = 1 összefüggést adja.
666–667. Hasonló az előzőhöz.
668. x 2 − 3x + 2 = (x − 1)(x − 2). f (1) = 0 és f (2) = 0, tehát az f (x) polinomból az
(x − 1) és az (x − 2) gyöktényező kiemelhető.
669. Az x = 1 helyettesítés után kapott érték adja a választ.
671. Mivel r gyöke az egyenletnek, így (r − a)(r − b)(r − c)(r − d) = 4. A bal oldalon
négy különböző egész szám szorzata áll; azonban négy különböző egész szám
szorzata csak úgy lehet 4, ha azok valamilyen sorrendben 1, 2, −1, −2. Ezért
(r − a) + (r − b) + (r − c) + (r − d) = 1 + 2 − 1 − 2 = 0, tehát 4r = a + b + c + d.
672–678. feladatok megoldásánál az a − b | f (a) − f (b) összefüggést célszerű fel-
használni, ahol f (x) egész együtthatós polinom, a és b egész számok; ill. azt a
tulajdonságot, hogy ha f (x) egy páratlan számra páros értéket vesz fel, akkor
minden páratlan számra páros értéket vesz fel, s ez hasonlóan így van egyéb
paritások esetén is.
679. Megmutatjuk, hogy bármely m-re 3 | f (m). Az előző feladatok megoldásából
tudjuk, hogy
m − k | f (m) − f (k),
m − (k + 1) | f (m) − f (k + 1),
m − (k + 2) | f (m) − f (k + 2).
Az m − k, m − (k + 1), m − (k + 2) három egymást követő egész szám valamelyike
osztható 3-mal, ezért ott az f (m) − f (k + i) is osztható 3-mal, s mivel f (k + i)
osztható 3-mal (0 ≤ i ≤ 2), így f (m) is osztható 3-mal.
680. Indirekt úton bizonyítsunk.
232 Megoldások, útmutatások

681. Tekintsük azt az esetet, amikor f (x) és g(x) fokszáma azonos:


f (x) = b0 x k + b1 x k−1 + · · · + bk , g(x) = c0 x k + c1 x k−1 + · · · + ck , és b0 · c0 = 0.
A feladat szerint minden n természetes számhoz van olyan αn egész, amelyre
f (x) b0 b0
f (n) = αn · g(n). Mivel lim = , így lim αn = . Az αn számok
x→∞ g(x) c0 x→∞ c0
egészek, ezért az egész számok αn sorozata csak úgy konvergálhat, ha bizonyos
b0
n ≥ K-tól kezdve αn állandó, vagyis αn = α = , n ≥ K. Ez utóbbi egyenlőség
c0
végtelen sok esetben teljesül, ezért minden x valós számra f (x) = αg(x). Ha
f (x) = h(x) · g(x) + r(x), ahol az r(x) polinom fokszáma kisebb, mint g(x)
fokszáma, és h(x), r(x) racionális együtthatós polinomok, akkor van olyan m
egész szám, melyre h1 (x) = m·h(x), r1 (x) = m·r(x) egész együtthatós polinomok.
Az m · f (x) = h1 (x) · g(x) + r1 (x) összefüggés és a feladat állítása miatt minden n
egész számra g(n) osztója r1 (n)-nek. Ahogyan a bizonyítás első részében tettük,
olyan okoskodással megmutatható, hogy r1 (x) ≡ 0.
682. Indirekt úton bizonyítjuk. Legyen r egész gyök, f (x) = (x − r) · g(x),
r = p + kq, 1 ≤ p ≤ k, de f (p) = (p − r)g(p) = −kqg(p), s így k | f (p) lenne.
683. Legyenek a, b, c, d gyökei az f (x) polinomnak!
Ekkor f (x) = (x − a)(x − b)(x − c)(x − d) · g(x). Ha f (x) = 7, akkor f (x) = 7 =
= (x − a)(x − b)(x − c)(x − d) · g(x), de ez nem lehet, hiszen a 7 nem állítható
elő négy különböző egész szorzataként.
684. Legyen g(x) a feladatban szereplő polinom, s f (x) = g(x) − 7. Ezzel a feladatot
visszavezettük az előzőre.
685. b − c = f (a) − f (b) = (a − b)C,
c − a = f (b) − f (c) = (b − c)A,
a − b = f (c) − f (a) = (c − a)B.
Ezekből |a − b| ≤ |b − c| ≤ |c − a| ≤ |a − b|, de ez páronként különböző a, b,
c számokra nem teljesül.
686. Tegyük fel, hogy f (x) végtelen sok helyen prím értéket vesz fel. Ezeken a helye-
ken a g(x) és a h(x) egyike prímet, a másik +1-et vagy −1-et vesz fel értékül.
Tehát a g(x) és h(x) polinomok egyike — mondjuk g(x) — végtelen sok he-
lyen +1-et (vagy −1-et) vesz fel értékül. Ez azt jelenti, hogy a g(x) − 1 (vagy a
g(x) + 1) polinomnak végtelen sok zérushelye van, ami nem lehetséges.
687. Tegyük fel, hogy f (x) = g(x) · h(x). Mivel f (x) hetedfokú, ezért g(x) és h(x)
egyike legfeljebb harmadfokú. Legyen ez a polinom g(x). A hét különböző hely-
ből legalább négy helyen a g(x) egyformán +1 vagy −1. Ha g(x) négy különböző
helyen +1, akkor a g(x) − 1 polinomnak négy különböző gyöke lenne.
688. Tegyük fel, hogy a polinom előállítható két másik polinom szorzataként, azaz
g(x) · h(x) alakban. Ekkor g(ai ) = −h(ai ) = +1 vagy −1. A g(x) + h(x) polinom-
15. Kombinatorika a számelméletben 233

nak van legalább n db zérushelye (a1 , a2 , . . . , an ), ezért a g(x) és h(x) polinomok


közül legalább az egyik n-ed fokú.
689. Tegyük fel, hogy a polinom előáll két másik polinom szorzataként, azaz p(x) =
= g(x) · h(x), ahol p(x) = (x − a1 )2 · (x − a2 )2 · . . . · (x − an )2 + 1. Ekkor
g(ai ) = h(ai ) = +1 vagy −1.
Lássuk be, hogy az előbbi „vagy” választás azt jelenti, hogy vagy minden ai -re
+1, vagy minden ai -re −1. Ugyanis ellenkező esetben g(ar ) = +1 és g(as ) =
= −1 azt jelenti, hogy a g(x) függvénynek van valamely α zérushelye ar és as
között. Ezért g(α) · h(α) = 0, s ez azért nem lehet, mert p(x) ≥ 1 minden x-re.
Tegyük fel például, hogy g(ai ) = h(ai ) = +1, i = 1, 2, 3, . . . , n. A g(x) − 1 és
h(x) − 1 polinomok mindegyikének legalább n zérushelye van, így g(x) és h(x)
mindegyike legalább n-ed fokú, magasabb fokú azonban egyik sem lehet, mert
akkor szorzatuk nem lenne 2n-ed fokú. Arra jutottunk, hogy g(x) és h(x) is n-ed
fokú, s g(x) − 1 = A(x − a1 ) · . . . · (x − an ), h(x) − 1 = B(x − a1 ) · . . . · (x − an ).
(A = 0, B = 0.) Könnyen belátható, hogy p(x) nem áll elő két ilyen szerkezetű
g(x) és h(x) polinom szorzataként.

15. Kombinatorika a számelméletben

690. A kiválasztott számok között lesz két szomszédos szám.


Megjegyzés. Több is igaz: van két olyan szám; ai és aj , melyekre (ai , aj ) = 1 és
ai ≤ n.
691. Legyenek a kiválasztott számok: 1 ≤ a1 < a2 < · · · < an+1 ≤ 2n. Tekintsük
a 2n > an+1 − a1 > an+1 − a2 > · · · > an+1 − an ≥ 1 számokat. Ez az n db
különböző szám és a kiválasztott n + 1 db szám mindegyike az 1, 2, 3, . . . , 2n
számok közül való, ezért a 2n+1 db szám között vannak egyenlők: an+1 −ai = ak ,
azaz ak + ai = an+1 .
692. A számok közül legalább kettő páratlan. A páratlanok közül legnagyobb b, a
többi páratlant jelölje b1 , . . . , br . Képezzük a b − bi különbségeket; ezek páros
számok. E különbségek és a halmaz páros számainak száma együttesen 51, ezért
valamelyik különbség megegyezik az 52 szám közüli egyik páros számmal.
693. A kiválasztott számok a1 < a2 < · · · < a69 ≤ 100. Nyilván a1 < 33, különben
a69 > 100 lenne.
Tekintsük az {a1 + a3 , a1 + a4 , . . . , a1 + a69 } és {a3 − a2 , a4 − a2 , . . . , a69 − a2 }
halmazokat. Mindkét halmaznak 67 eleme van, és az elemek legnagyobbika, a1 +
+ a69 legfeljebb 132. A két halmaznak tehát van közös eleme, azaz létezik olyan
n és m, amelyekre a1 + an = am − a2 , azaz a1 + a2 + an = am .
Az állítás 69 helyett 68-ra nem igaz, lásd: 33, 34, . . . , 100.
234 Megoldások, útmutatások

694. a1 = 2α1 · b1 , a2 = 2α2 · b2 , . . . , an+1 = 2αn+1 · bn+1 , ahol a bi számok páratlanok.


Mivel a bi páratlan számok 2n-nél kisebbek, ezért csak n-féle értékük lehet. Így
az n + 1 darab bi szám között van két egyenlő: bk = bl . Ekkor az ak és al számok
közül egyik a másiknak osztója, hiszen prímtényezős felbontásukban csak a 2
hatványkitevője különböző.
Más bizonyítás. A 2n db számot beosztjuk n db kupacba: {1, 2, 4, 8, . . . },
{3, 6, 12, 24, . . . }, {5, 10, 20, 40, . . . }, . . . , {2n − 5}, {2n − 3}, {2n − 1}. A ku-
pacoknak megvan az a tulajdonsága, hogy ha egy kupacból kiveszek két számot,
akkor az egyik szám osztója a másiknak. Mivel az 1, 2, 3, . . . , 2n számok közül
n + 1 számot választunk ki, ezért valamelyik kupacból két számot vettünk; így
van két olyan szám, hogy egyik osztója a másiknak.
Megjegyzés. A bizonyítás elvégezhető teljes indukcióval is, az indukciós lépés-
ben indirekt feltevést alkalmazva. Ezt leírtuk az 1861. feladat megoldásánál.
696. Azt kell belátni, hogy az ai -k között szerepel 2-nek pozitív egész kitevős hat-
ványa, hiszen akkor ennek a kétszerese is 2-hatvány; vagy pedig előfordul egy
2s − r, 2s + r alakú számpár.
   m    
N N N N
697. ai -nek többszöröse van N-ig. ≤ N. −1 < , így
ai ai ai ai
m   i=1
 N m
N m
1
− 1 < N, és mivel m ≤ N, ezért < 2N, azaz < 2.
i=1
a i i=1 i
a a
i=1 i

31
Megjegyzés. A pontos korlát .
30
698. Jelölésmódosítással kezdjük: N ≥ a1 > a2 > · · · > am ≥ 1. Állítjuk, hogy
N
ak ≤ . Ez k = 1-re igaz. Indukciós lépés (k + 1)-re:
k
ak · ak+1 ak · ak+1
ak − ak+1 ≥ (ak , ak+1 ) = ≥ .
[ak , ak+1 ] N
ak · N N 2
N2 N
Ebből ak+1 ≤ =N− ≤N− N = .
ak + N ak + N k +N
k+1
Ezzel igazoltuk az indukciós lépést.
N √
a[√N +1] ≤ √ < N . Tehát az N ≥ a1 > a2 > · · · > am ≥ 1 sorozatnak
√ [ N + 1] √ √
N és N között legfeljebb N eleme van, ezért m ≤ 2 N .
699. Felhasználjuk az egyik korábbi feladat állítását: Ha az 1, 2, 3, . . . , 2n számok
közül kiválasztunk n + 1-et, akkor azok között mindig lesz olyan kettő, hogy az
egyik osztója a másiknak.
2n
(Indirekt bizonyítás.) Ha a1 ≤ , akkor 2a1 < 2n, 3a1 ≤ 2n. Hagyjuk el
3
ekkor a kiválasztott számok közül a1 -et, helyette vegyük be a sorozatba a 2a1 ,
3a1 számokat. Az így kapott sorozat is teljesíti az [ai , aj ] > 2n feltételt, és a
15. Kombinatorika a számelméletben 235

sorozatnak n + 1 eleme van. Azonban az előbb említett állítás miatt van az n + 1


szám között olyan kettő, hogy egyik a másiknak osztója. Ekkor mégsem teljesül
2n
az [ai , aj ] > 2n feltétel, tehát hibás a kiinduló a1 ≤ feltevés.
3
700. Legyen a számok legkisebb közös többszöröse A. A feltétel azt jelenti, hogy
A A A
> > ··· > . Tudjuk, hogy A minden ai -nek többszöröse, így a most
a1 a2 am
felsorolt számok egészek. Mivel m darab különböző pozitív egész legnagyobbi-
A
ka legalább m, így ≥ m, ahonnan a1 -gyel szorozva a bizonyítandó állítást
a1
kapjuk.
A
Megjegyzés. Ha A = m! és ai = , akkor A az a1 , a2 , . . . , am számok
m+1−i
legkisebb közös többszöröse és éppen m · a1 -gyel egyenlő. Tehát az állítás nem
javítható.
701. Legyen aj −1 és aj legkisebb közös többszöröse t ≤ N. Ekkor t/aj −1 és t/aj
különböző egész számok, és közülük az első a kisebb, azaz
t t N N
1≤ − ≤ − . Ezeket az egyenlőtlenségeket j = 2, 3, . . . , i-re
aj aj −1 aj aj −1
N
a nyilvánvaló 1 ≤ N/a1 egyenlőtlenséghez hozzáadva kapjuk, hogy i ≤ +
    a 1
N N N N N
+ − + ···+ − = .
a2 a1 ai ai−1 ai
N
Megjegyzés. Ha N = m! és ai = , akkor a feltételben is és az állításban is épp
i
egyenlőség teljesül, tehát az állítás nem javítható.
702. Azt kell észrevenni, hogy minden ai -hez tudunk olyan pj prímet rendelni, melyre
α α
pj j | ai , de pj j  |a1 · a2 · . . . · ai−1 · ai+1 · . . . · ak . Különböző ai -khez különböző
pj -k tartoznak.
703. A sorozat minden eleméhez egy számpárt rendelhetünk: ai → (li+ , li− ), ahol li+
jelenti az ai -vel kezdődő leghosszabb növekvő részsorozat elemeinek számát, és
li− az ai -vel kezdődő leghosszabb csökkenő részsorozat elemeinek számát.
Az így képzett számpárok különbözők. Hiszen, ha i < j és ai < aj , akkor
li+ > lj+ ; ha pedig ai > aj , akkor li− > lj− .
li+ és li− pozitív egész számot jelöl. Az olyan különböző számpárok száma, mely-
ben li+ ≤ n, li− ≤ m, m · n. Mivel a sorozat elemeihez rendelt számpárok száma
m · n + 1, ezért valamelyik i-re vagy li+ > n, vagy li− > m. Ez a tétel állítását
jelenti.
704. ai + aj = ar + as , azaz ai − as = ar − aj . Tehát az ax−ay különbségeknek mind
k
különbözőknek kell lenniük. Egy k-tagú sorozatból különbség képezhető, e
2
236 Megoldások, útmutatások
 
k
különbségek abszolútértékei 1-től (n − 1)-ig terjedhetnek, ezért ≤ n − 1.
  2
k √
(k − 1)2 < 2 · ≤ 2(n − 1) < 2n, tehát f (n) < 2n + 1.
2 √
Megmutatjuk, hogy ha az a1 , a2 , . . . , ak a kívánt tulajdonságú, és k < 3 n, akkor
a sorozat bővíthető. Ugyanis találhatunk olyan a-t, amelyhez nincs olyan x, y,
z, hogy a − ax = ay − az lenne; azaz a = ax + ay − az bármely x, y, z esetén.
A k elemű sorozatból k 3 ilyen ax + ay − az szám képezhető (ez is csak akkor,
ha mind különböző számot eredményez). Mivel k 3 < n, így biztosan van olyan
n-nél nem nagyobb a pozitív egész szám, mely nem áll elő ax + ay − az alakban.
708. 1. megoldás. Tekintsük a három legnagyobb számot: a1 ≥ a2 ≥ a3 .
Ha a3 < 2, akkor a4 , . . . , a50 is kisebb 2-nél, tehát ezek összege kisebb 47 · 2 =
= 94-nél, azaz az első három szám összege nagyobb 6-nál.
2. megoldás. Legyen az 50 szám a1 , a2 , . . . , a50 . Legyen S1 = a1 + a2 + a3 ,
S2 = a2 + a3 + a4 , . . . , S49 = a49 + a50 + a1 , S50 = a50 + a1 + a2 . Ekkor S1 + S2 +
+ · · · + S50 = 3(a1 + a2 + · · · + a50 ) = 300 = 50 · 6. Nos, ha az 50 szám összege
50 · 6, akkor nem lehet mind kisebb 6-nál.
3. megoldás. Vegyük most azösszes  3-tagú összeget, s adjuk össze ezeket.  Eb-

49 49
ben az összegben egy szám -ször szerepel, vagyis az összeg 100 · .
2       2
50 49 50
Másfelől a 3-tagú összegek száma . Mivel 100 · = 6· , így nem
3 2 3
lehet minden 3-tagú összeg 6-nál kisebb.
709. Legyen a hét szám nagyság szerint: A > B > C > D > E > F > G.
Megmutatjuk, hogy A + B + C ≥ 50. Ha ugyanis C > 15, akkor A + B + C ≥
≥ (C + 2) + (C + 1) + C ≥ 51. Ha pedig C ≤ 15, akkor D + E + F + G ≤
≤ (C − 1) + (C − 2) + (C − 3) + (C − 4) ≤ 50, tehát ebben az esetben is teljesül
az A + B + C ≥ 50 egyenlőtlenség, hiszen a hét szám összege 100.
710. A 101 szám egyike a. A megmaradt 100 számot osszuk két 50-es csoportba,
melyekben a számok összege S1 , ill. S2 . A feltétel szerint S1 < a + S2 , és S2 <
< a + S1 . Ezeket összeadva kapjuk, hogy S1 + S2 < 2a + S1 + S2 , azaz a > 0.
Másik megoldás. Legyenek a számok nagyság szerint rendezve: a1 ≤ a2 ≤ . . . ≤
≤ a101 . Ekkor nyilván a2 + a3 + · · · + a51 ≤ a52 + a53 + · · · + a101 . Itt a bal oldalhoz
a1 -et adva a feltétel szerint megfordul az egyenlőtlenség iránya, tehát a1 pozitív,
és így a többi szám is pozitív.
711. Olyan különböző ai > ak , al > aj számokat kell találni, amelyekre ai − ak =
= al − aj . Képezzük a 16 szám közötti összes pozitív különbséget; ezek száma
120. Mivel mindegyik különbség kisebb 100-nál, így vannak egyenlő különbsé-
gek.
Ha van három egyenlő különbség, akkor kiválasztható közülük kettő, amelyek-
ben különböző számok különbségét képeztük. Legyen ugyanis ai −aj = ak −al =
16. Számkonstrukciók 237

= am − an , ai < ak < am . Az első és második pár csak akkor nem jó, ha ai = al ,


az első és a harmadik pedig akkor, ha ai = an . Ez a két eset nem lehetséges, így
a kívánt kiválasztás mindig lehetséges.
Ha nincs három, egymással egyenlő különbség, akkor van legalább 21, amely
kétszer lép fel. Tegyük fel indirekte, hogy a különbségpárok között nincs olyan,
melyben négy különböző szám szerepelne. Ekkor mind a 21 pár ai − am , am − aj
alakú volna. Mivel am legfeljebb 16 különböző értéket vehet fel (valójában 14-et,
mert am a legkisebb és a legnagyobb nem lehet), volna olyan am , amely két kü-
lönbségpárban is szerepel: aj > ak -ra ai − am = am − aj és ak − am = am − aj .
Ezekből ai − ak = al − aj a különböző ai , aj , al számokra, ami ellentmond az
indirekt feltevésnek. Így a 16 szám közül mindig kiválasztható négy olyan kü-
lönböző, amelyre ai + aj = ak + al .
712. Az a1 , a2 , . . . , 
an és az ai +aj összegeknek ki kell adniuk minden számot an+1 -ig;
n
így an+1 ≤ n + < (n + 1)2 .
2

16. Számkonstrukciók

713. 95 210.
714. 10 112 358.
715. 900 000.
716. 999 990.
717. 105.
718. 18, 36, 54, 72, 90.
719. 269, 807, 1345.
720. 129, 387, 645.
721. Több megoldás van:
(192, 384, 576); (219, 438, 657); (273, 546, 819); (327, 654, 981).
722. 652, 978, 1304.
723. 183 = 5832, 184 = 104 976. Könnyen belátható, hogy csak egy ilyen szám van.
724. 567 és 5672 = 321 489; vagy 854 és 8542 = 729 316.
238 Megoldások, útmutatások

725. 532 · 14 = 98 · 76 = 7 448, 584 · 12 = 96 · 73 = 7 008,


174 · 32 = 96 · 58 = 5 568, 158 · 32 = 79 · 64 = 5 056,
186 · 27 = 93 · 54 = 5 022, 259 · 18 = 74 · 63 = 4 662,
146 · 29 = 73 · 58 = 4 234, 174 · 23 = 69 · 58 = 4 002,
134 · 29 = 67 · 58 = 3 886, 138 · 27 = 69 · 54 = 3 726,
158 · 23 = 79 · 46 = 3 634.
726. 39 · 402 = 15 678, 27 · 594 = 16 038, 54 · 297 = 16 038,
36 · 495 = 17 820, 45 · 396 = 17 820, 52 · 367 = 19 084,
78 · 345 = 26 910, 46 · 715 = 32 890, 63 · 927 = 58 401.
727. 42·138 = 5796, 18·297 = 5346, 27·198 = 5346, 39·186 = 7254, 48·159 = 7632,
28 · 157 = 4396, 4 · 1738 = 6952, 4 · 1963 = 7852.
728. 21 · 87 = 1 827, 15 · 93 = 1 395, 27 · 81 = 2 187, 35 · 41 = 1 435.
729. 153 = 13 + 53 + 33 , 370 = 33 + 73 + 03 , 371 = 33 + 73 + 13 , 407 = 43 + 03 + 73 .
730. 1634 = 14 + 64 + 34 + 44 , 8208 = 84 + 24 + 04 + 84 ,
9474 = 94 + 44 + 74 + 44 .
731. 54 748 = 55 + 45 + 75 + 45 + 85 , 92 727 = 95 + 25 + 75 + 25 + 75 .
732. 96 433 469.
733. 142 857.
734. 549, 1449.
735. 7, 29, 1, 1, 1, . . . , 1 (167 db 1-es). Könnyű belátni, hogy hasonló megoldás
minden összetett számra megadható.
736. −2, 2, 25, 1, . . . , 1 (36 db 1-es), −1, . . . , −1 (61 db −1-es).
−4, −2, 25, 1, . . . , 1 (89 db 1-es), −1, . . . , −1 (108 db −1-es).
737. 1, 1, 1, . . . , 1 (998 db), 2, 1000.
739. vagy cd
2 3 20 a b
e
ab
4 5 6 c d
e
bd ac
15 8 1 e
e e
16. Számkonstrukciók 239

740.
27 10 1

5 3 18

2 9 15

741.
12 9 2
1 6 36
18 4 3

742. Ilyen táblázatot úgy is készíthetünk, hogy egy 3×3-as bűvös négyzet elemei he-
lyére olyan 2-hatványokat írunk, ahol a kitevők az eredeti bűvös négyzet megfe-
lelő elemei.

2 7 6 22 27 26 4 128 64

9 5 1 29 25 21 512 32 2

4 3 8 24 23 28 16 8 256

743.
27 7 2 16 7 18 20 2

8 18 14 3 10 4 21 6

28 4 9 6 3 5 12 28

1 12 24 21 24 14 1 15

744.
3 4 9 6

6 6 2 4

8 0 0 4
240 Megoldások, útmutatások

745.
1 −1 −1 1
−1 1 1 −1
1 −1 −1 1
−1 1 1 −1

746.
52 82 112 142 172
39 74 109 144 179
26 66 106 146 186
13 58 103 148 193
0 50 100 150 200

749. 1, 19, 2, 18, 3, 17, . . . , 8, 12, 9, 11, 10, 20.


750. 1, 2, 4, 8, 3, 6, 12, 11, 9, 5, 10, 7 vagy
1, 6, 10, 8, 9, 2, 12, 7, 3, 5, 4, 11.
751. (1, 2n, 2n + 1), (2, 2n − 1, 2n + 2), (3, 2n − 2, 2n + 3), . . . , (n − 1, n + 2, 3n − 1),
(n, n + 1, 3n).
752. 1, 2, 7, 11, 24, 27, 35, 42, 54, 56 vagy
1, 3, 9, 14, 23, 24, 41, 53, 57, 60.
753. Hamis. Ellenpélda: 2 · 3, 2 · 5, 3 · 5, 7 · 11, 7 · 13, 11 · 13. Kis módosítással azonban
már igaz az állítás: Bármely hat természetes számból vagy három, páronként
relatív prím választható ki, vagy három olyan, amelyek közül bármely kettőnek
van 1-nél nagyobb közös osztója. (Lásd a 1625. feladatot!)
754. Hamis. Ellenpélda: 2184, 2185, 2186, . . . , 2200.
755. 1939 db szám kiválasztható, több nem. A kiválasztott számok: 45, 46, 47, . . . ,
1982, 1983.
756. Hamis. Ellenpélda: 2, 3, 5, 7, 8, 11, 12, 26, 27, 28, 29, . . . , 50 (összesen 32 db
szám).
757. Az egyik számhalmaz álljon azokból az n természetes számokból, amelyekhez
található olyan k, melyre (2k)! + 1 ≤ n ≤ (2k + 1)!.
758. Az egyik halmaz {1, 2, 3} ∪ {9, 10, 11, 12, 13, 14, 15} ∪ {25, 26, . . . , 34, 35} ∪
∪{49, 50, 51, . . . , 63} ∪ . . .
760. A 2k, 3k, 4k + 1, 6k + 5, 12k + 7 alakú, a 2k, 3k, 4k + 1, 6k + 1, 12k + 11 alakú
vagy a 2k, 3k, 4k + 1, 8k + 7, 12k + 11, 24k + 19 alakú számok.
761. (09, 18, 27, 36, 45), (05, 16, 27, 38, 49), (01, 23, 45, 67, 89),
(18, 36, 54, 72, 90), (10, 32, 54, 76, 98), (54, 63, 72, 81, 90),
(50, 61, 72, 83, 94).
16. Számkonstrukciók 241

762. a) 6, 12, 18.


b) 120, 240, 360, 480, 600.
Általában 2n + 1 db számra:
A, 2A, 3A, . . . , (2n + 1)A, ahol A = (2n + 1)!
763. Például a 10k + 24, 10k + 25 alakú számpárok. (k ≥ 2)
764. 242, 243, 244, 245.
765. Van. Ha n osztható 2 · 3 · 5 · 7 = 210-zel, akkor az n − 10, n − 9, . . . , n + 10
számok mindegyike — kivéve az n − 1 és az n + 1 számokat — osztható a 2, 3,
5, 7 prímek közül legalább az egyikkel. Ha n = 9450, akkor n − 1 11-gyel, n + 1
pedig 13-mal osztható. Ily módon a 9440, 9441, . . . , 9460 számok teljesítik a
kívánt elvárásokat.
Megjegyzés. Vizsgáljuk meg a következő kérdést: „Van-e 14 olyan, egymás után
következő pozitív egész szám, hogy a számok mindegyike osztható a 2, 3, 5, 7,
11 prímek közül legalább eggyel?” Erre a válasz: Nincsenek ilyen számok.
766. m = 2k − 2, n = 2k (2k − 2), m + 1 = 2k − 1, n + 1 = (2k − 1)2 .
767. 2, 3, 7.
768. 1, 2, 3, 7 vagy 2, 3, 7, 43.
769. −1, 1, −2, 2; 1, 2, 3, 6; 1, 2, 6, 9; 1, 3, 8, 12; 1, 4, 5, 10; 1, 6, 14, 21; 2, 3, 10,
15.
770. Helyezzük el a 16 számot egy 4 × 4-es táblázatba az ábra szerint. A vastaggal
kiemelt számok alkotják az egyik csoportot, a többiek a másikat. (Ellenőrizzük,
hogy megfelelő-e a számok szétosztása.)
1 2 3 4
5 6 7 8
9 10 11 12
13 14 15 16

771. 1, 3, 4, 5 vagy 1, 2, 3, 5.
772. 1, 2, 5, 7.
773. ak = k · n! + 1, k = 1, 2, 3, . . . , n.
774. 2, 22 , 23 , . . . , 2n . (n ≥ 3)
775. Legyenek p1 , p2 , . . . , p8 prímszámok. A keresett nyolc szám:
a1 = p12 · p2 · . . . · p8 , a2 = p1 · p22 · . . . · p8 , . . . , a8 = p1 · p2 · . . . · p82 .
776. Válasszuk ki közülük a 3k + 1 vagy a 3k + 2 alakú számokat!
242 Megoldások, útmutatások

777. Ha már találtunk n db megfelelő számot, az a1 , a2 , . . . , an számokat, akkor a


következő módon tudunk megadni n + 1 db, a kívánt tulajdonsággal rendelkező
számot. Legyen N ennek az n db számnak és az n db számból képezhető páron-
kénti különbségeknek a legkisebb közös többszöröse! A keresett n + 1 db szám:
N, N + a1 , N + a2 , . . . , N + an .
778. Tekintsük ezeknek a számoknak a hármas számrendszerben felírt alakját, s azokat
a számokat válasszuk ki közülük, melyek felírásában a 2-es nem szerepel!
779. {1, 2}, {1, 3}, {1, 4}, {1, 5}, {1, 6}, {2, 3, 4, 5, 6} vagy {1, 2}, {1, 3}, {1, 4}, {1, 5},
{1, 6}, {1}.
780. A három halmaz lehet pl. a következő:
A = {1, 2, 4, 5, 7, 8, . . . }, B = {2, 3, 5, 6, 8, 9, . . . }, C = {3, 4, 6, 7, 9, 10, . . . }.
781. Álljon A azokból a természetes számokból, amelyeknek minden, 0-tól különbö-
ző számjegye a szám végétől visszafelé számítva páratlan helyen áll, B pedig
azokból a természetes számokból áll, melyeknek minden, 0-tól különböző jegye
páros helyen helyezkedik el.
782. Hasonló az előzőhöz.
783. Tekintsük a 781. feladat megoldásában leírt konstrukciót. Tetszőlegesen választ-
va egy b elemet B-ből, az A halmaz minden eleméhez hozzáadjuk ezt a választott
számot, így kapjuk az Ab halmazt. Ezt az Ab halmazt B minden elemére elké-
szítve megkapjuk a keresett halmazokat.
784. A = {1, 2, 5, 6}, B = {3, 4, 7, 8}.
Megjegyzés. Az 1, 2, 3, 4, 5, 6, 7, 8, 9 számokat a kívánt módon már nem
tudnánk két csoportba osztani.
785. Használjuk fel a 105. b) feladat megoldásánál leírt azonosságot!
786. Az n2 + (n + 3)2 + (n + 5)2 + (n + 6)2 = (n + 1)2 + (n + 2)2 + (n + 4)2 + (n + 7)2
azonosság következménye, hogy bármely 8 — és így bármely 16 — szomszédos
egész szám négyzete beosztható két egyenlő összegű csoportba. Elegendő tehát
az első 11 négyzetszámot szétosztanunk. Ebben segít az n2 + (n + 2)2 + (n + 6)2 +
+(n+7)2 +(n+8)2 +(n+10)2 = (n+1)2 +(n+3)2 +(n+4)2 +(n+5)2 +(n+9)2 +(n+11)2
azonosság.
788. 1210, 2020, 21 200, 3 211 000, 42 101 000, 521 001 000, 6 210 001 000.
789. 1, 7, 3, 2, 1, 1, 1, 2, 1, 1 vagy 1, 11, 2, 1, 1, 1, 1, 1, 1, 1.
790. x1 =0,95, x2 =0,05, x3 =0,34, x4 =0,74, x5 =0,58, x6 =0,17, x7 =0,45,
x8 =0,87, x9 =0,26, x10 =0,66.
17. Melyik szám a nagyobb? 243

17. Melyik szám a nagyobb?

791. 23 < 32 .
792. 210 = 1024 > 103 .
793. 23 < 32 miatt 230 < 320 , így 2 · 230 < 3 · 320 vagy 231 < 321
794. 2023 = 23 · 1013 > 32 · 1012 = 3032 .
795. 992 < 99 · 101 = 9999.
796. 920 = 340 > 339 = 2713 .
797. 35 = 243 < 343 = 73 .
798. 199010 + 19909 = 19909 · (1990 + 1) < 19919 · 1991.
 100
4
801. 2 100
+3 100 100
< 4 , mert 2 100
< 3 100
, és 2 · 3 100
< 4 100
; hiszen > 2,
3
 3
4 64
ugyanis már = > 2.
3 27
999 1 1 1000
802. =1− <1− = .
1000 1000 1001 1001
803. Hasonló az előzőhöz.
434343 43 10101 43
804. A két szám egyenlő. Pl.: = · = .
575757 57 10101 57
2 3 2 3
805. < , így 1 − >1− .
22222223 33333334 22222223 33333334
222221 444442 222222 222222 444443
806. = =1− <1− = .
333332 666664 666664 666665 666665
3 300000001 1500000003
807. : = < 1, tehát a második szám a nagyobb.
5 500000001 1500000005

√6 √ √ √
√6
808. 2 3 =8 2<9 3= 3 2 vagy

√3 √
√3 √ √
2 3 = 8 < 9 = 32 < 3 2 , így 2 3 < 3 2 .

√5 √
√5
809. 7 5 = 75 = 16807 > 15625 = 56 > 5 7 .

810. 3111 < 3211 = 255 < 256 = 1614 < 1714 .
811. 12723 < 12823 = 2161 < 2162 = 51218 < 51318 .
812. 113 = 1331 < 1369 = 372 .
244 Megoldások, útmutatások

 398  2148
813. 29 és 34 számoknál az alapokat és a kitevőket hasonlítsuk össze!
√ √
814. 2 · log12 145 > 2 · log12 144 = 4 = 16 > 15.
3
815. log4 9 > log4 8 =
= log9 27 > log9 25.
2
6 6 7
816. log5 > log6 > log6 , tehát log5 6 − log5 5 > log6 7 − log6 6, azaz log5 6 >
5 5 6
> log6 7.
817. lg2 11 = (1 + lg 1,1)2 > 1 + 2 · lg 1,1 = 1 + lg 1,21 = lg 12,1 > lg 12.

π 3 7 √
818. sin 1 < sin = < < log3 7, a legutóbbi egyenlőtlenség azért helyes,
3 2 8
mert 37 < 74 .
819. 1, 011000 > 1000. Ennek igazolásához felhasználjuk a Bernoulli-egyenlőtlenséget:
(1 + x)n ≥ 1 + nx, ha x ≥ −1 valós szám, n ≥ 1 egész szám. (Az egyenlőtlenség
igazolható teljes indukcióval.)
1, 018 ≥ 1, 08; 1, 011000 = (1, 018 )125 ≥ 1, 08125 . 1, 085 > 1, 4. Ezeket is fel-
használva: 1, 011000 > 1, 425 > 1, 424 > 2, 78 > 74 = 2401 > 1000.
 4 20
4
820. 2 · 3100 > 2100 + 3100 miatt elegendő belátni, hogy 480 > 2 · 3100 , vagyis =
35
 20
256
= > 2. Ennek igazolásához felhasználjuk a Bernoulli-egyenlőtlenséget:
243
(1 + x) ≥ 1 + nx, ha x ≥ −1 valós szám, n ≥ 1 egész szám. (Az egyenlőtlenség
n

igazolható teljes indukcióval.)


   
256 1 256 20 1 20
> 1 + , ezért > 1+ ≥ 2.
243 20 243 20
Most belátjuk, hogy 2100 + 3100 > 479 . Megmutatjuk, hogy 3100 > 479 , azaz
   20    10
480 480 256 20 19 361 10 9
100
< 4. Bizonyítás: 100
= < = < =
3 3 243 18 324 8
 5  5
81 9 59049
= < = < 4.
64 7 16807

18. Egyenletek és egyenletrendszerek

821. a) A bal oldalon 2 ≤ x ≤ 3 teljesül, de ilyen x-re a jobb oldal negatív.



b) A bal oldalon álló összeg értéke legalább 1 + 2, s ez > 2.
2 1
c) x 2 + 2 > x 2 + 2 ≥ 2.
x x
18. Egyenletek és egyenletrendszerek 245

2
d) 2sin x ≥ 20 = 1 ≥ sin x, így a sin x = 0, sin x = 1 egyenletrendszert kapjuk.
Ennek az egyenletrendszernek nincs gyöke.

e) 2 sin x cos x = 2 sin 60◦ , azaz sin 2x = 3.
f) Vagy mindkét tényező +1, vagy mindkettő −1, hiszen −1 ≤ sin x ≤ 1.
g) sin x · sin(x + π) ≤ 0.
h) Az egyenlet bal oldalán álló kifejezés mindig pozitív. Vizsgáljuk az x < 0,
0 ≤ x ≤ 1, 1 < x intervallumokon a kifejezést!
822. x = 1.
823. Nincs megoldás.
824. x = −1.
825. Az egyenlőség csak akkor teljesülhet, ha mindkét összeadandó nulla: x = 1.
826. 2x + 3x szigorúan monoton növekvő, ezért a 35 értéket legfeljebb egyszer veszi
fel: x = 3.
 x  x  x  x
2 3 2 3
827. + = 1. A + függvény szigorúan monoton csökken, ezért
5 5 5 5
legfeljebb egy megoldás van: x = 1.
828. A bal oldalon álló kifejezés szigorúan monoton növekvő, ha x ≥ 5: x = ±5, más
megoldás nem lehet.
829. Nyilván x ≥ 5, s ekkor a bal oldalon álló kifejezés szigorúan monoton növekvő,
míg a jobb oldalon álló szigorúan monoton csökken. Ezért legfeljebb egy gyök
lehet, s van is: x = 5.
830–831. Grafikus ábrázolás mutatja, hogy legfeljebb két gyök van: x = 2 és x = 3,
1
ill. x = 0 és x = .
2
832. A bal oldalon álló kifejezés szigorúan monoton növekvő, ezért az egyenletnek
1
legfeljebb egy megoldása van: x = .
3
833. Szorozzuk az egyenlet mindkét oldalát 2x -nel. Ekkor a bal oldalon álló kifejezés
szigorúan monoton növekvő lesz; a jobb oldalon konstans áll, tehát legfeljebb
egy megoldás van: x = −1.
834. Adjunk az egyenlet mindkét oldalához lg(x 5 − 24)-et. Ekkor a bal oldalon álló
kifejezés szigorúan monoton növekvő lesz; a jobb oldalon konstans áll, tehát
legfeljebb egy megoldás van. x = 4 gyök.
835. Az egyenletnek [0, 2π] intervallumon csak 0 és π/10 között lehet megoldása.
π/12 megoldás. Több megoldás nincs, mert a [0, π/10] intervallumon az egyen-
lőség két oldalán álló függvény eltérő monotonitású.
246 Megoldások, útmutatások

836. cos x ≤ 1 ≤ (y − 1)2 + 1, tehát a megoldás: y = 1, x = 2kπ, k ∈ Z.


837. A bal oldali kifejezés növekvő, a jobb oldali csökkenő függvénye x-nek, így
legfeljebb egy megoldás lehet. A megoldás: x = 1.
838. x = 105 megoldás, és ennek az elsőfokú egyenletnek több megoldása nem lehet.
839–841. Mindegyik egyenlet egyismeretlenes elsőfokú egyenlet, tehát legfeljebb egy
gyöke lehet (ugyanis egyik egyenlet sem azonosság), s ez a gyök megsejthető.
Az egyenletek gyökei: x = a + b + c, x = a + b + c, x = ab + bc + ca.
842. Az egyenletnek – ha nem azonosság – legfeljebb n számú különböző valós gyöke
lehet. x = 1, 2, 3, . . . , n mindegyike megoldás.
√ √
843. Nyilván 2 √≤ x ≤ 3. Ekkor 0 ≤ x − 2 ≤ 1, ezért x − 2 ≤ x − 2. Ugyanígy
3 − x ≤ 3 − x. Utóbbi két egyenlőtlenséget összeadva azt kapjuk, hogy az
egyenlet bal oldalán álló kifejezés legalább 1. Vizsgáljuk a jobb oldalt! x 2 − 5x +
+ 7 = (x − 2)(x − 3) + 1 ≤ 1, ha 2 ≤ x ≤ 3. Ezért az egyenletnek csak akkor
lehet megoldása, ha mindkét oldalon 1 áll.
 (x 2 + x − 1) + 1 x 2 + x
844. x 2 + x − 1 ≤ = ,
2 2
 (x − x + 1) + 1 x − x + 2
2 2
x − x2 + 1 ≤ = , tehát
2 2
  x + x x − x2 + 2
2
x2 − x + 2 = x2 + x − 1 + x − x2 + 1 ≤ + = x + 1;
2 2
az innen adódó x − x + 2 ≤ x + 1 egyenlőtlenséget átrendezve az (x − 1)2 ≤ 0
2

egyenlőtlenséghez jutunk, melynek csak x = 1 a megoldása, és ez megoldása az


egyenletnek is.
845. Átalakítás után az (x − 1)2 + (y − 1)2 = 0 egyenletet kapjuk.
846. Az egyenlet mindkét oldalát szorozzuk meg 2-vel, s egyszerű átalakítások után
a (4x 2 − 1)2 + (4y 2 − 1)2 = 0 egyenletet kapjuk.
847. Átalakítás után a következő egyenlethez jutunk:
 2  2
6 √ 2 
√ − − √ − −
4 4
2 x 2 + y 1 = 0.
4
x−2 4
y−1
848. Átalakítás után az (x + y)2 + (x + 1)2 + (y − 1)2 = 0 egyenletet kapjuk.
 
849. Átalakítás után a 3(x + 1)2 + 4+ 5(x + 1)2 + 9 = 5−(x +1)2 egyenletet kapjuk.
A bal oldal legalább 5, a jobb oldal legfeljebb 5. A megoldás x = −1.
850. Vonjuk ki a második egyenlet megfelelő oldalait az elsőéből, s a kapott egyen-
letet átalakítva az (x − y)2 + (y − z)2 = 0 egyenlethez jutunk.
851. (x 2 − a 2 )2 = 4ax + 1, (x 2 + a 2 )2 − 4a 2 x 2 = 4ax + 1,
(x 2 + a 2 )2 = 4a 2 x 2 + 4ax + 1, (x 2 + a 2 )2 = (2ax + 1)2 .
18. Egyenletek és egyenletrendszerek 247

 2  2  2
1 2 1 1 1
852. Az egyenletet átalakítva az x + x−y + x−z + x−t = 0
4 2 2 2
egyenletet kapjuk, tehát x = y = z = t = 0.
853. Az egyenletek megfelelő oldalait összeadva kapjuk az
(x −3)2 +(y +5)2 +(z −1)2 = 0 egyenletet. Ennek x = 3, y = −5, z = 1 megoldása
megoldása az egyenletrendszernek is.
854. Az egyenletek megfelelő oldalait összeadva és 2-vel szorozva:
(x − y)2 + (y − z)2 + (z − x)2 = 0.
855. Az első két egyenlet megfelelő oldalainak összegéből a harmadikét kivonva az
(x − y)2 + (y − z)2 + (z − x)2 = 0 összefüggéshez jutunk.
856. Az egyenletrendszer egy speciális megoldása az x = 0, y = 0, z = 0. Ha
(x0 , y0 , z0 ) az egyenletrendszer egy megoldása és x0 , y0 , z0 közül pl. x0 = 0,
akkor y0 = z0 = 0. Tehát feltehetjük, hogy x = 0, y = 0, z = 0. Szorozzuk 2-vel
az egyenletek megfelelő oldalainak reciprokait és adjuk össze őket! A következő
egyenletet kapjuk:
     
1 2 1 2 1 2
1− + 1− + 1− = 0.
x y z
857. x, y, z, t egyszerre pozitív, ill. egyszerre negatív. Nézzük azt az esetet, amikor
1
mindegyik pozitív. Ha a pozitív, akkor a + ≥ 2, ezért most x, y, z, t mindegyi-
a
kének értéke legalább 1. Szorozzuk meg az egyenletek megfelelő oldalait 2-vel,
1 1 1 1
és adjuk össze őket; ekkor az x + y + z + t = + + + egyenlethez jutunk. Az
x y z t
előbbi észrevétel miatt a bal oldal értéke legalább 4, a jobb oldalon pedig 4-nél
nagyobb szám nem állhat. Megoldások: x = y = z = t = 1, x = y = z = t = −1.
858. Tekintsük az egyenletek megfelelő oldalainak reciprokait.
x y 1 y
859. x, y, z egyike sem 0. Legyen xyz = a. Ekkor az egyenletrendszer: + = , +
a a 2 a
z 5 z x 2 x 1 y 1 z 1
+ = , + = . Ennek megoldása: = , = , = . Figyelembe véve
a 6 a a 3 a 6 a 3 a 2
a jelentését azt kapjuk, hogy yz = 6, zx = 3, xy = 2. Ez utóbbi egyenletrendszer
megoldása: x1 = 1, y1 = 2, z1 = 3 és x2 = −1, y2 = −2, z2 = −3.
3
860. Legyen y = tx. Ekkor az egyenletek: x 2 (1 + t) = 10, x 2 (t 2 + t) = 15. Tehát t = .
2
Megoldások: (2; 3), (−2; −3).

861. 3x 3 − 3x 2 − 3x = 1, 4x 3 = x 3 + 3x 2 + 3x + 1, 4x 3 = (x + 1)3 , 4x = x + 1,
3

1
x=√ .
4−1
3
248 Megoldások, útmutatások

3 1
862. Legyen f (t) = t 2 + t + . Ekkor az egyenletrendszer x = f (y), y = f (z), z =
3
= f (x) alakú. Mivel f (t) minden t-re pozitív, ezért x, y, z pozitív. t > 0 esetén
f (t) monoton növekvő, így ha x > y, akkor f (x) > f (y), azaz z > x, emiatt
z > y. f (z) > f (y), vagyis y > x. Ellentmondásra jutottunk, tehát x = y = z.
1
Az előző feladatra támaszkodva kapjuk a megoldást: x = y = z = √ .
4−1
3

863. (x − y + z)2 − z2 = x 2 − y 2 , (x − y)(x − y + 2z) = (x − y)(x + y). Megoldás: x = y


vagy y = z.
864. Legyen x = a 2 , y = b2 , z = c2 . Ekkor az egyenlet: (a − b + c)2 = a 2 − b2 + c2 , és
ez megegyezik az előző feladatban szereplő egyenlettel.
865. (x − 1)3 + 8 = 0.
866. Szorozzuk 2-vel az egyenlet mindkét oldalát, s a kapott kifejezést másképp írva
jutunk a (2x + 3)3 − 7 = 0 egyenlethez.
867. Átalakítás után az (x + 1)4 − 3 = 0 egyenletet kapjuk.
868. Átalakítás után az (x + 1)3 + 8x 3 = 0 egyenletet kapjuk.
869. Átalakítás után a 4x 4 − (x − 1)4 = 0 egyenletet kapjuk.
870. Átalakítás után az x 4 − (x − 1)2 = 0 egyenletet kapjuk.
871. Átalakítás után az (x 2 + 2x + 3)2 = 25 egyenletet kapjuk.
872–874. Jelöljük a gyökös kifejezéseket új változókkal, s ezekre írjunk fel egyenle-
teket. A kapott egyenletrendszert már könnyű megoldani.
875. Legyen y = x 5 . Ekkor az egyenlet az y y = 55 alakot ölti.
876. A bal oldalon egész szám áll, ezért a jobb oldalon levő kifejezés értéke −1, azaz
x egész szám. Az egyenlet megoldását az (x + 1)(x 2 + 1) = 0 egyenlet megoldása
adja.
√ √
877. A megoldások: 1, 2, 3.
√3
878. A megoldás: 4.
879. A jobb oldalon álló szám csak 0 vagy −1 lehet.
 
 √
√ √
880. Legyen y = x x x x . . .. Ekkor az egyenlet a xy = 1994 alakot ölti, és
√ √
y = 1994 miatt x = 1994.
18. Egyenletek és egyenletrendszerek 249

3
881. Az egyenletnek csak x ≥ esetén lehet megoldása.
4
√ 3
Tekintsük az f (x) = −3 + 4x függvényt. Ez a függvény x ≥ esetén értelmes
4
és szigorúan monoton nő. A feladat egyenlete x = f (f (f (x))) alakban írható.
Ha x > f (x), akkor f szigorú monotonitása miatt x > f (x) > f (f (x)) >
> f (f (f (x))). Ilyen x tehát nem lehet megoldása az egyenletnek. Hasonlóan
látható, hogy x akkor sem megoldása az egyenletnek,
√ ha x < f (x).
Marad tehát az az eset, ha x = f (x), azaz x = −3 + 4x. Ennek gyökei: x1 = 1,
x2 = 3, melyek kielégítik az eredeti egyenletet is.
882. Átalakítás után az (a − b)2 + (b − c)2 + (c − a)2 = 0 egyenletet kapjuk. Ezt
használjuk fel az egyenlet megoldásánál is.
883. sin2n x ≤ sin2 x, cos2n x ≤ cos2 x. Adjuk össze a két egyenlőtlenséget.
884. Használjuk az előbbi eredményeket is.
885. A második egyenletből kapjuk, hogy −1 ≤ x, y ≤ 1. Ezután az első egyenlet
miatt 0 ≤ x, y ≤ 1. Így már igazak az x 3 ≥ x 4 és y 3 ≥ y 4 , azaz 1 = x 3 + y 3 ≥
≥ x 4 + y 4 = 1 egyenlőtlenségek. Tehát x 3 = x 4 , y 3 = y 4 .
886. Indirekt úton bizonyítunk. Tegyük fel, hogy az egyenletnek van három különböző
valós gyöke: x1 , x2 , x3 . A gyökök és együtthatók közti összefüggés alapján: x1 +
+x2 +x3 = −4; x1 x2 +x2 x3 +x3 x1 = 6. Ezekből x12 +x22 +x32 −x1 x2 −x2 x3 −x3 x1 = −2,
azaz (x1 − x2 )2 + (x2 − x3 )2 + (x3 − x1 )2 = −4. Ez utóbbi egyenlőség nem lehet
igaz, emiatt hamis volt a kiinduló feltevés. Tehát igaz a feladat állítása.

887. Legyen f (x) = x 3 − 6, ekkor f −1 (x) = x + 6.
3

Az egyenlet f (f (x)) = f −1 (x) alakba írható, azaz f (f (f (x))) = x.


Belátjuk, hogy ez csak úgy teljesül, ha f (x) = x.
Ha f (x) = x, akkor f (f (x)) = f (x) = x, ezért f (f (f (x))) = f (x) = x.
Ha pl. f (x) < x, akkor — mivel az f (x) függvény szigorúan monoton növekvő
— f (f (x)) < f (x) < x, ezért f (f (f (x))) < f (x) < x. (Hasonlóképpen látható
be, hogy az f (x) > x feltevés is ellentmondásra vezet.)
Oldjuk meg az x 3 − 6 = x (f (x) = x) egyenletet!
x 3 − x − 6 = 0, (x − 2)(x 2 + 2x + 3) = 0. Az egyenlet egyetlen valós gyöke: x = 2.
888. Az egyenlet (x 2 + x − 2)3 + x 2 + x − 2 = x 3 + x alakban is írható. Legyen f (x) =
= x 3 + x, g(x) = x 2 + x − 2, h(x) = x. Az egyenlet f (g(x)) = f (h(x)) alakú.
Mivel az f (x) függvény szigorúan monoton növekvő, ezért √ √
g(x) = h(x), azaz x 2 + x − 2 = x. Ennek gyökei: x1 = − 2, x2 = 2.
889. A p és q gyökök kielégítik az egyenletet, p2 + p2 + q = 0, q 2 + pq + q = 0.
 tehát 
1 1
A (p; q) megoldások: (0; 0), (1; −2), − ; − .
2 2
250 Megoldások, útmutatások

890. Ha x páratlan szám, akkor az x 10 +px 7 +q = 0 egyenlet bal oldalán három páratlan
számot adunk össze; azok összege páratlan, így nem lehet egyenlő 0-val.
Ha x páros szám, akkor az x 10 + px 7 + q = 0 egyenlet bal oldalán két páros és
egy páratlan számot adunk össze; azok összege páratlan, így nem lehet egyenlő
0-val.
Így beláttuk, hogy az x 10 + px 7 + q = 0 egyenletnek nincs egész gyöke, ha p és
q páratlan egészek.

19. Egyenlőtlenségek

1 1 1 1
891. Lehet. Pl.: , , , ... , (1000 db szám).
1000 1000 1000 1000
892. Lehet. Pl.: 8 és −7.
893. 1 = (a1 + a2 + · · · + a100 )50 . A hatványozást elvégezve megkapjuk a feladatban em-
lített 50 tényezős szorzatokat, s az összegnek lesznek ezektől különböző pozitív
tagjai is.

894. Ha 0 < a < 1, akkor a < a < 1. Ezért az első 60 tizedesjegy mindegyike 9.
895. Az összeadandók mindegyike legalább 1.

896. Szorozzuk az egyenlőtlenség mindkét oldalát n-nel. Az összeadandók minde-
gyike legalább 1.
1
897. Az összeadandók mindegyike legalább .
2n
1
898. Az összeadandók mindegyike legalább 2 .
n
899. Hasonló az előzőkhöz.
1 1
900. Az összeg első ötven tagja legalább , a következő száz tag legalább .
100 200
901. Hasonló az előzőhöz, csak most a bontást 2-hatványok szerint végezzük.
1 1
902. Az első öt tag legfeljebb , a következő nyolc kisebb, mint , ill. az első hat
5 8
1 1
tag legalább , a következő hét összeadandó legalább .
10 17
903. Használjuk fel, hogy n! > 2n−1 , ha n > 2.
904. Használjuk fel, hogy n2 > n(n − 1), továbbá támaszkodjunk az 1040. feladat
eredményére.
19. Egyenlőtlenségek 251

906. Az összeg tagjai rendre kisebbek, mint 4, 5, 6.


 
 
 
 √
907. 100 + 99 + 98 + · · · + 2 + 1 <
 
 
 
 √
< 110 + 110 + 110 + · · · + 110 + 121 = 11.
Teljes indukcióval
 általánosabb állítás is igazolható:
  
 
 √ √
Ha An = n + n − 1 + n − 2 + · · · + 2 + 1, akkor An < n+1. Sőt (teljes

1 1
indukcióval is) igazolható, hogy An < n + + .
2 2
 
 
 
 √
908. 1 + n + n + n3 + · · · + nn =
2
 
  
   
   2 
√  n n n3 nn √
= n· + 2 +
1
4
+ 8
+ · · · + 2n < 2 n, hiszen a törtek mind-
n n n n n
annyian egy természetes szám reciprokai, és így biztosan kisebbek 2-nél; ami azt
jelenti, hogy minden gyökjel alatt 4-nél kisebb szám áll. (Ugyanis,
 ha ai < 2,
  
√ √ √ √
i = 1, 2, . . . , akkor a2 + a1 < 2 + 2 < 4 = 2, így a3 + a2 + a1 <
√ √
< 2 + 2 = 2, és így tovább.) 2 n ≤ n, ha n > 3. Ezért az n = 2, n = 3 esetekre
még ellenőrízni kell a feladat egyenlőtlenségét.
 

909. Legyen an = 1 + 2 + · · · + n és
 


bn = 1 + 2 + · · · + n − 2 + 2n − 1 (n ≥ 2).
Ha an kifejezésében a legbelső gyökjel alá n helyett n2 -et írunk, akkor éppen
bn -et kapjuk; és mivel ezzel an értékét növeltük, fennáll az an < bn (n ≥ 2)
egyenlőtlenség.
Ha most bn kifejezésében a legbelső gyökjel alá 2n − 1 helyébe (n − 1)2 -et
írunk, akkor éppen bn−1 -et kapjuk. Ezzel ismét növeltünk, vagy legalábbis nem
csökkentettünk; feltéve, hogy 2n − 1 ≤ (n − 1)2 , ami n ≥ 4 esetén teljesül.
 és így n ≥ 3 mellett b3 ≥
Eszerint a b3 , b4 , . . . , bn sorozat monoton fogyó,

≥ bn > an . Emiatt elég belátni, hogy b3 = 1 + 5 < 9/5 fennáll. Végül


a1 = 1 < a2 = 1 + 2 < b3 miatt igaz állításunk n = 1 és n = 2 mellett is.
252 Megoldások, útmutatások

910. 1.
 megoldás. Általánosabb állítást igazolunk: ha n ≥ 2, akkor
 

m (m + 1) . . . n < m + 1, ahol 2 ≤ m ≤ n.
A bizonyítást
√ „fordított indukcióval” végezzük, azaz lefelé lépkedünk. m = n
esetén a n < n + 1 állítás igaz. Feltehetjük, hogy lefelé, valamely m-ig igaz az
egyenlőtlenség, az indukciós lépésben m − 1-re látjuk be az állítást.

  
   √
 √ 
(m − 1) m (m + 1) . . . n < (m − 1)(m + 1) = m2 − 1 < m2 = m =
= (m − 1) + 1.
2. megoldás. A súlyozott számtani-mértani közép közötti egyenlőtlenség szerint,
ha a1 , a2 , . . . , ar > 0, p1 , p2 , . . . , pr > 0 és p1 + p2 + · · · + pr = 1, akkor
p p
a1 1 a2 2 . . . arpr ≤ 1 a1 + p2 a2 + · · · + pr ar .
p
  

 √ n−1
Legyen xn = 2 3 4 . . . n = 21/2 · 31/4 · . . . · n1/2 . A kitevők összege:
1 1 1 1
+ + · · · + n−1 = 1 − n−1 . Legyen a > 0.
2 4 2 2
n−1 1 1 1 1
Ekkor xn a 1/2 ≤ · 2 + · 3 + · · · + n−1 · n + n−1 · a.
2 4 2 2
1 1 1 n+2
Tudjuk, hogy · 2 + · 3 + · · · + n−1 · n = 3 − n−1 (ez igazolható pl. teljes
2 4 2 2
indukcióval).  
n+2 a
Így xn ≤ a −1/2
n−1
3 − n−1 + n−1 minden a > 0 esetén. Ha a = 1, akkor
2 2
n+1
megkapjuk a bizonyítandó egyenlőtlenséget: xn ≤ 3 − n−1 < 3.
2
a a b b
911. > és > .
1+a 1+a+b 1+b 1+a+b
Adjuk össze az egyenlőtlenségeket.
1 1 1 1 1
912. > , azaz + > 0, továbbá > 0. Utóbbi két
b−c a−c b−c c−a a−b
egyenlőtlenséget adjuk össze.
914. (a − b)(b − c)(a − c) > 0.
915. (a 2 − b2 )(a − b) ≥ 0.
916. (a 3 − b3 )(a − b) ≥ 0.
 
1 1
917. (a − b )
3 3
− ≥ 0.
b a
19. Egyenlőtlenségek 253

919. Belátjuk, hogy


a m + bm a n + bn a m+n + bm+n
· ≤ teljesül, ha m és n azonos párosságú.
2 2 2
Átrendezve az egyenlőtlenséget, előbb a
2(a m+n + bm+n ) − (a m + bm )(a n + bn ) ≥ 0, majd az (a m − bm)(a n − bn ) ≥ 0
egyenlőtlenséghez jutunk.
920. Az előbb leírt egyenlőtlenség szerint:
a + b a 3 + b3 a 4 + b4 a 2 + b2 a 4 + b4 a 6 + b6
· ≤ és · ≤ .
2 2 2 2 2 2
Ezekből megkapjuk a feladat állítását.
921. a) (x + 1)2 + (y − 2)2 + 1.
b) (x 3 + x + 1)2 − 2.
c) 2x 2 − 8xy + 17y 2 − 16x − 4y + 2062 = 2(x − 2y − 4)2 + 9(y − 2)2 + 1994, ill.
= (x − 4y)2 + (x − 8)2 + (y − 2)2 + 1994.
922. Átrendezés, átalakítás után az (a − b)2 + (b − c)2 + (c − a)2 ≥ 0 egyenlőtlenséget
kapjuk.
923. (2a − 1)2 + (2b − 1)2 + (2c − 1)2 ≥ 0.
924. (a 2 − b2 )2 + 2(ab − 1)2 ≥ 0.
   
1 2 1 2 1
925. a 4 − a + 1 = a 2 − + a− + > 0.
2 2 2
926. Átalakítások után: (a − b)2 + (a − 1)2 + (b − 1)2 ≥ 0.
a
2
927. Átalakítások után: − b + c ≥ 0.
2
 2  2
b b
928. a − +3 − 1 + (c − 1)2 ≥ 0.
2 2
929. (a − b)2 + (a − c)2 + (a − d)2 + (b − c)2 + (b − d)2 + (c − d)2 + (b + c + d − a)2 ≥ 0.
     
2 b 2 3 2c 2 2 3d 2
930. a 2 +b2 +c2 +d 2 −ab−bc−cd −d + = a − + b− + c− +
5 2 4 3 3 4
 
5 4 2
+ d− ≥0
8 5
a
2 a
2 a
2
931. a 2 + b2 + c2 + d 2 + e2 − a(b + c + d + e) = −b + −c + −d +
a
2 2 2 2
+ − e ≥ 0.
2
Megjegyzés. Az x12 + x22 + · · · + xn2 ≥ xn (x1 + x2 + · · · + xn−1 ) egyenlőtlenség csak
n = 2, 3, 4 és 5 esetén igaz.
254 Megoldások, útmutatások

Ugyanis a bal és jobb oldal különbsége:


xn
2 xn
2 xn
2 x2
x1 − + x2 − + · · · + xn−1 − + (5 − n) n .
2 2 2 4
 2  2

a b b c c a 2
932. − + − + − ≥ 0.
b c c a a b
 2 2  2 2    
a b a b 2 a b
933. −1 + −1 + − + + − 2 ≥ 0.
b2 a2 b a b a
1 1 4
935. Használjuk fel az + ≥ egyenlőtlenséget.
x y x+y
√ √ √
936. a + b ≥ 2 ab, b + c ≥ 2 bc, c + a ≥ 2 ca. Szorozzuk össze az egyenlőtlen-
ségeket.
ab a+b bc b+c ca c+a
938. ≤ , ≤ , ≤ . Ezeket adjuk össze.
a+b 4 b+c 4 c+a 4
939. a 2 + b2 ≥ 2ab, így (a 2 + b2 )c ≥ 2abc. Hasonlókat kapunk b, c-re és c, a-ra.
Ezeket adjuk össze.

940. ab(a + b) ≥ 2ab ab. Hasonlót írjunk fel a másik két tagra is, s adjuk össze a
három egyenlőtlenséget, s a kapott egyenlőtlenség jobb oldalára alkalmazzuk a
közepek közti egyenlőtlenséget.
941. Hasonló az előzőhöz.
942. A bal oldalon álló szorzat mindkét tényezőjére írjuk fel a közepek közti egyen-
lőtlenséget, s ezeket szorozzuk össze.

ab bc ab bc
943. + ≥2 · = 2b. Írjunk fel hasonlókat és adjuk össze ezeket.
c a c a
944. Legyen A = a + b − c, B = b + c − a, C = c + a − b. Ekkor az egyenlőtlenség
(A+B)(B +C)(C +A) ≥ 8ABC alakban írható, s ez megegyezik a 936. feladattal.
√ a 2 + (a 2 + c2 ) √ b2 + (b2 + c2 )
945. a a 2 + c2 ≤ , b b2 + c2 ≤ , ezeket adjuk össze.
2 2
957. A pozitív számok súlyozott számtani és mértani közepei közt fennálló egyenlőt-
lenség felhasználásával
8 + 2 + 11
ab5 + bc5 + ca 5 = (ab5 + bc5 + ca 5 ) =
21
8ab5 + 2bc5 + 11ca 5 11ab5 + 8bc5 + 2ca 5 2ab5 + 11bc5 + 8ca 5
= + + ≥
21 21 21
≥ [(ab5)8 (bc5 )2 (ca 5 )11 ]1/21 +[(ab5 )11 (bc5 )8 (ca 5 )2 ]1/21 +[(ab5 )2 (bc5 )11 (ca 5 )8 ]1/21 =
= a 3 b2 c + b3 c2 a + c3 a 2 b = abc(a 2b + b2 c + c2 a).
Egyenlőség pontosan akkor áll, ha ab5 = bc5 = ca 5 , azaz a = b = c.
19. Egyenlőtlenségek 255

958. Bizonyítandó, hogy a 4 + b4 + c4 ≥ abc(a + b + c) = a 2 bc + b2 ac + c2 ab, azaz


a 4 + b4 + c4 − a 2 bc − b2 ac − c2 ab ≥ 0.
Ez igaz, mert 4a 4 + 4b4 + 4c4 − 4a 2 bc − 4b2 ac − 4c2 ab = (a 2 − b2 )2 + (b2 − c2 )2 +
+ (c2 − a 2 )2 + 2(a 2 − bc)2 + 2(b2 − ca)2 + 2(c2 − ab)2 ≥ 0.
Második bizonyítás.
       
a 4 + b4 + c4 a+b+c 4 a+b+c a+b+c 3 a+b+c
≥ = · ≥ ·abc.
3 3 3 3 3
Harmadik bizonyítás.
a 4 + b4 + c4 = (a 2 )2 + (b2 )2 + (c2 )2 ≥ a 2 b2 + b2 c2 + c2 a 2 = (ab)2 + (bc)2 + (ca)2 ≥
≥ (ab)(bc) + (bc)(ca) + (ca)(ab) = abc(a + b + c).
963. A súlyozott számtani és mértani közép közötti egyenlőtlenséget fogjuk felhasz-
1 2 4
nálni az , , súlyokkal.
7 7 7
a 3
b + b3 c + c3 a =     
1 3 2 3 4 3 2 3 4 3 1 3 4 3 1 3 2 3
= a b+ b c+ c a + a b+ b c+ c a + a b+ b c+ c a ≥
7 7 7 7 7 7 7 7 7
≥ (a 3 b)1/7 (b3 c)2/7 (c3 a)4/7 + (a 3 b)2/7 (b3 c)4/7 (c3 a)1/7 + (a 3 b)4/7 (b3 c)1/7 (c3 a)2/7 =
= abc2 + ab2c + a 2 bc = abc(a + b + c).
Más bizonyítás. A bizonyítandó egyenlőtlenség a triviális
0 ≤ ab(a − c)2 + bc(b − a)2 + ca(c − b)2 egyenlőtlenség átrendezése.
977. Használjuk fel, hogy a pozitív x, y és z valós számokra
x 3 + y 3 + z3 x 2 + y 2 + z2
≥ .
x+y+z 3

a 3 + b3 + c3 b3 + c3 + d 3 c3 + d 3 + a 3 d 3 + a 3 + b3
+ + + ≥
a+b+c b+c+d c+d+a d +a+b
a 2 + b2 + c2 b2 + c2 + d 2 c2 + d 2 + a 2 d 2 + a 2 + b2
≥ + + + =
3 3 3 3
= a 2 + b2 + c2 + d 2 .
Igazolnunk kell még a felhasznált egyenlőtlenséget. Általánosabb egyenlőtlensé-
get igazolunk.
Legyen xi > 0, i = 1, 2, . . . , n, és k természetes szám. Ekkor
x1k + · · · + xnk x k+1 + · · · + xnk+1
≤ 1 .
n x1 + · · · + xn
Ezt igazoljuk.
xi
Az ai = helyettesítéssel egyenlőtlenségünk az
x1 + · · · + xn
a1k + · · · + ank
(1) ≤ a1k+1 + · · · + ank+1
n
256 Megoldások, útmutatások

alakot ölti. Elegendő belátni ezt az egyenlőtlenséget.


Teljes indukcióval bizonyítunk. k = 0 esetén igaz az állítás. Feltehetjük, hogy
az állítás teljesül (k − 1)-re. (1) bal oldalát átalakítva és a Schwarz–Cauchy–
Bunyakovszkij egyenlőtlenséget alkalmazva:
(k−1)/2 (k−1)/2
a1k + · · · + ank (k+1)/2 a1 an
(∗) = a1 · + · · · + an(k+1)/2 · ≤
n n n
 1/2
a1k−1 ank−1
≥ (a1k+1 + ···+ ank+1 )1/2 + · · · + .
n2 n2
a1k−1 + · · · + ank−1
Használjuk az indukciós feltevést: ≤ a1k + · · · + ank .
n
 1/2
k−1
k+1 1/2 a1 ank−1
(a1 + · · · + an )
k+1
+ ···+ 2 ≤
n2 n
 k 1/2
k+1 1/2 a1 ank
(a1 + · · · + an )
k+1
+ ···+ .
n n
(∗) és az utóbbi összevetéséből:
 k 1/2
a1k + · · · + ank k+1 1/2 a1 ank
≤ (a1 + · · · + an )
k+1
+ ···+
n n n
 k  1/2
a1 + · · · + ank
≤ (a1k+1 + · · · + ank+1 )1/2
n
a1k + · · · + ank
≤ a1k+1 + · · · + ank+1 .
n
Megjegyzés. Az igazolt általánosabb egyenlőtlenség mutatja, hogy a feladat ál-
talánosítható.
978. A számtani és mértani közép közti egyenlőtlenség alapján felírhatók az alábbi
egyenlőtlenségek:
a 3 + a 3 + b3 ≥ 3a 2 b
a 3 + a 3 + c3 ≥ 3a 2 c
b3 + b3 + c3 ≥ 3b2 c
a 3 + b3 + b3 ≥ 3ab2
a 3 + c3 + c3 ≥ 3ac2
b3 + c3 + c3 ≥ 3bc2
2(a 3 + b3 + c3 ) ≥ 6abc
Ezek összege a feladatbeli egyenlőtlenséget adja. (Vegyük figyelembe:
3(a + b)(b + c)(c + a) = 3a 2 b + 3ab2 + 3a 2 c + 3ac2 + 3b2 c + 3bc2 + 6abc.)
 (4a + 1) + 1
980. (4a + 1) · 1 ≤ = 2a + 1, s hasonlóan a többi.
√ √ 2 √
Így 4a + 1 + 4b + 1 + 4c + 1 < (2a + 1) + (2b + 1) + (2c + 1) = 5.
19. Egyenlőtlenségek 257

981. Hasonló az előzőhöz.



995. (a 2 + b2 ) + (c√2 + d 2 ) ≥ 2(ab + cd) ≥ 4 abcd = 4.
ab + cd ≥ 2√abcd = 2.
bc + ad ≥ 2√bcad = 2.
ac + bd ≥ 2 acbd = 2.

996. 1 + ai ≥ 2 ai , ezeket szorozzuk össze.
 
1 1 1 1
997. Legyen p = ,q = , ahol 0 ≤ x < 1/2. + = 1. Ekkor
1/2 + x  1/2 − x   p
 q 
1 1 1 1 1 1 1 1
+ = − + − = 1− + =
p(p + 1) q(q + 1) p p + 1 q q+1 p+1 q+1
1 1 3/4 − x 2
=1− + = 1 − 2 · .
1
1/2+x + 1
1
1/2−x + 1
9/4 − x 2
1 3/4 − x 2 1 1 3/4 − x 2
Könnyen ellenőrizhető, hogy ≥ > , ezért ≤ 1 − 2 · <
3 9/4 − x 2 4 3 9/4 − x 2
1 1 1 1 1
< , azaz ≤ + < .
2 3 p(p + 1) q(q + 1) 2
1 1
Most igazoljuk az + ≥ 1 egyenlőtlenséget!
p(p −  1) q(q −  1)  
1 1 1 1 1 1 1 1
+ = − + − = + −1=
p(p − 1) q(q − 1) p−1 p q−1 q p−1 q −1
1 1 1 1 1/2 + x 1/2 − x
= 1 + 1 − 1 = 1/2−x + 1/2+x − 1 = + −1 =
1/2+x
− 1 1/2−x − 1 1/2+x 1/2−x
1/2 − x 1/2 + x
1/4 + x 2
=2· − 1 ≥ 2 − 1 = 1.
1/4 − x 2
998. Legyen az n db kis négyzet oldala rendre a1 , a2 , . . . , an hosszúságú. Ekkor
a négyzetek területének összege, a12 + a22 + · · · + an2 legfeljebb a nagy négyzet
területét adja, ezért a12 + a22 + · · · + an2 ≤ 1. A számtani és a négyzetes közép
közötti egyenlőtlenségszerint
a12 + a22 + · · · + an2 n √
a1 + a2 + · · · + an ≤ n ≤ √ = n.
n n
999. Legyen a kis kockák éle rendre b1 , b2 , . . . , bn . Ekkor a kockák térfogatának
összege legfeljebb 1; b13 +b23 +· · ·+bn3 ≤ 1. A számtani és a harmadik hatványközép
közötti egyenlőtlenségszerint
3 b + b2 + · · · + bn
3 3 3 √
b1 + b2 + · · · + bn ≤ n 1 ≤ n2 = n2/3 .
3

n

a 2 + b2 (a − b)2 + 2ab 2 2 √
1000. = = (a − b) + ≥ 2 (a − b) = 2 2.
a−b a−b a−b a−b
258 Megoldások, útmutatások

a 2 − b2 2ab a 2 + b2
1001. =a+b >a+b− = .
a−b a+b a+b
1 1 1
1002. a) + = + log2 π > 2 (lásd a 934. a) feladatot!).
log2 π logπ 2 log2 π
1 1
b) + = logπ 2 + logπ 5 = logπ 10 > 2.
log2 π log5 π
1003. Aközepek közti egyenlőtlenség szerint a bal oldali összeg nagyobb, mint
4 3
4 > 4 · 1,1.
2
1004. Ha 0 < sin x < 1, akkor sin2 x < sin x, ugyanígy ha 0 < cos x < 1, akkor
cos2 x < cos x. Továbbá sin2 x + cos2 x = 1.
Ezért 1 = sin2 40◦ + cos2 40◦ < sin 40◦ + cos 40◦ .

1005. Ha 0 < sin x < 1, akkor√sin2 x < sin x < sin x, ugyanígy ha 0 < cos x < 1,
2
akkor cos2 x < cos x < cos x.√Továbbá sin 2
√ x + cos x = 1.
2 ◦ 2 ◦ ◦ ◦
Ezért 1 = sin 40 + cos 40 < sin 40 + cos 40 .
1006. Írjuk fel a közepek közti egyenlőtlenséget az 1, 2, 3, . . . , n számokra.
1007. Írjuk fel a közepek közti egyenlőtlenséget az a, b, b, . . . , b (n db b) számokra.
1 1 1
1008. Írjuk fel a közepek közti egyenlőtlenséget az n + 1 db 1 + , 1 + , . . . , 1 + ,
n n n
1
1 számra. (Az előző feladat speciális esete: a = 1, b = 1 + .)
n
1 1 1
1009. Írjuk fel a közepek közti egyenlőtlenséget az n + 2 db 1 + , 1 + , 1 + , . . . ,
n n n
1 1 1
1 + , , számra.
n 2 2
√ √ √ 3 √3 √3
1010. Írjuk fel a közepek közti egyenlőtlenséget a a, a, b, b, b számokra.
1011. Írjuk fel
 a közepek
 köztiegyenlőtlenséget az
 
a 1 b 1 b 1 3 c 1 3 c 1 3 c
, , , , , számokra.
b 2 c 2 c 3 a 3 a 3 a
1
1012. Mivel x(1 − x) ≤ (következik a közepek közti egyenlőtlenségből, vagy függ-
4
vényábrázolás is mutatja), így az egyenlőtlenségek bal oldalainak szorzatára
 3
1
a(1 − b)b(1 − c)c(1 − a) ≤ .
4

a a √
1014. a) x + ≥ 2 x = 2 a.
x x

x3 + x + 2 1 1 4 1 1
b) = x + 1 + + ≥ 4 x 2 · 1 · · = 4.
2
x x x x x
19. Egyenlőtlenségek 259

1015. (1 − x)3 (1 + 3x) = (1 − x)(1 − x)(1 − x)(1 + 3x) ≤


 
(1 − x) + (1 − x) + (1 − x) + (1 + 3x) 4
≤ = 1.
4
   4
(1 + x) + (1 + x) + (1 + x) + 3(1 − x) 4 3
1016. 3(1 + x) (1 − x) ≤
3
= .
4 2
1017. Legyen a négy gyök x1 , x2 , x3 , x4 . A gyökök és együtthatók közti összefüggé-
seket felhasználva írjuk fel a közepek közti egyenlőtlenséget.

x1 + x2 + x3 + x4 
1= ≥ 4 x1 x2 x3 x4 = 1,
4
tehát x1 = x2 = x3 = x4 = 1. Ahonnan a következő azonosságot kapjuk:
x 4 − 4x 3 + ax 2 − bx + 1 = (x − 1)4 .
1019. Az egyenlőtlenség négyzetreemelés és rendezés után
0 ≤ (n − 3)(a 4 + b4 + c4 ) + (a 2 − b2 )2 + (b2 − c2 )2 + (c2 − a 2 )2 alakban írható.
Innen látható, hogy a keresett érték n = 3.
2
1020. k = .
3
1 (k + 1)2 2(n + 1)
1021. 1 + = , a szorzat értéke .
k(k + 2) k(k + 2) n+2
1024. Az első egyenlőtlenség másképp írva: x 2 < (y − z)2 , azaz
(x + y − z)(x − y + z) < 0. Hasonló módon írjuk át a másik két egyenlőtlenséget
is, majd az így nyert három egyenlőtlenséget szorozzuk össze.
Így az ((x − y + z)(x + y − z)(y + z − x))2 < 0 egyenlőtlenséghez jutunk, melynek
nincs megoldása.
1026. n! < (n + 1)n , (n!)n · n! < (n!)n · (n + 1)n , (n!)n+1 < [(n + 1)!]n .
1028–1030. Képezzük a sorozat egymás utáni elemeinek hányadosát.
     
1 ak 1 1 ak 1 1 ak k+1 2
1031. 1 + 1+ 2 =1+ 2 + + ≥1+ 2 +2 · = .
ak k k ak k 2 k ak k 2 k
 
n     n 2
1 ak k+1
Ezért 1+ 1+ 2 ≥ = (n + 1)2 .
k=1
ak k k=1
k
1033. Sp + Sq − Spq = (Sp + Sq − Sp ) − (S2p − Sp ) − (S3p − S2p ) − · · · − (Sqp − S(q−1)p ) ≤
1 1 1
≤ Sq − p · −p· − ··· − p · = 1.
2p 3p qp
260 Megoldások, útmutatások

20. Különféle algebrai feladatok

1034. a) Nulla, hiszen szorzunk nullával.


b) Nulla.
1035. Használjuk az a 2 − b2 = (a − b)(a + b) azonosságot.
4
a) .
5
b) 96 · 104 = (100 − 4) · (100 + 4) = 10000 − 16 = 9984.
c) 899 = 900 − 1 = 302 − 12 = 29 · 31.
d) 77782 − 22232 = (7778 − 2223)(7778 + 2223) = 5555 · 10001.
1036. a) x 4 + x 3 + 2x 2 + x + 1 = (x 4 + x 3 + x 2 ) + (x 2 + x + 1) = (x 2 + x + 1)(x 2 + 1).
b) x 3 + 2x 2 + 2x + 1 = (x 3 + x 2 ) + (x 2 + x) + (x + 1) = (x + 1)(x 2 + x + 1).
c) x 4 + 2x 3 + 2x 2 + 2x + 1 = (x 4 + x 3 ) + (x 3 + x 2 ) + (x 2 + x) + (x + 1) =
= (x + 1)(x 3 + x 2 + x + 1) = (x + 1)2 (x 2 + 1).
d) x 4 + 2x 3 + 3x 2 + 2x + 1 = (x 4 + x 3 + x 2 ) + (x 3 + x 2 + x) + (x 2 + x + 1) =
2
= (x 2 + x + 1)(x 2 + x + 1) = (x 2 + x + 1) .
e) x 4 + x 2 + 1 = x 4 + 2x 2 + 1 − x 2 = (x 2 + 1)2 − x 2 = (x 2 + x + 1)(x 2 − x + 1).
f) x 10 + x 8 + x 6 + x 4 + x 2 + 1 = (x 4 + x 2 + 1)(x 6 + 1) =
= (x 2 + x + 1)(x 2 − x + 1)(x 2 + 1)(x 4 − x 2 + 1).
g) x 4 + 4 = x 4 + 4x 2 + 4 − 4x 2 = (x 2 + 2)2 − (2x)2 = (x 2 + 2x + 2)(x 2 − 2x + 2).
h) x 4 −7x 2 +1 = x 4 +2x 2 +1 −9x 2 = (x 2 +1)2 −(3x)2 = (x 2 +3x +1)(x 2 −3x +1).
i) x 5 + x 4 + 1 = (x 5 + x 4 + x 3 ) − (x 3 − 1) = x 3 (x 2 + x + 1) − (x − 1)(x 2 + x + 1) =
= (x 3 − x + 1)(x 2 + x + 1).
1037. x 9 + y 9 = (x + y)(x 8 − x 7 y + x 6 y 2 − x 5 y 3 + x 4 y 4 − x 3 y 5 + x 2 y 6 − xy 7 + y 8 ) =
= (x 3 + y 3 )(x 6 − x 3 y 3 + y 6 ) = (x + y)(x 2 − xy + y 2 )(x 6 − x 3 y 3 + y 6 ).
Innen
x 8 −x 7 y+x 6 y 2 −x 5 y 3 +x 4 y 4 −x 3 y 5 +x 2 y 6 −xy 7 +y 8 = (x 2 −xy+y 2 )(x 6 −x 3 y 3 +y 6 ).
1038. 2x 3 + 2y 3 − 3x 2 − 3y 2 + 1 = (x + y − 1)(2x 2 − 2xy + 2y 2 − x − y − 1).
1039. Az a 2 − b2 = (a − b)(a + b) azonosságot használva: 1002 − 992 + 982 − 972 + · · · +
+ 22 − 12 = 100 + 99 + 98 + 97 + · · · + 2 + 1 = 5050.
1 1 1 1 1 1 1 1 1
1040. Az = − azonosság alapján az összeg: − + − + − +
k(k + 1) k k + 1 1 2 2 3 3 4
1 1 1 n
+ ···+ − =1− = , (n = 99).
n n+1 n+1 n+1
2k + 1 1 1 1
1041. 2 = − . Végeredmény: 1 − .
k (k + 1)2 k 2 (k + 1)2 (n + 1)2
20. Különféle algebrai feladatok 261

1042. k · k! = (k + 1)! − k!. Végeredmény: (n + 1)! − 1.


k 1 1 1
1043. = − . Végeredmény: 1 − .
(k + 1)! k! (k + 1)! (n + 1)!
k+2 k+1
1044. = . Ezt alkalmazva, a feladat azonos az előzővel.
k! + (k + 1)! + (k + 2)! (k + 2)!
1 √ √ √
1045. √ √ = k + 1 − k. Végeredmény: n + 1 − 1.
k+ k+1
1 1 1 1
1046. √ √ =√ −√ . Végeredmény: 1 − √ .
(k + 1) k + k k + 1 k k+1 n+1
1047. tg 2k α = ctg 2k α − 2 ctg 2k+1 α. Végeredmény: ctg α − 2n+1 ctg 2n+1 α.
 
1 1 sin(k + 1)α sin kα
1048. = −
cos kα · cos(k + 1)α  sin α cos(k + 1)α  cos kα
1 sin(n + 1)α sin α
Végeredmény: − .
sin α cos(n + 1)α cos α
1
1049. = ctg 2k−1 α − ctg 2k α. Végeredmény: ctg α − ctg 2n α.
sin 2k α
    
1 1 1 3 4 5 101 101
1050. 1 + 1+ ... 1 + = · · · ...· = .
2 3 100 2 3 4 100 2
    
1 1 1
1051. 1 − 2 1 − 2 ... 1 − =
 2  3  1002    
1 1 1 1 1 1
= 1− 1+ 1− 1+ ... 1 − 1+ =
2 2 3 3 100 100
1 3 2 4 3 5 4 100 99 101
= · · · · · · ·...· · · =
2  2 3  3 4 4 5   100 100
99 
1 3 2 4 3 5 4 100 99 101
= · · · · · · ·...· · · =
2 2 3 3 4 4 5 99 100 100
1 101 101
= · = .
2 100 200
 
1 √ 1 √ 1 1
1052. √ = k · = k − =
(k + 1) k (k +1)k
 k  k+1  
√ 1 1 1 1 √ 1 1
= k √ +√ √ −√ . Mivel k √ + √ < 2, ezért
k k + 1 k  k+1 k k+1
1 1 1
√ <2 √ −√ .
(k + 1) k k k+1

k(k + 1) 1 k(k + 1) 1
1053. < , hiszen 2 < (mivel 4k(k + 1) < 4k 2 + 4k + 1).
2k + 1 2 4k + 4k + 1 4
Emiatt az egyenlőtlenség igaz, mivel az n-tagú összeg mindegyik tagja kisebb
1 n
-nél, tehát összegük értéke kisebb .
2 2
262 Megoldások, útmutatások

1054. Ha xk+1 = xk − xk2 , k = 1, 2, . . . , akkor xk2 = xk − xk+1 , k = 1, 2, . . . x12 + x22 + x32 +


+ · · · + xn2 = (x1 − x2 ) + (x2 − x3 ) + (x3 − x4 ) + · · · + (xn − xn+1 ) = x1 − xn+1 < x1 < 1.
1055. Látható, hogy a sorozat elemei pozitív számok, és a sorozat szigorúan monoton
csökkenő.
1 1 1 1 + xi 1 1
= = > = 1 + . Ebből következik, hogy >
xi+1 xi − xi2 xi (1 − xi ) xi xi x1001
1
> 1000 + = 2000, azaz x1001 < 0, 0005.
x1
√ √ √ √
(4 + 15)3/2 + (4 − 15)3/2 (8 + 2 15)3/2 + (5 − 2 15 + 3)3/2
1056. √ √ = √ √ =
(6 + 35)3/2 − (6 − 35)3/2 (12 + 2 35)3/2 − (7 − 2 35 + 5)3/2
√ √ √ √ √ √
( 5 + 3)3 + ( 5 − 3)3 2(5 5 + 9 5) 7
= √ √ √ √ = √ √ = .
( 7 + 5) − ( 7 − 5)
3 3 2(21 5 + 5 5) 13
 4 1  4 1  
2 + 4 4 + 4 · . . . · 204 + 14 (44 + 4)(84 + 4) . . . (404 + 4)
1057.       = =
14 + 14 34 + 14 · . . . · 194 + 14 (24 + 4)(64 + 4) . . . (384 + 4)
(32 + 1)(52 + 1)(72 + 1)(92 + 1) · . . . · (392 + 1)(412 + 1)
= = 841,
(12 + 1)(32 + 1)(52 + 1)(72 + 1) · . . . · (372 + 1)(392 + 1)
hiszen n4 + 4 = (n2 − 2n + 2)(n2 + 2n + 2) = [(n − 1)2 + 1][(n + 1)2 + 1].
 
1 · 2 · 4 + 2 · 4 · 8 + 3 · 6 · 12 + · · · + 100 · 200 · 400 1/3
1058. =
1 · 3 · 9 + 2 · 6 · 18 + 3 · 9 · 27 + · · · + 100 · 300 · 900
   
1 · 2 · 4(13 + 23 + 33 + · · · + 1003 ) 1/3 8 1/3 2
= = = .
1 · 3 · 9(13 + 23 + 33 + · · · + 1003 ) 27 3
1059. Hamis. Legyen x = y = 1. Erre a helyettesítésre az egyenlőségjel két oldalán
más-más szám áll.
1062. f (t) = t + et szigorúan monoton növekvő, ezért f (x) = f (y) csak úgy lehet, ha
x = y.
1063. 0 = (a + b + c)2 = a 2 + b2 + c2 + 2(ab + bc + ca).
1064. Átalakítás után: (a − b)2 + (b − c)2 + (c − a)2 = 0.
a b c
1065. Legyen k = = = . Ekkor kb = a, kc = b, ka = c. Szorozzuk össze ezeket
b c a
az egyenlőségeket, s azt kapjuk, hogy k 3 abc = abc, tehát k = 1.
1066. Mivel abc = 1 és ab + bc + ca = a + b + c,
ezért abc − ab − bc − ca + a + b + c − 1 = 0,
tehát (a − 1)(b − 1)(c − 1) = 0.
1067. Hasonló az előzőhöz.
20. Különféle algebrai feladatok 263

1068. Legyen p = a + b + c, q = ab + bc + ca, r = abc, f (x) = x 3 − px 2 + qx − r. Az


f (x) függvény x ≤ 0 esetén negatív értékeket vesz fel, tehát zérushelyei pozitív
számok, s ezek a zérushelyek a gyökök és együtthatók közti összefüggések miatt
az a, b, c számok.
1 1
1070. 0 = x + y + z − 2(xy + yz + zx) + 4xyz − = (2x − 1)(2y − 1)(2z − 1).
2 2
1072. x 2 + y 2 + z2 = (x + y + z)2 − 2(xy + yz + zx) = 0.
1073. Támaszkodjunk az előző feladatra.
1074. Az egyenlőségjel bal oldalán álló kifejezést átalakítjuk:
y+z z+x x+y y+z+x z+x+y x+y+z
+1+ + 1 + +1 = + + =
x  y  z x y z
1 1 1
= (x + y + z) + + = 0.
x y z
1075. Szorozzuk a megadott egyenlőség mindkét oldalát rendre x, y, z-vel, s adjuk
össze az így kapott három egyenlőséget.
x2 y2 z2
+ + +
y+z z+x x+y
     
xy xz yx yz zx zy
+ + + + + + =
y+z y+z z+x z+x x+y x+y
=x+y+z
Mivel a zárójelekben levő kifejezések értéke rendre x, y, z, így a bizonyítandó
állításhoz jutottunk.
1077. a) b5 = b3 · b2 = (b + 1)b2 = b3 + b2 = (b + 1) + b2 = b2 + b + 1. Mivel b4 + 1 =
= b3 · b + 1 = (b + 1)b + 1 = b2 + b + 1, így b5 = b4 + 1.
b) Szorozzuk meg a b5 = b4 + 1 egyenlőséget b-vel: b6 = b5 + b = b4 + 1 + b.
Innen b6 − 1 = b4 + b, azaz (b3 − 1)(b3 + 1) = b(b3 + 1). Mivel b3 + 1 = 0, így
b3 − 1 = b, azaz b3 = b + 1.
    
1 1 1 2
3
1078. x + 3 = x + x+ − 3 = 0.
x x x
   
1 2 2 1 1
1079. x + = x + 2 + 2 = 7 + 2 = 9, így x + = 3.
x x x
   
1 3 1 1 1
27 = x + = x3 + 3 + 3 x + , ezért x 3 + 3 = 18.
x  x  x  x
    
1 1 1 1 1
126 = 7 · 18 = x + 2 · x + 3 = x + 5 + x +
2 3 5 5
= x + 5 + 3,
x x x x x
5 1
emiatt x + 5 = 123.
x
264 Megoldások, útmutatások

1080. 0 = (a + b + c)2 = a 2 + b2 + c2 + 2(ab + bc + ca). Mivel a 2 + b2 + c2 = 1, így


2(ab + bc + ca) = −1. Innen 1/4 = (−1/2)2 = (ab + bc + ca)2 = a 2 b2 + b2 c2 + c2 a 2 +
+2(ab·bc+bc·ca+ca·ab) = a 2 b2 +b2 c2 +c2 a 2 +2abc(b+c+a) = a 2 b2 +b2 c2 +c2 a 2 ,
1
hiszen b + c + a = 0. Tehát a 2 b2 + b2 c2 + c2 a 2 = .
4
1
1 = (a + b + c ) = a + b + c + 2(a b + b c + c2 a 2 ) = a 4 + b4 + c4 + 2 · ,
2 2 2 2 4 4 4 2 2 2 2
4
4 4 4 1
azaz a + b + c = .
2
1081. a 4 + b4 + c4 = a 2 (b + c)2 + b2 (a + c)2 + c2 (a + b)2 =
= 2(a 2 b2 + b2 c2 + c2 a 2 ) + 2abc(a + b + c) = (a 2 + b2 + c2 )2 − (a 4 + b4 + c4 ), azaz
2(a 4 + b4 + c4 ) = (a 2 + b2 + c2 )2 .
1082. 0 = 2(a 8 + b8 + c8 ) − (a 4 + b4 + c4 )2 =
= −(a 2 + b2 + c2 )(a 2 + b2 − c2 )(a 2 − b2 + c2 )(b2 + c2 − a 2 ).
1083. Az x 2 + ax + b + 1 = 0 egyenlet gyökei x1 és x2 .
Ekkor a = −[x1 + x2 ], b + 1 = x1 x2 , b = x1 x2 − 1.
Innen a 2 + b2 = (−[x1 + x2 ])2 + (x1 x2 − 1)2 = x12 · x22 + x12 + x22 + 1.
Mivel a 2 + b2 = x12 · x22 + x12 + x22 + 1 = (x12 + 1)(x22 + 1), ahol (x12 + 1) > 1 és
(x22 + 1) > 1 egészek, ezért a 2 + b2 két, 1-nél nagyobb egész szám szorzata, tehát
összetett szám.
1085. a 3 = 3ab − 2c + 6xyz.
1086. (x + y + z)2 − (x 2 + y 2 + z2 ) = 9 − 5 = 4, tehát xy + yz + zx = 2.
(x 2 + y 2 + z2 )2 − (x 4 + y 4 + z4 ) = 25 − 17 = 8; innen kapjuk, hogy x 2 y 2 + y 2 z2 +
+ z2 x 2 = 4. Az (xy + yz + zx)2 = x 2 y 2 + y 2 z2 + z2 x 2 + 2xyz(x + y + z) azonosság
miatt xyz(x + y + z) = 0, azaz xyz = 0.
1087. Az egyenlet (x 2 − 2)2 = 5x 3 + 7x alakban is felírható.
1088. Ha teljesülne
√ a megadott egyenlőség, √ akkor az
(a − b · 3)2 = 99 999 − 111 111 · 3 egyenlőség is fennállna; de ez nem lehet,
mert itt a jobb oldalon negatív szám áll.
√ √
1089. Ha teljesülne a megadott egyenlőség, akkor az (5 − 3 · 2)m = (3 − 5 · 2)n
egyenlőség is fennállna; de ez nem lehet, mert a bal oldalon 1-nél kisebb, a jobb
oldalon 1-nél nagyobb abszolút értékű szám áll.
1090. Hasonló az előzőkhöz.
√ √
1091. ( 26 + 5)1993 − ( 26 − 5)1993 értéke egész szám. Ezért azt kell belátni, hogy
 1993
√ 1 √ 1 1 1
( 26 − 5)1993 < . 26 − 5 = √ < = .
10 26 + 5 5 + 5 10
20. Különféle algebrai feladatok 265
√ √
1092. (45 + 1975)30 + (45 − 1975)30 értéke páros szám (= 2n), továbbá
 30
√ 50 √
(45 − 1975)30 = √ < 1, emiatt [(45 + 1975)30 ] páratlan,
45 + 1975
hiszen = 2n − 1.
Az említett összeg páros, mert
√ √
(45 + 1975)30 + (45 − 1975)30 =
  28 30 26 30 2

= 2 4530 + 302 45 · 1975 + 4 45 · 1975 2


+ · · · + 28 45 · 1975 14
+ 1975 15
.
√ √ √ √
√− 220)
1093. [(15 + 220)19 + (15 + 220)82 ] + [(15 + (15√− 220)82 ] értéke 10-zel
19

osztható egész szám, továbbá (15 − 220)19 + (15 − 220)82 < 1.


Így a tizedesvessző előtti számjegy 9-es.
1094. A tizedesvessző melletti jegyek: . . . 7, 9 . . . .
1095. A diszkrimináns az m minden értékére pozitív, hiszen 4 az értéke, azonban szük-
séges még az m + 1 = 0 feltétel is.
1096. A gyökök és együtthatók közti összefüggés szerint a két gyök szorzata c, tehát c
mindig negatív.
1097. Helyettesítsünk az x helyére 0-t.
1098. Az y = ax 2 + bx + c függvénynek a feltétel szerint nincs zérushelye. Nézzük a
függvény értékét az x = 1 helyen: y = a + b + c < 0, tehát a parabola az y-tengely
alatt helyezkedik el, a függvény értékei negatív számok. Így az x = 0 helyen is
negatív értéket vesz fel, ez az érték c.
1099. Az f (x) = x 2 + bx + ac polinomra a feladat feltétele miatt f (c) = c2 + bc +
+ ac = (c + b + a)c < 0 teljesül. Az f (x) = x 2 + bx + ac függvényt ábrázoló
parabola álló helyzetű és f (c) < 0 miatt előjelet vált, tehát a függvénynek vannak
zérushelyei. Ez azt jelenti, hogy az x 2 + bx + ac = 0 egyenletnek vannak valós
gyökei, tehát D = b2 − 4ac > 0.
1100. Az ábra szerint mindegyik parabolának van két zérushelye; ezért a 2 > 4bc, b2 >
> 4ca és c2 > 4ab. Mivel a, b, c pozitív számok (hiszen a parabolák szárai
felfelé mutatnak), az egyenlőtlenségek összeszorozhatók: a 2 b2 c2 > 64a 2 b2 c2 .
Nem találunk olyan a, b, c valós számokat, amelyekre ez az egyenlőtlenség
teljesülne.
1101. Három különböző parabolának legfeljebb 6 metszéspontja van. Az x = 1 he-
lyen mindegyik függvény értéke a + b + c. Tehát az x = 1 helyen lennie kell
egy újabb közös pontnak, ami nem lehet: ugyanis ez már a 7. metszéspont len-
ne. Tehát nincsenek olyan a, b, c valós számok, amelyek kielégítenék a feladat
követelményeit.
Azért sem léteznek ilyen parabolák, mert az x = 1 helyhez tartozó pont mindhá-
rom parabola közös pontja, s ilyen esetben a három parabolának összesen legfel-
jebb 4 közös pontja lehetne.
266 Megoldások, útmutatások

1102. x = 1 mindig megoldás.


1104. Az f (x) = ax 2 + bx + c polinom 0 és 1 közötti zérushelyei x1 és x2 . Ekkor f (x) =
= a(x − x1 )(x − x2 ). f (0) és f (1) pozitív egészek, ezért f (0) · f (1) ≥ 1, azaz
1 1
a 2 x1 (1 − x1)x2 (1 − x2) ≥ 1. Mivel x1 (1 − x1) ≤ , x2 (1 − x2 ) ≤ , és nem állhat
4 4
fenn mindkét helyen egyenlőség, így a 2 > 16, a ≥ 5.
a = 5 lehetséges, pl.: 5x 2 − 5x + 1 = 0.
1105. Tekintsük az x = 0, x = 1, x = −1 helyettesítéseket.
 
2 1 a b
1107. f (x) = ax +bx +c, f (0) = c, f = + +c, f (1) = a +b +c. Mivel 0 = 2a +
  2 4 2  
1 1
+ 3b + 6c = f (0) + 4f + f (1), tehát az f (0), f , f (1) számok között
2 2
vannak ellentétes előjelűek, így a polinomnak van 0 és 1 közötti zérushelye.
1108. a) Vizsgáljuk a bal oldali kifejezést. Ez x = a-ra pozitív, x = b-re negatív, x = c-re
pozitív értéket vesz fel, tehát van zérushelye a és b, ill. b és c között.
b–c) Hasonló a)-hoz.
d) A bal oldali kifejezést jelölje f (x), s vizsgáljuk az f (a)·f (b) és az f (b)·f (c)
szorzatokat.
e–f) Vizsgáljuk a bal oldali kifejezés értékét x = 0 és x = 1 esetén.
1109. Legyen az Ai pont az (a1 + a2 + · · · + ai ; b1 + b2 + · · · + bi ) (i = 1, 2, . . . , n) koor-
dinátákkal megadva. Az egyenlőtlenség bal oldalán az OA1 A2 . . . An töröttvonal
hossza szerepel, amely nem rövidebb az OAn szakasznál, melynek hossza a jobb
oldalon áll. (O = (0; 0))
1110. Az a + b + c oldalú ABCD négyzet AB oldalára mérjük egymás után az a,
c, b szakaszokat, a BC oldalra a b, a, c szakaszokat, s az osztáspontokon át
húzzunk párhuzamosakat az oldalakkal. Kijelölhető A és C között egy három
szakaszból álló töröttvonal úgy, hogy ennek hossza az egyenlőtlenség bal oldalán
álló kifejezés, s ez nyilván legalább akkora, mint az AC átló.
1111. Vegyünk fel három, O pontból induló szakaszt! Az OA, OB, OC hossza rendre
a, b, c, továbbá OA és OB, valamint OB és OC között 60◦ -os szög van. Ekkor
a feladatban szereplő egyenlőtlenség az |AB| + |BC| ≥ |AC| egyenlőtlenséggel
azonos.
ab
1112. Itt is segít a geometriai megfogalmazás. A keresett érték: x = .
a+c
1113. Hasonló az előzőkhöz. Legyen P (0; −1), Q(0; 1), X(x; x).
A |P X| + |QX| ≥ |P Q| egyenlőtlenséget a koordináták segítségével felírva
kapjuk a minimumot, 2-t.
20. Különféle algebrai feladatok 267
 √   √ 
1 2 2 2
1114. Hasonló az előzőhöz. Itt P − ; ,Q ;− , X(x; 0).
3 3 3 3
1
Egyenlőség x = -nál lesz.
6
1115. O(0; 0), A(2a; 2b), B(a + c; b). Az |OB| + |BA| ≥ |OA| egyenlőtlenséget írjuk
fel a koordinátákkal.
1116. Az ABC derékszögű háromszög BC, CA, AB oldalainak hossza 3, 4, 5. A
háromszög belsejében levő P pont olyan, hogy a BP C szög derékszög, a CP A
y
szög 120◦ -os. A P A, P B, P C szakaszok hosszát x, √ , z jelöli. Ekkor a három
3
egyenlet a három kis háromszögben√ felírt koszinusz-tétel,
√ a keresett kifejezés
pedig a háromszög területének 4 3-szorosa, 24 3.
2x 2y 2z
1117. Az egyenleteket átalakítva: y = ,z= ,x= . A kifejezések
1−x 2 1−y 2 1 − z2
emlékeztetnek tg 2α képletére. Legyen x = tg α, akkor y = tg 2α, z = tg 4α,

x = tg 8α. Ez azt jelenti, hogy tg α = tg 8α, melynek megoldása α = . Az
7
egyenletrendszer megoldásai:
kπ 2kπ 4kπ
x = tg , y = tg , z = tg , ahol k egész szám.
7 7 7
1118. Legyen x = cos α, akkor 2x 2 −1 = cos 2α, 8x 4 −8x 2 +1 = cos 4α. Így az egyenlet
8·cos α·cos 2α·cos 4α = 1, ezt sin α-val szorozva a sin 8α = sin α egyenlethez
2kπ (2n + 1)π
jutunk. Ennek gyökei: α = és α = . Az egyenlet gyökei: x =
7 9
2kπ (2n + 1)π
= cos és x = cos , ahol k és n egész számok.
7 9
Legyen a = cos2 x, b= cos2 y, c = cos2 z(0◦ ≤ x, y, z ≤ 90◦√
1120.  ). Ekkor a
a(1 − b)(1 − c) + b(1 − a)(1 − c) + c(1 − a)(1 − b) − abc ≤ 1
egyenlőtlenség
cos x sin y sin z + cos y sin x sin z + cos z sin x sin y − cos x cos y cos z ≤ 1
alakban írható. Ez pedig igaz, mert a bal oldali kifejezés = − cos(x + y + z).
◦ ◦
 α ésβ olyan szögek, melyekre 0 < α, β < 45 és tg α = x, tg β = y.
1121. Legyenek
x−y 
Ekkor   = | tg (α − β)|, és ez valóban kisebb 1-nél.
1 + xy 
1122–1124. Hasonló az 1121. feladat megoldásához.
1125. Egységoldalú négyzetben kijelölhetünk a, b területű téglalapokat, melyek közös
részének területe ab. Ekkor a két téglalap együttes területe a közös rész nélkül
a + b − 2ab lesz, s ez kisebb a négyzet területénél.
268 Megoldások, útmutatások

1126. Hasonló az előzőhöz. Most egy egységoldalú szabályos háromszög egy-egy ol-
dalára felmérjük a c, b és az a hosszúságú szakaszt úgy, hogy a háromszög
csúcsainál rendre c és 1 − a, b és 1 − c, a és 1 − b hosszúságú szakaszok talál-
kozzanak. Így a háromszög sarkaiban kijelöltünk egy-egy kisebb háromszöget.
A bizonyítandó egyenlőtlenség geometriai tartalma már látható lesz: a három kis
háromszög együttes területe kisebb, mint a szabályos háromszög területe.
1127. A bal oldali összeg összeszámlálja, hogy n-ig hány négyzetszám (12 , 22 , . . . ),
hány köbszám (13 , 23 , . . . ), hány negyedik hatvány (14 , 24 , . . . ) stb. van. A jobb
oldalon megszámláljuk, hogy n-ig mennyi 2-hatvány (21 , 22 , . . . ), 3-hatvány
(31 , 32 , . . . ), 4-hatvány (41 , 42 , . . . ) stb. van. Ha a listákon álló első elemeket
elhagyjuk, akkor a bal oldali listákon ugyanazok a számok maradnak meg (és
mindegyik ugyanannyiszor), mint a jobb oldali listákon.
p(x)
1128. Az y = sin x függvénynek végtelen sok zérushelye van, míg az y = függ-
q(x)
vénynek véges sok: annyi, ahány gyöke van a p(x) = 0 egyenletnek; s ha a p(x)
polinom n-edfokú, akkor legfeljebb n.
1 1 1
1129. Legyen Sn = 1 + + + · · · + . Mutassuk meg, hogy lim Sn = ∞ és
2 3 n n→∞
Sn
lim = 0. Lássuk be, hogy nincsenek olyan p(x) és q(x) polinomok, ame-
n→∞ n
p(n) p(n)
lyekre lim = ∞ és lim = 0.
n→∞ q(n) n→∞ n · q(n)



3k + 1 ∞
k ∞
1
1130. S = k
= 3 k
+ k
. (Végtelen sok tagú összegen csak akkor
k=1
3 k=1
3 k=1
3
végezhetünk ilyen átalakításokat, ill. egyéb műveleteket, ha belátjuk, hogy az
összeg létezik, azaz az összeg értéke véges.)
∞
1 1
k
= (végtelen mértani sor)
k=1
3 2
Legyen |x| < 1, P = x + 2x 2 + 3x 3 + 4x 4 + . . . ,
x · P = x 2 + 2x 3 + 3x 4 + 4x 5 + . . . ,
x
P − xP = x + x 2 + x 3 + x 4 + · · · = ,
1−x
x
azaz P = .
(1 − x)2
∞ ∞  k
x 1 1/3 9
Tehát kx =k
. Ezért 3 k· =3· = .
k=1
(1 − x) 2
k=1
3 (1 − 1/3) 2 4
∞
3k + 1  ∞
k  ∞
1 9 1 11
Emiatt S = =3 + = + = .
k=1
3k k=1
3k k=1 3k 4 2 4
1131. Az összeg az abszolútértékek felbontása után az S = ±(a1 − a2 ) ± (a2 − a3 ) ±
±(a3 − a4 ) ± · · · ± (a1999 − a2000 ) ± (a2000 − a1999 ) alakot ölti, ahol a ± jelből a
21. Függvényegyenletek 269

megfelelőt választjuk. Ebben az összegben 2000 tag előtt + jel, 2000 tag előtt −
jel fog állni.
S ≤ 2(2000 + 1999 + · · · + 1001) − 2(1000 + 999 + · · · + 1) = 2(1000 + 1000 + · · · +
+ 1000) = 2 000 000. Ez az érték elérhető, ha pl. az a1 , a2 , a3 , . . . , a1998 , a1999 ,
a2000 sorozat a 2000, 1, 1999, 2, 1998, 3, . . . , 1002, 999, 1001, 1000 sorozatot
jelöli.
1132. f (x) = f (−x).
 2π
1133. f (x) dx = 0, ahol f (x) a feladatban megadott kifejezést jelöli.
0
1−x
1134. f1 (x) = − , f2 (x) = x, f3 (x) = f0 (x), fn+3 (x) = fn (x), f1994 (1994) = 1994.
x

21. Függvényegyenletek

1135. Nincs megoldás. Ha x = 1, akkor f (1) = 1, ha x = −1, akkor f (1) = −1.


1136. Helyettesítsünk x helyébe (1−x)-et, a 2f (1−x)+3f (x) = 4(x −1)−1 egyenletet
kapjuk. Ez és az eredeti egyenlet adja a megoldást:
11
f (x) = −4x + .
5
1137. Nincs ilyen függvény. Helyettesítsünk x helyébe (−x)-et.
1138. Helyettesítsünk x helyébe (−x)-et.
1139. Helyettesítsünk x helyébe (−x)-et.
1140. Helyettesítsünk x helyébe (−x)-et.
1141. Nincs ilyen függvény. Legyen t = x − 1, akkor
f (t) − f (−t) = t + 1,
f (−t) − f (t) = −t + 1
Ezek megfelelő oldalainak összege a 0 = 2 „egyenlőséget” adja.
1
1142. Helyettesítsünk x helyébe -et.
x
1143. f (x) = x − 1.
1144. Helyettesítsünk x helyébe (1 − x)-et.
1145. Helyettesítsük az egyenletbe az x = t és az x = 1 − t értékeket:
2f (t) + f (1 − t) = t 2 , 2f (1 − t) + f (t) = (1 − t)2 .
1
Ebből f (t) = (t 2 + 2t − 1).
3
270 Megoldások, útmutatások

x−1
1146. Legyen φ(x) = , és vizsgáljuk a következő sorozatot: 0 < x1 < 1, xn =
x
= φ(xn−1 ), n = 2, 3, . . . Ekkor a sorozat elemei felírhatók x1 segítségével: x1 ,
x1 − 1 1
x2 = , x3 = , x4 = x1 , . . . Ennek alapján:
x1 1 − x1
f (x1 ) + f (x2 ) = 1 + x1 , f (x2 ) + f (x3 ) = 1 + x2 , f (x3 ) + f (x1
) = 1 + x3 , ahonnan
1 1 1 1
f (x1 ) = (1 + x1 − x2 + x3 ), azaz f (x) = x+ + .
2 2 x 1−x
       
1 1 1 1 x 1
1147. f (x) + f 1 − = ; f 1− +f = ;f + f (x) =
x x x 1−x x−1 1−x
x −x +1
3 2
= 1 − x egyenletekből kapjuk, hogy f (x) = .
2x(1 − x)
1148. f (x) = 1 − x 2 .
1149. Helyettesítsünk x helyébe (1 − x)-et. A megoldás: f (x) = x + 1.
1 x−1
1150. Helyettesítsünk x helyébe -et, ill. -et. A megoldás: f (x) = 1 − x.
1−x x
1151. Legyen y = x. Így az egyenlet: f (2x) − f (0) = 4x 2 . Jelölje a az f (0) értékét. A
megoldás: f (x) = x 2 + a.
1152. Legyen y = 0.
1153. Legyen y = 1. Így az egyenlet: f (x) · f (1) = f (1), ezért f (x) ≡ 1.

1154. Legyen y = x. Így az egyenlet: f 2 (x) = f (0), vagyis f (x) = ± f (0) minden
x-re. A folytonosság miatt f (x) = a, a konstans. Ezt helyettesítve az eredeti
egyenletbe: a 2 = a. Tehát f (x) ≡ 0 vagy f (x) ≡ 1.
1155. f (x) ≥ x − 1 és f (x) ≤ x − 1, tehát f (x) = x − 1.
1156. A bal oldali egyenlőtlenségből f (x) ≤ x 2 + x. Míg a jobb oldali egyenlőtlenség
szerint f (x − 1) ≥ x 2 − x. Írjunk x helyébe (x + 1)-et, ekkor az f (x) ≥ x 2 +
+x egyenlőtlenséget kapjuk. A két egyenlőtlenség egybevetése adja a megoldást:
f (x) = x 2 + x.
1157. x = y = 0 adja, hogy egyrészt f (0) ≤ 0, másrészt f (0) ≤ 2f (0), tehát f (0) ≥ 0,
s ezekből f (0) = 0. f (x) + f (−x) ≥ f (0) = 0, azaz f (x) ≥ −f (−x) ≥ x. A
megoldás f (x) = x.
1158. x = y = 0 adja, hogy f (0) értéke 0, −1 vagy 1. Az y = −x helyettesítés
alkalmazásával eljutunk a megoldáshoz: ha f (0) = 0, akkor f (x) = 0; ha f (0) =
= 1, akkor f (x) = 1; ha f (0) = −1, akkor f (x) = −1.
1159. Legyen y = 1. Ekkor az egyenlet az xf (x) = f (1) alakot ölti. x = 0 adja, hogy
f (1) = 0, innen f ≡ 0.
21. Függvényegyenletek 271

1160. Legyen y = 0. A megoldás: f (x) = 1 vagy f (x) = 0.


1161. Legyen x = y = 1. Ekkor f (1) = 0 vagy f (1) = −2. Most az y = 1 helyettesítés
adja, hogy az első esetben f (x) = x − 1, a második esetben pedig f (x) = −x − 1.
1162. Hasonló az előzőhöz.
1163. Legyen x = y = 1. Ekkor f (1) = 0 vagy f (1) = 2. Most az y = 1 helyettesítés
adja, hogy az első esetben f (x) = 1 − x, a második esetben pedig f (x) = 1 + x.
1164. Legyen y = x. Ekkor: 2xf (x) = 2xf 2 (x), azaz xf (x)(f (x) − 1) = 0. Ennek
folytonos megoldásai: f (x) ≡ 0, f (x) ≡ 1. Második megoldás. Legyen x =
= y = 1. Innen kapjuk, hogy f (1) = f 2 (1), tehát vagy f (1) = 0, vagy f (1) = 1.
Ha f (1) = 0, akkor legyen y = 1, s kapjuk a megoldást: f (x) ≡ 0. Ha f (1) = 1,
akkor is legyen y = 1; ekkor az egyenlet x(f (x)−1) = 0 alakra hozható, melynek
egyetlen folytonos megoldása van: f (x) ≡ 1.
1165. Legyen y = x. Ekkor az egyenlet [f (x) − x]2 − 1 = 0 alakra hozható.
1166. Végezzük el az x = 0, y = t, majd az x = t, y = 2t és végül az x = t, y = −2t helyette-
sítéseket! A kapott három egyenletből kifejezhető f (t). A megoldás: f (x) = x+a.
1167. Hasonló az előzőhöz.
π

1168. Tekintsük az alábbi helyettesítéseket, ahol a = f (0), b = f :


2
x = 0, y = t helyettesítéssel az f (t) + f (−t) = 2a cos t,
π π
x = + t, y = helyettesítéssel az f (π + t) + f (t) = 0,
2 2
π π
x = , y = + t helyettesítéssel az f (π + t) + f (−t) = −2b sin t.
2 2
egyenleteket kapjuk. Az első két egyenlet megfelelő oldalainak összegéből ki-
vonjuk a harmadik egyenletéit, s osztunk 2-vel.
A megoldás: f (t) = a cos t + b sin t.
!
1169. Legyen a = f (0) és y = 0. Ekkor a következő egyenlethez jutunk: a f (x)+cos x =
= 0. Tehát vagy minden x-re f (x)+cos x = 0, vagy a = 0. Utóbbi esetet vizsgálva,
tekintsük az x = y helyettesítést.
!2
Ekkor 0 = f (0) = f 2 (x) + 2f (x) cos x − sin2 x, tehát f (x) + cos x = 1, s itt
csak f (x) = 1 − cos x a megoldás, hiszen az f (x) = −1 − cos x függvényre
f (0) = 0. Tehát két megoldás van: f (x) = − cos x és f (x) = 1 − cos x.
Megjegyzések. Az egyenlet más úton történő megoldásairól és egyéb vizsgála-
tokról olvashatunk A Matematika Tanítása 1984. augusztusi számában, ill. a Ma-
tematikai Lapok (Kolozsvár) 1985. októberi számában Vincze Endre: Egy függ-
vényegyenlet folytonos és nemfolytonos megoldásai c. cikkében.
272 Megoldások, útmutatások
π
π
π
π π
π

1170. f (x) = f x − + = f x− cos + f cos x − =


π
π

2 2

π
2 2 2 2
=f cos x − =f sin x. A megoldás: f (x) = a sin x.
2 2 2
1171. f (2x) = f (x + x) = f (x) + f (x) = 2f (x), f (3x) = f (2x + x) = f (2x) + f (x) =
= 2f (x) + f (x) = 3f (x) és általában: f (nx) = nf (x), n ∈ N .
Lássuk be, hogy az utóbbi egyenlőség igaz minden n ∈ Z esetén, felhasználva
x
a könnyen látható f (0) = 0 tulajdonságot. A qf = f (x) egyenlőségből
   q   
x 1 p x p
az f = f (x) azonosságot nyerjük. Így f x = pf = f (x),
q q q q q
p, q ∈ Z, tehát minden r racionális számra f (rx) = rf (x). A folytonosság miatt
ez tetszőleges r valós számra is igaz. Tehát f (r) = f (r · 1) = rf (1), azaz a
megoldás: f (x) = cx, c = f (1).
1172. Ha valamely x-re f (x) = 0, akkor f (x) ≡ 0. A többi megoldást keresve, ha
x = y = 0, akkor f (0) = 1. Az előző
!r feladat megoldásához hasonló lépésekben
beláthatjuk, hogy f (rx) = f (x) és a megoldás: f (x) = a x , a = f (1).
1173. Legyen b > 0, b = 1, t = logb x, s = logb y, ekkor az egyenlet az f (bt bs ) =
= f (bt ) + f (bs ) alakot ölti; s a g(z) = f (bz ) jelölést használva ez az egyenlet
g(t + s) = g(t) + g(s) alakban írható, melynek az 1171. feladat szerint a megol-
dása g(x) = cx, tehát f (t) = f (blogb t ) = g(logb t) = c logb t, c = f (b).
1174. Hasonló az előzőhöz.
1175. Legyen g(x) = f (x) + 1, ekkor g(x + y) = g(x) + g(y). (Lásd az 1171. feladatot!)
 
a+b f (a + b) + f (0)
1176. Az x = a + b, y = 0 esetben f = . Innen és az eredeti
2 2
egyenletből az f (a + b) = f (a) + f (b) − A egyenletet kapjuk, ahol A = f (0). Az
előző feladat megoldását követve f (x) = cx − A.
1177. Az 1171. és az 1172. feladat megoldásával rokon. Lássuk be előbb, hogy f (x r ) =
rx r−1 f (x) igaz minden r természetes számra, minden r egészre (itt alkalmazzuk
1
az y = helyettesítést), továbbá minden racionális számra is. f (x) folytonossága
x
miatt az egyenlőség minden r valós számra igaz. Legyen r = logx y. Ekkor
y lg y f (y) f (x)
f (y) = f (x) , melyből = ≡ a, a állandó. A keresett megoldás:
x lg x y lg y x lg x
f (x) = cx lg x, ahol c állandó.
1178. y = 1 helyettesítés adja, hogy f (x + 1) = f (x) + 1. A megoldás további lépései:
1
f (0) = 1; f (x + n) = f (x) + n; f (n) = n + 1. Az x = n, y = helyettesítés adja,
  n  
1 1 1 p p
hogy f = 1 + . Az x = p, y = helyettesítésből pedig az f = +1
n n q q q
összefüggéshez jutunk.
A folytonosság miatt f (x) = x + 1, x ∈ R.
21. Függvényegyenletek 273

1179. Nincs.    
1 1 1
1. megoldás: Az x = y = − helyettesítésből f − g − = 0 adódik. Ha
  2 2 2
1 1 1
f − = 0, akkor tekintsük az x = − , y = helyettesítést, melyből 0 = 1
2 2 2
következne.
2. megoldás: f (0)g(y) = y + 1, f (x)g(0) = x + 1, f (0)g(0) = 1. Az első két
egyenlőség megfelelő oldalainak szorzatát elosztva a harmadikkal az f (x)g(y) =
= xy + x + y + 1 egyenletet kapjuk, s ezt vessük egybe az eredeti egyenlettel!
3. megoldás: f (5)g(3) = 9, f (3)g(5) = 9, f (5)g(5) = 11, f (3)g(3) = 7. Az
első két egyenlőség bal oldalának szorzata egyenlő az utolsó két egyenlőség bal
oldalának szorzatával, azonban 9 · 9 = 11 · 7.
1180. Nincs. Hasonló az előző feladathoz.
1181. Helyettesítsük (x; y) helyére rendre a (0; 0), (0; 1), (1; 0), (1; 1) számokat.
1182. Nincs. x = 1-re h(4) + h(2) = 14, x = −1-re h(2) + h(4) = 4.
1183. Nincs. f (g(x)) > g(x) > g(f (x)).
1184. Nincs. A feltétel miatt nyilván f (x) > x minden x-re, vagy f (x) < x minden
x-re. Vegyük az első esetet. Ekkor f (f (x)) > f (x) > x.
1185. Készítsük el az f (x) függvény gráfját. Az a valós számból vezessen nyíl az f (a)
számhoz. Egy pontból kövessük a nyilak útját. Ha f (x) gráfját már sikerült meg-
szerkeszteni, abból f (f (x)) gráfját könnyen megkaphatjuk, ha az eredeti gráf
két, egymáshoz csatlakozó nyílból álló irányított útját egyetlen nyíllal helyette-
sítjük. Könnyen belátható, hogy f (x) gráfjának egy lánca az f (f (x)) gráfjában
két láncra bomlik, ugyanis páratlan hosszúságú körnek ugyanolyan hosszúságú
kör felel meg, viszont egy 2k hosszúságú kör két, k hosszúságú körre bomlik fel.
Mindez azt jelenti, hogy f (f (x)) gráfjában minden páros hosszúságú körből pá-
ros soknak kell lennie. Ha tehát egy függvény gráfjában valamilyen k-ra páratlan
sok 2k hosszúságú kör van, akkor ez a függvény biztosan nem áll elő f (f (x))
alakban. x 2 − 2 gráfjában egyetlen 2 hosszúságú kör van, hiszen az x 2 − 2 = y,
y 2 − 2 = x egyenletrendszer, azaz az (x 2 − 2)2 − 2 = x, vagyis a 0 = x 4 − 4x 2 −
− x + 2 = (x 2 − x − 2)(x 2 + x − 1) egyenlet megoldásaiból csak egy 2 hosszúságú
kör állítható össze.
1186. A bizonyítást többféle módon is végezhetjük. Kiszámolhatjuk f (f (f (x)))−x-et,
majd az így kapott nyolcadfokú egyenletet megoldjuk (könnyen felírhatjuk első-
és harmadfokú tényezők szorzataként). Az is elegendő, ha mutatunk csatlakozó
intervallumokat, ahol a függvény előjelet vált.
Kevésbé számolós megoldás a következő. Vizsgáljuk az f (x) függvény grafikon-
ját a [−2; 2] intervallumon. Ezt követően ugyanezen az intervallumon fel tudjuk
vázolni az f (f (x)) függvény, majd pedig az f (f (f (x))) függvény grafikonját.
274 Megoldások, útmutatások
x
x
x

1187. f (x) = f =f = · · · = f n . A függvény folytonossága miatt f (x) =


2 4 2
= f (0) állandó.
1188. f (x, y, z) = 2f (y, z, x) = 4f (z, x, y) = 8f (x, y, z), innen f (x, y, z) ≡ 0.
1189. f (x) páratlan, g(x) páros függvény, hiszen
f (x) = f (2x − x) = f (2x)g(x) − g(2x)f (x),
f (−x) = f (x − 2x) = f (x)g(2x) − g(x)f (2x),
g(x) = g(2x − x) = g(2x)g(x) − f (2x)f (x),
g(−x) = g(x − 2x) = g(x)g(2x) − f (x)f (2x).
Innen f (0) = 0, g(0) = g(0 − 0) = g 2 (0), amiből g(0) = 0 vagy g(0) = 1.
1190. Az x = y = 0 helyettesítés adja az f (0) = 2f (0)g(0) és a g(0) = g 2 (0) − f 2 (0)
1
összefüggéseket. Ha f (0) = 0, akkor az első összefüggésből g(0) = következik,
2
ami ellentmond a második összefüggésnek, így f (0) = 0. Ezért g(0) = 1 vagy
g(0) = 0. Az utóbbi eset az f (x + y) = f (x)g(y) + g(x)f (y) összefüggés miatt
f (x) ≡ 0 következményt vonja maga után (y = 0).
1192. Teljes indukcióval mutassuk meg a könnyen megsejthető megoldást: f (n) =
= logn (n!).
1194. Ha n = m = 1, akkor f (2) = 4. n = 2, m = 1 helyettesítésre f (3) = 8. n = 3,
m = 1 helyettesítésre f (4) = 13. Esetleg még néhány próbálkozás, s megsejthető,
n(n + 3)
hogy f (n) = − 1, és ez teljes indukcióval igazolható.
2
1195. 1. megoldás: Előbb teljes indukcióval lássuk be, hogy f (2n ) = 2n . Majd a
2n = f (2n ) < f (2n + 1) < f (2n + 2) < · · · < f (2n+1 ) = 2n+1 egyenlőtlenséglánc
miatt kapjuk, hogy f (2n + i) = 2n + i, i = 1, . . . , 2n − 1. Tehát: f (n) = n.
2. megoldás: Itt is teljes indukcióval bizonyítunk. f (n) = n nyilvánvaló n = 1 és
n = 2 esetén.
n = 2k-ra f (n) = f (2k) = f (2)f (k) = 2k = n.
Ha n = 2k + 1, akkor n − 1 = 2k = f (2k) < f (2k + 1) < f (2k + 2) = n + 1 adja,
hogy f (2k + 1) = 2k + 1.
1196. Ha m = n = 0, akkor f (0) = 0. Legyen m = n, ekkor f (2n) = f (3n), tehát
f (4k) = f (6k) = f (9k). Másrészt, ha n = 3m, akkor f (4m) + f (2m) = f (9m),
tehát f (2m) = 0. Ha m = 0, akkor 2f (n) = f (3n),
1 1
azaz f (n) = f (3n) = f (2n) = 0, így f (n) ≡ 0.
2 2
1197. f (n) = 1.
21. Függvényegyenletek 275

1198. Legyen n ≤ 100 és n + 11 > 100, azaz 90 ≤ n ≤ 100.


f (n) = f (f (n + 11)) = f (n + 11 − 10) = f (n + 1),
így f (90) = f (91) = · · · = f (100) = f (101) = 91.
Legyen most n < 90; akkor van olyan m, hogy 90 ≤ n + 11m ≤ 100, és f (n) =
= f 2 (n + 11) = · · · = f m+1 (n + 11m) = f m (f (n + 11m)) = f m (91) = 91, (ahol
definíció szerint f k+1 (x) = f (f k (x))).
Tehát f (18) = 91.
1199. f (32) = 63, f (63) = f (f (32)) = 125, f (125) = f (f (63)) = 249, f (249) =
= f (f (125)) = 497, f (497) = f (f (249)) = 993, f (993) = f (f (497)) = 1985,
f (1985) = f (f (993)) = 3969. Látható, hogy az f (x) = 2x − 1 függvény kielégíti
a felírt egyenleteket.
1 1
1200. f (999) = f (f (1000)) = = . Az f (x) függvénynek két értékét már
f (1000) 999
1
ismerjük: f (1000) = 999 és f (999) = . A folytonosság miatt van olyan a,
999
1 1
amelyre f (a) = 500. De akkor f (500) = f (f (a)) = = .
f (a) 500
1201. x = 1, x = 0, x = −1 az f (x) = 0 egyenlet gyökei, ezért f (x) = (x 3 − x)g(x).
Ezt az egyenletbe helyettesítve a g(x + 1) = g(x) összefüggést nyerjük. Akkor a
g(x + 1) − g(x) polinomnak végtelen sok zérushelye van, ezért g(x) ≡ konstans,
tehát f (x) = c(x 3 − x).
1202. x = 0, x = 1 az f (x) függvény zérushelyei, ezért f (x) = (x 2 − x) · g(x). Ezt a
feltétel egyenletébe helyettesítve a g(x) ≡ g(x − 1) összefüggést nyerjük. Akkor
a g(x)−g(x −1) polinomnak végtelen sok zérushelye van, ezért g(x) ≡ konstans,
tehát f (x) = c(x 2 − x).
1203. Helyettesítések mutatják, hogy az f (x) polinomnak zérushelyei a 0, 1, 2, 3, 4,
. . . számok; s ez azt jelenti, hogy f (x) ≡ 0, hiszen csak ennek a polinomnak
van végtelen sok zérushelye.
1204. Ha x = 0, akkor 0 = (−12) · f (0), tehát a polinomnak gyöke a 0. A feltételből
látható, hogy ha (α − 1) gyöke a keresett polinomnak — akkor a bal oldal nulla
— és α = 12, akkor a jobb oldalon f (α) = 0, tehát α is gyök. A 0-val együtt
tehát az 1, 2, . . . , 11 számok is gyökök, tehát
f (x) = x(x − 1) · (x − 2) · . . . · (x − 11) · g(x). Ezt az alakot a feltételbe írva,
majd alkalmasan egyszerűsítve azt kapjuk, hogy például minden 12-nél nagyobb
x-re g(x) = g(x − 1). Ez azt jelenti, hogy g(x) = c (=konstans), így f (x) =
= c · x(x − 1) · (x − 2) · . . . · (x − 11), ahonnan f (12) = c · 12!, vagyis c = 1.
Emiatt f (x) = x(x − 1) · (x − 2) · . . . · (x − 11).
1205. Teljes indukcióval megmutatható, hogy f (n) = nf (1), n ∈ N . Ez azt jelenti,
hogy az f (x) − xf (1) polinomnak végtelen sok zérushelye van, így a megoldás:
f (x) = cx, ahol c = f (1).
276 Megoldások, útmutatások

1206. Legyen g(x) = (x + 1)f (x) − x, g(x) (n + 1)-edfokú polinom. Tudjuk, hogy
g(x) értéke az x = 0, 1, . . . , n helyeken nulla, ezért g(x) = (x + 1)f (x) − x =
= Ax(x − 1)(x  − 2) . . . (x − n). x = −1 esetén 1 = A(−1)n+1 (n + 1)!. Ezért
1 (−1) x(x − 1)(x − 2) . . . (x − n)
n+1
f (x) = x+ .
x+1 (n + 1)!
n
Innen f (n + 1) = 1 vagy , attól függően, hogy n páratlan vagy páros.
n+2
1207. Helyettesítsünk az f (x 2 −x +1) = g(x 2 +x +1) egyenlőségben x helyébe (−x)-et:
f (x 2 + x + 1) = g(x 2 − x + 1).
Emiatt f (x 2 − x + 1) = g(x 2 + x + 1) = g((x + 1)2 − (x + 1) + 1) = f ((x + 1)2 +
+ (x + 1) + 1) = f (x 2 + 3x + 3).
Így f (1) = f (7) = f (21) = . . . , azaz az f (x) polinom végtelen sok helyen
ugyanazt az értéket veszi fel, ezért a polinom azonosan konstans.
   
f (x) + f (−x) f (x) − f (−x)
1208. f (x) = + .
2 2
1209. Tegyük fel, hogy x 2 = g(x) + h(x), g(x + p) = g(x), h(x + q) = h(x), p, q > 0
minden x ∈ R esetén. Írjunk x helyébe rendre 0, p, q, p + q értékeket.
0 = g(0) + h(0) = g(p) + h(q),
p2 = g(p) + h(p),
q 2 = g(q) + h(q),
(p + q)2 = g(p + q) + h(p + q) = g(q) + h(p).
Az első és az utolsó egyenlőség megfelelő oldalainak összegéből kivonjuk a má-
sodik és harmadik egyenlőséget, s ekkor a 2pq = · · · = 0 összefüggést nyerjük.
Tehát az f (x) = x 2 függvény nem állítható elő két periodikus függvény össze-
geként. Ugyanezt állíthatjuk az f (x) = x 3 függvényről is. A bizonyítás hasonló-
képpen végezhető.
1210. Nincs. Egy ilyen függvény értékkészlete megszámlálható, mert racionális helyen
felvett értékei megszámlálhatók, a többi helyen racionális értékeket vesz fel, tehát
értékei ott is megszámlálhatók. Azonban egy nem konstans folytonos függvény
értékkészlete kontinuum számosságú halmaz.
1211. Vizsgáljuk a g(x) = f (x +1)−f (x) függvényt. Ez a függvény egyrészt folytonos,
másrészt csak irracionális értékei lehetnek. Ilyen (nem konstans) függvény pedig
nem létezik.
1 1
1212. f (x + 2a) = − , f (x + 4a) = − = −(−f (x)) = f (x).
f (x) f (x + 2a)
21. Függvényegyenletek 277
√ √
1213. f (x + 2) + f (x) = 2f (x + 1), f (x) + f (x√− 2) = 2f (x − 1), !
összegük: 2f (x) + f (x + 2) + f (x − 2) = 2 f (x + 1) + f (x − 1) = 2f (x),
tehát f (x + 2) + f (x − 2) = 0, f (x + 4) = −f (x), f (x + 8) = −f (x + 4) = f (x).
π
Megjegyzés. Létezik az egyenletet kielégítő függvény: f (x) = sin x.
4
1214. f (x + a)f (x) = b és b = f (x + 2a)f (x + a) szorzata bf (x) = bf (x + 2a).
f (x) cos kφ − sin kφ
1215. Az f (x + ka) = összefüggést igazoljuk, s így
f (x) sin kφ + cos kφ
f (x + na) = f (x).
1216. a) 1. megoldás: Ha p > 0 a periódus, akkor cos(x + p)2 = cos x 2 minden x-re,
ezért (x + p)2 = x 2 + 2kπ, vagy (x + p)2 = −x 2 + 2kπ. Rögzített k mellett
a két egyenletet legfeljebb három x érték elégíti ki (hiszen az első egyenlet
elsőfokú, a második másodfokú), ezért az összes k egészre adott p mellett
megszámlálható sok x-re teljesülhet csak az elsőnek felírt egyenlőség.
2. megoldás:
 A cos x 2 függvény k-adik zérushelye:
π
xk = (2k − 1) . Azonban az xk+1 − xk különbségek nullához tartó, csök-
2
kenő sorozatot alkotnak.
b) Hasonló a)-hoz.
√ √
1217. a) Legyen p > 0 a periódus: cos x + p = cos x. Legyen  x = 0, √ ekkor
√ √
cos p = 1, tehát p = 2kπ.√Legyen x = p, ekkor cos 2p = cos p =
 2p √ n √
= 1, tehát 2p = 2nπ. Innen √ = 2 = ; s ez azt jelentené, hogy 2
p k
racionális.
b) Hasonló a)-hoz.
1218. a) Ha f (x + p) = f (x), p > 0, akkor cos x + cos ax = cos(x + p) + cos a(x + p).
Legyen x = 0, ekkor 2 = cos p + cos ap, ami csak úgy lehet, ha mindkét
n
összeadandó 1; azaz p = 2kπ, ap = 2nπ, tehát a = , így a racionális lenne.
k

b) Ha f (x) periodikus, akkor f (x) = − sin x − a sin ax is periodikus, és ekkor
2

periodikus f (x) + f  (x) = (1 − a 2 ) sin ax is, tehát sin ax periodikus, s az ed-


dig felsorolt függvényeknek van közös periódusa. Azonban a sin x és sin ax
függvényeknek nem lehet közös periódusa, mert a irracionális.
1219. Hasonló az 1218. a) feladat megoldásához.
278 Megoldások, útmutatások

22. Vektorok a geometriában

−−→ 2π
1220. a) Forgassuk el az OAi vektorokat szöggel O körül. Az összegben az össze-
n
adandók csak felcserélődtek, de az összeg ugyanaz maradt. Az összegvektort
ez az elforgatás nem változtatta meg, ezért az nullvektor.
−−→ −−→ −−→ −→ −−→ −→ −−→ −→ −−→
b) XA1 + XA2 + · · · + XAn = (XO + OA1 ) + (XO + OA2 ) + · · · + (XO + OAn ) =
−→ −−→ −−→ −−→
= n · XO + (OA1 + OA2 + · · · + OAn ), s a zárójelben levő összeg az a) feladat
állítása szerint nullvektor.
1221. Az előző feladat b) állítását felhasználva a kérdésre a válasz: Ha A közelebb van
O-hoz, mint B.
1222. Helyezzük a sokszöget koordináta-rendszerbe, legyen a sokszög középpontja az
origóban és egyik csúcsa se legyen rajta az x-tengelyen. A sokszög csúcsaiba a
csúcsok első koordinátáit írjuk. Ezek a számok megfelelő számok, gondoljunk
csak az 1220. a) feladat állítására.
−−→ −→ −−→ −→
1223. |A1 X|2 = (A1 O + OX)2 = |A1 O|2 + |OX|2 + 2A1 O · OX. Ezért |A1 X|2 + · · · +
−→ −−→ −−→ −−→
+|An X|2 = n(r 2 +d 2 )+2OX·(A1 O + A2 O +· · ·+ An O). A jobb oldali összeg utolsó
tagja 0, hiszen a zárójelben levő összeg az 1220. a) feladat szerint nullvektor.
1224. Az előző feladat speciális esete.
−−→ −→ −−→
1226. Legyen a kör középpontja O, a kör sugara r. P Ai = P O + OAi .
−−→ −−→ −→ −−→ −→ −−→
|P Ai |2 = P Ai · P Ai = (P O + OAi )2 = 2r 2 + 2P O · OAi .
−→ −−→ −→ −−→
|P Ai |4 = (|P Ai |2 )2 = (2r 2 + 2P O · OAi )2 = 4r 4 + 8r 2 · P O · OAi + 4r 4 · cos2 φi ,
−→ −−→
(ahol φi a P O és OAi által bezárt szöget jelenti) ugyanis
−→ −−→ −→ −−→ −→ −−→
(P O · OAi )2 = (|P O| · |OAi | · cos φi )2 = |P O|2 · |OAi |2 · cos2 φi = r 2 · r 2 · cos2 φi .
Így a vizsgált összeg:
−→ −−→ −−→ −−→
|P A1 |4 + |P A2 |4 + · · · + |P An |4 = 4n · r 4 + 8r 2 · P O · (OA1 + OA2 + · · · + OAn ) +
+ 4r · (cos φ1 + cos φ2 + · · · + cos φn ).
4 2 2 2
−−→ −−→ −−→
Ismert, hogy OA1 + OA2 + · · · + OAn = 0,
−→ −−→ −−→ −−→
ezért 8r 2 · P O · (OA1 + OA2 + · · · + OAn ) = 0.
Már csak azt kell megmutatni, hogy a cos2 φ1 + cos2 φ2 + · · · + cos2 φn összeg
értéke állandó.    
2π 2π
cos φ1 + cos φ2 + · · · + cos φn = cos α + cos α +
2 2 2 2 2
+ cos α + 2 ·
2
+
  n n

+ · · · + cos2 α + (n − 1) ·
n
Vizsgáljuk az utóbbi összeget. A 2 cos2 x = 1 + cos 2x összefüggés alapján
22. Vektorok a geometriában 279
     
2π 2π 2π
2 2
cos α + cos α + + cos α + 2 ·
2
+ · · · + cos α + (n − 1) ·
2
=
 n   n   n
1 2π 2π
= n + cos 2α + cos 2 α + + · · · + cos 2 α + (n − 1) · .
2 n n
Nem nehéz belátni,
 hogy    
2π 2π 2π
cos 2α+cos 2 α + +cos 2 α + 2 · +· · ·+cos 2 α + (n − 1) · = 0.
n n n
Az eddigi megállapítások összegzésével eljutunk a kérdezett összeg értékéhez:
n
|P A1 |4 + |P A2 |4 + · · · + |P An |4 = 4n · r 4 + 4r 4 · = 6n · r 4 .
2
1228. x = 2b − a, y = 2c − b, z = 2a − c.
x 2 = 4b2 + a 2 − 4a · b, a
y 2 = 4c2 + b2 − 4b · c, z
z2 = 4a 2 + c2 − 4c · a.
Innen x b a
x 2 + y 2 + z2 = 5(a 2 + b2 + c2 )−
c
− 4(a · b + b · c + c · a). c
Mivel a + b + c = 0, így ennek négyzete y
b
a 2 + b2 + c2 + 2(a · b + b · c + c · a) = 0,
és ezt felhasználva a korábban kapott
egyenlőségben, adódik a kívánt eredmény.
−→ − → − → −→ − → − → −→ − → −

1232. P A = P S + SA, P B = P S + SB, P C = P S + SC.
Ezek négyzeteit összeadva:

→ − → − → − →
P A2 + P B 2 + P C 2 = 3 · P S 2 + SA2 + SB 2 + SC 2 + 2P S · (SA + SB + SC). Az
összeg utolsó tagja nulla, mert a zárójelben levő összeg értéke nullvektor.
1233. Használjuk fel az előző feladat eredményét.
1234. Hasonló az előző feladathoz.
−→ −→
1240. Legyen az adott paralelogramma ABCD. Akkor |AC| · |BD| > |AC · DB| =
−→ −→ −→ −→
= |(AB + BC)(AB − BC)| ≥ ||AB|2 − |BC|2 |.
−→ −→ −→ −→
1241. Legyen a tér egy pontja O, jelölje az OA, OB, OC, OD vektorokat rendre a,
b, c, d. Ekkor az AC 2 + BD2 + AD2 + BC 2 ≥ AB 2 + CD2 egyenlőtlenség átírható
a következő alakba:
(c − a)2 + (d − b)2 + (d − a)2 + (c − b)2 ≥ (b − a)2 + (d − c)2 .
Négyzetreemelés és átrendezés után nyerjük a nyilvánvalóan igaz
(a +b−c−d)2 ≥ 0 egyenlőtlenséget. Így igazoltuk a feladat állítását. Egyenlőség
akkor és csak akkor áll fenn, ha a + b − c − d = 0, azaz ha az ACBD négyszög
paralelogramma.
280 Megoldások, útmutatások

−−→ −→ −−→ −→ −−→ −→


1242. |P A1 | + |P A2 | + · · · + |P An | = |OA1 − OP | + |OA2 − OP | + · · · + |OAn − OP | =
−−→ −→ −−→ −−→ −→ −−→ −−→ −→ −−→
= |OA1 − OP | · |OA1 | + |OA2 − OP | · |OA2 | + · · · + |OAn − OP | · |OAn | ≥
−−→ −→ −−→ −−→ −→ −−→ −−→ −→ −−→ −−→
≥ (OA1 − OP ) · OA1 + (OA2 − OP ) · OA2 + · · · + (OAn − OP ) · OAn = OA1 2 +
−−→ −−→ −→ −−→ −−→ −−→
+ OA2 2 + · · · + OAn 2 − OP · (OA1 + OA2 + · · · + OAn ) = n, hiszen a feladat
−−→ −−→ −−→ −−→
feltételei szerint OA1 + OA2 + · · · + OAn = 0, és OAi 2 = 1.
−→ −→ −→ −→ −→ −→ −→ −→ −→ −→ −→
1243. a) Legyen OP = OA+ OB + OC. CP = OP − OC = OA+ OB ⊥ OA− OB =
−→ −→ −→
= BA, így (CP ⊥ BA miatt) P rajta van a C-ből induló magasságegyenesen.
Hasonlóan mutatható meg, hogy P rajta van a többi magasságegyenesen is,
ezért valóban P ≡ M.
−−→ −→ −→ −→
b) Az előbbi OM = OA + OB + OC összefüggés mindkét oldalát négyzetre
−→ −→ −→ −→ −→ −→ −→ −→
emelve: d 2 = 3r 2 + 2(OA · OB + OB · OC + OC · OA). Mivel AC = OC −
−→ −→ −→
− OA, így ennek mindkét oldalát négyzetre emelve: b2 = 2r 2 − 2OA · OC;
hasonlókat felírva a másik két oldalra megkapható a kívánt eredmény.
1244. Következik az 1243. b) feladatból.
1245. Következik az 1243. a) feladatból.
  
1 1 1 1
1249. s a = a+b, s b = b+a, s a ⊥ s b , emiatt s a ·s b = 0, azaz a+b b + a = 0,
2 2 2 2
4 a 2 + b2 4
melyből rövid számolással kapjuk a cos γ = · ≥ összefüggést.
5 2ab 5
4
1250. Az előző megoldást követve a cos γ = eredményt kapjuk.
5
1251. a) Tekintsük a háromszögbe írt kör középpontjából az érintési pontokba mutató
vektorok összegének négyzetét, amely nemnegatív: (x + y + z)2 = 32 −
− 22 (cos α+ cos β+ cos γ ) ≥ 0. Ezt rendezve a kívánt eredményhez jutunk.
( a háromszögbe írt kör sugara.)
b) Tekintsük a háromszög köré írt kör középpontjából a csúcsokba mutató vek-
torok összegének négyzetét, amely nemnegatív:
(x + y + z)2 = 3r 2 + 2r 2 (cos 2α+ cos 2β+ cos 2γ ) ≥ 0. Ezt rendezve a kívánt
eredményhez jutunk. (r a háromszög köré írt kör sugara.)
1253. Tekintsük a p(a; b), q(c; d), r(e; f ), s(g; h) vektorokat. A négy vektor között
van kettő, melyek által bezárt szög legfeljebb 90◦ , s ezek skaláris szorzata nem-
negatív. (A feladatban szereplő kifejezések e négy vektorból képezhető párok
skalárszorzatai.)
1256. Legyenek az A, B, C, P pontokba mutató helyvektorok rendre a, b, c, p. Fejez-
zük ki ezek segítségével a tükrözések után kapott pontokba mutató helyvektoro-
kat.
22. Vektorok a geometriában 281

1261. A hatszög szomszédos oldalait tekintsük mint egymás után fűzött vektorokat.
Ezek összege nullvektor. Továbbá e hat vektorból nullvektor annak a három vek-
tornak az összege is, melyek az adott háromszög oldalvektorai. Ezért a megma-
radó három vektor összege is nullvektor.
−→ −→ −→ −→ −→ −→
1268. AG = EF = LK = CB = DA = F H .
1270. Fejezzük ki az eredeti háromszög oldalvektoraival a keletkezett háromszögek
oldalvektorait.
R
1274. Legyen az x vektor 60◦ -os elforgatottja N
(az óramutató járásával ellentétesen) x  . Az P M A
ábrán látható vektorok szakaszfelező pon- a
−→ −−→ a 0

tokat kötnek össze, és DN = b + a, DM =


= a  + b . Mivel (a + b) = a  + b , így állí- b 0 b
tásunkat igazoltuk. B C
D

1278. Előbb mutassuk meg, hogy ha az ABCD B1


(konvex) négyszög AD, ill. BC oldalá-
−→ C1 B2
nak felezőpontja E, ill. F , akkor EF =
1 −→ −→ F
= (AB + DC). A feladat megoldásá-
2 C2
nál lássuk be azt, hogy EF ⊥ BC és c 0

b A c
2 · |EF | = |BC|. A fenti állítás szerint b 0

−→ 1  −→
EF = (b + c ), valamint igaz a BC = B E C
2
= b + c egyenlőség is. (Itt az x vektor
90◦ -os elforgatottja az óramutató járásával ellentétesen: x  .)
1279. Legyen az x vektor 90◦ -os elforgatottja F
az óramutató járásával megegyezően x  .
Nyilván igaz az (x + y) = x  + y  egyen- E
lőség. a 0

−→
a + b = 2s, EF = a  + b . ;b 0

Tehát |EF | = 2s és EF ⊥ s. C
a s b
A B
282 Megoldások, útmutatások

1281. A vektorok szakaszfelező pontokat kötnek B P C


össze, és x  az x vektor 60◦ -os elforgatott-
ját jelöli az óramutató járásával ellentéte- a b
0

−→ −→
sen. Látható, hogy P S  = P R, és ez állí- b a 0

tásunk igazolását jelenti. A D


c c 0

S R

F
E
1282. Legyen az x vektor 60◦ -os elforgatottja (az
óramutató járásával ellentétesen) x  . Ennek a a c0
műveletnek a tulajdonságai: (x + y) = x  + y  ,
a0
x  = −x, x + x  = x  . Most a + a  + b + b + c + c
+c = 0, azaz (a +b+c)+(a +b +c) = 0, innen
a + b + c = 0. Ezeket felhasználva belátható
a −(a  + b) = (b + c) összefüggés, melyből b
b0
következik a feladat állítása.

23. Területátalakítások

1284–1292. Néhány vonal berajzolása után páronként egyenlő területű vonalkázott és


jelöletlen részeket kapunk.
1284. A belső ponton át húzzunk párhuzamosakat az oldalakkal.
1285. Rajzoljuk meg a négyszög vonalkázott részén áthaladó átlóját.
1286. A középvonalak metszéspontját kössük össze a csúcsokkal.
1288. A vonalkázott háromszög csúcsaiból indítsunk a hatszög oldalaival pár-
huzamos egyeneseket.
1292. A belső ponton át húzzunk párhuzamosakat az oldalakkal.
1293–1300. Ha egyenlő területű idomokból egyenlő területű részeket veszünk el, egyen-
lő területű részeket kapunk.
23. Területátalakítások 283

1296. Az ábrán két egyenlő területű négyszöget találhatunk.


1300. Használjuk fel az 1284. feladatot.
1301. Két szemközti pöttyözött háromszög területének összege a négyszög területének
negyede.
1302. A vonalkázott téglalap átdarabolható a „nagy” téglalap területének felét kitevő
háromszögbe.
1303. A jelöletlen részek átdarabolhatók a vonalkázott körcikkbe.
1304. A területképletekből kapjuk, hogy a két félkör együttes területe megegyezik a
negyed körív területével.
1307. A két szemközti vonalkázott háromszög együtt a pöttyözött paralelogramma felét
teszi ki.
1308. Használjuk fel az 1285. feladatot.
1309. Hosszabbítsuk meg a „kis” négyzet oldalait!
1311. Hasonló az 1288. feladathoz.
1312. 1/2.
1313. 1/2.
1314. 7/9.
1315. 5/9.
1316. 2/9.
1317. 2/8.
1318. 1/5.
1319. 1/2.
1320. 5/12.
1321. 1/3.
1322. 1/3.
1323. 1/2.
1324. 1/3.
1325. 1/3.
1326. 5/24.
1327. 1/7.
284 Megoldások, útmutatások

1328. 7/60.
1329. a) 7,
b) 19,
c) 5,
d) 3.
1331. Nem. Használjuk az 1284. feladatot.
1332. Tükrözzük a félkört az átmérőre, s lássuk be, hogy a körbe írt téglalapok közül
a négyzet területe a legnagyobb. Ezt úgy bizonyíthatjuk be a legkönnyebben, ha
berajzoljuk a téglalap egyik átlóját.
1333. Használjuk az 1284. feladatot.
1 1 1
1334. P A2 + P B 2 + P C 2 + P D2 = (P A2 + P B 2 ) + (P B 2 + P C 2 ) + (P C 2 + P D2 ) +
2 2 2
1
+ (P D + P A ) ≥ P A · P B + P B · P C + P C · P D + P D · P A ≥ 2TAP B +
2 2
2
+ 2TBP C + 2TCP D + 2TDP A = 2T .
Az egyenlőtlenségek helyén csak akkor áll egyenlőség, ha P A = P B = P C =
P D és AP B ^ = BP C ^ = CP D^ = DP A^ = 90◦ . Ezekkel a tulajdonságokkal
a P pontból indítva a P A, P B, P C, P D szakaszokat, a kapott ABCD négyszög
négyzet lesz, a P pont pedig a négyzet középpontja.
1335. P az adott pont, F az AC oldal felezőpontja, C
e  f . P R egyenes felezi a háromszög terüle-
tét. F f
R
P
e B
A

1336. F a BD átló felezőpontja, P R AC. AR D


felezi a négyszög területét.

P F R
A C

B
24. Geometriai konstrukciók 285

24. Geometriai konstrukciók

1337. Igen (konkáv négyszög).


1338.

1339. 1340.

1341. a) b)
6 4
4 3
4
6 3
4
4
3
6

1342.

+ + =

1 2
1 2
vagy

+ + =
1 1 p
2
2
286 Megoldások, útmutatások

1343. 1344.

vagy vagy

vagy

1345. 1346.

1347. 1348.
24. Geometriai konstrukciók 287

1349. Legkevesebb 6 kockából állhat az építmény. Az el-


helyezést felülnézetből mutatja az ábra. Pl. a 2 szám 1
azt jelzi, hogy ott két kocka van egymáson. (Legfel-
jebb 20 kocka lehet.) 2
1350. Pl. a térbeli kereszt, azaz ragasszunk egy kocka min-
den lapjára egy vele egybevágó kockát! Egységkoc- 2
kákból egyéb testek is készíthetők, melyek teljesítik
a kívánt feltételt. 1

1351. 1352. Az ábrán a szakaszok 1 egység hosz-


szúak.

1353. Igen. Az ábrán a szakaszok 1354. Van.


1 egység hosszúak.

2 2

1355. Hamis. Ellenpélda a 3 egység átmérőjű körön a szabályos 12-szög.


1356. Egy lehetséges konstrukció: egy félkörívnél rövidebb íven vegyünk fel n pontot.
1357. Szabályos ötszög csúcsai és a köré írt kör középpontja.
1358.
288 Megoldások, útmutatások

1359.

40 40

1360. 1361. E és F oldalfelező pontok.

E F

1362.

1363.

30
30 1 2

1364.

1
1 2 4
24. Geometriai konstrukciók 289

1365. 1366.

1367. 1368.

1369. A feldarabolás során van olyan háromszög, melynek van csúcsa a négyzet bel-
sejében (különben lenne derékszögű háromszög), sőt van két háromszögcsúcs
is a négyzet belsejében. Mindegyik csúcs legalább öt háromszögnek csúcsa, ez
összesen 2 · 5 = 10 háromszög, legfeljebb két háromszöget számoltunk kétszer,
így a háromszögek száma legalább 8.

B B

A C A D C
8 9 10 11
1370.
4
9 3
1
5 7 5
9
4
1 6 2
8 2
3 6
7 8
290 Megoldások, útmutatások

1371.

1372. Néhány megoldás.

36◦ 36◦ 72◦ 72◦

36◦
36◦ 72◦ 72◦ 180◦ /7

360◦ /7 540◦ /7 540◦ /7

1373. Elvégezhető a felosztás; az ábrák mutatnak egy-egy megoldást.

1374.
24. Geometriai konstrukciók 291

1375.

C
B

1376.

1377.
A

B C

1378.

1379. Ötszög alapú gúla.


292 Megoldások, útmutatások

1380. Van.

1381. Írjunk 1/2 sugarú köröket az (n; n2 ) koordinátájú pontok köré, ahol n pozitív
egész szám.
1382. Nem lehet. Vegyünk egy olyan egyenest, mely egyik parabola-tengellyel sem
párhuzamos. A parabolatartományok ennek az egyenesnek véges hosszúságú ré-
szét fedik le.
1383. Vegyünk egy pontot, körülötte egy tetraédert, és vegyük fel a tetraéder oldallapjai
köré írt köröket. A pont és egy-egy kör meghatároz egy kúppalástot. Ezekbe a
kúpokba „ejtsünk bele” egy-egy gömböt; úgy választva a sugarukat, hogy ne
érintsék egymást.
1384. a) Nem lehet. P -ből a körök mindegyike 180◦ -nál kisebb szögben látszik. Így e
szögek összege kisebb 4 · 180◦ = 720◦ -nál. Míg, ha lenne megfelelő elrende-
zés, akkor e szögtartományok kétszeresen fednék le a síkot, tehát összegük
legalább 720◦ lenne.
b) Lehet. Vegyünk fel 5 db, P -ből induló félegyenest úgy, hogy bármely két
szomszédos félegyenes 72◦ -os szöget zárjon be egymással! Két-két ilyen
szomszédos, 72◦ -os szögtartományból álló 144◦ -os tartományba írjunk a szög-
szárakat érintő kört!
1386. a) Lehet.

b) Nem lehet. Ugyanis 1991 páratlan szám, ezért bárhogyan is veszünk fel egy
egyenest, annak valamelyik oldalára a sokszögnek több csúcsa esik, mint a
másikra.
25. Invariáns tulajdonságok 293

1387. a) Igen, lehet. Az első ábrán látható


alakzatból 496 db-ot (4·496 = 1984),
a másodikból egyet véve, ezeket
egymás után helyezzük, úgy hogy a
szélső érmék érintsék egymást, s ala-
kítsunk ki egy záródó kör alakú lán-
cot!
b) Nem lehet. Számoljuk meg az érintési pontokat! Minden érme 3 másikat
érint; ez összesen 1991·3 érintési pont, de minden pontot kétszer számoltunk,
tehát ennek a számnak párosnak kell lennie.

25. Invariáns tulajdonságok

1388. Nem lehet. A papírlapok száma 9-esével nő.


1389. Nem lehet. Egy törlés-felírás alkalmával a páratlan számok száma nem válto-
zik vagy kettővel csökken. Kezdetben öt páratlan szám volt, így az utolsóként
megmaradó szám páratlan lesz.
1390. A törlések során a 0 számjegyek száma nem változik, vagy kettővel csökken
aszerint, hogy különböző vagy egyenlő két jegy helyébe írunk. Így a századik
törlés előtt a még meglévő két számjegy páros sok (0 vagy 2) 0 jegyet ad, tehát
a két számjegy egyenlő, vagyis a századik törlés után az 1 számjegy marad a
táblán.
1391. Ha egy tárgyat 6 részre osztunk, akkor 1-ből 6 rész lesz, a részek száma tehát 5-
tel nő. Ha a tárgyat 11 részre daraboljuk, akkor a darabok számát 10-zel növeltük,
minthogy 1 helyett 11 darabbal rendelkezünk. Ezek szerint Béla „osztogatásai”
során minden alkalommal az 5 többszörösével (1-szeresével vagy 2-szeresével)
növelte a részek számát, végül tehát a részek számának egy 5-többszörösnél 1-
gyel nagyobb számnak kell lennie. Az 1993 nem ilyen, Béla tehát hibázott a
számlálás során.
1392. Minden törlés-felírás után a táblán levő számok összege 1-gyel csökken. 19
törlés-felírás után a táblán egy szám marad, mégpedig:
(1 + 2 + · · · + 20) − 19 = 191.
1393. Ha a táblán levő számok a1 , a2 , . . . , ak , akkor az S = (a1 +1)·(a2 +1)·. . .·(ak +1)
kifejezés értékét vizsgálva, az a műveletek során változatlan marad. Kezdetben
S = 21!, ugyanez S értéke akkor is, amikor egyetlen szám marad a táblán; ezért
ez a szám: 21! − 1.
294 Megoldások, útmutatások

1394. A fehér babszemek száma egy-egy kivételnél vagy nem változik, vagy kettővel
csökken. Kezdetben 75 fehér babszem volt, ezért az utolsónak megmaradó bab-
szem fehér lesz.
1396. Nem, hiszen a középső mezőben mindig olyan szám áll, amely a négy sarokban
levő szám összege.
1397. Nem lehet. Legyen a és b a két kupacban levő gyufák száma és S = a − b.
A változtatások során S értékének hárommal való osztásakor a maradéka nem
változik.
1398. Nem lehet. Legyen a, b és c a csúcsokban levő gyufák száma és S = a + b + c.
A változtatások során S értékének hárommal való osztásakor a maradéka nem
változik.
1399. Nem lehet. Legyen a, b, c és d a csúcsokban levő gyufák száma és S = a − b +
+ c − d. A változtatások során S értékének hárommal való osztásakor a maradéka
nem változik.
1400. Nem lehetséges. Figyeljük az asztalon levő kupacok számának és az asztalon
maradt kavicsok számának S összegét. Ez az S érték a lépések során nem vál-
tozik. Kezdetben S = 1001 + 1 = 1002. Ha a kívánt állapotban n kupac van,
akkor az elvárások miatt S = n + 3n = 4n, de nincs olyan n egész szám, amelyre
4n = 1002.
1401. Nem lehet. Legyen a, b és c a szürke, a barna, ill. a zöld kaméleonok száma
és S = a − b. A változtatások során S értékének hárommal való osztásakor a
maradéka nem változik.
1402. A megmaradó szám az 1-es. Egy-egy alkalommal megváltozik a 0-k, 1-esek,
2-esek számának paritása.
1403. Nem változik a felírt számok összegének 13-as maradéka.
1404. A táblán levő három szám négyzetösszege a lépések során nem változik, hiszen
   
a+b 2 a−b 2
a +b = √
2 2
+ √ . Mivel a megadott számhármasok négyzetösszege
2 2
nem egyenlő, ezért célunk nem érhető el.
1405. Nem kaphatjuk meg. Figyeljük a táblára felírt számok négyzetének összegét. Ez
az összeg minden lépésben nő.
1406. Nem lehet.
a) A csúcsokban levő számok összege mindig páratlan.
b) A négyzet egyik átlójának két végén levő szám összege S1 , a másik két szám
összege S2 . A változások során |S1 − S2 | értéke nem változik.
25. Invariáns tulajdonságok 295

1407. Nem lehet.


a) A csúcsokban levő számok összege mindig páratlan.
b) Osszuk két csoportba a kocka nyolc csúcsát úgy, hogy az ugyanabban a
csoportban levő négy csúcs között ne legyen kettő, mely ugyanannak az élnek
két végpontja (ez a felosztás egy-egy szabályos tetraéder csúcsait jelöli ki).
Az azonos csoportban levő számok összege legyen S1 , ill. S2 . A változások
során |S1 − S2 | értéke nem változik.
1408. Válasszunk ki 3, nem egymás mellett ülő személyt, s úgy figyeljük az előttük
levő tányérok számának változását.
1409. Hasonló az előzőhöz.
1410. Válasszunk ki 3 nem szomszédos személyt, és figyeljük a következő S összeg
változását. S értékét úgy kapjuk, hogy a 3 személy előtt levő tányérok összegéből
levonjuk a másik 3 személy előtt levő tányérok összegét. S értéke a megengedett
lépések során nem változik. Emiatt nem érhető el a kívánt állapot.
1411. Nem lehet. Legyen S a nők előtt levő poharak számának összege! Az átalakulá-
sok lépéseiben S vagy nem, vagy 2-vel változik. Mivel kezdetben S = 5, ezért
nem érhető el, hogy S = 0 legyen.
1412. Nem lehet. A fákat számozzuk meg valamilyen körüljárás szerint, rendre az 1,
2, 3, . . . , 44 számokkal. Az i-edik fához hozzárendeljük az Si = iai számot,
ahol ai az i-edik fán ülő majmok száma. Figyeljük az S összeg változását, ahol
S = S1 + S2 + · · · + S44 . Az S értéke egy-egy alkalommal vagy nem, vagy 44-gyel
változik az 1. és 44. fák közti átugráskor. Kezdetben S = 1 + 2 + · · · + 44 = 22 · 45,
az elérni kívánt állapotban S = 44 · k (ha a majmok a k-adik fán vannak), de ez
nem érhető el, mert 44  | (22 · 45 + 44n).
1413. Nem lehet. Lásd az előző két feladatot.
1414. A matematikusnak igaza van. Ugyanis megadható olyan, a klubokba sorolástól
függő, természetes szám értékű függvény, melynek értéke minden egyes javító
lépés során legalább 1-gyel csökken. Ezért az algoritmus véges sok lépés után
megszakad, ami azt jelenti, hogy létrejöttek a kívánt tulajdonságú klubok.
A = 7 × (a 11-esek klubján belüli ismeretségek száma) + 11× (a 7-esek klubján
belüli ismeretségek száma). Egyszerű számolás mutatja, hogy a függvény értéke
11-es → 7-es csere esetén legalább 7-tel, 7-es → 11-es csere esetén legalább
11-gyel csökken.
Ugyancsak megfelelő függvény: A = (a klubokon belüli ismeretségek összes
száma) + 4 × (a 7-esek klubjának létszáma).
1415. Nem lehetséges. Tekintsük ugyanis a gyomos parcellákat körbevevő kerület hosz-
szát. Ez a kerület a további gyomosodás során nem nő. (Miért?) A 9 mező kerü-
lete legfeljebb 9 · 4 = 36 egység, míg a 10 × 10-es tábla kerülete 40 egység.
296 Megoldások, útmutatások

Megjegyzés. Látható, hogy 10 gyomos parcella esetén már megtörténhet, hogy


az egész tábla elgazosodosik.
1417. Legyen a sáska, szöcske, tücsök jele: A, B és C, és a kezdeti sorrendjük ABC.
Az ABC, BCA, CAB, ACB, BAC, CBA sorrendek közül az első hármat ne-
vezzük szabályosnak, az utolsó hármat szabálytalannak. Az ugrálások során sza-
bályos és szabálytalan sorrendek váltakozva követik egymást. Emiatt 1999 (azaz
páratlan számú) ugrás után nem állhat vissza az eredeti sorrend.
1418. Igen, elérhető. Ha egy poharat páratlan sokszor forgatunk meg, akkor meg lesz
fordítva. Hatszor fordítunk meg öt-öt poharat, először az első, majd a második,
. . . , végül a hatodik pohár kivételével a többi poharat.
1419. Igen, elérhető (hasonló az előző feladathoz). Ha egy-egy alkalommal négy elem
előjelét változtatjuk meg, akkor a kívánt állapot nem érhető el, mert a változtatá-
sok száma páros (4 többszöröse), míg az elérni kívánt állapothoz páratlan számú
változtatást kell végezni.
1420. 1. megoldás: Helyettesítsük a + jelet +1-gyel, a − jelet −1-gyel. A számok
szorzata az átalakítások során nem változik.
2. megoldás: Helyettesítsük a + jelet 0-val, a − jelet 1-gyel! A számok összege
minden átalakítás után páratlan marad.
3. megoldás: Egy-egy lépésben a − jelek száma vagy nem változik, vagy 2-vel
csökken.
1421. Helyettesítsük a + jelet +1-gyel, a − jelet −1-gyel. Az * *
átalakítások során nem változik a megjelölt mezőkben álló
számok szorzata. * *
* *
* *

1422. Nem mindig. A megjelölt mezőkön álló számok összege


minden lépésben 3k-val változik. • • • • • • • •
• • • • • • • •
• • • • • • • •

• • • • • • • •
• • • • • • • •
• • • • • • • •

1423. a) Nem lehet. Állítsuk párba a szemközti csúcsokat. Egy-egy átalakítás min-
den egyes pár egyik elemének előjelét változtatja meg. Az eljárást páratlan
sokszor elvégezve, a párba állított csúcsokban az előjelek különbözők lesz-
nek, páros sokszor elvégezve pedig azonosak — ezek alól kivétel az (A1 , A7 )
páros. Ezért nem lehet, hogy egyidejűleg az (A2 , A8 ) párban különböző, az
(A3 , A9 ) párban azonos legyen az előjel.
25. Invariáns tulajdonságok 297

1424. Nem lehet. Vegyük fel a koordináta-rendszerben a négyzet három csúcsát: (0; 1),
(1; 1), (1; 0). Egy tükrözés alkalmával a tükrözött pont egy-egy koordinátája vagy
változatlan marad, vagy páros számmal változik. Mivel a megadott pontok mind-
egyikének van páratlan szám a koordinátái között, így nem kaphatunk olyan pon-
tot, melynek mindkét koordinátája páros.
1425. Nem lehetséges. Figyeljük meg, hogy az adott négy pont mindegyikénél, és a
keletkező új pontoknál is a pont két koordinátájának különbsége osztható 3-mal.
1426. Nem lehet. Könnyű belátni, hogy a sorozatban négy páratlan szám után egy
páros, majd négy páratlan, egy páros, . . . követi egymást ismétlődve.
1427. Legyen a sorozat hét egymást követő eleme x1 , x2 , x3 , x4 , x5 , x6 , x7 . Tekintsük az
S(a, b, c, d, e, f ) = 2a +4b+6c+8d +10e+12f függvényt. S(x1 , x2 , x3 , x4 , x5 , x6 )
és S(x2 , x3 , x4 , x5 , x6 , x7 ) utolsó jegye ugyanaz (lássuk be!). Emiatt a sorozatban
bármely k esetén az S(xk+1 , xk+2 , xk+3 , xk+4 , xk+5 , xk+6 ) utolsó számjegye mindig
ugyanaz.
S(1, 0, 1, 0, 1, 0) = 18, míg S(0, 1, 0, 1, 0, 1) = 24. Ezek utolsó számjegye kü-
lönböző, így a sorozatban nem szerepelhet a 0, 1, 0, 1, 0, 1 részsorozat.
1428. a) Nem lehet. A műveletek megőrzik az első számpár legnagyobb közös osztóját
közös osztóként.
b) Lehetséges.
1429. a) Lehetséges.
b) Nem lehet. Az átalakítások során a számpár két elemének különbsége mindig
osztható 7-tel.
1430–1433. feladatokban az inverziók számának párosságát vizsgáljuk. Az 1, 2, 3, 4,
5 számok 3, 4, 5, 2, 1 sorrendjében inverzióban áll pl. a 3 a 2-vel (mert ebben a
számpárban a nagyobb szám áll elöl), míg pl. a 3 a 4-gyel nincs inverzióban. A
számok felírt sorrendjében az inverziók száma 7.
1430. Egy-egy csere megváltoztatja az inverziók számának párosságát, így a 15.
csere után páratlan sok inverzió lesz, tehát nem juthatunk el az eredeti
sorrendhez.
1431. Az első kérdésre nem a válasz, mivel egy ilyen átrendezés nem változtatja
meg az inverziók párosságát, s a kiinduló helyzetben az inverziók száma
páros, míg az elérni kívánt állapotban páratlan.
1432. Nem lehet. Egy művelet az inverziók számát nem változtatja, ha vízszin-
tes irányban toltunk el egy lapocskát, s páratlan számmal változtatja, ha
függőlegesen toltuk el. Kezdetben az inverziók száma páros, az elérni kí-
vánt állapotban páratlan, s ez azért nem érhető el, mert a függőleges irá-
nyú mozgatások száma — ha az üres mezőt ismét az alsó sorban akarjuk
látni — páros.
1433. Hasonló az előzőhöz.
298 Megoldások, útmutatások

1434. Nem lehet. Adott irányt kijelölve haladjunk végig pozitív körüljárás szerint egy
sokszög kerületén! Utunk során kizárólag a megadott iránnyal párhuzamos ol-
dalak hosszát vegyük figyelembe, összegezzük pozitív előjellel, ha a megadott
iránnyal egyezően; negatív előjellel, ha azzal ellentétesen jártuk be a szóban for-
gó oldalt! A sokszögre így számolt érték invariáns a feldarabolással és az elto-
lással szemben. Háromszögnél három (valójában hat) irány is van, mikor ez az
érték nem nulla; míg téglalapnál minden irányra nullát kapunk.
1435. x = −1-re az egyik polinom 11-et, a másik −9-et vesz fel értékként. A megadott
változtatás a polinom x = −1 helyen felvett értékét 1-gyel változtatja. Ezért lesz
olyan polinom, melynek x = −1 gyöke.
1436. Az előírt művelet növeli a sokszög területét. Mivel csak véges sok különböző
sokszöget kaphatunk, így az eljárás néhány lépés után véget ér.
1437. Legyen S a táblázatban levő számok összege! Ha valamely sorban vagy oszlop-
ban a számok összege negatív, akkor ott változtassuk meg az előjeleket; ezzel
S nő. S-nek véges sok értéke lehetséges. Az eljárások befejeztével S nem lehet
negatív, hiszen (az előbbiek szerint) ekkor növelhető lenne.
1438. Legyen az S állapotjelző függvény értéke a szomszéd számpárok szorzatának
összege! Ebben az összegben az a, b, c, d sorrendre az ab + bc + cd szerepel,
míg a leírt cserét követően ac + cb + bd, mely (a − d)(b − c) < 0 miatt nagyobb a
korábbi részösszegnél (ab + cd < ac + bd). Tehát egy ilyen lépésben S értéke nő,
de S-nek csak véges sok értéke lehet (n db adott szám esetén), ezért a növekedés
egyszer véget ér.
1439. Vegyük észre, hogy egy-egy lépés nem változtatja meg a csúcsokban levő szá-
mok összegét; míg S értéke minden lépésben csökken, ahol S a másodszomszé-
dos csúcsokban levő számok különbségei négyzetének összege. Mivel S értéke
pozitív egész, így az eljárás néhány lépés után véget ér.
1440. Az n-edik lépésben kapott számnégyest jelölje (an , bn , cn , dn ).
Nyilván an + bn + cn + dn = 0, és így
an2 + bn2 + cn2 + dn2 ≥ 2(an−1
2 2
+ bn−1 2
+ cn−1 2
+ dn−1 ).
1441. Tekintsük a fekete mezőket körbefogó, rácsszakaszokból álló, legrövidebb zárt
töröttvonal S kerületét. Az átalakítás lépéseiben S értéke mindig csökken.
1443. Tekintsük az „igazi” barát-párok S számát, tehát azokét a párokét, melyekben a
két törpe egymás barátja, s ugyanolyan színű házikóban laknak. Az átfestések
során ez az S szám mindig nő.
1445. Helyezzük el tetszőlegesen a manókat és építsünk kerítést két parcella közé, ha
azok tulajdonosai utálják egymást. Megmutatjuk, hogy minden kerítés felszámol-
ható két manó parcellájának megcserélésével úgy, hogy közben nem keletkezik
új kerítés. Innen már nyilván következik, hogy néhány csere után az összes ke-
rítés megszüntethető.
25. Invariáns tulajdonságok 299

Legyen A és B két szomszédos manó, akik utálják egymást. Keresünk olyan C


manót, akivel B felcserélhető anélkül, hogy új kerítést kellene építeni, s közben
az A és B közötti kerítés lebontható. Azaz olyan, A-tól különböző C manót kell
keresni,
(1) aki B eddigi szomszédait nem utálja;
(2) akinek egyetlen szomszédját sem utálja B.
Az (1) feltétel legfeljebb 3 · 4 = 12 manót zár ki, hiszen B négy szomszédja
legfeljebb ennyi különböző manót utálhat. A (2) feltétel ismét 12 manót zár ki a
C-ként szóbajöhetők közül, ugyanis a B-t utálók szomszédjai legfeljebb ennyien
vannak. Vegyük észre azonban, hogy a B manót mindkétszer kizártuk. (Ugyan-
is B utálja B valamely szomszédját — nevezetesen A-t; továbbá B valamely
szomszédját — A-t — utálja B, ez utóbbi a (2) feltétel.)
Így ez a két feltétel legfeljebb 23 manót zár ki; köztük van B manó is, tehát
C = B teljesül. A C = A feltétel esetleg kizár még egy manót, de így is csak 24
manót zártunk ki; tehát van (1)-nek és (2)-nek eleget tevő C manó, s ekkor B
ezzel a C-vel parcellát cserélhet.
1446. Valahogyan szétosztjuk őket. Ha valamelyik csoportban valakinek két ellenfele
van, akkor őt áttesszük a másik csoportba. Ez a művelet fokozatosan csökkenti
az azonos csoportban levő ellenfélpárok S számát.
1447. Ültessük le őket tetszőlegesen. Ha ez még nem
megfelelő, akkor keressünk két szomszédot, A-
A B
t és B-t, akik ellenségek. Keressük meg A-nak
olyan C barátját, hogy C-nek jobb oldali D szom-
szédja B-nek barátja legyen (ez a feladat feltétele
miatt létezik)! A BC íven ülőket ültessük fordí-
tott sorrendben. Egy ilyen transzformáció során
csökken az ellenfél-szomszédpárok S száma.

C D
1448. Színezzünk 4 színnel tetszőlegesen. Ha az egyszínű pontokat összekötő szaka-
szok száma nagyobb n-nél, akkor van olyan pont, melyből legalább három, egy-
színű pontokat összekötő szakasz indul. Ezt a pontot színezzük át, s ekkor csök-
ken az egyszínű szakaszok száma.
1449. A piros és a kék pontokat kössük össze szakaszokkal. Ha valamely két szakasz
metszi egymást, akkor a metszéspont megszüntethető, ha a között a négy pont
között a két szakaszt másképp vesszük fel. Kínálkozik, hogy itt az állapotjelző
függvény a metszéspontok számát jelentse, de az előbbi lépés nem biztos, hogy
300 Megoldások, útmutatások

csökkenti annak értékét. Figyeljük inkább a szakaszok összhosszának változását.


Ez az átalakítások során mindig csökken, azonban a csökkenésnek egyszer véget
kell érnie, s ekkor nem lehet, hogy két szakasz metssze egymást.
1450. Hasonló az előzőhöz.
1451. Írjunk a táblázat mezőibe betűket, majd ezekkel fejezzük ki az átalakítások során
kapott táblázatokat!
1452. Nem. Ha az utolsó átalakítás után csupa 0-t kaptunk, akkor előzőleg mindegyik
szám egyforma volt, mindegyik 1-es. Ez azt jelenti, hogy ezt megelőzően a szá-
mok 0, 1, 0, 1, . . . , ami nem lehet, mert 9 páratlan.
1453. Hasonló, mint az 1451. feladat.
1454. Ha a sorozat elemeinél páros szám helyett +1-et, páratlan helyett −1-et írunk,
akkor az átalakítások megegyeznek az előző feladat átalakításaival. Ez azt jelen-
ti, hogy néhány lépés után mindegyik szám páros lesz. Figyeljük meg, hogy a
számok közt a legnagyobb nem nőhet.

26. Feladatok a sakktáblán

1455. A huszár fekete mezőről fehérre, fehérről feketére lép. Ha a bal alsó sarok fekete,
akkor innen indulva a 63. lépésben nem léphet a jobb felső fekete mezőre, hiszen
ekkor fehér színű mezőre lép.
1456–1459. Hasonló az előzőhöz.
1460. 1 20 5 14 9
6 15 10 19 4
11 2 17 8 13
16 7 12 3 18
1461. Tekintsük a négysoros táblát, két belső és két szélső sorral. Egy szélső mező-
re a huszár csak belső mezőről léphet, szélső mezőről pedig belsőre lép. Mivel
ugyanannyi szélső mező van, mint belső, ezért a huszár minden második lépé-
sében szélső mezőre lép, s mivel a huszár felváltva lép fehérről feketére, így a
szélső mezők mindegyikének fehérnek (vagy feketének) kellene lennie.
1462. Hasonló az előzőhöz: a 4 szélső sorból a huszár a három középső valamelyikére
léphet.
1463. Nem. Legyen a jobbra, a felfelé, a balra lefelé történő lépések száma rendre x,
y, z. A feltétel szerint: x + y + z = n2 − 1, y = z, x = z + 1. Ezekből 3z + 2 = n2 ,
de négyzetszám 3-mal osztva nem adhat 2 maradékot.
26. Feladatok a sakktáblán 301

1464. a) Nem lehet. Fekete (fehér) mezőn álló bábu ismét fekete (fehér) mezőre lép.
b) Nem lehet. Színezzük a tábla sorait felváltva fehérre, feketére! Ekkor a 9
figurából pl. 6 áll fekete, 3 fehér mezőn. Ez a lépések során nem változik,
pedig a célul kitűzött állapotban 6 figurának fehéren, 3-nak feketén kell állnia.
Ez egyben újabb indoklása az a) feladat megoldásának.
1465. 32 huszár elhelyezhető, pl. ha mindegyiket fehér mezőre tesszük. 1 2
Többet nem lehet elhelyezni, hiszen a tábla ábra szerinti 4 × 2-es
részére legfeljebb 4 huszár helyezhető (ugyanis az azonos számmal 3 4
jelölt mezőkből legfeljebb az egyiken állhat huszár), így az egész 2 1
táblán legfeljebb 8 × 4 huszár állhat.
4 3

1466. Az egyszínű mezők száma 12 és 13, ezért 13 huszár elhelyezhető a kívánt módon.
Többet nem lehet feltenni, mert — az ábrákon látható bejárási sorszámozás miatt
— nem állhatnak huszárok szomszédos sorszámú mezőn.
1 20 9 14 3 1 10 15 22 3
10 15 2 19 24 16 21 2 9 14
21 8 25 4 13 11 8 17 4 23
16 11 6 23 18 20 25 6 13 18
7 22 17 12 5 7 12 19 24 5

1467. • •
• •• •
••• •• ••• •• •••••
• ••
•••••
• •

• ••
•• •• •• •• •• ••
• • • ••

•• •• • •
• •• •• ••
•• ••
• ••
1468. Osszuk a táblát 2 × 2-es részekre. Egy 2 × 2-es részben legfeljebb egy király
állhat, így a táblán legfeljebb 16 király lehet. Ennyit el is lehet helyezni.
302 Megoldások, útmutatások

1469. • 1470.
• •

• •
• •
• •

• •
1471. A bekeretezett területeken kell állnia királynak; különben lenne olyan mező, mely
nem áll ütés alatt. Tehát legalább 9 király szükséges; s az ábra mutatja, hogy
ennyivel a cél el is érhető.
• • •
• • •
• • •

1472. 10-et el lehet helyezni (az ábrák mutatják). Többet nem, még bástyát sem. Ha
11 bástya van, akkor kell lennie 3 sornak, melyben 2–2 bástya áll. Ezeknek
különböző oszlopokban kell elhelyezkedniük, azonban a maradék két oszlopban
csak 2 × 2 bástya állhat.

• ••
• •
• • •

• • •
• •
• ••
• ••
• •
•• •
• •
• • •
• •
• •
•• •
• ••
26. Feladatok a sakktáblán 303

1473.
• • • • • •
• • • •
• • • • • •

• • • • • •
• • • •
• • • • • •

1474–1475. Figyeljük meg a „közlekedési szabályokat”.

• • 1 4 7 8
1
2
6 2
7 3
◦ ◦ 3 8 5
6 4
5

...
1476. Mindig ugyanazt a számot kapjuk az ösz- 0+1 0+2 0+3 0+8
szegzéskor. Ugyanis a számokat az ábra ...
szerint felbontva két szám összegére, lát- 8+1 8+2 8+3 8+8
...
ható, hogy a nyolc szám összege: 0 + 8 + 16+1 16+2 16+3 16+8
+ 16 + · · · + 56 + 1 + 2 + 3 + · · · + 8. .. .. .. .. .. .. . .. ..
. . . . . . .
...
56+1 56+2 56+3 56+8
...
1477. Az előző feladat eredményét alkalmazzuk. Számozzuk meg a táblázat mezőit,
soronként kitöltve az 1, 2, 3, . . . , 8, 10, 11, 12, . . . , 17, 19, 20, . . . számokkal
(vigyázat!, pl. a második sort nem 9-cel kezdtük, hanem 10-zel). Így értük el,
hogy fekete mezőkön mondjuk páratlan számok, a fehéreken páros számok áll-
nak. Ha elhelyeztük a nyolc bástyát, adjuk össze az e mezőkön álló számokat, a
végeredmény mindig ugyanaz a páros szám, tehát az összeadandók között párat-
lan számból páros sok van, ezért a fekete mezőkön álló bástyák száma páros.
1478. Figyeljük a tábla színezését; s gondoljunk arra, hogy a fehér és fekete mezők
száma eltérő; s a bogarak fehér mezőről feketére, feketéről fehérre lépnek.
1479. Nem. Ha a táblázatban levő számokat soronként adjuk össze, akkor pozitív az
összeg; ha oszloponként, akkor ugyanez az összeg negatív lenne.
304 Megoldások, útmutatások

1480. Lehet.
1 1 1 1 1 3 3 3 3 3
1 −4 1 −4 1 −4 −4 −4 −4 −4
1 1 1 1 1 3 3 3 3 3
1 −4 1 −4 1 −4 −4 −4 −4 −4
1 1 1 1 1 3 3 3 3 3

1481. Nem. Lásd az 1479. feladatot.


1482. Válasszuk ki valamelyik számot, pl. az 1-et; s úgy állítsuk párba azokat a me-
zőket, melyen az 1 áll, hogy a párban álló mezők egymás tükörképei legyenek,
ha a főátlóra tükrözünk. Mivel öt ilyen mező van, valamelyik mezőnek önmaga
a képe; ezért a főátlóban ott áll az 1-es szám. A hasonló állítás általában is igaz
n × n-es táblázatra, ha az n páratlan szám, s ha n páros, akkor nem igaz.
1483. A táblázat bármelyik mezőjéről bármelyik másikra eljuthatunk legfeljebb 2n − 2
lépésben úgy, hogy minden lépésben egy mezőről valamelyik szomszédjára lé-
pünk. Mivel a szomszédos mezőkbe írt számok különbsége legfeljebb 1, így
a táblázatba írt számok maximumának és minimumának különbsége legfeljebb
2n − 2. Ez azt jelenti, hogy a táblázatban legfeljebb 2n − 1 különböző szám for-
dul elő.
Ebből következik,
 hogy a táblázatba írt számok között van
 olyan,  amelyik lega-
n2 n2 n2 n n2 n
lább helyen szerepel. > = , így ≥ .
2n − 1 2n − 1  2n 2 2n − 1 2
n
Van tehát olyan szám, amelyből legalább darab van a táblázatban.
2
Megjegyzés. Több is igaz. Van olyan szám, amelyik minden sorban szerepel;
vagy pedig olyan szám, amelyik minden oszlopban előfordul. Van tehát olyan
szám, amelyik legalább n-szer szerepel.
1484. Tekintsük a sakktábla egy tetszőleges számozását; és válasszuk ki azt a két me-
zőt, amelyen az 1, ill. a 64 áll. Az 1-est tartalmazó mezőről oldalszomszédos
mezőkön lépkedve legfeljebb 14 lépésben eljutunk a 64-es számhoz. Ha egy-egy
lépésben legfeljebb 4-gyel nagyobb számra lépünk, akkor a 14. lépésben legfel-
jebb az 57 = 1 + 14 · 4 számra léphetünk. Emiatt legalább az egyik lépésben
(legalább) 5-tel nagyobb számra léptünk.
1485. Tekintsük a sakktábla egy tetszőleges számozását, és helyezzünk el egy királyt
— amelyik egy lépésben szomszédos mezőkre léphet — azon a mezőn, amelyik
az 1-es számot kapta. Bárhol áll is a király, legfeljebb 7 lépésben a sakktábla
akármelyik mezejére, így a 64-es számúra is eljuthat. Az útja során megtett tá-
volságok összege legalább 63, így a király legalább egy alkalommal „9-et lépett”,
hisz 7 · 8 csak 56. Ez azt jelenti, hogy bárhogyan is számozzuk meg a sakktáb-
lát, van két olyan szomszédos mező, melyek távolsága legalább 9, így a keresett
szám legalább 8.
26. Feladatok a sakktáblán 305

Azt viszont könnyű belátni, hogy a keresett szám éppen a 8. Ha ugyanis „fo-
lyamatosan” számozzuk meg a sakktáblát, azaz az n-edik sor k-adik mezőjére az
(n−1)8+k számot írjuk, akkor a szomszédos mezők távolsága 1, 8, 7 vagy pedig
9 lehet. Van tehát olyan számozás, amikor nem lép fel 9-nél nagyobb távolság a
szomszédos mezők között.
1486. Azt bizonyítjuk, hogy létezik olyan (a1 , b1 ), . . . , (an , bn ) diszjunkt, szomszé-
dos mátrixelemekből álló pár, hogy max aj < min bj . Ekkor azon t indexre,
1≤j ≤n 1≤j ≤n
melyre bj maximális, fennáll
bt = max bj ≥ n − 1 + min bj ≥ n + max aj ≥ n + at ,
1≤j ≤n 1≤j ≤n 1≤j ≤n
és ezért (at , bt ) a kívánt tulajdonságú szomszédos elempár.
Jelölje α a mátrix sormaximumainak minimumát. Tegyük fel, hogy ez a ν-edik
sorban vétetik fel.
1. eset. Minden sorban van α-nál kisebb elem. Ekkor α definíciója szerint minden
sorban van α-nál nem kisebb elem is, így minden sorból kiválasztható olyan
szomszédos elempár, melynek egyik tagja α-nál kisebb, a másik nem kisebb. Az
így kiválasztott n pár megfelel a követelményeknek.
2. eset. Van olyan sor, mondjuk a μ-edik, hogy minden eleme legalább α; α
definíciója miatt μ = ν és ezért a mátrixelemek különböző volta miatt a μ-
edik sor minden eleme nagyobb, mint α. Ekkor tehát minden oszlopban van
α-nál nagyobb és α-nál nem nagyobb elem egyaránt. Innen az állítás ugyanúgy
bizonyítható, mint az 1. esetben.
1487. a) Az ábra mutatja a kitöltést.
0 0 0 0 8 8 8 9 9 9
b) Jelölje qk és rk azoknak az oszlopoknak, ill.
0 0 0 1 1 1 1 9 9 9
soroknak a számát, amelyben előfordul a k szám-
jegy. 0 0 0 1 1 1 2 2 2 2
3 3 3 1 1 1 2 2 2 3
Minden oszlop fölé odaírjuk, hogy mely szám- 3 3 4 4 4 4 2 2 2 3
jegyekből található legalább egy darab az
3 3 4 4 4 5 5 5 5 3
adott oszlopban. Így az oszlopok fölé nyilván
6 6 4 4 4 5 5 5 6 6
q0 + q1 + · · · + q9 darab számot írtunk. Ugyan-
6 7 7 7 7 5 5 5 6 6
akkor, ha feltételezzük, hogy minden oszlop-
6 7 7 7 8 8 8 8 6 6
ban legfeljebb három különböző szám van, ak-
9 7 7 7 8 8 8 9 9 9
kor az összes oszlop fölé legfeljebb 30 darab
szám kerülhet. Így q0 + q1 + · · · + q9 ≤ 30. A sorokra hasonlóan kapjuk, hogy
r0 + r1 + · · · + r9 ≤ 30.
Mivel minden számjegy tízszer fordul elő, így qk rk ≥ 10. Ezért qk + rk ≥

≥ 2 qk rk > 6, azaz (q0 + r0 ) + (q1 + r1 ) + · · · + (q9 + r9 ) > 60, ami ellentmond az
előző két egyenlőtlenségnek.
Az ellentmondás azt igazolja, hogy van legalább négy különböző számot tartal-
mazó sor vagy oszlop.
306 Megoldások, útmutatások

1488. k ≥ 3, hiszen az 1-est tartalmazó mezőnek négy másik mezővel van közös oldala,
ugyanakkor a papírlapon csak 2 darab 2-es szerepel. Így az egyik szomszédos
mezőbe kettőnél nagyobb szám kerül, tehát különbségük legalább 2. A k = 3
értékhez tartozó kitöltés többféle módon is megadható.
1489. Az 1, 2, . . . , 120 számok mindegyikét ötször írhatjuk be, tehát e számok össze-
sen 600 mezőt fednek le. Másrészt minden olyan mezőt e számokkal kell lefed-
nünk, amelyekre soruk és oszlopuk sorszámát összeszorozva 120-nál nem na-
120
gyobb számot kapunk. Az első sorban 120 ilyen mező van, a másodikban =
  2
120 120
= 60, a harmadikban , . . . , az i-edik sorban olyan mező van, amely
3 i
megfelel. Ezeket összeadva 602 mezőt számolunk meg.
Ezek szerint több mezőt kell 120-nál nem nagyobb számokkal lefednünk, mint
ahány szám rendelkezésünkre áll; így a táblázat nem tölthető ki a feladat kívá-
nalmainak megfelelően.
1491. Számozzuk meg a végtelen négy- 1 2 1 2 1 2 1 2 1 2 1 2 1 2
zetrács mezőit az ábra szerint az 3 4 3 4 3 4 3 4 3 4 3 4 3 4
1, 2, 3, 4 számokkal! A 100 me- 1 2 1 2 1 2 1 2 1 2 1 2 1 2
zőből legalább 25 mezőn ugyan-
3 4 3 4 3 4 3 4 3 4 3 4 3 4
az a szám áll, s ezeknek a me-
1 2 1 2 1 2 1 2 1 2 1 2 1 2
zőknek megvan a kívánt tulaj-
3 4 3 4 3 4 3 4 3 4 3 4 3 4
donságuk.
1 2 1 2 1 2 1 2 1 2 1 2 1 2

1492. A 29 × 29-es négyzetrács mezőit jelöljük meg az ábra szerint pontokkal! Ösz-
szesen 100 db 2 × 2-es részt jelöltünk meg. Vegyük észre, hogy ha bármiképpen
vágtunk ki egy 2 × 2-es részt, az csak egy, a pontokkal megjelölt 2 × 2-es részt
metsz.
...
•• •• •• ... •• ••
•• •• •• ... •• ••
...
•• •• •• ... •• ••
•• •• •• ... •• ••
.. .. .. .. .. .. .. .. .. .. .. . .. .. .. .. .. ..
. . . . . . . . . . . . . . . .
...
...
•• •• •• ... •• ••
•• •• •• ... •• ••
26. Feladatok a sakktáblán 307

1493. Általánosabban vizsgáljuk a feladatot, a tábla mérete legyen m × m. Helyezzünk


a kiválasztott mezőkbe csillagot. Ha az oszlopokban levő csillagok száma rendre
m    
xi m
x1 , x2 , . . . xm , akkor ≤ .
i=1
2 2
Ennek oka: ha egy oszlopban kiválasztunk két csillagot, akkor másik oszlop-
ban ugyanannak a két pozíciónak mindegyikében nem lehet csillag (mert ekkor
felbukkanna a tiltott téglalap); tehát oszloponként a csillagpárok más-más pozíci-
ókat (sor-párokat) jelölnek ki, így ezeknek
  a pároknak a száma legfeljebb annyi,
m
mint az összes sor-pár száma, azaz .
2
  xi     
m m
m m
xi (xi − 1) m(m − 1)
Legyen xi = k. ≤ , azaz ≤ . Innen:
i=1 i=1
2 2 i=1
2 2
m  m
xi2 ≤ m(m − 1) + xi = m(m − 1) + k.
i=1 i=1

m
Mivel xi2 ≥ (x1 + x2 + · · · + xm )2 /m = k 2 /m, így k 2 /m ≤ m(m − 1) + k,
i=1 √
m + m 4m − 3
ahonnan k ≤ . Ez m = 7 esetén a k ≤ 21 egyenlőtlenséget
2
jelenti. A mellékelt ábra mutatja, hogy 21 mező kijelölhető a kívánt módon.


1494. Játsszuk a játékot fekete-fehér sakktáblán. Látható, hogy


9 figura fekete, 6 pedig fehér mezőn áll, s ez is marad
minden lépés után. Ezért a kívánt állás nem valósítható
meg.

1495. Jelöljük a három színt az 1-es, 2-es, 3-as számokkal. Színezzük a tábla mezőit az
ábra szerint 3 színnel úgy, hogy az egyik átlós irányban (a vízszintessel 45◦ -os
szöget bezáró irányban) azonos színűek legyenek a mezők. Mindegyik színnel
11–11 mezőt színeztünk ki. Kezdetben az azonos színű mezőkön álló bábuk
száma: a = 11 (az 1-es színű mezőkön álló bábuk száma), b = 11 (a 2-es színű
mezőkön álló bábuk száma), c = 10 (a 3-as színű mezőkön álló bábuk száma).
308 Megoldások, útmutatások

Hogyan változnak ezek a számok egy lépés során? Ahonnan ugrunk és amelyik
mezőt átugorjuk, e két szín esetén eggyel–eggyel csökken az ilyen színű mezőn
álló bábuk száma; míg ahová érkezünk, az olyan színű mezőkből eggyel több
mezőn lesz bábu. Tehát a, b, c mindegyike 1-gyel változik, ezért c párossága
mindig különbözni fog a és b párosságától. Ha csak egy bábu marad, az úgy
lehet, hogy a = 0, b = 0, c = 1. Azaz a megmaradó bábu csak olyan mezőn
állhat, amelynek a színe ugyanolyan, mint a középső mező színe. Ez 11 mezőt
jelent.
Most színezzük újra a játéktáblát, hasonlóan az elő-
zőhöz, csak a másik átlós irányban. Ismét azt kapjuk, 1 3 2
hogy az egyedül maradó bábu 11 mező valamelyi- 2 1 3
kén állhat. Azonban csak 5 olyan mező van, amely 2 1 3 2 1 3 2
az előző 11 mező között is szerepelt; csak ezeken áll- 3 2 1 3 2 1 3
hat a megmaradó bábu (vagy a tábla közepén, vagy 1 3 2 1 3 2 1
a négy oldal valamelyikének középső mezőjén). 3 2 1
1 3 2
1496. A hiányos sakktáblán nem egyenlő a fekete
és a fehér mezők száma, s a dominók ugyan-
annyi fehér és fekete mezőt fednek le. Ha egy
fekete és egy fehér mezőt „tiltunk le”, mint pl.
az ábrán látható, akkor a megrajzolt „útvonal”
mutatja a lefedés módját.

1497. A sakktáblát az ábra szerint számozva, bárhogyan 1 2 3 1 2 3 1 2


is helyezünk a táblára 1 × 3-as dominót, az mind 3 1 2 3 1 2 3 1
a három számból lefed egyet-egyet. Ha a táblát le- 2 3 1 2 3 1 2 3
fedtük volna, akkor ugyanannyi lenne a táblán az 1 2 3 1 2 3 1 2
1, 2, 3 számok mindegyikéből (mindegyikből 21), 3 1 2 3 1 2 3 1
de pl. az ábrán az 1-esből 22 db van. Tehát a tábla 2 3 1 2 3 1 2 3
nem fedhető le. 1 2 3 1 2 3 1 2
1 2 3 1 2 3 1
26. Feladatok a sakktáblán 309

Tekintsük a 8 × 8-as sakktáblának a következő ábrák szerinti két számozását.

1 2 3 1 2 3 1 2 1 2 3 1 2 3 1 2
3 1 2 3 1 2 3 1 2 3 1 2 3 1 2 3
2 3 1 2 3 1 2 3 3 1 2 3 1 2 3 1
1 2 3 1 2 3 1 2 1 2 3 1 2 3 1 2
3 1 2 3 1 2 3 1 2 3 1 2 3 1 2 3
2 3 1 2 3 1 2 3 3 1 2 3 1 2 3 1
1 2 3 1 2 3 1 2 1 2 3 1 2 3 1 2
3 1 2 3 1 2 3 1 2 3 1 2 3 1 2 3

Ezekre is bárhogyan helyezünk egy 1 × 3-as domi-


nót, az mind a három számból lefed egyet-egyet.
Az első táblán az 1-ből, a másodikon a 2-ből van
22 db. Tehát, ha a táblára feltettünk 21 db 1 × 3-
as dominót, akkor a lefedetlen mező az egyik táb-
lán az 1-es, a másikon a 2-es számot viseli. Négy
ilyen mező van; ezek bármelyikének „letiltása” ese-
tén már lefedhető a tábla.

1498. Nem lehet. Hasonló az előzőhöz, csak most a mezőket az 1, 2, 3, 4 számokkal


számozzuk.
1499. Nem lehet. Színezzük a sakktáblát a szokásos módon. Az adott síkidom 1 vagy
3 fekete mezőt fed le; a 10 × 10-es tábla lefedéséhez szükséges 25 db dominó
páratlan számú fekete mezőt fed le, tehát nem fedheti le az 50 fekete mezőt.
1500. Nem lehet. A megadott dominók páratlan számú fekete mezőt fednek le.
1501. Nem lehet. Színezzük a sakktábla sorait felváltva fehérre, feketére. A dominók
páratlan számú fekete mezőt fednek le.
1502. Nem lehet. Tekintsük a táblának az ábra sze-
rinti színezését, s vizsgáljuk a dominók által
fedett fekete mezők számának párosságát.
310 Megoldások, útmutatások

1504. Nem lehet. Tekintsük a téglalapnak az ábra sze-


rinti színezését. Mivel 1 × 4-es dominókkal le-
fedhető, ezért a fekete mezők száma páros. Azon-
ban, ha 1 db 2 × 2-es dominót elhelyeztünk, ak-
kor páratlan számú fekete mező marad, s a tégla-
lap többi része csupán 1 × 4-es dominókkal már
nem fedhető le. (A feladatban levő egyik feltétel-
re nem volt szükség. Melyikre?)

1505. Nem lehet. A keresett téglalapot sakktáblaszerűen színezve 10 fehér és 10 fekete


mező lesz a táblán. A megadott alakzatokkal nem lehet lefedni 10 fekete mezőt.
1506. Színezzük a tábla sorait felváltva fehérre, feketére. Ekkor 5 sor fehér (= 70 fehér
mező), 4 sor fekete (= 56 fekete mező). A 11 db 2×3-as dominó 33 fekete mezőt
fed, a 10 db 3 × 2-es dominó pedig páros számú fekete mezőt. Tehát a dominók
páratlan számú fekete mezőt fednek, miközben a fekete mezők száma 56, ami
páros szám.
1507. A 101 × 101-es tábla mindegyik sorába írjuk be az 1, 2, 1, 2, 1, . . . , 1, 2,
1 számokat ebben a sorrendben. A táblára egy 2 × 2-es dominót helyezve, a
dominó alatt levő számok összege 6, egy 3 × 3-as dominó alatt pedig 12 vagy 15
a számok összege — azaz a dominók alatt levő számok összege osztható 3-mal.
Azonban a 101 × 101-es táblán levő számok összege nem osztható 3-mal, ezért
a tábla nem fedhető le a kívánt módon.
1508. Nem lehet. Ugyanis a 39 = 5a + 11b egyenletnek nincs megoldása a nemnegatív
egészek körében.
1509. Nem lehet. Tekintsük a táblának az ábra sze-
rinti színezését. Egy-egy dominó legfeljebb egy
fekete mezőt fed le, s mert 20 fekete mező van,
így legalább 20 dominó szükséges a tábla lefe-
déséhez.

1510. Legfeljebb 14 dominó helyezhető el a táblán. Osszuk fel a táblát 2 vízszintes és 3


függőleges egyenessel részekre úgy, hogy a szomszédos párhuzamos egyenesek
távolsága 3 egység legyen. Ezután színezzük ki a részeket sakktábla-szerűen
26. Feladatok a sakktáblán 311

feketére, fehérre. Így 48 fekete és 42 fehér mezőt kapunk (vagy fordítva). Minden
42
dominó három fehér és három fekete mezőt fed le. Ezért legfeljebb = 14
3
dominót tehetünk a táblára. (14 dominót el is tudunk helyezni a táblán.)
1511. A feladat „csak akkor” részét szükséges bizonyítani, a másik irány ugyanis nyil-
vánvaló. Töltsük ki a táblázat mezőit a 0, 1, 2, . . . , k − 1 számokkal úgy, hogy a
táblán bárhogyan helyezünk el egy (k × 1)-es dominót, az csupa különböző szá-
mot fedjen le („átlós színezés”). Ennek vizsgálatával is eredményre juthatunk.
Hasonló ötlettel, komplex számok segítségével gyorsabban érünk célba. Legyen
ε = cos(2π/k) + i sin(2π/k) az első k-adik egységgyök, és tegyük fel, hogy az
n × m-es táblát lefedtük (k × 1)-es dominókkal. Ha a tábla mezőit oly módon
töltjük ki komplex számokkal, hogy az r. sor s. mezőjére az εr+s−2 egységgyököt
írjuk, akkor minden (k×1)-es dominó a k. egységgyököknek egy teljes rendszerét
fedi le, amelyeknek 0 az összege. Így a táblán levő összes egységgyök is 0-t ad
összegül, azaz
    
n m
1 − εn 1 − εm
0= εr+s−2 = εr−1 · εs−1 = · .
1≤r≤n r=1 s=1
1−ε 1−ε
1≤s≤m

Az egyenlőség két oldalát összehasonlítva kapjuk, hogy εn = 1 vagy εm = 1,


azaz k | n vagy k | m.
1512. Helyezzük a téglalapot olyan sakktáblára, melyben az alapnégyzetek oldalhosszai
1/2 hosszúságúak. Könnyű belátni, hogy a felosztásban szereplő kis téglalapok
mindegyike a sakktáblából ugyanakkora területű fehér részt fed le, mint fekete
részt. (Ez abból következik, hogy mindegyik kis téglalapnak valamelyik oldala
egész.) Tehát a nagy téglalap is ugyanakkora területű fehér részt fed le, mint
feketét. Így a feladat állítását bebizonyítjuk, ha megmutatjuk, hogy ez csak abban
az esetben lehetséges, amikor a nagy téglalap valamelyik oldala egész.
Tegyük fel, hogy ez nem így van. Vágjuk ki a téglalap jobb alsó sarkából azt a
téglalapot, melynek oldalai olyan hosszúak, amennyivel a két oldal hosszúsága
meghaladja a hozzá legközelebb eső egészt. A megmaradt konkáv hatszögben
ugyanakkora területű fehér, mint fekete rész van (miért?), ezért ez a kivágott
részben is így van. Azonban ha ezt megvizsgáljuk, meglátjuk ennek lehetetlen-
ségét.
1513. a) Ha egy rácsegyenest keresztez egy dominó, akkor keresztez még legalább
egy másik dominó is, hiszen az egyenes bármelyik oldalán páratlan számú
fedetlen mező maradt. Ha mind a 10 rácsegyenest keresztezi dominó, akkor
ez legalább 20 db dominót jelent, azonban a tábla lefedéséhez 18 db dominó
szükséges.
b) Hasonló az a)-hoz.
1514. Nem lehet, ugyanis egy-egy átfestés nem változtatja meg a fekete mezők számá-
nak párosságát.
312 Megoldások, útmutatások

1515. Igen, lehet. Bármely színezés megvalósítható; ugyanis néhány átfestéssel elér-
hető, hogy csak egy tetszőlegesen választott mező színe változzék meg. Ehhez
színezzünk át minden olyan sor–oszlop párt, mely a kiválasztott mezőt tartal-
mazza.
1518. Nem, hiszen bármely 7 × 11 × 1-es rétegben a kis téglák 3n helyet foglalnak el,
azonban 3  | 7 · 11.
1519. Nem. A kocka 2 × 2 × 2-es részeit felváltva színezzük sakktáblaszerűen fehérre,
feketére! Egy 1 × 2 × 4-es téglát bárhogyan is helyezünk el, annak fele fekete,
fele fehér lesz. Tehát, ha a kirakás megvalósítható, akkor a 6 × 6 × 6-os kocka
fele fehér, fele fekete — s ez nem teljesül.
1520. Azt lássuk be, hogy a láda három, a, b, c élhossza közül legalább kettő páros, és
ezek egyike osztható 4-gyel! Ha pl. a és b is páratlan lenne, akkor a láda nem
lenne kitölthető a megadott téglákkal, hiszen bármely a × b × 1-es rétegből a
téglák páros számú egységkockányi helyet foglalnak el. Az előző feladat megol-
dását követve belátható: az nem lehetséges, hogy a, b, c mindegyike páros, ám
egyik sem osztható 4-gyel. A láda térfogata osztható 16-tal. Ezekből következik
a várt állítás.
1521. Színezzük sakktáblaszerűen az egységkockákat felváltva fehérre, feketére! A 26
kockát úgy vesszük el, hogy felváltva veszünk fehér és fekete kockát, tehát a
kockák felének feketének kell lennie, azonban ez nem teljesül.
1522. Nem. Helyezzük a Rubik-kockát egy Descartes-féle derékszögű koordináta-rend-
szerbe oly módon, hogy a merőleges oldalélek illeszkedjenek egy-egy tengelyre,
és válasszuk a kocka egy kis négyzetének élhosszát 1-nek! Tekintsük a koordi-
náták összegének változását, miközben egyik lapcsúcsból a másikba lépünk. Az
összeg mindig páros számmal változik, tehát megőrzi paritását. A lapcsúcsok fe-
lénél a koordináták összege páros, másik felénél páratlan, ezért az útvonal bejárja
az összes azonos paritású csúcsot. Tehát át kell haladnunk a kocka valamely csú-
csán, de a csúcs három négyzetlapra illeszkedik, így a harmadik átló berajzolása
után nem tudunk tovább haladni.
1523. A térfogatokat összehasonlítva legfeljebb 54 db 1 × 1 × 4-es téglát helyezhetünk
a kockába. Azonban ez nem érhető el, legfeljebb 52 db 1 × 1 × 4-es téglát
helyezhetünk el.
Színezzük a kocka 27 db 2 × 2 × 2-es részét (sakktáblaszerűen) felváltva fehérre,
feketére. Ekkor egy 1×1×4-es tégla két fehér és két fekete 1×1×1-es kockából
áll. Mivel a színezés során 104 db 1 × 1 × 1-es kocka fehér és 112 fekete (vagy
104
fordítva), ezért legfeljebb = 52 1 × 1 × 4-es téglát helyezhetünk az adott
2
kockába. Nem nehéz megmutatni, hogy ennyit el lehet helyezni.
1524. Kirakható, de csak úgy, hogy az 1 × 1 × 1-es kockák a 3 × 3 × 3-as kocka
valamelyik testátlóján vannak. Ha a kockát felszeleteljük 3 rétegre valamelyik
27. Skatulyaelv 313

oldallapjával párhuzamosan, akkor mindegyik réteg térfogata 9 egység, és egy


ilyen réteg nem tölthető ki 2 × 2 × 1-es téglákkal. Ezért mindegyik rétegben kell
lennie 1 × 1 × 1-es kockának. Ha a kockát más oldallapjával párhuzamosan sze-
leteljük fel, ugyanezt állapíthatjuk meg. Ugyanezt megtesszük még a harmadik
irányból is. Csak úgy lehet minden rétegben 1×1×1-es kocka, ha ezek a kockák
valamelyik testátló mentén helyezkednek el.
1525. Ha k páros, akkor a színezés elvégezhető. A kockát osszuk fel 2 × 2 × 1-es
blokkokra, s ezeket váltakozva színezzük feketére, fehérre.
Belátjuk, hogy ha a színezés elvégezhető, akkor k páros. Kössük össze a szom-
szédos fehér színű kockák középpontját; minden középpontból két szakasz in-
dul. Ezek a szakaszok zárt töröttvonalat alkotnak; egy vagy több ilyen zárt vonal
keletkezik. Minden zárt vonalon páros számú csúcspont van, hiszen a szaka-
szok három, páronként merőleges irányban futhatnak. Ha egy csúcspontból pl.
3 egységnyit távolodunk előre a szakaszokon, akkor ugyanennyit meg kell tenni
visszafelé is, hogy záródjon az útvonal. Emiatt a fehér kockák száma páros, és
páros a fekete kockák száma is.
1526. Nem lehet. A fekete átlók száma az egyik irányban 7, a másikban 8. Így, ha
megszámláljuk a fekete mezőkön álló bábukat, egyik irányú átlókon haladva
7 páratlan szám összege adja a végeredményt, másik irányú átlókon haladva 8
páratlan szám összege a végeredmény. Ez a két összeg nem lehet egyenlő, ezért
nem valósítható meg a kívánt elrendezés.
1527. Legfeljebb 48 bábu helyezhető el. Ugyanis van 16 átló, melyek mindegyike pá-
ratlan számú mezőből áll, és ezeknek az átlóknak nincs közös mezőjük. Ezért
mindegyik átlónak legalább egyik mezőjén nem áll bábu, emiatt legfeljebb 48
bábut tehetünk fel. Ez meg is valósítható, ha a két, 8 mezőből álló átlóba nem
teszünk bábut.
1528. Lehet. Az első sor: 1, 1, 0, −1, −1, 0,1, 1, 0, −1, −1, 0, 1, 1, 0, −1, −1. A
második sorban minden szám nulla, a harmadik sorban az első sor számainak
(−1)-szerese áll, a negyedik sorban megint csupa nulla, stb.
1529. Ha 50 mezőn átmegy az egyenes, akkor a tábla belsejében 49 rácsegyenest metsz,
azonban csak 48 ilyen egyenes van.

27. Skatulyaelv

1530. 4 · 9 + 1 = 37.
1531. a) 4 · 3 + 1 = 13.
b) 20 + 20 + 20 + 4 = 64.
314 Megoldások, útmutatások

1532. a) 12,
b) 20,
c) 12,
d) 23,
e) 24,
f) 7.
1533. a) 3,
b) 22,
c) 5,
d) 7,
e) 22,
f) 24,
g) 11,
h) 26.
1534. a) 4,
b) 52,
c) 42,
d) 32,
e) 6,
f) 12,
g) 56,
h) 46,
i) 54,
j) 54,
k) 2k + 2.
1535. 10 + 10 + 1 = 21.
1536. A fogak számára 33 lehetőség van: 0, 1, 2, 3, . . . , 32. Ha 33 skatulyába 34 nevet
helyezünk, akkor lesz olyan skatulya, melyben van két név.
1537. A fogazat azonosítását megtehetjük egy 32-jegyű, 0 és 1 jegyekből álló szám-
mal: 0 áll a k-adik helyen, ha a k-adik fog hiányzik, különben 1-es. Az ilyen
különböző sorozatok — így a lehetőségek — száma: 232 . A válasz: 232 + 1 =
= 4 294 967 297 (nincs ilyen ország a Földön, melynek ennyi lakosa van).
1538. Igaz.
27. Skatulyaelv 315

1539. Válasszunk ki egy cédulát, és legyen az erre írt szám m. Ekkor a többi cédulán
szereplő n számok mindegyikére: m−1 000 000 ≤ n ≤ m+1 000 000. Ha minden
ilyen n szám véges sokszor szerepel, akkor csak véges sok cédula lehet.
1540. Két szelvény kevés, három elegendő.
1541. Osszuk fel a sakktáblát 16 db 2 × 2-es részre! Nem állhat mindegyiken 2 bábu.
1542. Van 17 azonos színű tehén, mivel 4 szín van és 4 · 16 < 65. Ezek között —
mivel csak legfeljebb négyféle korú tehén van — találhatunk 5 azonos korút,
hiszen 4 · 4 < 17. Az 5 tehén között van 3, melyek ugyanabba a faluba tartoznak.
1543. A körlapot osszuk fel 6 db egybevágó körcikkre! Lesz két pont, mely ugyanabba
a körcikkbe esik.
1544. Az előbbi felosztást most úgy végezzük el, hogy az egyik pont rajta legyen
valamelyik, a felosztásban résztvevő egyenesen.
1545. A körbe írjunk szabályos hatszöget, és az oldalak, mint átmérők fölé írjunk 1 egy-
ség átmérőjű köröket, továbbá még egy ilyen kört rajzoljunk a kör középpontja
körül. Ezt a hét kört úgy forgatjuk el az adott kör középpontja körül, hogy az
adott 8 pontból egyik se illeszkedjék e körök valamelyikére. Így elértük, hogy a
8 pontból legalább kettő ugyanannak az 1 egység átmérőjű körnek a belsejében
van, ezért távolságuk 1-nél kisebb.
1546. Osszuk fel a háromszöget 9 db egybevágó háromszögre.
1554. Az ábra szerint öt részre bontjuk a téglalapot. Ezek
valamelyikében a 6 pontból legalább kettő van, s egy
ilyen részben
√ lévő pontok között fellépő legnagyobb
távolság: 5.

865 − 1
1555. Elegendő megmutatnunk, hogy a háromszög felosztható = 432 olyan
2
részre, amelyek mindegyike lefedhető egy 1 átmérőjű zárt félkörlemezzel, mivel
akkor a 432 rész közül legalább egyben legalább 3 pontnak kell lennie.
Ehhez osszuk az eredeti háromszög mindegyik oldalát 12 egyenlő részre, és az
osztáspontokon keresztül húzzunk párhuzamosakat a háromszög oldalaival. Így
144 darab egybevágó, az eredetihez hasonló kis háromszögre daraboltuk fel a
háromszöget. Mindegyik kis háromszög feldarabolható 3 kisebb, az adott há-
romszöghöz hasonló, egybevágó háromszögre, s ezeknek a derékszögű három-
szögeknek az átfogója 1 egység hosszú. Ily módon eljutunk a kívánt felosztáshoz.
316 Megoldások, útmutatások

1556. Osszuk fel a háromszöget 12 részre úgy, hogy a részek minde-


gyike lefedhető legyen egy 5/17 átmérőjű körrel. Ehhez előbb 30◦
rajzoljuk be a háromszög középvonalait, majd a kapott négy
háromszög mindegyikét osszuk fel három egybevágó három-
szögre.
30◦
30◦

1557. Tekintsük a háromszög ábra szerinti szabályos


hatszöglemezekkel történő lefedését! Megmu-
tatjuk, hogy az állítás 20 pont helyett√már 17-
re is igaz. A hatszögek átmérője: 1/ 3 . Ha
valamelyik fél-hatszögben 3 pont van, akkor
készen vagyunk. Ellenkező esetben valamelyik
hatszögben 4 pont van.
Vegyük a hatszög köré írt körnek a 4 pont egyikén átmenő átmérőjét! Az átmérő
egyik oldalára legalább két pont fog esni.
1558. Fedjük le a háromszöget az ábrán látható módon 6 darab
egybevágó szabályos hatszöggel. Rövid számolás mutat-
ja, hogy a hatszögek lefedhetők egy-egy, 0,2 m-nél ki-
sebb sugarú körrel. A céltáblát 7 találat érte, ezért a 6
hatszög közül van olyan, amelyik a találatok közül lega-
lább kettőt tartalmaz. Ennek a két találatnak a távolsága
kisebb 40 cm-nél.
1559. Osszuk fel a négyzetet 16 db 2 cm oldalú négyzetre! Ezek egyikében lesz 3 pont.
Lássuk be, hogy e pontok mint csúcsok által meghatározott háromszög területe
legfeljebb fele a négyzet területének.
1560. Osszuk fel a négyzetet 25 db egybevágó, 1/5 oldalú√ négyzetre. Ezek egyikében
lesz 3 pont, s e négyzet köré írható kör sugara 1/ 50 < 1/7.
1561. Osszuk fel a táblát 16 db egybevágó négyzetre.
1563. Osszuk fel a kockát 125 db egybevágó kockára.
1564. Indirekt módon bizonyítjuk. Tegyük fel, hogy bármely két pont között a távolság
legalább egy egység. Rajzoljunk mindegyik pont köré 1/2 sugarú köröket; ezek
nem metszik egymást, s benne vannak az eredetivel koncentrikus, 10,5 sugarú
körben. Akkor a területeket összehasonlítva 450π(1/2)2 ≤ (10,5)2 π, azaz 450 ≤
≤ 441 lenne.
1565. Tegyünk mindegyik pontra mint középpontra ilyen gyűrűt. Ezek összterülete 650·
· 5π = 3250π. E körgyűrűk az adott körlemezzel koncentrikus, 19 egység sugarú
körön belül vannak. Ennek a körnek van olyan pontja, melyet a körgyűrűk közül
legalább 10 lefed, hiszen 9 · 192 π = 3249π < 3250π. E pont mint középpont
köré írt körgyűrű lefedi a (legalább) tíz körgyűrű középpontját.
27. Skatulyaelv 317

1566. A kör sugara legyen r. Tegyük fel, hogy az állítás nem igaz. Akkor a pontok
köré írt r/3 sugarú körlemezek nem metszik egymást. E 17 körlemez terüle-
17π 2
tének összege 17π(r/3)2 = r . A 17 körlemez lefedhető az eredeti körrel
9
r 16π 2
koncentrikus r + sugarú körrel. Ennek a körnek a területe r , ami kisebb
3 9
17π 2
r -nél, tehát nem fedi le a 17 körlemezt. Ellentmondásra jutottunk, hamis
9
volt a feltevés. Ezzel igazoltuk az állítást.
1567. Tekintsük a körnek a szabályos n-szög csúcsai által létrehozott n ívét. Ezek

közül az ívek közül az egyikben, pl. A1 A2 -ben legalább két pont van, amelyek

a szabályos (n + 1)-szög csúcsai, pl. B1 és B2 . Mivel az A1 A2 ív hossza ,a
n
2π 2π
B1 B2 ív hossza , az A1 B1 és a B2 A2 ívek hosszának összege .
n+1 n(n + 1)
1568. Válasszuk azt az Ai Aj átlót, melyhez O a legközelebb van.
1569. A 8 csúcsból legalább 3 azonos színű, s ezek közt van kettő, melyeket nem köti
össze a kocka valamely éle.
1570. Tekintsük a „nagy” háromszög három csúcsát és a három oldalfelező pontot. Ezt
a 6 pontot lefedi az 5 „kis” háromszög, tehát valamelyikük két pontot is lefed;
ezért ez a háromszög legalább a negyedét lefedi a „nagy” háromszögnek.
1571. Lássuk be, hogy egy ilyen egyenes a négyzet egyik középvonalát 2:3 arányban
osztja. A két középvonalon összesen 4 ilyen pont van, ezért ezek valamelyikén
legalább 3 átmegy a 9 egyenesből.
1572. Osszuk a körívet négy negyedkörívre úgy, hogy az egyik osztópont a 4 pont
egyike legyen! (A megoldás menete hasonló az 1544. feladatéhoz.)
1573. Az öt pontból egyet válasszunk Északi-sarknak, s vegyük fel azokat a főköröket,
melyek e ponton és egy-egy további ponton mennek át. Tekintsük e négy főkör
és az Egyenlítő metszéspontjait. Vagy az Északi-sark és egy másik pontból álló
páros a megfelelő, vagy az Egyenlítőn levő metszéspontokra alkalmazzuk az
előző feladat megoldását.
1574. A köröket merőlegesen vetítsük a négyzet valamelyik oldalára. A vetületek hosz-
szának összege 1,02, ezért a vetületek között van átfedés. Az átfedett szakasz va-
lamely pontjában állítsunk merőlegest a négyzet oldalára, ez az egyenes legalább
két kört fog metszeni.
10
1575. A körök átmérőinek összege > 3. Innen a feladat megoldása hasonló az
π
előzőhöz.
318 Megoldások, útmutatások

1576. Vetítsük a gömböket egy kiválasztott egyenesre. A „nagy” gömb 6 egység hosszú
vetületének szakaszán lesz a többi vetülete, melyek hosszának összege 50. Mivel
50 > 6 · 8, ezért van olyan pont, mely legalább 9 vetületnek is közös pontja. E
pontban állítsunk merőleges síkot az egyenesre.
1577. Vetítsük a töröttvonal szakaszait merőlegesen a négyzet két, egymásra merőleges
oldalára: az ai szakasz vetületei xi , yi . Nyilván xi + yi ≥ ai . Ezért
n n n n
xi + yi = (xi + yi ) ≥ ai > 1000.
i=1 i=1 i=1 i=1

Tehát a bal oldali két összeg közül legalább az egyik > 500, így azon az egység-
nyi hosszú négyzetoldalon a vetületek az oldal valamely pontját legalább 501-
szer lefedik. E pontban állítsunk merőlegest a négyzetoldalra, s ez az egyenes
legalább 501 pontban metszi a töröttvonalat.
1578. Hasonló az előzőhöz.
1579. Csak a tábla peremét figyeljük. Itt körben 28 mező, ill. megjelölt pont van, s már
ezek elválasztásához is kevés 13 egyenes.
1580. Válasszunk két pontot, A-t és B-t, melyek távolsága nem kisebb 1-nél (ha nincs
ilyen pontpár, akkor tetszőlegesen választhatunk), s rajzoljunk mindegyik körül
1 egység sugarú kört! A 25 pont bármelyike benne van valamelyik körben, ezért
az egyik körben mindenképp lesz 13 pont.

1000 
1000 
1000
1581. Válasszuk ki a kör egy AB átmérőjét. Ekkor |APi | + |BPi | = (|APi | +
i=1 i=1 i=1

1000
+|BPi |) ≥ 2000, (hiszen |APi |+|BPi | ≥ |AB| = 2), ezért vagy |APi | ≥ 1000,
i=1

1000
vagy |BPi | ≥ 1000.
i=1

1582. Vegyünk fel egy 1 egység oldalú szabályos háromszöget, ennek lesz két azonos
színű csúcsa.
1583. Vegyünk két különböző színű pontot, melyek távolsága kisebb, mint 1 egység.
Ezek köré írjunk 1 egység sugarú kört, s figyeljük a körök valamelyik metszés-
pontját.
1584. Használjuk fel az 1353. feladat megoldásának ábráját.
1585. Figyeljük egy szabályos hatszög csúcsait és a hatszög köré írt kör középpontját.
Megjegyzés. A sík két színnel kiszínezhető úgy, hogy ne legyen olyan egységnyi
oldalú szabályos háromszög, amelynek csúcsai ugyanolyan színűek.
27. Skatulyaelv 319

1587. Tekintsük az öt piros pont által meghatározott ötszöget. A tízszög középpontja


körül ezt az ötszöget forgassuk el valamelyik irányba 36◦ − 36◦ -kal, amíg önma-
gába vissza nem tér. Ha az így meghatározott ötszögek valamelyikének csúcsai
között található három kék, akkor a feladatbeli állítás nyilván teljesül.
Tegyük fel, hogy egyik ötszögben sincs három kék csúcs. Ekkor a teljes körül-
forgatás közben összesen legfeljebb 2 · 9 = 18 kék csúcspont található. A körül-
forgatás során egy-egy tízszög-csúcson áthaladt mind az öt ötszög-csúcs, ezért
minden egyes kék pontot ötször számoltunk. Mivel öt kék pont van, ezért össze-
sen huszonöt pontot kellett volna összeszámolnunk. Itt ellentmondásra jutottunk,
tehát a feltételezett eset nem állhat fenn.
1588. Válasszunk két szomszédos, azonos színű csúcsot. Ha a két pont bal vagy jobb
szomszédja ugyanolyan színű, már megtaláltuk a kívánt háromszöget. Ellenkező
esetben az eredetileg kiválasztott két pont és az általuk meghatározott szakasz
felezőmerőlegesén levő csúcs, vagy a két pont két szomszédja és az utóbbi csúcs
alkotta háromszög valamelyike megfelelő.
Másik
 megoldás.
 Tekintsünk 7 egyszínű csúcsot a 13-szög csúcsai közül. Össze-
7
sen = 21 olyan szakasz van, amelyeknek ezek a csúcsok a végpontjai. Mi-
2
vel a szabályos 13-szög csúcsai között csak 6 különböző hosszúságú szakasz
húzható, ezért a 21 szakasz között van legalább 4, amelyeknek hossza egyenlő.
Mivel ezek 7 pontot kötnek össze, ezért nem lehet minden végpontjuk külön-
böző. Egyenlő hosszúságú szakaszok csatlakozása esetén viszont egyenlő szárú
háromszög keletkezik, melynek csúcsai az előzők értelmében egyező színűek.
Megjegyzés. A feladat állítása igaz marad, ha 13 helyébe 5-öt, 7-et vagy tetsző-
leges, 8-nál nagyobb egész számot mondunk.
1589. Tekintsük a sokszög három szomszédos csúcsát. Összesen 13 ilyen hármas van.
Ezek két színnel való színezése 23 = 8-féleképp történhetik. Így találunk a 13
háromszög között kettőt, melyek ugyanúgy vannak színezve.
1590. Tekintsük az (1; i), (2; i), (3; i), (4; i), (5; i), (6; i), (7; i) 7 pontból álló rendsze-
reket. Ezeknek 67 -féle különböző színezése lehet. Vegyünk 67 + 1 db ilyen 7
pontból álló rendszert. Ezek közt két lesz azonosan színezett. A 7 pont között
van két azonos színű pont. Ezt a két pontot válasszuk ki mind a két, 7 pontból
álló rendszerből.
1591. Egy rácspont két koordinátája párosság szerint négyféleképp viselkedhet: páros-
páros, páros-páratlan, páratlan-páros, páratlan-páratlan. Az öt rácspont között van
két azonos típusú. A két pontot összekötő szakasz felezőpontja szintén rácspont.
1592. Egy-egy rácspont három koordinátájának párossága szempontjából 8-féle osz-
tályba sorolható. Mivel 9 rácspontunk van, lesz közöttük kettő, amelyeknek első
koordinátája azonos párosságú; ugyanígy a második koordináták, és a harmadik
320 Megoldások, útmutatások

koordináták is azonos paritásúak. Ezért e két pontot összekötő szakasz felező-


pontjának koordinátái egész számok lesznek.
1594. Indirekt úton bizonyítunk. Tegyük fel, hogy nincs háromszög a kívánt tulajdon-
sággal. Ekkor van olyan ABC háromszög, melynek csúcsai kékek. Ha nem len-
ne, akkor a síknak legfeljebb egyetlen egyenese tartalmazhatna kék pontokat, s
ekkor lenne piros csúcsokkal és súlyponttal háromszög.
Legyen ABC súlypontja S, amely az indirekt feltevés miatt piros. Nagyítsuk S-
ből négyszeresére az ABC háromszöget, a képháromszög A B  C  . Az S pont az
A B  C  háromszögnek is súlypontja. Ekkor az A BC háromszögnek A súlypontja
(miért?). Indirekt feltevésünk miatt A piros. Hasonlóan mutatható meg, hogy
B  is, C  is piros. Így az A B  C  háromszög csúcsai is, súlypontja is pirosak,
ellentmondásra jutottunk.
1595. E prímek utolsó jegye csak 1, 3, 7 vagy 9 lehet. Tehát az öt szám között lesz
kettő, melyek ugyanarra a számjegyre végződnek.
1596. Négyzetszámok utolsó jegye csak 0, 1, 4, 5, 6 vagy 9 lehet. Tehát a hét szám
között lesz kettő, melyek ugyanarra a számjegyre végződnek.
1597. Négyzetszám 3-mal osztva 0 vagy 1 maradékot adhat.
1598. Négyzetszám 4-gyel osztva 0 vagy 1 maradékot adhat.
1599. Általában is igaz: n + 1 db egész szám között van kettő, melyek különbsége oszt-
ható n-nel. Ugyanis a számok n-nel osztva n-féle maradékot adhatnak; így lesz
köztük kettő, melyek ugyanazt a maradékot adják, s ezek különbsége osztható
n-nel.
1600. Általában is igaz: n + 2 db egész szám között van kettő, melyek összege vagy
különbsége osztható (2n + 1)-gyel.
1601. Egy szám n-nel osztva 0, 1, 2, . . . , n − 1 maradékot adhat. Ezekbe a skatulyákba
megfelelően elhelyezve a 2-hatványokat, lesz olyan skatulya, melyben végtelen
sok szám lesz, s ezek közül bármely kettő különbsége osztható n-nel.
1602. Az 1, 11, 111, 1111, 11111, . . . , 111 . . . 11 (1992 db 1-es) számok között van
kettő, melyek 1991-gyel osztva ugyanazt a maradékot adják; ezek különbsége
osztható 1991-gyel, s ez a különbség 111 . . . 11100 . . . 000 alakú. Ha 111 . . . 11·
· 10k osztható 1991-gyel, akkor 111 . . . 11 is osztható 1991-gyel.
1603. Készítsük el a következő számokat: a1 , a1 + a2 , a1 + a2 + a3 , . . . , a1 + a2 +
+a3 + · · · + an . Ha e között az n db szám között nincs n-nel osztható, akkor
van kettő, melyek n-nel osztva ugyanazt a maradékot adják. Ezek különbsége:
ak+1 + ak+2 + · · · + am osztható n-nel.
1604. Három egész szám között van kettő, melyek ugyanolyan párosságúak.
27. Skatulyaelv 321

1605. Egy szám 3-mal osztva 0, 1 vagy 2 maradékot adhat. Ha az öt szám között van
három, melyek ugyanazt a maradékot adják, akkor ezek összege osztható 3-mal.
Ellenkező esetben kiválasztható három szám, melyek rendre 0, 1 és 2 maradékot
adnak 3-mal osztva. Ezek összege osztható 3-mal.
1606. Három egész szám között van kettő, melyek összege osztható 2-vel; így van az
adott 7 szám között is, legyenek ezek a és b. A megmaradó öt szám között is
van kettő, c és d, melyek összege páros; végül az utolsó három között is találunk
a+b c+d e+f
kettőt, e-t és f -et. Most , , egész számok, tehát közöttük van
2 2 2
a+b c+d
kettő, melyek összege osztható 2-vel. Legyenek ezek és . Így tehát
  2 2
1 a+b c+d a+b+c+d
+ = is egész, vagyis a + b + c + d osztható 4-gyel.
2 2 2 4
1607. Az a1 , a2 , . . . , a37 számok között van 13 olyan, mely 3-mal osztva ugyanazt
a maradékot adja (hiszen 37 > 3 · 12). Ha e 13 szám indexét vesszük, az ilyen
indexű bi -k között van 5 olyan, melyek 3-mal osztva ugyanazt a maradékot adják
(hiszen 13 > 3 · 4). Ha ennek az öt számnak az indexét vesszük, az ilyen indexű
ci -k között (lásd 1605. feladat!) van három, melyek összege osztható 3-mal. Ez
az ilyen indexű bi -kre és ai -kre is igaz.
1608. Az 1605. feladat mutatja, hogy legfeljebb 4 prímszámot lehet így megadni. Négy
ilyen prím pl.: 7, 11, 13, 23 vagy 19, 23, 37, 41.
1609. A 2-hatványok utolsó két jegyéből álló szám legfeljebb 100-féle lehet (ez a kor-
lát könnyen csökkenthető). Tehát, ha veszünk 101 különböző 2-hatványt, lesz
köztük kettő, melyek utolsó két jegye azonos. Ez már a periodikusságot jelenti,
hiszen, ha 2n és 2m utolsó két jegye megegyezik, akkor 2n+1 és 2m+1 utolsó két
jegye is azonos.
1610. Figyeljük az (fk , fk+1 ) számpár utolsó három jegyéből álló számpárt! Ezek leg-
feljebb 106 -félék lehetnek. Tehát van két olyan számpár, melyek utolsó három
jegye megegyezik. Ez a sorozat képzési szabálya miatt jelenti a periodikusságot.
1611. Lássuk be, hogy a sorozat elemei kisebbek egy a0 -tól függő korlátnál.
1612. Az 1608. feladat megoldásához hasonlóan van két 3-hatvány is, 3k és 3n , melyek
utolsó négy számjegye megegyezik. Ez azt jelenti, hogy
104 | 3k − 3n = 3n (3k−n − 1), tehát 104 | 3k−n − 1, s ez pontosan a feladat állítása.
1613. Hasonló az előző feladathoz, ill. a következő feladatnak speciális esete.
1614. Az m, m2 , m3 , . . . , mn+1 számok között van kettő, melyek n-nel osztva ugyanazt
a maradékot adják; tehát n | ma − mb , azaz
n | mk − 1.
322 Megoldások, útmutatások

1615. 100-ig 25 prím van. Ezek adnak 25 skatulyát, s még vegyük hozzá az 1-esét.
A 27 számot elhelyezzük ezekbe a skatulyákba. Egy számot annak a prímnek a
skatulyájába teszünk, mely osztója neki; ha több lehetőség is van az elhelyezésre,
akkor is csak egyet választunk, tetszés szerint. Az 1-es skatulyájába csak az 1
kerülhet. Lesz két szám, mely ugyanabban a skatulyában van.
1616. Tegyük fel, hogy nincs három egyforma számjegy a 20-jegyű számban. Ekkor
a számban mind a 10 számjegy pontosan kétszer szerepel. Ebben a számban a
számjegyek összege: 2(0+1+· · ·+9) = 90. A szám osztható 3-mal, ami ellentmond
annak, hogy a szám egy 3-tól különböző prímszámnak a hatványa.
1617. A bizonyítást Laczkovich Miklós: Sejtés és bizonyítás (TypoTEX, 1998) c. kitűnő
könyvéből vettük.
Például 10001 = 1002 + 12 pozitív osztói 1, 73, 137 és 10001. Ezek mindegyike
előáll két négyzetszám összegeként: 1 = 12 + 02 , 73 = 82 + 32 , 137 = 112 + 42 .
Az általános tételt bizonyítandó tegyük fel, hogy n osztója a 2 + b2 -nek. Nyilván
feltehetjük, hogy n maga nem négyzet. Azt fogjuk belátni, hogy vannak olyan
x, y egészek,
√ melyek nem mindegyike nulla, és amelyekre teljesül, hogy |x|,
|y| ≤ 2[ n], továbbá
√ 2 n osztója x 2 + y 2 -nek. E feltételekből következik, hogy
x + y ≤ 2[ n] < 2n, valamint x 2 + y 2 nullától különböző többszöröse n-nek.
2 2

Ez csak úgy lehetséges, ha x 2 + y 2 = n, ez tehát bizonyítani fogja állításunkat.


Mivel (a, b) = 1, ezért (n, b) = 1 (miért?), tehát n akkor és csak akkor osztója
x 2 + y 2 -nek, ha n osztója a
b2 x 2 + b2 y 2 = b2 x 2 − a 2 y 2 + (a 2 + b2 )y 2 = (bx − ay)(bx + ay) + (a 2 + b2 )y 2
számnak. Így n | x 2 + y 2 biztosan teljesül, ha n | bx √ − ay.
Tekintsük
√ a bx − ay számokat, ahol 0 ≤ x, y ≤ [ n]. Ezek száma
([ n] + 1)2 > n. Mivel ezen számok n-nel osztva csak n-féle maradékot adnak,
ezért a skatulya-elv szerint van két különböző (x1 , y1 ) és (x2 , y2 ) pár úgy, hogy
bx1 − ay1 és bx2 − ay2 azonos maradékot adnak n-nel osztva. Legyen x = x1 − x2
és y = y1 −y2 . Ekkor bx √ −ay = (bx1 −ay1 )−(bx√ 2 −ay2 ) osztható n-nel. Másrészt
|x| ≤ max(x1 , x2 ) ≤ [ n], és hasonlóan |y| ≤ [ n]. Végül x és y nem lehetnek
mindketten nullák, mert az (x1 , y1 ) és (x2 , y2 ) párok különbözőek voltak. Ezzel
a bizonyítást befejeztük.
1619. Ha nincs 8 páronként metsző egyenes, akkor legfeljebb 7 olyan irány van, melyek
valamelyikével az 50 egyenes bármelyike párhuzamos. 50 = 7 · 7 + 1, ezért van
8 egyenes, mely egymással párhuzamos.
1620. Minden intervallum befoglalható nem-csökkenő intervallumok egy sorozatába a
tartalmazás szerint, így „láncokat” alkothatunk. Egy intervallumot k-adrendűnek
nevezünk, ha azok a láncok, melyeknek ő a végső intervalluma, maximálisan
k-hosszúak. Ha van az intervallumok között (n + 1) olyan, hogy egyik sincs a
másikban, akkor készen vagyunk. Ha ez nem teljesül, akkor van (n + 1) hosszú-
ságú lánc, hiszen 1, 2, 3, . . . , n-edrendű intervallumokból — feltevésünk szerint
27. Skatulyaelv 323

— legfeljebb n–n darab van, s ezeknek az intervallumoknak a száma legfeljebb


n2 , noha az intervallumok száma n2 + 1.
1621. Minden helyiértéken a megadott 11 számból legalább két számban ugyanaz a
tizedesjegy szerepel.
1622. Három egymást követő számjegyből álló számhármas összesen 103 lehet, míg a
tizedesjegyek végtelen sorozatában végtelen sok, egymás utáni három számjegy
van.
1623. Válasszuk ki a poliéder egyik legtöbb oldalú lapját! Ha ez a lap egy n-oldalú
sokszög, akkor ehhez n db másik lap illeszkedik. Azonban ezek oldalainak száma
csak 3, 4, 5, . . . , n lehet. Ezért van közöttük két azonos oldalszámú.
1624. Egy n csúcsú egyszerű gráfban a fokszámok a 0, 1, 2, . . . , n−1 értékeket vehetik
fel, azonban a 0 és az n − 1 nem léphet fel egyszerre a gráfban. Az n csúcson
a fokszámok tehát n − 1 különböző értéket vehetnek fel, ezért van két azonos
fokszám.
1625. A 6 személyből egyet választva, az a többi öt közül hármat ismer (vagy hármat
nem ismer). Ha e 3 személy közül valamely kettő ismeri egymást, akkor ők ketten
a kiszemelt személlyel egy 3 fős klikket alkotnak (a 3 fő kölcsönösen ismeri
egymást). Ha a 3 személy közül semelyik kettő nem ismeri egymást, akkor ez
egy 3 fős antiklikk. Az állítás 5 főre nem igaz.
1626. Használjuk az előző és a 635. feladat állítását!
1628. Valamelyik tudóst kiválasztva, az a többi 16-tal három nyelven levelezik, így van
6 olyan partnere, akivel ugyanazt a nyelvet (pl. angol) használja, hiszen 3·5 < 16.
Ha e 6 személy között van kettő, akik egymás között angolul leveleznek, akkor
megvan a keresett hármas. Ellenkező esetben a 6 személy egymás között a német
vagy a francia nyelvet használja. Közülük az egyik az öt másikból legalább 3-
mal ugyanazt a nyelvet (pl. német) használja, hiszen 2 · 2 < 5. Ha e három
közül valamelyik kettő egymás között a németet használja, akkor megtaláltuk
a keresett három főt, különben pedig ez a 3 levelező partner egymás között a
francia nyelvet használja.
1629. Ugyanolyan, mint az előző vagy az 1625. feladat.
1630. Jelöljük az i-edik napon lejátszott játszmák számát ai -vel; i = 1, 2, . . . , 77.
Képezzük az s1 = a1 , s2 = a1 + a2 , . . . , sk = a1 + a2 + · · · + ak , . . . , s77 = a1 +
+ a2 + · · · + a77 összegeket. Mivel mindegyik nap játszik a sakkmester, ai ≥ 1
(i = 1, 2, . . . , 77). Emiatt s1 < s2 < · · · < s77 . Képezzük most az s1 + 21,
s2 + 21, . . . , s77 + 21 számokat. Így az si és az si + 21 számokból összesen 2 · 77 =
= 154 darab van. Mivel a sakkmester hetenként legfeljebb 12 mérkőzést játszik,
s77 ≤ 11 · 12 = 132, s77 + 21 ≤ 153. Azaz (a1 ≥ 1 miatt) a 154 db si és si + 21
szám bármelyikének értéke 1 és 153 közé esik (a határokat is megengedve). Ezért
324 Megoldások, útmutatások

lesz közöttük kettő, amelyik megegyezik. Az si számok mind különbözők, így


különbözők az si + 21 számok is, tehát az egyezés valamelyik sk és valamelyik
si +21 szám között van. sk = si +21-ből sk −si = 21, azaz ak +ak−1 +· · ·+ai+1 = 21.
Sakkmesterünk tehát az (i + 1)-edik nappal kezdve, a k-adikkal bezárólag éppen
21 partit játszik le.
1631. Ha a kiinduló sortól különböző sorok bármelyikében megjelenik egy n szám, az
azt jelenti, hogy az előző sorban valamelyik számból n darab egyenlő található.
Jelölje k azoknak a különböző számoknak a számát, amelyek mindegyikéből az
előző sorban pontosan n darab található. Ha k = 1, akkor a tekintett sor azon he-
lyei alá, ahol n áll; a következő sorban is n-et írunk. Ha k > 1, akkor a következő
sorban n alá kn kerül. Tehát a kiinduló sor számai alatt lefelé induló sorozatok
mindegyike nemcsökkenő. Ha találunk két olyan, egymást követő sort, amelyek
megegyeznek; akkor az ismétlődés minden további sorban be fog következni. Az
első ismétlődő sort véges sok lépés után megtaláljuk, hiszen addig minden sor-
ban legalább egy szám alá írt érték nő, másrészt viszont a kiinduló sor után leírt
számok értéke legfeljebb 1000.
Megjegyzés. Érdekes megvizsgálni hasonló módon pl. a verseket. Leírjuk egy
vers első sorát, majd a sor alá számokat írunk. Minden betű alá azt a számot,
mely megmutatja, hogy az a betű hányszor szerepel ebben a sorban. A számso-
rozatból feladatunk szabályai szerint képezzük az újabb számsorozatokat. Leg-
feljebb hányadik sor az, ahonnan kezdve már biztosan ismétlődnek a sorok?
1632. Válasszuk ki a sorok közül azt, amelyiken legtöbb bábu van, és hagyjuk el. (Ha
több ilyen sor van, ezek bármelyikét választhatjuk.) Ezt ismételjük, amíg n sor
nem marad. Ha az utolsó lépésben legalább 2 bábut hagytunk el, akkor az előző
lépésekben is legalább 2–2 bábut hagytunk el, összesen legalább 2n bábut, így
legfeljebb n bábu maradt az n × 2n-es táblán. Ha az utolsó lépésben 1 bábut
hagytunk el, akkor a maradék n sor mindegyikén legfeljebb 1–1 bábu van. Tehát
az n sor elhagyása után az n × 2n-es táblán legfeljebb n bábu maradt, ezeket
pedig n oszlop elhagyásával eltüntethetjük.
1633. Az összegek lehetséges értékei: 5, 3, 1, −1, −3, −5. Mivel tíz összeg van, ezek
között van két azonos.
1634. Az összegek száma: 2n + 2. Az összegek lehetséges értékei: n, n + 1, . . . , 3n,
azaz összesen 2n + 1 lehetőség.
1635. Legyen m a legkisebb szám a táblázatban! Abból a mezőből, ahol ez a szám
áll, oldalszomszédos mezőkön lépkedve legfeljebb 198 lépésben bármely me-
ző elérhető, ezért a legnagyobb szám legfeljebb m + 198 · 20 = m + 3960.
A táblázatban tehát legfeljebb 3961 különböző szám lehet. Mivel 2 · 3961 <
< 100 · 100, így valamelyik szám legalább háromszor szerepel.
28. Matematikai játékok 325

28. Matematikai játékok

1636. A kezdő az asztal szimmetria-középpontjába helyezi az első érmét, majd mindig


az ellenfele által elfoglalt helynek a középpontra való tükrös képére tesz.
1637. Először egy főátlót rajzol meg, majd mindig azt az átlót húzza meg, amelyik kö-
zéppontosan tükrözött képe az ellenfél által legutóbb húzott átlónak. Különben
mindegy, hogy követ-e valamilyen stratégiát, mert mindig a kezdő nyer. Ha az
oldalak száma 2n + 1, akkor a második nyer. Ugyanis egy N oldalú sokszögbe
a feltételek szerint átlókat rajzolva, a játék végére a sokszöget háromszögekre
bontottuk. A háromszögek száma N − 2, hiszen a háromszögek szögeinek össze-
ge egyenlő a sokszög belső szögeinek összegével. Mindez azt jelenti, hogy a
sokszögbe N − 3 átlót rajzoltunk (lásd a 113. feladatot!).
1638. Ha n páros, akkor a kezdő nyerhet, ha úgy játszik, ahogy az előző feladat meg-
oldásánál leírtuk. Ha n páratlan, akkor a második nyerhet, ha mindig ellenfele
lépésének a sokszög középpontjára tükrözött képét rajzolja be.
1640. A kezdő első lépése: a középső, ill. a középső két bábut dönti föl. A továbbiakban
mindig ellenfele lépésének a középső(k)re vett tükörképét dönti el.
1641. A másodiknak van nyerő stratégiája, hiszen a kezdő első lépése után a játék
megegyezik az előző feladat játékával.
1642. A másodiknak van nyerő stratégiája. Mindig ellenfele lépésének a kocka közép-
pontjára tükrözésével kapott csúcsba helyezi az ugyanolyan színű korongot.
1643. Mindegyik játékos megakadályozhatja ellenfelét a győzelemben, ha első lépése-
ként három kitérő élt színez.
1644. A kezdő nyerhet, ha a (60, 70) állapotból a (60, 60) állást alakítja ki első lépés-
ben, majd mindig az (m, m) állapotra törekszik.
1645. A kezdő nyerhet, ha első lépésében az (1, 64, 65) állást alakítja ki; majd mindig
az (1, 2m, 2m + 1) vagy a (0, m, m) állapot valamelyikére törekszik.
1646. Ha kezdetben mindkét kupacban páros számú kavics volt, akkor a második nyer-
het; különben a kezdő játékos. A nyerő stratégia az, hogy az asztalon mindkét
kupacban páros számú kavicsot kell hagyni.
1647. A második játékos mindig „szimmetrikusan lép”: az első játékos által elfoglalt
mezőnek a tábla középpontjára vett tükörképét foglalja el.
1648. Az első játékos első lépésében elfoglalja a tábla középső mezőjét, majd mindig
„szimmetrikusan lép”: a második játékos által elfoglalt mezőnek a tábla közép-
pontjára vett tükörképét foglalja el.
1649. A második játékosnak van nyerő stratégiája: a tábla középpontjára szimmetriku-
san játszik (tehát ellenfele utolsó lépésének tükörképét lépi).
326 Megoldások, útmutatások

1650. Az a) és b) esetben a második játékosnak van nyerő stratégiája. (Középpontos


szimmetria.) A c) esetben az első játékosnak van nyerő stratégiája: első lépésében
elfoglalja azt az oszlopot, amely tartalmazza a téglatest szimmetriaközéppontját.
Mindegyik esetben a második játékosnak van nyerő stratégiája.
1651. A kezdő játékosnak van nyerő stratégiája: előbb elfoglalja a tábla középpontját,
majd arra szimmetrikusan játszik (tehát az ellenfél utolsó lépésének tükörképét
lépi).
1652. a) Nyerhet a kezdő. Lásd az 1636. feladatot.
b) A második nyerhet. Az ábra középpontjára szimmetrikusan játszik.
1653. A kezdő nyerhet. Elsőként 1 cukorkát áttesz, majd a 10 cukorkát tartalmazó ku-
pacból 2 cukorkát elvesz; így mindegyik kupacban 8–8 cukorka marad. Legköze-
lebb a cukorkák számát 6–6 darabra állítja be, s később is a 2n–2n darabszámot
éri el.
1654. A 10 × 10-es táblázaton ábrázolhatjuk a játék pozícióit. Legyen a bal alsó sarok
a nulladik sor nulladik mezője. Az m-edik sor és az n-edik oszlop közös mezője
a játéknak azt az állapotát tükrözi, amikor az első kupacban m kavics, a második
kupacban pedig n kavics van (0 ≤ m, n ≤ 9).
+ + + +
+ + +
+ + +
+ + + +
+ + +
+ + +
+ + + +
+ + +
+ + +
+ + + +

Visszafelé okoskodva (a bal alsó sarokból indulva) megtaláljuk a nyerő pozí-


ciókat, ezeket a + jelek jelzik. Innen láthatjuk, hogy a kezdő játékos nem tud
nyerő pozícióra lépni; ám a második igen, ha szemmel tartja a nyerő pozíciókat.
A második játékosnak van nyerő stratégiája.
1655. A második játékos nyer, ha mindig ellenfele lépésének a tábla középpontjára
tükrözésével kapott mezőre teszi bábuját.
1656. A második játékos nyer, ha mindig ellenfele lépésének a tábla középpontjára
tükrözött mezőit színezi ki.
1657. A második nyerhet, ha mindig az átlóra tolja fel a bábut.
28. Matematikai játékok 327

1658. A kezdő mindig a rajz szerinti állapotot alakítja ki.


... ...
a) b)
... ...
...
.. ..
. .

1659. Ha n = m, akkor a kezdő nyerhet; ő mindig k × k-s méretű táblát hagy meg.
Különben pedig a második számára követhető ez a nyerő stratégia.
1660. A kezdő nyerhet. Először két darab 5×5-ös táblára töri a csokit; majd mindaddig,
amíg nem tud 1×1-est letörni, megismétli ellenfele törését a másik csokidarabon.
1661. A kezdő nyerhet. Először két darab 5 × 5-ös táblára töri a csokit, majd mindig
megismétli ellenfele lépését a másik csokidarabon.
1662. Ezzel kapcsolatos az 1625. feladat. 5 pont esetén ügyes játékkal mindegyik játé-
kos megakadályozhatja a másik győzelmét, s ekkor döntetlennel ér véget a játék.
6 pont esetén a kezdőnek van nyerő stratégiája.
1663. Felvehetjük a hat pontot úgy is, hogy azok egy szabályos hatszög csúcsait al-
kossák. A hatszög egyik, átlótól különböző szimmetriatengelye legyen t. A t
tengelyre a hatszögnek két oldala (legyen ez a és b) és egy átlója merőleges. A
kezdő játékos első húzása legyen a t-re merőleges átló. A továbbiakban mindig
az ellenfél szakaszának a t egyenesre vonatkozó tükörképét rajzolja, illetve ha az
ellenfél a vagy b egyikét húzza, ő berajzolja a másikat. Az adott eljárásnak meg-
felelő szakasz nyilván mindig behúzható. Lássuk be, hogy a kezdő nem zárhat
háromszöget elsőként! Ez azt jelenti, hogy a kező nyer.
1664. A második nyerhet. Kiválasztja valamelyik 1 × 1-es rácsnégyzet középpontját, s
ellenfele lépésének e pontra tükrözésével kapott pontot választja.
1665. A kezdő nyerhet, ha az adott szám utolsó számjegye nem nulla; különben a
második nyerhet. Mindig a táblán levő szám utolsó jegyét vonja le a számból
(így ő mindig 0-ra végződő számot hagy a táblán).
1666. Okoskodjunk „visszafelé”. Aki 4 gyufát hagy az asztalon, az már nyert. Előzőleg
8, korábban rendre 12, 16, 20, 24 szálat kell meghagyni. Ezt a kezdő megteheti.
1667. A második játékos el tudja érni azt, hogy a kezdő ne nyerhessen: mindig 4-re
egészíti ki a kezdő által elvett gyufák számát.
1668. Az előbbi megoldáshoz hasonló. A kezdő nyerhet: 25, 20, 15, 10, 5 szál gyufát
kell az asztalon hagynia.
328 Megoldások, útmutatások

1669. Az előző feladatok megoldásához hasonló. A kezdő nyerhet: 35, 28, 21, 14, 7
szál gyufát kell az asztalon hagynia.
1670. Az előző feladatokhoz hasonló. A kezdő nyerhet: 37, 30, 23, 16, 9, 2 szál gyufát
kell az asztalon hagynia.
1671. Mindig annyi szál gyufát hagy az asztalon, amely szám a 6 többszöröse (ugyanis
vehet 1, 2, 3, 4, ill. 5 szál gyufát).
1672. Bárhogyan játszanak is a játékosok, mindig a kezdő játékos a győztes; hiszen ha
(a, b) = d, akkor d | a + b = 101, de 101 prímszám, ezért d = 1.
(Érdekes megvizsgálni a játékot 101 helyett összetett számot választva.)
1673. Ha n osztható 3-mal, akkor a második nyerhet. Az első játékos vételét mindig
3-mal osztható számra egészíti ki.
Ha n nem osztható 3-mal, akkor az első nyerhet. Első alkalommal az el nem vett
gyufák számát 3-mal osztható számra állítja be, s a további lépéseiben is erre
törekszik.
1674. Ha n páratlan, akkor a kezdő nyerhet; hiszen ő mindig léphet páros számra, de
innen a második csak páratlanra léphet. Ha n páros, akkor (az előzőek szerint) a
második nyer.
1675. Két esettel foglalkozunk aszerint, hogy a golyók g száma néggyel osztva 1 vagy
3 maradékot ad.
1. eset. Ha g = 2k + 1 és k páratlan, akkor mindegyik játékos legfeljebb k golyót
vehet el egyszerre.
Legyen a kezdő húzó A, a második húzó B.
Ha A elsőre k darab (tehát páratlan sok) golyót húz , akkor B válasza után
legalább 1, de legfeljebb k darab golyó marad. Így másodszorra attól függően,
hogy B páros vagy páratlan sok golyót húzott, A egy kivételével valamennyi
golyót elveszi, illetve egyetlen golyót sem hagy B-nek.
Ez esetben tehát A mindig nyer, hiszen B-nek páratlan sok golyója lesz.
2. eset. Ha g = 2k + 1 és k(> 2) páros, akkor A előszörre k − 2 golyót vegyen el!
Ha most B a maradék k + 3 golyóból 3-nál többet és legfeljebb k darabot vesz
el, akkor ha az így adódó maradék páros, A ezt mind felveszi; ha pedig páratlan,
akkor 1 kivételével veszi fel A ezeket.
Ha B a maradék k + 3 golyóból csak egyet vesz el, akkor másodjára A is csak
egyet húzzon, s ezzel előállt az 1. esetben tárgyalt helyzet, hiszen a maradék k +
+ 1 golyóból most B kénytelen húzni és legalább egyet meghagyni. Ha tehát a
maradék páros, akkor A egy kivételével valamennyit vegye el; ha pedig páratlan
a maradék, akkor valamennyit fel kell vennie.
Ezzel beláttuk, hogy ha g ≥ 3 és g = 5, akkor mindig A nyer. A g = 1 esetben
A veszít. Ha g = 5, akkor is A veszít, mert bármennyit is húzzon elsőre, B is
ugyanannyit fog húzni, s ezzel eléri, hogy A-nál 1 vagy 3 golyó legyen.
28. Matematikai játékok 329

1678. Összesen 1990 két részre osztás történik (lásd a 113. feladatot!), ezért minden-
képp a második játékos győz.
1679. a) A második nyerhet.
b) A kezdő nyerhet.
Cél: a legnagyobb kupacban mindig 2k − 1 db kavics legyen.
1680. Okoskodjunk „visszafelé”. Akinél a szorzat 56 és 111 között van (a határokat is
beleértve), az ellenfelének következő lépése után 1000 fölé jut. Ugyanígy látható,
hogy az előző nyerő intervallum a 4, 5, 6 számokból áll. Tehát a kezdő nyerhet.
1681. Ismét „visszafelé” okoskodunk. A nyerő számok (fordított sorrendben): 100, 90,
80, 71, 62, 53, 44, 35, 30, 23, 20, 14, 11, 5, 2. Tehát a második játékosnak van
nyerő stratégiája.
1682. „Visszafelé” okoskodva megkereshetők a
nyerő mezők. Az ábrán ezek a megjelölt
mezők. A kezdő játékos mindig ezek va-
lamelyikére lép. Egyszerűbben is leírható,
hogyan lépjen a kezdő játékos: első lépé-
se egyet jobbra fel, majd mindig ellenfele
lépését ismétli meg.

1683. Belátjuk, hogy ha n · m páros, akkor a kezdő játékosnak van nyerő stratégiája,
ha n · m páratlan, akkor a másodiknak.
Ha n · m páros, akkor n vagy m páros. Ezért a tábla felosztható (2 × 1)-es
téglalapokra. A kezdő játékos első lépése az induló mezőt tartalmazó téglalap
második mezője, ezután is mindig a királyt tartalmazó téglalap másik mezőjére
lép az első játékos.
Ha n · m páratlan, akkor n is, m is páratlan. A sakktáblát, a bal felső sarok
kivételével, lefedjük (2 × 1)-es téglalapokkal. Bárhová is lép a kezdő, a második
játékos a királyt tartalmazó dominó másik mezőjére lép.
1686. Készítsük el a játék fáját! A fagráf csúcsai a játék egy-egy állapotát tükrözik; s
a gráf élei mutatják, hogy mely állapotból mely állapotba lehet eljutni. A gráf
magasságára teljes indukcióval igazolható az állítás.
1687. A játékosok lépésétől függetlenül a táblán egyedül maradó szám 1-es, ha m
páratlan; ha pedig m páros, akkor 2-es marad a végén. Ennek oka, hogy a táblán
levő 1-esek száma lépésenként vagy nem változik, vagy 2-vel csökken.
330 Megoldások, útmutatások

1688. Az 1686. feladat szerint valamelyik játékosnak biztosan van nyerő stratégiája.
Megmutatjuk, hogy a második játékosnak nem lehet. Ugyanis, ha lenne, akkor
a kezdő bármilyen kezdése esetén meg tudná nyerni a játékot. Kezdjen most
az első játékos az 1 számmal! Ezzel szerepcsere történt: az első ezek után úgy
játszhat, ahogyan a második játszana, ha az első kezdene azzal a számmal, amit a
második írt az 1 után a táblára. Ellentmondásra jutottunk, hiszen ebben a játékban
most a kezdőnek és a másodiknak is nyerő stratégiája lenne. Ha n = 10, akkor a
kezdő első lépése a 6. Ezután az ellenfele által választott számnak a következő
párosításban kijelölt párját választja: (4, 5), (7, 9), (8, 10).
1689. Ahogyan az előző feladatban, itt is a „helycserés” bizonyítás mutatja, hogy a
kezdőnek van nyerő stratégiája. Ha n = 72, akkor a kezdő stratégiája a következő:
Első választása a 6. Ha a második első lépése a (36, 24) valamelyike, akkor ő
a másik számot választja, s megnyerte a játékot. Ha a második első választása
a (9, 8) valamelyike, akkor ő a megmaradt számot választja; majd a (12, 18),
(24, 36) párokból mindig azt, melynek a társát ellenfele elvitte. Ha a második
első választása a (18, 4) valamelyike, akkor ő a megmaradt számmal válaszol;
aztán a (8, 12), (24, 36) párokból a megmaradt számot veszi el. Ha a második a
12-t választja, akkor ő 9-cel válaszol, majd a (8, 18), (24, 36) párokból mindig a
megmaradt számot veszi el.
1690. Mindenképpen a második játékos nyer. A felírt számok csak a kiinduló két szám
(25 és 36) legnagyobb közös osztójának többszörösei közül kerülhetnek ki (euk-
lideszi algoritmus). A 36-nál kisebb összes szám felírható, így 34 számot írnak
fel a játékosok (hiszen két szám már fel volt írva), az utolsó számot a második
játékos írja.
1691. Ha a négy együttható a, b, c, d lesz, a második játékosnak elegendő úgy meg-
választani az utoljára felírt együtthatót, hogy a + b + c + d = 0 legyen. Ennek az
egyenletnek x = 1 gyöke.
1692. Elérhető a cél: x 3 + kx 2 − x − k = (x 2 − 1)(x + k) = 0.
1693. Elérhető a cél: x 3 + ax 2 + x + a = (x 2 + 1)(x + a) = 0.
1694. Elérhető a cél. Az első játékos olyan a, b, c racionális számokat választ, ame-
lyekre a + b + c = 0 és a = 0. Ekkor az ax 2 + bx + c = 0 egyenlet gyökei: x1 = 1,
c
x2 = .
a
1695. A kezdő játékos elérheti, hogy az egyenletrendszer megoldása (0, 1, −1) legyen.
Ehhez az szükséges, hogy mindegyik egyenletben y és z együtthatója ugyanaz a
szám legyen.
1696. Ahogyan az 1689. feladatban, itt is a „helycserés” bizonyítás mutatja, hogy a
kezdőnek van nyerő stratégiája.
1697. Itt is a „szerepcsere” segít. A kezdő kilép, majd visszalép a lóval.
28. Matematikai játékok 331

1698. Itt is a „helycsere” segít. A kezdő egy „távoli” mezőre lép.


1699. A kezdő után a második 2 jelet üres sorba helyez, hogy azok egyike se legyen
a kezdő jelével egy oszlopban. A következő alkalommal üres sorba, saját osz-
lopaiba helyezi jeleit. Ekkor a másodiknak 4 vonala van 2–2 jellel, s ezeknek
legfeljebb az egyikében van a kezdőnek 1 jele. A második játékos harmadik lé-
pésében két szabad vonalának mindegyikére egy-egy jelet tesz.
1700. Vagy a második nyer, vagy döntetlen lesz. A második minden jel írása előtt
megnézi, hogy tud-e harmadik azonos jelet írni. Ha nem, akkor a kezdő utolsónak
írt jelével ellentétes jelet ír a tábla középpontjára szimmetrikus mezőre. Így a
kezdő a következő lépésében szintén nem tud harmadik jelet sorba tenni, tehát
nem győzhet. Arra viszont törekedhet, s el is érheti, hogy ne jöjjön létre sorban
egymás mellett három azonos jel.
1701. A másodiknak van nyerő stratégiája. Mindig a kezdő lépésének a tábla közép-
pontjára tükrözésével kapott mezőre lép.
1702. 1 × 2-es dominókkal való lefedés segít. Ha a sarokmező szomszédjáról indul,
akkor mindig a kezdőnek van nyerő stratégiája.
1703. Ha m · n páratlan, akkor a kezdőnek; ha páros, akkor a másodiknak van nyerő
stratégiája. Ehhez elegendő megmutatni, hogy ha m·n páros, akkor a tábla mezői
párba állíthatók úgy, hogy a pár két tagja „huszárlépésre” legyen egymástól. Ha
pedig m · n páratlan, akkor egy mező kivételével ez a párosítás elvégezhető.
1704. Képzeljük el, hogy nem sima négyzetrácsos papíron játszunk, hanem egy „sakk-
táblaszerű” játékmezőn, azaz a négyzetek ki vannak színezve felváltva feketére
és fehérre. Kezdetben a fekete bábuk a fekete mezőkön, a fehér bábuk a fehér
mezőkön állnak. Mivel minden lépés szomszédos mezőre történik, ezért a lyuk
(az a mező, ahol nem áll bábu) minden lépés után más színű mezőre kerül. Ebből
következik, hogy a kezdő minden lépése előtt a fekete mezőn lesz a lyuk, a má-
sodik játékos lépései előtt pedig mindig fehéren. Ebből az is következik, hogy a
játék során minden figura legfeljebb egyszer mozoghat; ugyanis pl. a kezdő vala-
melyik figurája, ha egyszer fehér mezőről feketére lép, akkor többet nem léphet;
hiszen csak a kezdő tolhatná tovább, de mint láttuk, ő mindig csak fehér mezőről
tolhat feketére bábut. Így kiderült, hogy a játék véges; mivel mindkét játékosnak
12 bábuja van, s minden bábu legfeljebb egyszer mozoghat, így legkésőbb 12
lépéspár után a játék biztosan véget ér.
A második játékosnak van nyerő stratégiája. Ehhez képzeletben (2 × 1)-es do-
minókkal fedjük le a táblának azt a részét, ahol kezdetben a bábuk állnak. A
stratégia a következő: a kezdő játékos egyik bábuját elmozdítja valamelyik kép-
zeletbeli dominóról, ekkor a második játékos az abban a dominóban levő bábuját
tolja az üresen maradt helyre. Ily módon a második játékos mindig tud válaszolni
a kezdő játékos lépésére, s a második játékos lép utoljára.
332 Megoldások, útmutatások

1705. A másodiknak van nyerő stratégiája: mindig a kezdő által választott szám bal
szomszédját foglalja el, ha az még választható, különben a jobb szomszédot, ha
már az is foglalt, akkor tetszőlegesen választ.
1707. Ha n = 3k, akkor a másodiknak van nyerő stratégiája. Ha a kezdő a (6, 9), (7, 8)
párokból az egyiket választotta, akkor ő a másikat írja.
Ha n = 3k + 1, akkor a kezdőnek van nyerő stratégiája. Első lépésként egy 9-től
különböző számjegyet ír, a továbbiakban pedig úgy játszik, ahogyan a második
az előbb leírt esetben.
Ha n = 3k + 2, akkor a kezdőnek van nyerő stratégiája. Első lépésében a 9-et írja.
1708. Ha n = 3k, akkor a másodiknak van nyerő stratégiája: ellenfele lépését mindig
kiegészíti 6-ra. Ekkor a számjegyek összege 6 · 2n = 36k, ezért a kapott szám
osztható 9-cel.
Ha n nem osztható 3-mal, akkor a kezdő játékosnak van nyerő stratégiája. Első
alkalommal 3-ast ír, majd ezután ellenfele lépését mindig kiegészíti 6-ra. Ekkor
az első 2n − 1 számjegy összege 3 + 6(n − 1). Ha n = 3k + 1, akkor 3 + 6(n − 1) =
= 18k + 3, amit a második játékos utolsó lépésében nem tud kiegészíteni 9 több-
szörösére. Ha n = 3k + 2, akkor 3 + 6(n − 1) = 18k + 9, s ezt sem tudja kiegészíteni
9 többszörösére a második játékos az utolsó lépésben.
1709. Tekintsük a számokat 2-től 100-ig! Azt állítjuk, hogy ha a feladatbeli játékot csak
erre a 99 számra korlátozzuk, akkor az itteni játékban második játékos el tudja
érni, hogy a műveleti jelek beírása után páros legyen az eredmény, bárhogyan is
játsszék ellenfele.
Ebből már következik, hogy a kezdő meg tudja nyerni az eredeti játékot: első
lépésében vagy az összeadás, vagy a szorzás jelét írja az 1 és a 2 közé, s ettől
függően páratlan, ill. páros lesz a végeredmény.
A fenti, korlátozott játékban a második stratégiája: ha ellenfele felírt egy műve-
leti jelet, akkor ő ezzel a jellel szomszédos páratlan szám másik oldalára írja fel
a szorzás jelét. Ezt mindig megteheti, hiszen a 99 szám közti 98 hely 49 párba
sorolható: az egyes párok a páratlan számokat fogják közre. Mivel egy-egy lé-
pésben egy pár két helyére kerülnek műveleti jelek, így a második játékos tud
a fenti utasítás szerint játszani. A kitöltés befejezésekor a végeredmény valóban
páros lesz, hisz minden egyes páratlan szám egy szomszédjával — ami páros —
megszorozva vesz részt a számolásban.
1710. A fehér színével felfelé levő korong helyére írjunk 1-est, a fekete helyére 0-t; így
kapunk egy kettes számrendszerbeli számot, nevezzük ezt állapotjelző számnak.
Ha egy fehér korongot megfordítunk, akkor az állapotjelző szám értéke csökken.
Ez azt jelenti, hogy a játék biztosan véget ér néhány lépés után, hiszen az ál-
lapotjelző szám értéke minden lépésben csökken, egyszer csak nulla lesz, azaz
mindegyik korong a fekete oldalával felfelé lesz.
Nincs nyerő stratégia. Ha az utolsó korong fekete oldalával felfelé van, akkor a
második játékos nyer; ha az utolsó korong fehér, akkor a kezdő játékos. Ugyan-
28. Matematikai játékok 333

is minden lépésben egy korong, az utolsó biztosan megfordul. Az lépésenként


váltakozva lesz fehér és fekete oldalával felfelé. Ha kezdéskor például a fehér
oldalával van felfelé, akkor mindig a kezdő játékos lépését követően fordul fe-
kete oldalára, a játék — mint láttuk — egyszer véget ér, s az utolsó lépés a
kezdőé lesz, ő nyer.
1711. Legyen Béla stratégiája az, hogy mindig arra a lépcsőfokra rak követ, ahonnan
András az előző lépésben elvett. Ezt Béla mindig megteheti, hiszen a következő
lépcsőfokon vagy András lépése előtt is állt már kő, vagy éppen András tett oda
követ. Egyszerűen belátható, hogy ha Béla így játszik, akkor András bármely
lépése előtt az alsó lépcsőfokon van kő, és két egymás utáni lépcsőfok nem lehet
üres (kivéve persze a felső lépcsőfokokat). Így András bármely lépése előtt a
legfelső két lépcsőfok üres; hiszen összesen n kő van és közöttük legfeljebb
n − 1 üres lépcsőfok. Ezért András soha nem rakhat a lépcső tetejére. (Célszerű
játszani a játékot néhány lépésen át, ill. játszunk a megadott stratégiát követve
is.)
1712. Vizsgáljuk a következő játékot. Az alábbi 3 × 3-as táblázatban ketten felváltva
foglalnak el egy-egy mezőt. Az a játékos nyer, akinek először lesz három saját
mezője egy vonalban (egy sorban, egy oszlopban vagy egy átlóban).

2 9 4
7 5 3
6 1 8

Mindkét játékos megakadályozhatja ellenfelét a győzelem elérésében, tehát egyik-


nek sincs nyerő stratégiája.
1713. A kezdő nyerhet. Először a 47, 48, . . . , 55 számokat veszi el, majd ellenfele
lépését úgy egészíti ki választásával, hogy az elvett számok között mindegyiknek
a párja is szerepeljen, ha a párbaállítás módja (k, 55 + k).
1715. A kezdő minden alkalommal legalább megkétszerezheti a még megmaradó szá-
mok közti legkisebb különbséget (ha minden választása alkalmával minden má-
sodik számot kihagy). Így elérheti, hogy a játék végén m legalább 32 legyen. A
második pedig minden lépése alkalmával a megmaradó számok közti legnagyobb
különbséget felezheti; így elérheti, hogy m legfeljebb 32 legyen.
334 Megoldások, útmutatások

29. Különféle kombinatorikai feladatok

1716–1720. Használjuk fel, hogy egy gráfban a páratlan fokszámú csúcsok száma pá-
ros.
1721. Nem lehet. Tekintsük például a piros színű éleket! Ha három piros él van, ak-
kor az egyik csúcsból nem indul piros él; ha a piros élek száma négy, akkor
valamelyik csúcsból két piros él indul.
1722. Ha lehetne, akkor a 0 szám kilenc másikkal lenne párban. Ha azonban négy
csúcsban szerepel a 0, akkor csak 8 szomszédja van; ha pedig öt csúcsban sze-
repel a 0, akkor 10 másik számmal lesz párban.
1723. Tekintsük az öt vonalon álló számok összegét; s tegyük fel, hogy ezek minde-
gyike páratlan! Ekkor az öt számot összeadva páratlan összeget kapunk; azonban
ez az összeg a körökben álló számok összegének kétszerese, tehát páros. Ellent-
mondásra jutottunk; ennek oka, hogy hibás volt feltételezésünk.
1724. Több különböző elrendezés is lehetséges.
1725. Nem lehet.
1726. Nem lehet. Adjunk össze minden három szomszédos csúcsban levő számot, s szá-
moljuk ki ezek összegét. Ha a háromtagú összegek mindegyike 13-nál nagyobb,
akkor ezek összege legalább 8 · 14 = 112, azonban ez az összeg: 3 · (1 + 2 + 3 +
+ · · · + 8) = 108.
1727. Nem lehet. Ha összeadjuk a csúcsokba összefutó éleken levő összegeket — s ha
azok egyenlők —, akkor a kapott összeg osztható 8-cal; azonban ez az összeg
2 · (1 + 2 + · · · + 12) = 2 · 6 · 13, de ez nem osztható 8-cal.
1730. Ha összeadjuk a 77 kéttagú összeget, akkor a gondolt számok összegének két-
szeresét, tehát páros számot kapunk, s ez nem lehet 77 · 1991.
1731–1734. Hasonló az előzőkhöz.
1735. Ha a kiválasztott érme valódi, akkor maradt 5 hamis és 5 valódi. Ebből a 10
érméből 5 db-ot az egyik, 5-öt a másik serpenyőbe teszünk. Ha a bal serpenyőben
k db hamis érme volt, akkor a jobb oldaliban 5 − k. A két oldal közti különbség
(5 − k) − k = 5 − 2k gramm, s ez a szám páratlan. Ha hamis pénzt választottunk,
akkor az előbbi különbség 4 − 2k, azaz páros szám.
1736. a) Nem lehet. A mondott összeg valójában a táblázatban levő számok összegé-
nek (ami 625 db páratlan szám összegeként páratlan) kétszerese.
b) Nem lehet.
1. megoldás. Legyen a táblázatban mindenhol +1; számoljuk ki a mondott össze-
get, majd változtassuk a +1-eket −1-ekre, s figyeljük az összeg változását!
29. Különféle kombinatorikai feladatok 335

2. megoldás. Az 50 szám szorzata +1, hiszen ez megegyezik a táblázatban levő


számok szorzatának négyzetével. Tehát az 50 szám között páros sok −1 van;
míg az 50 szám összege csak akkor lehetne 0, ha közöttük 25 db +1 és 25 db
−1 lenne.
1737–1738. Az előző b) feladat mindkét megoldása követhető.
1739. Nem igaz. Bármely n-re (n ≥ 4) adható ellenpélda. Ha n páros, akkor vegyünk
egy (n − 1)-oldalú sokszöget, ez lesz egy gúla alaplapja, s írjunk a gúla minde-
gyik csúcsába −1-et. Ha n páratlan, akkor vegyünk két, egybevágó (n − 2)-szög
alapú gúlát, ezeket alaplapjuknál illesszük össze! Az egyik csúcsra +1-et írunk,
a másikra és az „alaplapon” levő csúcsokra −1-et.
1740. Kezdetben minden élre +1-et írunk. Egy csúcsot nevezzünk +1 vagy −1 csúcs-
nak aszerint, hogy a vizsgált szorzat +1 vagy −1. Megadunk egy eljárást, mely
kettővel növeli a −1 csúcsok számát: válasszunk ki két +1 csúcsot; jelöljünk ki
egy, a két csúcsot összekötő, a poliéder egymáshoz csatlakozó éleiből álló út-
vonalat, s ezeknek az éleknek mindegyikén változtassuk meg az ott álló szám
előjelét!
1741. Követhető az 1736. b) feladat mindkét megoldása.
1742. b) Ha n = 4k, akkor a lefedés megvalósítható. . .
Ha n páratlan, akkor még 1 × 2-es dominók- −1 .. +1 +1 +1 .. −1
...
kal sem lehet lefedni a táblát. Ha n páros, de .
−1 −1 −1 −1 .. +1
nem osztható 4-gyel, akkor a dominók száma ... ...
.
páratlan, így a számok szorzata negatív. +1 +1 −1 +1 .. −1
...
. .
+1 .. −1 +1 −1 .. +1

1743. A szorzat kitalálásához legalább 50 kérdést kell feltenni.


1744–1745. Válasszuk ki a számok közül az egyik legkisebbet; látható, hogy ennek
szomszédai egyenlők ezzel a számmal. Hasonló okoskodással kiderül, hogy min-
den szám egyenlő egymással.
1746. Nem lehet. Az 1, 2, 3, 9, 10 számok között nem lehet szomszédos, így ezek
minden második helyet foglalnak el valamilyen sorrendben. A 8-nak nincs helye,
csak a 3 mellett állhat, a többi nem lehet szomszédja.
1747. Nem lehet. Az 1, 2, 3, 11, 12, 13 számok nem lehetnek szomszédosak. E hat
szám közé kell helyezni a maradék hetet. Figyeljük a 4 és 10 számokat! Ezeknek
a fenti hat számból csak egy-egy szomszéd jut, ezért ezek ketten szomszédosak
lesznek (a hét számból kettőnek szomszédosnak kell lennie).
1748. Igen, lehet. Például: −9, 5, 5, −9, 5, 5, . . . , −9, 5, 5, −9. Egy kör kerületén nem
lehetséges ez a felírás, mert akkor a 25 szám összegének egyidejűleg pozitívnak
és negatívnak is kellene lennie.
336 Megoldások, útmutatások

1749. Nem lehet. a1 + a2 + · · · + a34 > 0 és a1 + a2 + a3 + · · · + a30 < 0 miatt a31 + a32 +
+ a33 + a34 > 0.
Ugyanígy a35 + a36 + a37 + a38 > 0, . . . , a47 + a48 + a49 + a50 > 0, melyekből
a31 + · · · + a50 > 0, azonban az egyik feltétel szerint ez az összeg negatív.
1750. Egy 8-tagú sorozat: −1, −1, −1, 2, 2, −1, −1, −1.
Egy 9-tagú sorozat: −1, −1, −1, −1, 5, −1, −1, −1, −1.
Egy 10-tagú sorozat: 5, −7, 5, −7, 5, 5, −7, 5, −7, 5.
A sorozat nyilván nem lehet 5 · 7 = 35 elemű. Ugyanis ekkor a sorozat elemeit
„szakaszoljuk” 5-ösével (az első öt elem, a következő öt, . . . ), és így összeadva a
számokat az eredmény pozitív lesz. Ha ugyanezt 7-esével tesszük meg, akkor az
összegzésben negatív számokat adunk össze, az eredmény negatív. Ugyanannak
a 35 számnak az összege nem lehet pozitív is és negatív is.
Tegyük fel, hogy a sorozat 11 tagú és elemei: a1 , a2 , a3 , . . . , a11 . Rendezzük el
a sorozat elemeit az alábbi módon:
a1 a2 a3 a4 a5 a6 a7
a2 a3 a4 a5 a6 a7 a8
a3 a4 a5 a6 a7 a8 a9
a4 a5 a6 a7 a8 a9 a10
a5 a6 a7 a8 a9 a10 a11

A táblázat elemeit soronként összeadva, a feltétel szerint negatív számokat adunk


össze (minden sorban negatív a hét szám összege), a táblázatban levő számok
összege emiatt negatív lesz. Ha oszloponként adjuk össze a számokat, akkor po-
zitív részösszegeket adunk össze, így most a táblázat elemeinek összege pozitív
lett. Az ellentmondás oka, hogy feltettük: megadható 11-tagú sorozat a kívánt
feltételek szerint.
1751. A hat szám szorzata = −(a11 a12 . . . a33 )2 nempozitív, ezért van közöttük nempo-
zitív szám.
1752. A sokszög csúcsai között 6 különböző távolság mérhető, míg az 5 pont 10 távol-
ságot határoz meg.
 
10
1753. A 10 számból = 45 kéttagú összeg képezhető. A megadott számokból
2
készíthető legnagyobb kéttagú összeg 39.
1754. Nem lehet. A 100 főből válasszunk ki egyet. Ha a kívánt módon megszervezhető
lenne az ügyelet, akkor a megmaradt 99 fő diszjunkt párokba lenne osztható.
29. Különféle kombinatorikai feladatok 337

1755. 1 + 3 + 3 · 2 = 10. Tehát a maximum 10. Ez — ahogyan az ábra mutatja — el is


érhető.

vagy

1756. Válasszuk ki a sokszög egyik csúcsát, s innen indulva járjuk végig a sokszög ke-
rületét. Ha vízszintes oldalon indulunk el, akkor egymás után vízszintes–függő-
leges, vízszintes–függőleges, . . . , vízszintes–függőleges oldalpárokon haladunk.
(A kiinduló csúcsba nyilván függőleges állású oldalon érkezünk meg.)
1757. Figyeljük a kocka belsejében levő 1 × 1 × 1-es kockát. Ezt csak úgy tudjuk
kifaragni a 3 cm élű kockából, ha az 1 cm élű kocka mindegyik lapja mentén
vágunk. Emiatt szükség van a 6 vágásra.
1758. Mivel a kocka konvex, egy vágással két részre tudjuk szétvágni, vagyis minden
egyes vágással legfeljebb megkétszerezhetjük az addigi darabok számát. Ez azt
jelenti, hogy 6 vágásra mindenképpen szükségünk van, mert 5 vágás után legfel-
jebb 25 = 32 darabot kaphatunk. A 6 vágás elegendő, ilyen megoldást nem nehéz
találni.
1759. A szalagot legfeljebb 21 részre vághatjuk szét.
1760. Az első, a második, . . . , a tizedik lépcsőfokra rendre 1, 2, 3, 5, 8, 13, 21, 34,
55, 89-féleképpen lehet feljutni. Hiszen mondjuk az ötödik lépcsőfokra vagy a
negyedikről vagy a harmadikról érkezhetünk, azaz az ötödikre vivő utak száma
megegyezik a negyedikre és a harmadikra vivő utak számának összegével. Az 1,
1, 2, 3, 5, . . . sorozatot Fibonacci-sorozatnak nevezik. A sorozat minden eleme
— a harmadikkal kezdődően — az előző két elem összege.
1761. Nézzük az egyszerűbb eseteket! A 2 × 1-es téglalapot 2 × 1-es dominókkal 1-
féleképp, a 2×2-es téglalapot 2-féleképp és a 2×3-as téglalapot 3-féleképp lehet
kirakni.
A téglalap kirakásainak száma következik a kisebb téglalapok kirakásaiból. Pél-
dául egy 2×6-os téglalap kirakása származtatható úgy, hogy egy 2×5-ös téglalap
mögé állítva tesszük a 2 × 1-es dominót, ill. egy 2 × 4-es téglalap mögé fektetve
teszünk 2 db 2 × 1-es dominót. (Ha állítva tesszük, azt már megszámoltuk ak-
kor, amikor a 2 × 5-ös mögé állítva tettünk egy dominót.) Így minden kirakást
megszámolunk, és minden kirakást egyszer számolunk.
Arra a megállapításra jutunk, hogy a 2 × n-es téglalap kirakásainak száma mege-
gyezik a 2×(n−1)-es és a 2×(n−2)-es téglalap kirakásai számának összegével.
Így a 2 × n-es téglalap kirakásainak száma n = 1, 2, 3, 4, 5, 6, 7, 8, 9, 10 esetén
rendre 1, 2, 3, 5, 8, 13, 21, 34, 55, 89. (Ez a Fibonacci-sorozat.)
338 Megoldások, útmutatások

Tehát a 2 × 10-es téglalapot 2 × 1-es dominókkal 89-féle módon lehet kirakni.


1762. A kívánt tulajdonságú 0–1 sorozatok számát állapítsuk meg, ha a sorozat 2, 3, . . .
számból áll. A 2 számból álló sorozatok: 00, 01, 10; 3 számból álló sorozatok:
000, 100, 010, 001, 101. A 4-jegyű sorozatok mindegyike megkapható úgy, hogy
a 2-jegyű sorozatokat 01-gyel egészítjük ki, ill. a 3-jegyű sorozatok végére 0-t
írunk. Emiatt a 4-jegyű sorozatok száma:
3 + 5 = 8. Hasonlóan kapjuk meg pl. az 5-jegyű sorozatokat a 3- és 4-jegyű
sorozatokból. Így a 2, 3, 4, 5, 6, 7, 8-jegyű sorozatok száma rendre: 3, 5, 8, 13,
21, 34, 55.
Tehát 55 kívánt tulajdonságú sorozat van.
1763. Vizsgáljunk egyszerűbb eseteket! 9 kavicsot 3 halomba, 16-ot 4, 25-öt 5 kupac-
ba. Az {1, 3, 5}, {1, 3, 5, 7}, {1, 3, 5, 7, 9} felosztásokhoz jutunk. Hasonló módon
kapjuk a keresett felosztást is.
Az 1, 3, 5, . . . , 199 szétosztás megfelel a feltételeknek. 1 + 3 + · · · + 199 =
(1 + 199)
= · 100 = 10 000. Ha valamely kupacot két részre osztjuk, akkor az
2
egyikben páros, a másikban páratlan számú kavics lesz. Mivel az első száz pá-
ratlan szám adja a kupacokban levő kavicsok számát, emiatt a két részre osztással
keletkezett, páratlan számú kavicsot tartalmazó halom valamely másik halommal
egyező számú kavicsot tartalmaz. Így a szétosztás teljesíti a kiírt feltételeket.
1764. Akár a megnyert, akár az elvesztett játszmák számát adjuk össze, eredményül a
lejátszott mérkőzések számát kapjuk. Ezért:
0 = (x1 +x2 +· · ·+x10 )−(y1 +y2 +· · · +y10 ) = (x1 −y1 )+(x2 −y2 )+· · ·+(x10 −y10 ).
Mindenki egyformán 9 mérkőzést játszott, tehát
x1 + y1 = x2 + y2 = · · · = x10 + y10 = 9.
Ezekből: 0 = 9(x1 − y1 ) + 9(x2 − y2 ) + · · · + 9(x10 − y10 ) = (x1 + y1 )(x1 − y1 ) +
+(x2 + y2 )(x2 − y2 )+· · ·+(x10 +y10 )(x10 −y10 ) = (x12 −y12 )+(x22 −y22 )+· · ·+(x10
2
−y10
2
).
Ezt kellett igazolnunk.
1765. Tegyük fel az ellenkezőjét. Ez azt jelenti, hogy minden percben minden csa-
ládban pontosan egy lovagnak volt aranyserlege, hiszen az aranyserlegek és a
családok száma megegyezik.
13 perc alatt mindenki kezében minden aranyserleg pontosan egyszer volt, azaz
mindenki kezében pontosan k percen át volt aranyserleg. Valamelyik családnak
legyen n lovagja. Mivel minden percben pontosan egy lovag kezében volt közü-
lük aranyserleg és minden lovag k percig tartott aranyserleget, így nk = 13. De
a feltétel szerint 1 < k < 13, s ez 13 prímszám voltának mond ellent.
Ha a lovagok r száma osztható k-val, azaz r = kn, és minden családból pontosan
n lovag ül az asztalnál; akkor könnyű olyan elrendezést mutatni, hogy minden
percben minden család valamely tagjánál legyen aranyserleg. Ehhez elég pl. az
egy családba tartozókat egymás mellé ültetni, az aranyserlegeket pedig egy sza-
bályos k-szög csúcsaiban elhelyezni a lovagok között.
29. Különféle kombinatorikai feladatok 339

1766. Nevezzük V -lovagnak azokat, akiknél éjfél előtt vörös bor volt, és tekintsük a V -
lovagok jobb oldali harmadszomszédjait. Nem lehet, hogy minden V -lovag jobb
oldali harmadszomszédja V -lovag legyen, hiszen ekkor V -lovagtól V -lovagig
lépkedve, az asztalt háromszor körüljárva végül is a társaság minden tagjához
eljutnánk, és a társaságban csak V -lovag volna. Van tehát legalább egy olyan
V -lovag, akinek a jobb oldali harmadszomszédjánál éjfél előtt fehér bor volt.
Éjfélkor ez a harmadszomszéd is, a V -lovag is a V -lovag jobb szomszédjának
nyújtja serlegét, ez a lovag tehát két serleget kap. Így van olyan tagja is a társa-
ságnak, akinek nem jut serleg.
1767. A sorok és oszlopok közül válasszunk ki egy olyat, amelyikben a számok össze-
ge a legkisebb. Legyen ez valamelyik sor, és ebben a számok összege k. Ebben a
sorban legalább n − k db 0 áll. Tekintsük azokat az oszlopokat, amelyek ezeket a
nullákat tartalmazzák. Egy-egy ilyen oszlopban a számok összege a feladat felté-
tele miatt legalább n − k. A táblázatban levő számokat oszloponként összegezve
n2
n2 n
2 n2 n2
S ≥ (n − k)2 + k 2 = 2 k 2 − kn + + =2 k− + ≥ .
4 2 2 2 2
1768. A 12. sorozatban a k-adik helyen álló számjegyet jelölje xk , melynek értéke vagy
0, vagy 1. Legyen − →xk = 1 − xk . x1996 = x1024+972 = −→=x =−
x972 460
→=x =
x204 76
−→
= x12 = 0.
1769. Megadunk egy eljárást, amely a kártyákat 50 dobozba teszi.
Minden kártyáról töröljünk le egy jegyet úgy, hogy a megmaradt 2 számjegy
összege páros legyen, majd tegyük a kártyát abba a dobozba, amelynek száma
az így kapott kétjegyű szám. Ezt a törlést minden kártya esetén megtehetjük (mi-
ért?). Mivel a 00, 01, . . . , 99 számoknak pontosan a felében páros a számjegyek
összege (ugyanis egyértelmű megfeleltetés létesíthető a páros és a páratlan szám-
jegyösszegű számok között: (a, b) ↔ (9 − a, b), ahol (a, b) olyan szám, melynek
első jegye a, második b), ezért valóban legfeljebb 50 dobozba tettük a kártyákat.
Bebizonyítjuk, hogy kevesebb doboz nem elég.
Tegyük fel, hogy a kártyákat a feltételeknek megfelelően betettük a dobozokba.
Osszuk a 100 db dobozt 10 db 10-es csoportba úgy, hogy minden csoportban az
első számjegyek azonosak legyenek (mint pl. 00, 01, . . . , 09). Tekintsük azt az
a0, a1, . . . , a9 csoportot, amelyben a legtöbb, k db üres doboz van. Legyenek
ezek sorszámai aa1 , aa2 , . . . , aak . Figyeljük meg, hogy ekkor az összes aai aj
(i, j = 1, 2, . . . , k és i, j lehet egyenlő is) számú kártya csak az ai aj dobozba
tehető, azaz az összes ai aj doboz nem üres. Ilyen ai aj feliratú dobozból nyilván
k 2 db van.
Tekintsük most azokat a csoportokat, amelyekben a felirat első jegye nem ai
(i, j = 1, 2, . . . , k), ilyen csoport (10 − k) db van. Ezekben a csoportokban —
mint minden csoportban — legalább (10 − k) db nem üres doboz van, hiszen k
megválasztása miatt k-nál több üres doboz nem lehet egy csoportban sem. Tehát
ezekben a csoportokban összesen legalább (10 − k)2 db nem üres doboz van.
340 Megoldások, útmutatások

Az előbb talált k 2 és a most talált (10 − k)2 nem üres doboz közt nincs átfedés,
hiszen első jegyeik biztosan különböznek. Ezzel beláttuk, hogy összesen lega-
lább k 2 + (10 − k)2 nem üres doboz van. Ez nem lehet kisebb 50-nél, hiszen
k 2 + (10 − k)2 = 2(k − 5)2 + 50.
1770. 7 lépésben leszedhetők a kavicsok. Első lépésben a 63-nál több kavicsot tar-
talmazó halmokból elveszünk 64-et. A kupacokban legfeljebb 63 kavics marad.
Másodjára, ahonnan lehet, elveszünk 32 kavicsot. Ekkor egy-egy halomban leg-
feljebb 31 kavics lehet. Ezután 16 kavicsot vegyünk el a megfelelő halmokból.
Ekkor mindenhol legfeljebb 15 maradhat. A következő lépésekben 8-at, 4-et, 2-t,
végül 1-et veszünk el. 6 lépésben a leszedés nem valósítható meg. Tegyük fel,
hogy mégis. Rendeljünk minden kupachoz egy 6 elemű 0–1 sorozatot: aszerint
áll 1 vagy 0 az i-edik helyen, hogy az i-edik elvételkor vettünk-e a kupacból
vagy sem. Két különböző kupachoz különböző 0–1 sorozat tartozik. Azonban a
6 elemű 0–1 sorozatok száma 64, miközben 100 kupac van.
1771. (1, 0, 1, 0, 1, 0, 1, 0, 1) + (0, 2, 2, 0, 0, 2, 2, 0, 0) + (0, 0, 0, 4, 4, 4, 4, 0, 0) +
+(0, 0, 0, 0, 0, 0, 0, 8, 8) = (1, 2, 3, 4, 5, 6, 7, 8, 9). Tehát 4 sorozattal elvégezhető
a kívánt előállítás.
Megmutatjuk, hogy 3 sorozattal nem oldható meg a feladat. Ugyanis ekkor az
„összeg” elemei csak 8-félék lehetnek, míg a megadott sorozatnak 9 különböző
eleme van: 1, 2, . . . , 9. Azért csak 8-féle, mert egy-egy elemet úgy kapunk
az összegben, hogy az első sorozat egyik vagy másik elemét vesszük, ehhez
hozzáadjuk a második sorozat egyik vagy másik elemét, majd ehhez adjuk a
harmadik sorozat egyik vagy másik elemét: ez összesen 2·2·2 = 8-féle lehetőség.
1772. Igen, mindig lehetséges. Állítsuk a 100 számot 50 párba úgy, hogy minden pár-
ban 101 legyen a két szám összege. A kiválasztott 25 számmal ezekből a szám-
párokból k db pár mindkét tagját kivettük; a további 25 − 2k számhoz válasszuk
ki a számokból a párokban hozzárendelt társait, és a még háborítatlan párokból
válasszunk ki k darabot. Így a 25 szám egy része párokba állt, a többit kiegészí-
tettük a párjával, s újabb számpárokat is választottunk. Ily módon 25 számpárt
vettünk ki, s ennek az 50 számnak ugyanannyi az összege, mint a megmaradó
50 számnak.
1773. A 100 mérősúlyt foglaljuk 50 párba úgy, hogy egy-egy párba két szomszédos
súly kerüljön. 25 párból a súlyokat rakjuk szét két kupacba: a könnyebbet balra, a
nehezebbet jobbra. A megmaradó 25 párban levő mérősúlyokat pedig úgy rakjuk
szét, hogy a könnyebbet jobbra, a nehezebbet balra tesszük. Ezzel a mérősúlyok
megfelelő szétosztásához jutottunk.
1774. Az alábbi eljárással k mérés után el tudjuk dönteni, hogy vannak-e különböző
súlyú érmék; és ez utóbbi esetben ki is tudunk jelölni majd egy könnyebbet és
egy nehezebbet.
Osszuk el az érméket két egyenlő számú csoportra, és tegyük a két részt a két
serpenyőbe. Ha a mérleg egyensúlyban van, akkor az egyik részt tegyük félre, a
29. Különféle kombinatorikai feladatok 341

másikkal pedig ismételjük meg az eljárást, azaz ismét felezzük meg és hasonlít-
suk össze a két részt. Folytassuk ezt mindaddig, amíg a mérleg ki nem billen.
Ha erre a k-adik mérésben sem kerül sor, akkor az utolsó, k-adik mérésre 2
érme bizonyult egyenlő súlyúnak. Ebben az esetben minden érme egyenlő súlyú.
(Miért?)
Ha az l-edik mérésben a mérleg kibillen, és l = k; akkor a serpenyőkben 1–1
érme maradt, így a kezünkben van egy könnyebb és egy nehezebb érme. Ha
l < k, akkor a következő méréstől kezdve módosítjuk eljárásunkat.
Tegyük fel, hogy a bal oldali serpenyőben levő érmék a nehezebbek. Mindegyik
serpenyő tartalmát két egyenlő számú érméből álló csoportra osztjuk, s méréssel
összehasonlítjuk a bal és a jobb oldali serpenyő tartalmának egyik-egyik felét
(az érmék megmaradó részét tegyük félre). Ha most is a bal oldali rész a ne-
hezebb, akkor a most összehasonlított két részen ismételjük meg a leírt eljárást.
Ha egyensúly van, vagy a bal oldali a könnyebb, akkor az eljárást az utóbbi
mérésnél félretett két részen folytatjuk. Ezen a módon nem veszítjük szem elől
a különböző súlyú érméket, az eljárás végén fel tudunk mutatni két különböző
súlyú érmét.
1777. 12 golyó közül is kiválasztható a 2 radioaktív golyó hét méréssel. Készítsünk
három, 4–4 golyóból álló csoportot, s mindegyiket mérjük meg (3 mérés). Ha
csak az egyik csoport radioaktív, abból további 4 méréssel (sőt 3 mérés is elég)
kiválasztjuk a keresett 2 golyót. Ha két csoportot is radioaktívnak találunk, akkor
az azokban levő 1–1 radioaktív golyót 2–2 méréssel könnyű megtalálni.
1778. Legyen az elsőnek fölrakott, legnehezebb súly tömege M gramm. Az eljárás so-
rán a serpenyők eltérése soha nem haladhatja meg az M grammot, hisz ha éppen
nincs egyensúly; akkor az újabb, legfeljebb M grammos súlyt a könnyebbik ser-
penyőbe kell tennünk. Ez igaz akkor is, amikor már valamennyi, 1 grammnál
nehezebb súly a mérlegre került. Azt állítjuk, hogy 1 grammos súlyaink száma
legalább M, vagyis ezekkel a mérleg kiegyensúlyozható.
Valóban, ha az 1 grammos súlyok számát k-val jelöljük, akkor az egy darab M
és a k darab 1 grammos súlyon kívül minden további legalább 2 grammos, így
1 · M + k · 1 + (101 − k − 1) · 2 ≤ 200, ahonnan M ≤ k.
Miután így alkalmas számú 1 grammos súlyt a könnyebbik serpenyőbe téve
egyensúlyba kerül a mérleg, a rajta levő súlyok összege páros, így az esetleg
fennmaradó — lehetséges, hogy valamennyi — 1 grammos súlyok száma is pá-
ros, a súlyok tömegének összege ugyanis 200 gramm. A megmaradt 1 grammos
súlyokat viszont ezután a feladat előírása szerint felváltva tesszük a bal, illetve
a jobb oldali serpenyőbe, tehát az eljárás végén a mérleg valóban egyensúlyban
lesz.
1779. Ha minden egyes kupacban 2 kavics van, akkor a feladat állítása nyilvánvaló,
hisz 25–25 kupacot egy-egy csoportba gyűjtve mindkét csoportban 50 kavics
lesz.
342 Megoldások, útmutatások

Ha a kupacok között vannak különböző elemszámúak, akkor jelölje a1 , a2 , . . . ,


a50 az egyes kupacokban levő kavicsok számát, és tegyük fel, hogy például
a1 = a2 .
Tekintsük ezután az a1 , a2 , a1 + a2 , a1 + a2 + a3 , . . . , a1 + a2 + · · · + a50 szá-
mokat. Ezen 51 szám között feltétlenül van kettő, b1 és b2 , amelyek ugyanazt a
maradékot adják 50-nel osztva. Mivel a felsorolt számok különbözők (a1 = a2 és
ai > 0), továbbá mindegyikük 1 és 100 között van, ez csak úgy lehetséges, ha
|b1 − b2 | = 50. Ez a két szám így nem az a1 és az a2 , hisz az ai -k között nincs
50-nél nagyobb. A felsorolt 51 szám közül viszont bármely további kettőnek a
különbsége különböző ai -k összege. Ha pedig ilyen számok összege 50, akkor a
megfelelő kupacokat egybegyűjtve, a kapott csoportra — és így a kimaradókból
összegyűjtött másikra — teljesül a feladat állítása.
1781. Az általánosság csorbítása nélkül föltehetjük, hogy a kezdetben utolsó helyen
álló érme aranyból van. Legyen az ezüst érmék kezdeti sorszámainak összege
S. Az átrendezés folyamán minden ezüst érme 1-gyel nagyobb sorszámú helyre
kerül (hiszen az utolsó helyen nem ezüst áll), így az ezüst érmék sorszámainak
összege (S + k)-ra változik.
Ha az átrendezés során ezüst érmét mindig csak ezüsttel cserélnénk fel, akkor
a fenti összeg nem változhatna. Az átrendezés folyamán ezért olyan lépésnek
is kell lennie, amikor ezüst érmét arannyal cseréltünk fel, és éppen ezt akartuk
bizonyítani.
1782. A zárójelezéssel elérhető legkisebb eredmény 1 − (2 + 3 + 4 + · · · + 1999), a
legnagyobb pedig 1−(2−3−4−· · ·−1999). Ezek között a határok között lehetnek
a végeredmények, továbbá tudhatjuk, hogy az eredmények páros számok (hiszen
999 páros és 1000 páratlan számot adunk össze).
Számoljuk ki, hogy hányféle olyan zárójelezés van, ahol a zárójeleken belül
pontosan két tag van: minden (2k, 2k + 1) párnál vagy elhelyezünk zárójelet,
vagy nem. Mivel 999 ilyen pár van, ez 2999 zárójel elhelyezést jelent, és ezek
mind különbözők.
A számolások mutatják, hogy 2999 nagyobb, mint a zárójelezésekkel kapható
eredmények száma, ezért van két olyan zárójelezés, melyek ugyanazt az ered-
ményt adják.
1783. A sokszög valamely oldalával legfeljebb n − 2 átló párhuzamos. Ezért a valame-
lyik oldallal párhuzamos átlók száma legfeljebb 2n(n−2), míg a sokszög átlóinak
száma n(2n − 3), s n(2n − 3) > 2n(n − 2) = n(2n − 4), tehát valóban van kívánt
tulajdonságú átló.
1784. Egy adott színezésből átszínezésekkel nyerhető sakktáblák számára adunk egy
felső becslést. Az átszínezési eljárás során az oszlopok, sorok átfestésének sor-
rendje nem befolyásolja a végeredményt. Másrészt ugyanannak a sornak, ill.
oszlopnak kétszeri átfestése is elhagyható; ezért feltehetjük, hogy minden sort
és oszlopot legfeljebb egyszer festünk át. Az oszlopok és sorok együttes száma
29. Különféle kombinatorikai feladatok 343

24, így összesen 224 módon jelölhetünk ki átszínezési eljárást (a végeredményül


nyert táblaszínezések között persze lehetnek egyformák). Ugyanakkor a sakk-
tábla 144 mezője 2144 -féleképp színezhető két színnel. 2144 > 224 miatt vannak
olyan színezések, melyekből a kívánt sakktábla színezést nem lehet elérni.
1785. A táblázatba írt számok mindegyike helyett írjuk be, hogy az adott szám 10-zel
osztva milyen maradékot ad. (Feladatunk szempontjából elegendő csak a számok
tízes maradékát figyelni.) Így egy mezőbe 10-féle szám kerülhet, a 8 × 8-as
táblázat 1064 -féle különböző módon tölthető ki.
A megadott műveletekkel hányféle különböző műveletsorrend készíthető? A táb-
lának 36 db 3 × 3-as része van, bármelyik ilyen részben az 1-gyel növelésre 10
(lényegesen) különböző lehetőségünk van. Összesen 1036 -féleképpen változtat-
hatunk meg egy táblázatot. A 4 × 4-es részekből 25 van a táblán, ezekkel 1025 -
féleképpen változtathatunk. A kétfajta művelettel egy adott táblázatból kiindulva
összesen (legfeljebb) 1036 · 1025 = 1061 különböző táblázathoz juthatunk.
1061 < 1064 , azaz az elérhető lehetőségek száma kisebb, mint a kitöltések száma;
emiatt a feladat kérdésére tagadó a válasz.
1786. A diákok száma n. Őket a két terembe 2n -féleképp lehet szétosztani. A tanulók
két teremben való szétosztásai között vannak „rossz” elhelyezések, amelyekben
valamelyik (vagy több) csapatnak minden tagja ugyanabba a terembe kerül. Ha
a rossz elhelyezések száma kisebb, mint 2n ; abból következik, hogy van jó elhe-
lyezés is.
Tekintsünk egy tetszőleges csapatot, és számítsuk ki, hogy hány olyan rossz
elhelyezés van, amikor ennek a csapatnak minden tagja ugyanabba a terembe
kerül. Először el kell döntenünk, hogy melyik ez a terem; ez 2 lehetőség. Ezután
a többi n − 10 embert kell szétosztanunk, amit 2n−10 -féleképp tehetünk meg.
Összesen tehát 2 · 2n−10 = 2n−9 olyan rossz elhelyezés van, amikor egy kiszemelt
csapat minden tagját ugyanabban a teremben helyeztük el.
500 csapat lévén az összes rossz elhelyezés száma legfeljebb 500·2n−9 . (Lehetnek
esetek, melyeket többször is megszámoltunk.)
500 n
Mivel 500 · 2n−9 = · 2 < 2n , ezért a feladat állítása igaz.
512
1787. A bástyák olyan elhelyezéseinek száma, amikor nem ütik egymást (a színezéssel
nem törődünk): n!.
A bástyák olyan elhelyezéseinek száma, amikor nem ütik egymást, s van legalább
n2
két azonos színű mezőn álló bástya: (n − 2)! · (hiszen valamelyik színt kivá-
2
lasztva, arra a két mezőre két bástyát helyezünk, a többit pedig (n − 2)!-féleképp
helyezhetjük el).
n2
Az állítást igazoltuk, mivel n! > (n − 2)! · .
2
344 Megoldások, útmutatások
 
3n + 1
1788. A társaság 3 fős csoportjainak száma . Az olyan hármasokat számoljuk
3
össze, melyeknek tagjai legfeljebb kétféle játékot játszanak
  egymás között. A
n
társaság egy A tagja n emberrel pingpongozik, így olyan hármasban van
2
benne, amelyikben mind a két másik tagjával   pingpongozik. Ugyanezt kapjuk a
n
sakk és a tenisz esetén is. Ez összesen 3 · . Ugyanez igaz a társaság bármely
2
tagjára, tehát a „rossz” hármasok
  száma legfeljebb (hiszen egyes hármasokat
n
többször is számolhattunk) 3 · (3n + 1).
2
Számolással ellenőrizhető, hogy az összes hármas száma nagyobb a „rossz” hár-
masok számánál, tehát van „jó” hármas.
Megjegyzés. Ehhez a feladathoz kapcsolódik az 1805. feladat.
1789. Az 1, 2, . . . , 1986 számok 21986 -féleképpen színezhetők két színnel. A „rossz”
színezések (amelyekben van egyszínű, 18 elemű számtani sorozat) számára fel-
ső becslést adunk. Jelölje S a különböző, 18 elemű számtani sorozatok számát.
S kiszámítása: 1 különbségű, 18 elemű számtani sorozat 1986 − 17 van; 2 kü-
lönbségű ilyen sorozat 1986 − 2 · 17 van; 3 különbségű ilyen sorozat 1986 −
− 3 · 17; 4 különbségű ilyen sorozat 1986 − 4 · 17; . . . ; 116 különbségű ilyen
sorozat 1986 − 116 · 17 van; 117 különbségű ilyen sorozat nincs, amelynek
elemei az 1, 2, . . . , 1986 számok közül valók. Ebből következik, hogy az
összes különböző, 18 elemű számtani sorozat száma: (1986 − 17) + (1986 −
−2 · 17) + (1986 − 3 · 17) + · · ·+ (1986 − 116 · 17) = 115 014. Egy „rossz” színezést
kapunk, ha veszünk egy 18 elemű számtani sorozatot; azt kékre (vagy pirosra)
színezzük, a többi 1968 számot pedig tetszőlegesen. Ezért a „rossz” színezések
száma legfeljebb 2 · S · 21968 = 21969 · 115 014. Mivel 21969 · 115 014 < 21986 ,
azaz 115 014 < 217 = 131 072, ezért az összes „rossz” színezés száma kisebb az
összes színezés számánál, így van „jó” színezés is.
1790. Itt is határozzuk meg az összes színezés számát, s adjunk felső becslést a „rossz”
színezések számára.
1791. Megmutatjuk, hogy már 9 nyelv is kiválasztható a kívánt módon. Bármekkora
is a feladatban szereplő n, és bárhogyan veszünk is ki (n + 1)-et a szóban forgó
nyelvek közül, biztosan mindenki beszéli ezek valamelyikét, hiszen a választot-
takon kívül n-nél kevesebb nyelv van, és mindenki legalább n nyelvet beszél.
Az n < 9 esetben ebből már következik állításunk, hiszen veszünk (n + 1)-et
a szóban forgó nyelvek közül, és ha ez kisebb 9-nél, hozzájuk veszünk tetszés
szerint (8 − n) nyelvet.
Ha n ≥ 9, akkor tekintsük a 2n nyelvbőlkiválasztható
 9 elemű részhalmazokat.
2n
Nevezzük ezeket blokkoknak, a számuk . Mondjuk azt, hogy egy dolgozó
9
30. Konstrukciók 345

elront egy blokkot, ha a benne szereplő nyelvek egyikét sem beszéli. Készen va-
gyunk, ha megmutatjuk, hogy az összes dolgozó együttesen sem tudja elrontani
az összes blokkot.
 Mivel
 egy dolgozó legfeljebb n nyelvet nem beszél, egy dol-
n
gozó legfeljebb blokkot ronthat el. Ha mind különböző blokkot rontanak is
9  
n
el, ez összesen legfeljebb 500 · blokk.
    9
n 2n
Mivel 500 · < , így igaz az állítás.
9 9
1792. Legyen |X| = t. Ekkor az összes kétszínezések száma 2t . Azoknak a kétszínezé-
seknek a száma, amelyekre az n elemű Ai ∈ F egyszínűvé válik, nyilván 2t−n+1 .
Minden egyes Ai ∈ F esetén kidobva ezeket a rossz kétszínezéseket, összesen
legfeljebb |F | · 2t−n+1 kétszínezést dobtunk el. A maradék viszont pontosan a jó
kétszínezések halmaza, s ennek van eleme, hiszen 2t > |F | · 2t−n+1 .

30. Konstrukciók

1793. 10 korong nyilván nem tehető fel. 9 korongot fel tudunk helyezni a követke-
ző módon: az első korongot tetszőlegesen tesszük fel; ezután a következőt úgy
rakjuk fel, hogy oda érkezzen, ahonnan az előző korongot indítottuk; ily módon
tesszük fel a többit is.
1794. 1, 3, 9, 27.
1795. „Kieséses versenyt” szervezünk. A mérleg serpenyőibe egy-egy golyót teszünk,
s ezek súlyának összehasonlítása után a könnyebbet hagyjuk a mérlegen. Mindig
a következő golyót hasonlítjuk össze az addigi legkönnyebbel és a könnyebbet
hagyjuk a mérlegen. 99 mérés szükséges.
1796. A 100 golyót párokba állítjuk, s 50 méréssel két csoportba osztjuk: a párokból
a nehezebb a „nehezek” csoportjába, a másik a „könnyűek” csoportjába kerül.
Az 50 „nehéz” golyóból (az 1795. feladat szerint) 49 méréssel megtaláljuk a
legnehezebbet, s 49 méréssel a másik csoportban megtalálható a legkönnyebb.
Összesen 148 mérés.
1797. Párokba rendezzük a golyókat, és 50 méréssel a párokból kiválasztjuk a nehe-
zebbiket. Most ezeket rendezzük párokba, és 25 mérés után ezek közül is ki-
választjuk a nehezebbeket. Így folytatva 12, 6, 3, 2, majd végül 1 mérés után
megtaláljuk a legnehezebb golyót. (Összesen 99 mérés.) A második legnehezebb
golyó csak akkor eshetett ki, amikor a legnehezebb golyóval együtt került mér-
legre. A legnehezebb golyót 7 alkalommal tettük fel a mérlegre, az ekkor mért
7 golyó között van a keresett. E hét golyóból a legnehezebbet 6 mérlegeléssel
346 Megoldások, útmutatások

megtalálhatjuk. Összesen 99 + 6 = 105 mérés szükséges a két legnehezebb golyó


megtalálásához.
1798. Tegyünk a serpenyőkbe 3–3 érmét! A mérlegelés után tudjuk, hogy mely 3 érme
között van a hamis. Ebből a 3 érméből egyet-egyet tegyünk a serpenyőkbe, s
most már kiderül, hogy melyik a hamis.
1799.

I. m r s
1 2 3 4 5 6 7 8 9 10 11 12

II. m r s

1 2 5 3 4 6 9 10 11 1 2 3 1 2 5 3 4 6

III. m r s

1 2 7 8 3 4 9 10 12 1 9 10 3 4 7 8 2 1

1 6 2 8 7 3 5 4 9 11 10 12 12 10 11 9 4 5 3 7 8 2 6 1
N K N K K N K N N N N N K K K K K N K N N K N K

1800. Előbb valamelyik 64 utasból álló csoportot vizsgáljuk meg. Ezt követően tud-
juk, hogy melyik 64 személy között van a terrorista, ismét a csoport egyik felét
vizsgáljuk meg és így tovább. Összesen 7 vizsgálat elegendő.
1801. Tegyünk a mérlegre az első ládikóból 1 db érmét, a másodikból 2 db-ot, a har-
madikból 3-at, . . .
1802. Hármuk közül az egyik három részre osztja a zsákmányt, a másik kettő pedig
választ. Ha két különböző részt választottak, akkor elteszik a kiválasztott részt,
a megmaradó harmadik részt pedig a zsákmányt szétosztó rabló kapja. Ha mind-
ketten ugyanazt a részt választják, akkor a másik két rész egyikét odaadják a
zsákmányt szétosztó rablónak, és a megmaradó két részből álló zsákmányt a két
rabló egyike újra két részre osztja, a másik rabló pedig választ ezekből.
1803. Elegendő egyszer át- és visszaevezni. A 49 vezetékből sorbakapcsolva egyetlen
kábelt lehet készíteni úgy, hogy az egyik drótvéget rákötjük a fázisra, a többit
pedig páronként összekötjük, majd átevezünk. Fázisceruza segítségével a túlsó
oldalon össze tudjuk fűzni a vezetékeket egyetlen kábellé. Még a túloldalon,
30. Konstrukciók 347

az összefűzés sorrendjében sorszámozzuk a drótvégeket. Visszatérünk a folyó


innenső oldalára, megszakítjuk az összeköttetéseket, megtaláljuk az egymáshoz
tartozó drótvégeket.
1804. A csapatokat ábrázolják egy szabályos 17-szög csúcsai és középpontja! Egy oldal
és a vele párhuzamos összes átló, valamint a középpont és a kiválasztott oldallal
szemközti csúcs összekötése jelöl ki egy fordulót. Ezután ismételten forgassuk
el a szakaszrendszert.
1805. A 3n + 1 főből válasszunk egyet: X-et, a többieket osszuk szét n–n fős csopor-
tokra, a T , S, P halmazokba. Az X játékos a T halmazba tartozó játékosokkal
teniszezik, az S halmazban levőkkel sakkozik, a P -beliekkel pedig pingpongozik.
Ha két másik játékost veszünk, s azok ugyanabban a halmazban vannak, akkor
egymás közt is ezt a játékot játszák; ha különböző halmazban vannak, akkor a
harmadik játékot játszák. (Lássuk be, hogy ez jó konstrukció!)
1806. Van két sor, mely legalább 4 csillagot tartalmaz. A megmaradó két csillagot két
oszloppal lefedhetjük.
∗ ∗ ∗ ∗ ∗ ∗
∗ ∗ ∗ ∗ ∗ ∗ ∗
∗ ∗ ∗ ∗ ∗ ∗ ∗
∗ ∗ ∗ ∗ ∗ ∗ ∗ ∗

1807. Átjuthatnak 12 perc alatt. Adél és Bori átmegy (2 perc), visszajön Adél (1 perc),
átadja Csillának és Dórinak a lámpát, átmennek (5 perc), Bori visszajön a lám-
pával (2 perc), majd Adéllal együtt átmennek az alagúton (2 perc).
1808. A világ körüli útra készülő gépen kívül elég 2 másik gép. Ezek egymást követően
úgy fordulnak vissza, hogy üres tartállyal érkezzenek vissza a szigetre, majd
később tele tankkal az érkező gép elé mennek, hogy haza segítsék.
1809. A kutató átjuthat a sivatagon. 60 km megtétele után az egyik teherhordó feltölti a
kutató és a másik teherhordó készletét 4 napi adagra, és visszafordul. (Ha visszaér
a kiinduló állomásra, ott keveset pihen; majd készleteit felfrissíti és társa elé siet,
hogy hazasegítse.) A kutató a vele maradó teherhordóval megtesz még 80 km-
t; ekkor a teherhordó feltölti a kutató készletét, aki innen már eljut a céljához.
A második teherhordó itt fordul vissza, készlete segítségével eljut az első (a 60
km-es) elválási pontra, ahonnan társa segítségével hazatér.
1810. Három segédautóval megoldható az A autó átjuttatása a szomszédos városba.
1811. Lehetséges, ha mindenki az ábra szerint a belül álló
két személy valamelyikére lő.
∗ ∗
348 Megoldások, útmutatások

31. Teljes indukció

1812. A hiba oka az, hogy az indukciós lépésben felhasználtuk, hogy két egymás utáni
értékre igaz az állítás (az (n − 1) és (n − 2) értékekre), de az indukciós lépés
igazolásában csak egy n értékre (n = 1-re) ellenőriztük az állítást.
1813. Itt helyes az indukciós lépés igazolása. A hibát az indukciós feltevés ellenőrzé-
sénél követtük el: a bal oldal nem értelmezhető n = 1 esetén, csak n > 1-re.
1814. Az az okoskodás, ami a 3-ról átsegített a 4-re, alkalmazható az n-ről (n + 1)-re
való átmenetnél egy kivétellel: nem megy, mint ahogy nem is mehet az 1-ről
2-re való átmenetnél.
1821. Mutassuk meg, hogy an = 4an−1 − an−2 .
1842. Lássuk be teljes indukcióval, hogy
1 1 1 1
+ + ···+ =1− .
f1 f3 f2 f4 fn fn+2 fn+1 fn+2
1850. Egy kockát könnyű feldarabolni 8, 27, 64, . . . kisebb (egybevágó) kockára. Ha
egy kockát feldaraboltunk n kisebb kockára, akkor a kis kockák valamelyikét
8 kockára darabolva, az adott kockát máris n + 7 kockára daraboltuk. Tehát
elegendő 7 egymást követő egész szám esetén megvalósítani a darabolást, mert
azokról 7-esével haladhatunk felfelé.
1852. Bizonyítsunk teljes indukcióval és használjuk fel, hogy
k 2 − (k + 1)2 − (k + 2)2 + (k + 3)2 = 4.
1853. Indukció az egyenesek számára.
1857. Az indukciós feltevés: 2k | (k + 1)(k + 2) . . . (2k).
Az indukciós lépés: 2k+1 | (k + 2)(k + 3) . . . (2k + 1)(2k + 2) = 2(k + 1)(k + 2)(k +
+3) . . . (2k)(2k+1), s innen a feltevést felhasználva következik az állítás igazsága.
1858. Lásd a Skatulyaelv c. fejezet 1604–1606. feladatait!
1859. n = 1-re 2 jó szám. Az indukciós feltevés szerint az A n-jegyű szám 1 és 2
jegyekből áll, s többszöröse 2n -nek. Az indukciós lépésben megmutatjuk, hogy
vagy az 1A, vagy a 2A n + 1-jegyű szám osztható 2n+1 -nel. A két szám másképp
írva: 10n +A, ill. 2·10n +A. Ezekből 2n -t kiemelve, 5n +B, ill. 2·5n +B számokhoz
jutunk, melyek közül az egyik páros.
Más megoldás. Legyen k pozitív egész, és írjuk fel azokat az n-jegyű számokat,
amelyeknek minden jegye 1 vagy 2. Azt állítjuk, hogy ezek 2n -nel osztva csupa
különböző maradékot adnak (s ebből már következik az állításunk).
Legyen x és y két szám az n-jegyű, 1-esekből és 2-esekből álló számok közül.
Azt látjuk be, hogy 2n  |x − y. A különbség végén lehet néhány nulla (m db), s
az utolsó nemnulla számjegy vagy 1-es, vagy 9-es. Tehát x − y = p · 10m , ahol
p páratlan szám. Ezért x − y osztható 2m -nel, és 2m+1  |x − y. Mivel m < n, így
a különbség nem osztható 2n -nel.
31. Teljes indukció 349

1860. Az indukciós feltevésben szereplő egyenletnek legyen megoldása u és v. Ekkor


az
 indukciós lépésbenfelírtegyenlet megoldása 
1 1 1 1
(u − v); (7u + v) és (u + v); (7u − v) is.
2 2 2 2
A számpárok egyike páratlan számokból álló pár.
1861. Tegyük fel, hogy az 1, 2, . . . , 2n + 2 számokból ki tudunk választani n + 2
db számot úgy, hogy közülük egyik sem osztója valamely másiknak! Ekkor az
indukciós feltevéssel összhangban az 1, 2, . . . , 2n számokból legfeljebb n db
számot választottunk ki, tehát a kiválasztott számok között szerepelnie kell a
2n + 1 és a 2n + 2 számoknak; ám az indirekt feltevés miatt n + 1 nem lehet a
kiválasztottak között, hiszen akkor osztaná a 2n + 2 számot. Cseréljük ki a 2n + 2
számot az n + 1 számra, s hagyjuk el a számok közül a 2n + 1 számot! Az így
kapott n+1 db szám közül egyik sem osztója valamely másiknak, ami ellentmond
az indukciós feltevésnek, tehát hamis a kezdeti indirekt feltevés.
1862. Tegyük fel, hogy igaz az állítás n = 2k-ra! Tekintsük az indukciós lépést 2k +1-re
(a 2k + 2-re történő indukciós lépés triviális, hiszen ekkor az egyenlőtlenségben
csak a jobb oldalon levő szám növekszik):  
   2k + 1
p= p· p < 4k+1 < 4k+1 · 4k = 42k+1 .
k
 p≤k+1
p≤2k+1 k+1<p≤2k+1
Itt p < 4k+1 az indukciós feltevés miatt teljesül;
p≤k+1
  
2k + 1
p < < 4k pedig a binomiális együttható definíció szerin-
k+1<p≤2k+1
k
(k + 2) · (k + 3) · . . . · (2k + 1)
ti felírásából látható, hiszen a tört számlálójában
1 · 2 · ...· k
szerepelnek a kérdéses prímek, a nevező viszont ezek egyikével sem osztható.
1864. Indukcióval előbb az ak+1 −ak ≥ 2k , majd az ak ≥ 2k egyenlőtlenséget igazoljuk.
1
1865. ak2 = ak−1
2
+2+ 2
2
, azaz ak−1 + 2 < ak2 ≤ ak−1
2
+ 3. Indukcióval megmutatható,
ak−1
hogy 2n − 1 < an2 < 3n − 2, ha n ≥ 3,
tehát 14 < a100 < 18.
1866. Indukcióval lássuk be, hogy an3 > 3n, ha n ≥ 2.
√ k k
1867. A k + 1-re történő indukciós lépés: k + 1 − 1 = √ < √ ≤
k+1+1 k+1
k k √ √ √ k √
≤ ≤ √ = k < k + 1, innen k + 1 < 1 + = ak+1 < k + 1 + 1.
ak k ak
350 Megoldások, útmutatások

1869. a12 ≤ a1 − a2 < a1 , tehát a1 < 1.


1 1
2 1 1 1
a2 ≤ a1 − a12 = − a1 − ≤ < . Mivel f (x) = x − x 2 a (0; )
4 2 4 2 2
intervallumon szigorúan monoton növekvő, ez lehetővé teszi az indukciós lépést:
1
1 1 1
an+1 ≤ f (an ) < f = − 2 < .
n n n n+1
2
1876. 22 < 33 ; ezt követően könnyű megmutatni indukcióval, hogy a második szám a
nagyobb (hiszen nagyobb számot nagyobb kitevőre hatványozunk).
1877. Itt az első szám a nagyobb. Jelölje An az n db 3-asból álló hatványt, Bn−1 pedig
az n − 1 db 4-esből álló hatványt! Az An > Bn−1 állítás helyett az indukcióval
már bizonyítható erősebb An > 2Bn−1 egyenlőtlenséget lássuk be! Az indukciós
lépés:
An+1 = 3An > 32Bn−1 = 9Bn−1 = 2,25Bn−1 · 4Bn−1 > 2 · 4Bn−1 = 2Bn .
1878. Az egyenlőtlenségre Cauchy adott egy nagyon szép bizonyítást:
Mutassuk meg, hogy a pozitív A, B, C, D, . . . számok mértani közepe mindig
kisebb a számtani közepüknél, ha a számok között van két különböző. Legyen
az A, B, C, D, . . . mennyiségek száma n, s igazoljuk, hogy
 
A+ B +C + D + ... n
ABCD · · · < .
n
Először, n = 2 esetén nyilvánvalóan igaz, hogy
     
A+B 2 A−B 2 A+B 2
AB = − < ,
2 2 2
amiből sorra feltéve, hogy n = 4, n = 8, . . . , végül n = 2m , ezt kapjuk:
    
A+B 2 C+D 2 A+B +C+D 4
ABCD < <
2 2 4
 4  4
A+B+C +D E+F +G+H
ABCDEF GH < <
4 4
 
A+B +C +D+E+F +G+H 8
<
8
..
.
 
A+ B +C + D +... n
ABCD · · · < (1)
n
Másodszor, ha n nem tagja a 2, 4, 8, 16, . . . mértani sorozatnak, jelöljük a
sorozat egy n-nél nagyobb tagját 2m -mel, és legyen
A+ B +C + D + ...
K= ;
n
31. Teljes indukció 351

feltéve, hogy az (1) képletben az utolsó (2m − n) tag K-val egyenlő, a képlet ezt
adja:
 m
A + B + C + D + · · · + (2m − n)K 2
ABCD . . . K 2 −n <
m
,
2m
m
−n m
vagy más szóval ABCD . . . K 2 2
< K . 
A+B + C +D + ... n
Következésképpen ABCD · · · < K =
n
, amit bizonyí-
n
tanunk kellett.
1879. Jelölje Sn a keresett összeget! Lássuk be, hogy Sn+1 = 2Sn + Sn ; s így Sn = 2 · 3n .
1880. Igen, lehet. A k + 1-edik pontot ne kössük össze egyikkel sem, ha azok között
van olyan, mely a többivel össze van kötve; különben pedig összekötjük minde-
gyikkel.
Megjegyzés. Lehet úgy is, hogy a pontokat sorszámozzuk az 1, 2, . . . , n szá-
n
mokkal, s az i-edik és j -edik pontot összekötjük, ha |i − j | ≤ . Ekkor azonos
2
számú szakasz csak a k-adik és (n − k)-adik pontból indul.
1881. A szorzatok összege nem függ az eljárás során végrehajtott felosztások soroza-
tától. Esetünkben
  mindig 500 500 lesz a végeredmény, általában pedig n kavics
n
esetén . Indukciós lépésben az eljárás első lépése után, mikor az n elemű
2
kupacot felbontottuk egy k és egy n − k elemű kupacra, az indukciós feltevést
felhasználva tudjuk számolni a szorzatok összegét. Ebben az összegben szerepel
(az első felosztás miatt) k(n − k), valamint a 
k és azn − k 
elemű csoportokhoz
k n−k
tartozó szorzatok összege, tehát: k(n − k) + + , s ezt kiszámolva
  2 2
n
kapjuk az eredményt.
2
Megjegyzés. Képzeljük el, hogy a kavicsok páronként zsineggel vannak össze-
kötve! Minden egyes kettéosztásnál a különböző csoportokba tartozó kavicsok
között elvágjuk a zsineget. A szétosztásnál
  számolandó szorzat pontosan az el-
n
vágandó zsinegek száma. Összesen zsineg van; ezeket mind elvágjuk, tehát
2
ez a keresett végeredmény.
1882. Teljes indukcióval megadunk egy algoritmust, mely a kívánt alsó korlátot biz-
tosítja. k = 2, 2k − 1 = 3, k · 2k−1 = 4. Vegyük pl. az x1 , x2 és az x3 , x4
számpárt! Ezek között egy-egy összehasonlítás megadja a nagyobbat, melyek le-
gyenek most x2 és x4 . Ha ez utóbbi két számot összehasonlítjuk, s x4 a nagyobb,
akkor nyertünk egy újabb egyenlőtlenséget is: x1 < x4 . Tehát 4 számot kiválaszt-
va, 3 alkalmas összehasonlítás után 4 egyenlőtlenségünk van; s ráadásul tudjuk,
hogy melyik a legnagyobb szám ezek közül.
352 Megoldások, útmutatások

Ez lesz indukciós feltevésünk is: 2k db szám között 2k − 1 alkalmas összehason-


lítást végezve, ismerünk legalább k ·2k−1 egyenlőtlenséget; s tudjuk, hogy melyik
a legnagyobb szám.
Az indukciós lépésben a 2k+1 db számot két, egyaránt 2k db számot tartalmazó
részre osztjuk. Ezek között az indukciós feltevésnek megfelelően külön-külön
2k−1 kérdés után tudjuk, hogy egyikben, másikban melyik a legnagyobb szám; s
ismerünk mindegyik részben legalább k·2k−1 egyenlőtlenséget. Ez eddig összesen
(2k − 1) + (2k − 1) = 2k+1 − 2 kérdés, s legalább 2 · k · 2k−1 egyenlőtlenség.
Most hasonlítsuk össze a két halmaz legnagyobb számát! Ezzel a kérdések száma
2k+1 −1 lesz; s az egyenlőtlenségek száma az utolsó kérdés után 2k -nal növekszik,
így összesen legalább 2 · k · 2k−1 + 2k = (k + 1) · 2k egyenlőtlenséget ismerünk; s
most is tudjuk, hogy a megkérdezett számok közül melyik a legnagyobb.
1883. n = 4 esetén a négy személy: A, B, C, D között a beszélgetések sorrendje:
(A, B), (C, D), (A, C), (B, D). Az indukciós lépésben szervezzük a beszélge-
téseket úgy, hogy az n + 1-edik személy beszél valakivel, majd az n személy
egymás között az indukciós feltevés szerint 2n − 4 hívással kicseréli az összes
pletykát (n + 1 pletykát!), s ezután valaki felhívja az n + 1-edik személyt.

32. Kombinatorika a geometriában

1885. b) Vegyük fel a pontpárok közötti szakaszfelező merőlegeseket. Ez véges sok


egyenes, ezért van olyan P pontja a síknak, melyen egyik szakaszfelező me-
rőleges sem halad át. Vegyünk fel egy elegendően nagy sugarú kört P közép-
ponttal, mely tartalmazza mind a 2000 pontot, majd a kör sugarát folytonosan
csökkentsük. Így egymás után, egyesével hagyunk el pontokat a körből. P vá-
lasztása miatt nem eshet meg, hogy egyszerre 2-vel (vagy többel) csökkenne
a körben levő pontok száma. Ily módon elérhetjük, hogy a kör belsejében
1000 pont maradjon.
1895. A nyolcszög egyik oldalától indulva vezessünk
egy vonalat az egymáshoz csatlakozó paralelo-
grammákon keresztül. Amelyik paralelogrammán
a vonal áthalad, annak van párhuzamos oldala a
...

kiválasztott nyolcszögoldallal. Ugyanezt tegyük ...


meg a nyolcszög egy másik, erre merőleges ol- ...
dalától indulva is. Ez a két vonal találkozik vala-
...

melyik paralelogramma belsejében, s ez a para-


lelogramma téglalap lesz.
A nyolcszög eddig figyelembe nem vett oldalaira
is elvégezve az előbbi eljárást, megtaláljuk a másik téglalapot is.
32. Kombinatorika a geometriában 353

1897. Tekintsük azokat az eseteket, amikor a ponthalmaz konvex burka hatszög, ötszög,
négyszög, ill. háromszög.
1898. Ha az ABC háromszög belsejében van a D pont, akkor az ABC és az ABD
háromszögek legkisebb szöge különböző. Ha a 6 pont konvex burka háromszög,
négyszög vagy ötszög, akkor az előbbi megállapítás segít. Ha a konvex burok az
ABCDEF hatszög, és az ACE háromszög legkisebb szöge A-nál van, akkor a
DEA háromszögnek az A-nál levő szöge kisebb.
1901. Tekintsük az öt pont konvex burkát. Ha ez négyszög vagy ötszög, akkor készen
vagyunk. Ha háromszög, akkor annak belsejében van a maradék két pont. Ezen
a két ponton átmenő egyenes egyik partjára a háromszögnek két csúcsa esik; ez
a két csúcs és a háromszögben levő két pont egy konvex négyszög négy csúcsa.
Megjegyzés. Azt sejtik, hogy ha adott a síkon 2n−2 + 1 pont úgy, hogy közülük
semelyik három sem esik egy egyenesre, akkor kiválasztható közülük n olyan
pont, amelyek egy konvex sokszög csúcsait alkotják.
 
n
1903. n pont közül -féleképpen választhatunk ki öt pontot. Egy korábbi feladat
5
állításaalapján
 minden pontötösben legalább egy konvex négyszög van. Így leg-
n
alább konvex négyszöget kapnánk. Egy négyszöget azonban esetleg több-
5
ször is figyelembe vettünk. Mégpedig maximálisan (n − 4)-szer számíthattuk be,
hiszen az ötödik pontot (n − 4)-féleképpen választhatjuk.
   Tehát a kiválasztható
1 n 1 n
konvex négyszögek száma legalább = .
n−4 5 5 4
1905. Tegyük fel, hogy k háromszöget sikerült kijelölni az előírt módon. Mivel három-
szögeink közül semelyik kettőnek nincs közös oldala (éle), ezért együttvéve 3k
élük van. Eza szám nem nagyobb,
  mint az n pontból képezhető pontpárok, élek
n n
száma, ami ; így 3k ≤ .
2 2
Másrészt nézzük azt az esetet, amikor kijelöltünk k darab háromszöget, s töb-
bet
 már nem tudtunk. Ez azt jelenti, hogy végigmenve a még föl nem használt
n
− 3k élen, és mindegyik ilyenhez hozzápróbálva harmadik csúcsnak a vég-
2
pontjaitól különböző (n − 2) pont mindegyikét, e próbák mindegyike tiltott há-
romszöghöz vezet; olyanhoz, melynek legalább egy éle a már kijelölt 3k él közül
való.  
n
A ki nem jelölt háromszögek száma − k. Fenti próbáink során mindig ilyen
3
háromszöget kapunk, és ha egy ki nem jelölt háromszögnek 2 olyan oldala van,
amely még ki nem jelölt él; azt 2-szer is megkaptuk, mindkét ilyen oldala révén.
(3-szor azonban nem kaphatjuk.)
354 Megoldások, útmutatások
     
n n
Ezek szerint: (n − 2) − 3k ≤ 2 − k . Innen megkapjuk a kívánt
  2 3
1 n
k> egyenlőtlenséget.
9 2
1906. n kör a síkot legfeljebb n2 − n + 2 részre osztja. Ez teljes indukcióval igazolható.
1907. Nem lehet. A lapok találkozási pontjainál levő szögek 108◦ , 144◦ és 180◦ lehet-
nek. Ezekből a 360◦ -ot 2 · 108◦ + 144◦ alakban állíthatjuk elő, tehát két ötszög- és
egy tízszöglap találkozik. Ezért egy ötszöglap körül felváltva ötszög- és tízszög-
lapok vannak. Ez a váltakozás nem lehetséges, hiszen 5 páratlan.
1908. A test minden csúcsában q darab p oldalú szabályos sokszög találkozik. A sok-
2

szög csúcsaiban levő szög nagysága 1 − π. A test egy-egy csúcsa kö-


p
2

rül a lapokon levő szögek összege kisebb 2π-nél: q 1 − π < 2π, azaz
p
(p − 2)(q − 2) < 4.
(p − 2)(q − 2) = 3: p = 5, q = 3 (dodekaéder); p = 3, q = 5 (ikozaéder).
(p − 2)(q − 2) = 2: p = 4, q = 3 (kocka); p = 3, q = 4 (oktaéder).
(p − 2)(q − 2) = 1: p = 3, q = 3 (tetraéder).
Tehát (legfeljebb) öt szabályos test létezik.
1909. Ha a ponthalmaz konvex burka egy n-oldalú sokszög, akkor a sokszög belsejébe
1000 − n db pont került. Amikor az utolsó szakaszt is berajzoltuk, akkor az n-
oldalú sokszöget olyan háromszögekre daraboltuk fel, amelyek csúcsai az adott
1000 pont közül kerülnek ki.
Hány háromszög keletkezett? Legyen a háromszögek száma m. A háromszögek
belső szögeinek összege m · 180◦ = (n − 2) · 180◦ + (1000 − n) · 360◦ , tehát
m = 1998 − n.
A berajzolt szakaszok a háromszögek oldalai, számuk:
3m − n
+ n = 2997 − n.
2
1910. A válasz: legfeljebb n. Ötlet a megoldáshoz: a vizsgált sokszög az általánosság
megszorítása nélkül lehet szabályos is. Soroljuk csoportokba a szabályos n-szög
átlóit; egy csoportba tartozzanak az egymással párhuzamos átlók. Lássuk be,
hogy a csoportok száma legfeljebb n.
1911. Konvex
  sokszög esetén legfeljebb 3, konkáv sokszög esetén pedig legfeljebb
2n
+ 1 hegyesszöge lehet az n oldalú sokszögnek.
3
Tegyük fel ugyanis, hogy az n-oldalú sokszögnek k számú hegyesszöge van,
és ennélfogva n − k szög nem hegyes. Mivel a hegyesszögek kisebbek 90◦ -nál,
a többi szög pedig 360◦ -nál, ezért k · 90◦ + (n − k) · 360◦ nagyobb a sokszög
32. Kombinatorika a geometriában 355

szögösszegénél: k · 90◦ + (n − k) · 360◦ > (n − 2) · 180◦ ; ebből 3k < 2n + 4,


2n
3k ≤ 2n + 3, és így k ≤ + 1. Ez az érték elérhető.
3
1912. Igen, lehetséges. Legyen például az i-edik rész mindazoknak az (x, y, z) pontok-
nak az összessége, melyekre [x] ≡ i (mod 2000).
1913. 1. bizonyítás. (Indirekt.) Legyenek a fellépő távolságok d1 , d2 , . . . , dn . Vegyünk
két pontot, P -t és Q-t az adottak közül, s rajzoljunk mindegyik körül d1 , d2 ,
. . . , dn sugarú köröket. A ponthalmaz bármely pontja rajta van valamelyik P és
valamelyik Q körüli körön. Csakhogy a P és Q körüli köröknek legfeljebb 2n2
közös pontja van.
2. bizonyítás. (Indirekt.) Véges sok távolság lép fel; ezek minimuma m, maxi-
muma M. Vegyünk egy négyzetrácsot a síkon, melyben egy négyzet átlója m.
Egy négyzetben legfeljebb egy pont lehet. Valamelyik pont körül vegyünk fel
egy M sugarú kört. Mindegyik pontnak ebben a körben kell lennie, azonban a
körben véges sok négyzet, és így véges sok pont van.
1914. A fellépő háromszögek közül válasszuk ki az egyik legnagyobb területűt. Húz-
zunk párhuzamosakat ennek csúcsain át a szemközti oldalakkal. Az így kapott
háromszög tartalmazza az összes pontot, s a háromszög területe legfeljebb 4 egy-
ség.
1915. Tekintsük az 5 piros pontból álló ötszöget. Ezt forgassuk körbe a tízszög csú-
csain (összesen 10 állapot van). Ha valamely elforgatásnál az ötszögben van 3
kék csúcs, akkor készen vagyunk. Indirekt feltevéssel igazoljuk, hogy ez mindig
teljesül. Ha egyik ötszögben sincs 3 kék csúcs, akkor a 9 elforgatott ötszöggel
legfeljebb 9 · 2 = 18 kék csúcsot számoltunk. Minden csúcsot ötször számoltunk
(hiszen egy csúcson az elforgatott ötszög mindegyik csúcsa „végigmegy”), így a
18
kék csúcsok száma ≤ , pedig a kék csúcsok száma 5.
5
1916. Válasszunk ki egy sokszöget, melyet a 13 oldalú sokszög átlói határolnak. A sok-
szög egy oldalához a 13-szög két csúcsa rendelhető, s egy csúcshoz a sokszög-
nek legfeljebb két oldalegyenese fut be. Ez azt jelenti, hogy a sokszög oldalainak
száma legfeljebb 13.
1917. Lássuk be, hogy a piros és fekete pontok konvex burkai diszjunktak. Ezek szétvá-
laszthatók pl. a két burok közti legkisebb távolság szakaszfelező merőlegesével.
(Miért?)
1
1918. Tegyük fel, hogy lehetséges az elrendezés. Írjunk mindegyik pont köré r =
2
sugarú köröket. Ezek nem metszik egymást, s rajta vannak a 10,5 sugarú „kibő-
vített” asztallapon; ezért területük összege nem nagyobb ennek az asztallapnak a
1
2
területénél, tehát 450 · π · ≤ (10, 5)2 · π, azaz 450 ≤ 441 lenne.
2
356 Megoldások, útmutatások

1919. Az egységnégyzetek oldalától 1/2 távolságban húzzunk vonalat. Az így kapott


π
alakzat területe 3 + . A téglalapot is „keretezzük”, mindegyik oldalától 1/2
4
távolságban befele
húzunk egyenest. Ennek a 19 × 24-es téglalapnak a területe
π

456. Mivel 120 · 3+ < 456, ezért a négyzetek között „van hely” egy körnek.
4
1920. A pénzérmék együttes területe kisebb az asztal területénél: n · π · r 2 < π · R 2 ;
√ R
innen n < . Most duplázzuk meg mindegyik érme sugarát, így ezek lefedik
r
√ 1 R

az R −r sugarú asztalt; tehát n·π ·(2r)2 > π ·(R −r)2 , amiből n > −1 .
2 r
1921. A test mindegyik oldallapjához rendeljük hozzá a csúcsok számozásának meg-
felelő körüljárási irányt. Ekkor egy közös él mentén csatlakozó két háromszög
ellentétes körüljárású lesz.
Az óramutató járásával megegyező körüljárású lapokat színezzük az egyik, az
ellentétes körüljárású lapokat a másik színnel.
1922. Azt kell csak megfigyelnünk, hogy az egyes tartományok hány körön belül van-
nak. A csupa homorú vonallal határolt tartomány (egy körívet a kör belsejéből
szemlélve nevezzük domborúnak, kívülről pedig homorúnak) kívül esik minden
körön, tehát 0-val jelölhetjük. A többi tartomány belül esik egy, két, három vagy
tetszés szerinti számú körön. Lehet, hogy ez a kör nem is metszi a határát, hanem
kívülről foglalja be. Ha a befoglaló körök számát beírjuk a tartományokba, meg-
oldottuk a színezés gondját: a páros számúak kapják az egyik színt, a páratlanok
a másikat. (Miért lesz jó a színezés?)
1923. Egy konfiguráció (a körök közül egy és a berajzolt húr) három részre osztja a
síkot. Számozzuk e három részt a 0, 1 és 2 számokkal! Minden tartományba
(országba) írjuk azt a számot, amelyikkel megjelölt részbe esik a három rész
közül. Ezt a számozást minden konfigurációra hajtsuk végre! Minden lépésnél
egy-egy újabb szám kerül minden országba. Az egy országon belüli számokat
adjuk össze, osszuk el az összeget 3-mal, és írjuk az országba a maradékot (a
korábbi számokat töröljük le). Ezután aszerint színezünk egy országot fehérre,
feketére vagy pirosra, hogy a beleírt maradék 0, 1 vagy 2. Be kell látnunk, hogy
ekkor bármely két szomszédos ország különböző színű (azaz különbözők a ma-
radékaik). Legyen két szomszédos ország L1 és L2 , határaik közös éle pedig h.
Vegyük el a térképről a h élt tartalmazó K konfigurációt. Ekkor L1 és L2 kö-
zött nincs határ. Tehát L1 és L2 a K-tól különböző konfigurációk mindegyikének
ugyanabba a részébe esik, a K által felosztott 3 rész közül pedig különbözőbe.
Ezért L1 és L2 maradékai nem egyezhetnek meg. Így tehát térképünk 3 színnel
jól színezett.
1924. A Diofantoszi egyenletek c. fejezetben már találkoztunk a végtelen leszállás
módszerével. Ezt alkalmazzuk itt is.
Használjuk fel a négyzetrács következő tulajdonságát: ha egy rácspontot tükrö-
zünk két rácspontot összekötő szakasz felezőpontjára, a tükörkép is rácspont lesz.
32. Kombinatorika a geometriában 357

Ha van egy szabályos rácssokszögünk, abban minden csúcsot tükrözzünk a vele


szomszédos két csúcs közti átló felezőpontjára. A szabályos háromszög, hatszög
és a négyzet kivételével minden más szabályos sokszög esetén a tükörképek
egymástól különböző pontok a sokszög belsejében, melyek maguk is szabályos
sokszöget alkotnak. (Miért?)
Ily módon újabb és újabb, egyre kisebb szabályos rácssokszöget kapunk; de a
kiinduló sokszög belsejében véges sok rácspont volt, így nem létezhet a rácssok-
szögeknek ez a végtelen sorozata. Az ellentmondás oka a kiinduló feltevés hibás
volta.
1925. Az előző feladathoz hasonlóan itt is a végtelen leszállás módszerét alkalmazzuk.
Használjuk fel a négyzetrács következő tulajdonságát: ha egy rácspontot egy
másik rácspont körül 90◦ -kal elforgatunk, akkor a kapott képpont is rácspont
lesz.
Tegyük fel, hogy egy szabályos ABCDEF hatszög csúcspontjai a négyzetrács
rácspontjai. Ekkor a hatszög A csúcsát B körül, B csúcsát C körül, . . . , F
csúcsát A körül elforgatjuk 90◦ -kal úgy, hogy a képpontok a hatszög belsejébe
essenek. Ezek a pontok is egy szabályos hatszög csúcsai (miért?), valamint előbbi
megjegyzésünk szerint rácspontok is.
Ily módon újabb és újabb, egyre kisebb szabályos rácshatszöget kapunk; de a
kiinduló hatszög belsejében véges sok rácspont volt, így nem létezhet a rácshat-
szögeknek ez a végtelen sorozata. Az ellentmondás oka a kiinduló feltevés hibás
volta.
Mivel nincs szabályos rácshatszög, ezért szabályos rácsháromszög sem lehet. Ha
lenne szabályos rácsháromszög, akkor lenne szabályos rácshatszög is. Ugyanis a
háromszög oldalainak harmadolópontjai szabályos hatszöget alkotnak, és ha ezt
a háromszög egyik csúcsából háromszoros arányban nagyítjuk, szabályos rács-
hatszöget kapunk.
1926. Vegyünk fel egy, az O szimmetriaközépponton áthaladó húrt, és rá merőlegesen
egy másik húrt. A húrok hossza legyen a és b, s tudjuk, hogy O felezi a húro-
kat. Ezt a merőleges húrpárt forgassuk O körül, s a forgatás szöge folytonosan
változzék 90◦ -ig. Forgatás közben figyeljük az a − b különbség változását. Ez a
különbség folytonosan változik és előjelet vált, ezért valamely köztes állapotban
értéke 0, azaz a = b. Ebben a helyzetben a húrok végpontjai egy négyzet csúcsai.
Megjegyzés. Tetszőleges zárt síkgörbén is mindig található 4 olyan pont, ame-
lyek egy négyzet csúcsai.
1927. Válasszuk ki a görbe három tetszőleges pontját, s tekintsük az ezekre illeszkedő
síkot; s figyeljük, hogy a görbe ezeken a pontokon áthaladva a síknak mikor me-
lyik oldalán tartózkodik. Könnyen látható, hogy a görbe ezt a síkot egy negyedik
pontban is metszeni fogja.
Most megmutatjuk, hogy öt pontra nem igaz ez az állítás. Konstruálunk egy olyan
térgörbét, melyen nincs öt komplanáris pont. Vegyünk egy gömböt és a gömböt
358 Megoldások, útmutatások

nem metsző S síkot. A gömb felületén vegyünk fel egy olyan L görbét, melynek
S-re való merőleges vetülete ellipszis — ez lesz a kívánt tulajdonságú görbe. A
vetítéssel megadtunk egy ellipszis alapú hengert. Tegyük fel, hogy L-en van öt
komplanáris pont. Nos, az öt pont síkja a gömbből kört, a hengerből ellipszist
metsz ki, ami ellentmondásra vezet, mert egy körnek és egy ellipszisnek nem
lehet öt közös pontja.
1929. A színeket nevezzük a, b, c, d, e-nek, és A, B, C, D, E jelentsen a továbbiakban
egy-egy a, b, c, d, ill. e színű pontot.
Egy egyenest, síkot, ha van rajta 3, 4, ill. 5 különböző színű pont, röviden 3-, 4-,
ill. 5-színűnek fogunk nevezni.
Ha egy 3-színű síknak és egy olyan egyenesnek, amelyiken van a maradó 2 színű
pont, van közös pontja, akkor van a térben 4-színű sík. Ugyanis, legyen az S
síkon a, b és c színű pont, a v egyenesen d és e színű pont, és P legyen a sík és
az egyenes közös pontja. Ha P színe a, b vagy c, akkor v 3-színű, s ekkor v-hez
választjuk a térnek valamely negyedik színű O pontját, a v-re és O-ra illeszkedő
sík 4-színű; ha pedig P d vagy e színű, akkor S 4-színű.
Nos, ezek után igazoljuk a feladat állítását.
Ha A, B, C, D, E közül valamely négyen át fektethető sík, akkor igaz a feladat
állítása. Ha nem, akkor tekintsük az ABCD tetraéder oldallapjainak a síkját. Ha
valamelyiknek, pl. a B, C, D pontokat tartalmazó S síknak ellenkező oldalára
esik a tetraéder és E, akkor AE metszi S-t.
Ha viszont bármelyik síkot véve, annak ugyanarra az oldalára esik a tetraéder és
E, akkor a tetraéder tartalmazza E-t. E különbözik A-tól, mert más színű, így
közelebb van S-hez, mint A; tehát ebben az esetben is metszi AE az S síkot.
Mindkét esetben az előzetesen bizonyított állítást felhasználva láthatjuk, hogy
igaz a feladat állítása.
1930. Igen, például a legkésőbb érkező vendég érkezésének (vagy a legkorábban el-
menő távozásának) pillanatában. (Ha egy egyenes véges sok intervalluma közül
bármely kettőnek van közös pontja, akkor olyan pont is található, amely az összes
intervallumban benne van.)
1931. Előbb n = 4-re lássuk be az állítást. Legyen a négy halmaz közül három-három
halmaz közös pontja P123 , P124 , P134 , P234 . Ha e négy pont konvex burka négy-
szög, akkor átlóinak metszéspontja; ha pedig háromszög, akkor a negyedik pont
a négy halmaz közös pontja lesz.
Az n > 4 eset — az előbbieket felhasználva — igazolható teljes indukcióval.
1932. Bármely ponttól legfeljebb hat másik lehet minimális távolságra, és a konvex
burok pontjaitól még kevesebb.
1933. Lássuk be, hogy a pontpárok között fellépő maximális, d hosszúságú szakaszok
közül bármely kettőnek van közös pontja.
32. Kombinatorika a geometriában 359

Mutassuk meg, hogy ha valamelyik pontra legalább három maximális szakasz


illeszkedik, akkor van olyan pont is, amelyre legfeljebb egy.
Ha az n pont között van olyan, amelyre legfeljebb egy d hosszúságú szakasz il-
leszkedik, azt hagyjuk el. Ezt az eljárást a maradék pontrendszerre is ismételjük
meg, amíg lehet. Ha így m pontot hagyunk el, akkor legfeljebb m db d hosszúsá-
gú szakaszt szüntetünk meg. A maradék n − m pont közül bármelyikre legalább
két d hosszúságú szakasz illeszkedik; de több a korábbi megjegyzés miatt nem
lehet, hiszen akkor lenne olyan, melyre csak egy illeszkedik. Ezért ezek között
n − m db d hosszúságú szakasz van. A pontok között összesen legfeljebb m +
+ (n − m) = n-szer lép fel a maximális távolság.
1934. Legyen A, B, C a ponthalmaz három olyan pontja, amelyek nincsenek egy  egye-
nesen. |AB| = k, |AC| = m, és P egy tetszőleges pont a halmazból. |P A| −


− |P B| = |q| ≤ |AB| = k, így P rajta van az A, B fókuszú hiperbola valame-
lyik ágán, q = −k, . . . , −1, 0, 1, . . . , k; azaz a 2k + 1 hiperbola valamelyikén.
Ugyanígy illeszkedik a P pont az A, C fókuszú 2m + 1 hiperbola valamelyikére
is. Ennek a két hiperbola-seregnek véges sok — legfeljebb 4(2k + 1)(2m + 1) —
közös pontja van.
Megjegyzés. Létezik olyan végtelen, nem egy egyenesbe eső ponthalmaz, amely-
ben minden távolság racionális.
1935. Tegyük fel, hogy létezik ilyen pontrendszer. Tekintsük a pontok által kifeszített
egyenesek és az egyenesen nem levő pontok távolságait. Ezek közül válasszuk
ki a legkisebb távolságot d-t, legyen ez a P pont és az e egyenes távolsága.
Bocsássunk P -ből merőlegest e-re; ezzel az egyenest két részre osztottuk, s az
egyenesen levő 3 pontból legalább kettő az egyik félegyenesre esik. A két pont-
ból (nevezzük őket A-nak, B-nek) a P -től távolabbi B. Most jutunk ellentmon-
dásra, mert a P B egyenes és az A pont közti távolság kisebb d-nél. Tehát nem
létezik a kérdezett tulajdonságú pontrendszer.
Megjegyzés. A Sylvester-feladatot Erdős Pál tette ismertté az 1930-as években.
Azóta a Sylvester-feladatra újabb és újabb bizonyítások születnek és különböző
rokon problémákat fogalmaznak meg. Legyen adva a síkon n pont úgy, hogy
nincs mind egy egyenesen, s jelölje f (n) azoknak az egyeneseknek a minimális
számát, mely ezekből a pontokból pontosan kettőn megy át. A Sylvester-feladat
szerint f (n) ≥ 1. Bebizonyították, hogy f (n) ≥ 3; s ez a becslés
√ pontos is, ha n
értéke 3, 4, 6 vagy 7. Később megmutatták, hogy f (n) > n; majd azt is, hogy
3 n
f (n) ≥ n, sőt n > 7 esetén f (n) ≥ .
7 2
Vizsgáljuk a két-pontú egyenesek helyett a négy-pontúakat. Erdős a következő
problémát tűzte ki. Legyen adva n pont a síkon úgy, hogy nincs közte öt, melyek
kollineárisak. Jelölje g(n) a négy pontot tartalmazó egyenesek számának maxi-
g(n)
mumát. Igaz-e ekkor, hogy 2 → 0, ha n → ∞? A kérdés eldöntésére 100
n
360 Megoldások, útmutatások

dollárt ajánlott fel. Előbb azt igazolták, hogy g(n) > c · n · log n; majd pedig
erősebb állítást, mégpedig azt, hogy g(n) > c · n3/2 . Talán ez utóbbi már pontos
korlát.
A hárompontú egyenesek számának maximuma c · n2 , ahol c pozitív konstans.
1936. Vegyük észre azt, hogy a pontok közülbármely  két pontra legfeljebb két egység-
n
sugarú kör illeszkedhet. Az n pontból db pontpár választható, ekkor ezekre
  2
n
legfeljebb 2 · egységsugarú kört tudunk illeszteni; azonban így minden kört
2
3-szor számoltunk (hiszen az A, B, C pontokon átmenő kört megszámoltuk, mint
az A és B, a B és C, valamint a C és A pontokra illeszkedő kört). Ezért az egy-
ségsugarú
 körökszáma  legfeljebb
1 n 2 n
·2· = .
3 2 3 2
Megadunk egy konstrukciót, melyben az egységkörök száma c · n3/2 .
Legyenek − →
a1 , −→
a2 , . . . , −
a→N egységvektorok a síkban. A ponthalmazt, melyek
köré a köröket −
→ −

 írjuk,
 az ai + aj helyvektorok   végpontjai alkotják. Ezért a pontok
N N
száma m = . E pontok köré n = egységkör írható, ugyanis minden
2 3
i = j = k esetén az − →
ai + − →
aj , −

aj + −

ak , −

ak + −→
ai ponthármas köré írt kör sugara
egységnyi; hiszen a kör középpontjától, a − →
ai + −

aj + −

ak ponttól mindhárom pont
egységnyire van.
1937. Vergilius híres eposzában dolgozta fel Didó, türoszi királylány, Karthágó alapí-
tójáról szóló következő görög-római mondát. Eszerint Didó szellemes furfanggal
egy országrészt vásárolt, amely Byrsa vagy Bikabőr néven ismert. Didó valójá-
ban csak annyi földre alkudott, amely körülkeríthető egy bikabőrrel. A bikabőrt
azonban vékony csíkokra szabdalta, és ezeket egymáshoz illesztve hatalmas terü-
letet kerített be, amit ma Byrsanak neveznek, és amely Karthágót magába ölelte.
Innen származtatható feladatunk, melynél a keresett síkidom a kör; ezt könnyű
megsejteni, de az első bizonyításra sokáig kellett várni. A XIX. században Jacob
Steiner (1796–1863), a kor egyik legnagyobb geométere oldotta meg a feladatot.
Bizonyítása, a „négycsuklós eljárás”-nak nevezett módszer a következő: belátja,
hogy ha a síkidom nem kör, akkor a a területe növelhető úgy, hogy a kerülete
nem nő. Ezért, ha létezik az extremális tulajdonsággal rendelkező síkidom, akkor
az csak a kör lehet. A bizonyítás állomásai: a síkidom konvex; a síkidomnak van
szimmetriatengelye; ha nem kör, akkor (Thalész tétele miatt) van olyan P pontja,
melyből a szimmetriatengely A és B végpontjai nem derékszög alatt látszanak.
Ekkor a P AP  B négyszöget (a P  pont a P tükörképe a szimmetriatengelyre)
mint egy csuklós szerkezetet tekintjük, és ezt a csuklós szerkezetet mozgatjuk (a
fölötte levő ívekkel együtt) úgy, hogy P -nél (és így P  -nél is) derékszög legyen
— ezzel növeltük a területét, a kerület nem változott.
33. Gráfok 361

Jacob Steiner fenti bizonyítása azonban hiányos. Ő azt bizonyította be, hogy kör-
lemeztől különböző síkidom területe nem lehet legnagyobb a vele egyenlő kerü-
letűek között; de nem bizonyította be, hogy a körlemeznek csakugyan megvan
ez a maximum-tulajdonsága.
Hogy mennyire hiányos ez a bizonyítás, azt a következő példán mutathatjuk be:
Ehhez hasonló gondolatmenettel „bizonyította be” Perron, hogy 1 a legnagyobb
természetes szám. Azt mondta ui., hogy a természetes számok 1, 2, 3, 4, . . .
sorozatában semelyik, 1-nél nagyobb k szám nem lehet a legnagyobb, mert ebből
a feltevésből ellentmondásra jutunk.
Ha ti. k > 1 lenne a legnagyobb természetes szám, akkor négyzete: k 2 =
= k ·k > k ·1 = k; tehát már találtunk is egy számot, amely nagyobb a feltételezett
legnagyobb természetes számnál. Emiatt k > 1 biztosan nem lehet legnagyobb a
természetes számok közül.
Mivel pedig a négyzetreemeléssel az 1-ből ismét 1 lesz, azaz előbbi okosko-
dásunk ebben az egyetlen esetben nem vezet ellentmondásra; ebből következik,
hogy 1 a legnagyobb természetes szám.
Visszatérve az izoperimetrikus problémára, megnyugtatásul közöljük a követke-
zőket. Egyrészt a későbbiekben sikerült teljessé tenni Steiner hiányos bizonyí-
tását (a kiegészítés ismertetésétől — annak bonyolult volta miatt — ehelyütt le
kell mondanunk), másrészt más helyes, pl. analitikus bizonyítással is igazolták,
hogy az egyenlő kerületű síkidomok közül a körlemez területe a legnagyobb.

33. Gráfok

1938. n csapat esetén egy-egy csapatnál a győzelmek száma 1 és n − 1 között lehet.


Ezért van két csapat, mondjuk A és B, melyeknek ugyanannyi győzelme van.
Vegyük azt, hogy A győzte le B-t. Azok között, amelyeket B legyőzött, van
olyan csapat, legyen ez C, amelyet A nem győzött le (hiszen, ha nem lenne
ilyen, akkor A több csapattal szemben győzött volna, mint B).
1939. Összesen 28 meccs volt; így van olyan csapat, mely legalább 28/8 = 3, 5, az-
az legalább 4 meccset nyert. Legyen ez A, és válasszunk ki 4 csapatot azok
közül, amelyeket legyőzött. Ezek egymás között 6 mérkőzést játszottak, valame-
lyik legalább 6/4 = 1, 5, azaz legalább 2 mérkőzést nyert. Legyen ez B, és az
általa legyőzött (amelyeket persze A is legyőzött) csapatok C és D. A C és D
jelölést úgy választjuk, hogy C legyőzte D-t.
Megjegyzés. Általánosítható az állítás 2n−1 csapatra. Közülük kiválasztható A1 ,
A2 , . . . , An csapat, hogy i < j esetén Ai legyőzte Aj -t.
1940. Legyen A az első csapat egyik legeredményesebb játékosa. A-nak is volt vere-
sége, kikapott B-től. Mivel B megverte A-t, ezért ő csak egy A-tól különböző
362 Megoldások, útmutatások

C-től kaphatott ki. Mivel A a csapatának legjobb játékosa, van a második csapat-
ban olyan D játékos, akit A megvert, és akitől C kikapott. Ha nem lenne ilyen,
akkor C több győzelmet aratott volna, mint A, ami ellentmondás.
1941. Legyen A az (egyik olyan), akinek legtöbb győzelme van. Ha B legyőzte A-t,
akkor a B által legyőzöttek száma nem nagyobb, mint az A által legyőzöttek
száma (az A választása miatt). Ezért van olyan C, akit A legyőzött, de B nem.
1942. Válasszunk egy tetszőleges csapatot, A-t. A-n kívül 17 csapat van. A eddig 8-cal
mérkőzött, 9-cel nem. Ha e között a 9 csapat között van kettő, melyek nem ját-
szottak egymással, akkor készen vagyunk. Ennek így is kell lennie, hiszen külön-
ben a 9-es csoport minden csapata a 8 mérkőzést a 9-es csoporton belül játszotta;
ami nem lehet, mert ekkor egy-egy fordulóban a 9 csapat valamelyikének nem
jutna ellenfél.
1943. Pingpongmérkőzés nem végződhet döntetlenül, így két-résztvevős „körmérkő-
zés” esetén az állítás nyilvánvalóan igaz. Erre támaszkodva, az állítás helyessé-
gét a résztvevők száma szerinti teljes indukcióval bizonyítjuk. Feltesszük, hogy
az állítás helyes, ha a résztvevők száma legföljebb n.
n + 1 résztvevő esetén szemeljük ki egyiküket, legyen ez B. A többi n játé-
kos feltevésünk szerint sorba állítható a kívánt módon az egymás közt lejátszott
mérkőzések alapján. Állítsuk most B-t ebbe a sorba az első olyan elé, akit ő
legyőzött — amennyiben ilyen van —, különben a sor végére. Így B helyzete
nemcsak a hátsó, hanem elülső szomszédjához viszonyítva is megfelelő, hiszen
az előtte állók egyikét sem győzte le, tehát vagy az áll, hogy azok mind legyőz-
ték B-t, vagy pedig senki sem győzte le, akkor pedig őt az első helyre állítottuk.
B ilyen beállítása sem az előtte, sem a mögötte állók egymás közti elhelyezését
nem változtatta meg, így példát adtunk az n + 1 játékos megfelelő sorbaállítására.
Másik bizonyítás. Állítsuk sorba találomra a versenyzőket, majd az első kettőt
cseréljük fel, ha az első kikapott a másodiktól, különben hagyjuk őket az eredeti
sorrendben; majd sorra a másodikat és harmadikat, harmadikat és negyediket,
. . . , cseréljük meg, ha nem megfelelő sorrendben állnak. Előfordulhat, hogy így
egy cserével előbbre került versenyző és az előtte álló közt helytelen sorrend
keletkezik, ezért a sor végére érve térjünk vissza az elejére és ismételjük meg
újra és újra az eljárást.
Belátjuk, hogy véges sokszor végighaladva a soron, eljutunk egy olyan elren-
dezéshez, amely már kielégíti a feladat követelményeit. Valóban, a versenyzők
csak véges számú különböző sorrendben állhatnak; így elég sokszor végigmenve
a soron, elő kell fordulnia egy olyan sorrendnek, amelyik már korábban is elő-
fordult. Ez azonban csak akkor lehetséges, ha végighaladva már nincs szükség
cserére, azaz ha a sorrend kielégíti a feladat feltételeit. Ha ugyanis valakit felcse-
réltünk a rákövetkezővel, az minden későbbi sorrendben is hátrább marad nála,
hiszen őket ellenkező irányban nem cserélhetjük meg, így a korábbi sorrend nem
térhet vissza.
33. Gráfok 363

1944. 1. megoldás. Legyen n résztvevő. Jelölje ai az i-edik résztvevő azon ismerőse-


inek számát, akik előtte állnak, bi pedig legyen az utána álló ismerősök száma.
Nyilván a1 + a2 + · · · + an = b1 + b2 + · · · + bn , s mivel 1 < i < n esetén ai = bi ,
ezért a1 + an = b1 + bn , viszont a1 = bn = 0. Tehát an = b1 .
2. megoldás. Minden résztvevő adjon 1–1 Ft-ot a mögötte álló ismerőseinek.
Az első és az utolsó kivételével mindenki ugyanannyit adott, amennyit kapott.
Tehát amennyit az első adott, annyit kapott az utolsó, hiszen az összvagyon nem
változott.
1945. Lehetséges. 1. megoldás. Számozzuk meg őket az 1, 2, . . . , n számokkal. Az
n
i-edik és a j -edik ismeri egymást, ha |i − j | ≤ . Ekkor azonos számú ismerőse
2
csak a k-adik és (n − k)-adik személynek van. (Miért?)
2. megoldás. Teljes indukcióval adunk eljárást az ismeretségek szervezésére. In-
dukciós lépésben (k + 1)-re az új, (k + 1)-edik személy ne ismerje a többit, ha
azok között van olyan, aki a többit ismeri; különben pedig ismerjen mindenkit.
1946. Az lenne a kedvező, ha mindenkinek legfeljebb 6 ismerőse lenne, s ez így is
van. Az A lakó ismerősei között nem lehet kettő, akik ismernék egymást (az
első feltétel szerint), ugyanakkor már 7 ember között vannak ismerősök (második
feltétel), tehát A-nak legfeljebb 6 ismerőse lehet.
1947. Belátjuk, hogy legalább 97 tanuló ismeri az összes többit. Tegyük fel, hogy nem
ismer mindenki mindenkit. Ekkor legyenek A és B olyanok, akik nem ismerik
egymást. A, B-hez válasszunk tetszőlegesen X, Y -t. Ekkor a feltétel szerint pl.
X ismeri A, B, Y -t. Most A, B, X, Z-re kapjuk, hogy X mindenkit ismer. A,
B-n kívül legfeljebb egy olyan tanuló lehet, aki nem ismer mindenkit.
1948. Legyen A egy tetszőleges diák. Megmutatjuk, hogy van olyan B diák, aki A-
nak két barátjával is barátkozik; ez együtt négy jó fő. Tegyük fel, hogy nincs
ilyen B. Ekkor A mind a 10 barátjának van 8 másik barátja úgy, hogy ezek a
második barátok különbözők. (Nézzük meg, miért 8 másik barátot vizsgálunk.)
Ha nem lennének különbözők, akkor A, valamint az a két barát, akinek van közös
második barátja, megfelelő négy személy lenne. Ez együtt 1+10+80 = 91 diákot
jelentene, miközben a társaság 90 főből áll. Tehát kiválasztható a megfelelő négy
személy.
1949. 1. megoldás. Legyen A az, akinek legtöbb ismerőse van, összesen n. Az A is-
merőseinek a feltétel szerint mind különböző számú ismerőse van. Ezek minde-
gyikének legalább 1 ismerőse van, hiszen A-t ismerik. Mivel közöttük bármely
kettőnek van közös ismerőse (az A), ezért nincs köztük kettő, akinek ugyanannyi
ismerőse van. Ez csak úgy lehet, ha ezen n személy ismerőseinek száma valami-
lyen sorrendben 1, 2, . . . , n.
2. megoldás. Teljes indukció a létszámra.
364 Megoldások, útmutatások

1950. Ha az állítás nem igaz, akkor a társaságban volnának ismerősök; és olyanok


is, akik nem ismerik egymást. Mivel az nem lehet, hogy egyesek mindenkit
ismerjenek, mások pedig senkit sem; így volna olyan ember is — jelöljük őt
E-vel —, akit a társaság bizonyos tagjai ismernek — legyen ezek egyike I —,
mások viszont, pl. N, nem.
Ha E kimegy, akkor I ismerőseinek száma eggyel csökken, míg N-é nem válto-
zik. A feltételek alapján tehát I pontosan eggyel több embert ismer a társaságban,
mint N.
A társaság akármelyik, E, I , N-től különböző tagjának távozása után I -nek és N-
nek csak úgy lehet ugyanannyi ismerőse; ha I ismeri az illetőt, N pedig nem. Ez
viszont azt jelenti, hogy N legfeljebb I -t ismeri a társaságból, I pedig legfeljebb
N-et nem ismeri.
Ha most I és N ismerik egymást, akkor I mindenkit ismer — azaz ismerőseinek
száma (n − 1) —, N viszont csak I -t ismeri. Ha pedig I és N nem ismerősök,
akkor I ismerőseinek száma n − 2, N-é pedig 0. Tudjuk, hogy I eggyel több
embert ismer, mint N, tehát mindkét esetben n = 3 adódik. Ez viszont ellentétben
áll az n > 3 feltétellel, s így a feladat állítását igazoltuk.
Megjegyzések. Az állítás n = 3-ra nem igaz: ha E, I és N közül csak E és I
ismerik egymást, akkor a feltételek teljesülnek.
Az állítás érvényben marad akkor is, ha az ismeretségekről nem tesszük fel, hogy
kölcsönösek.
1951. A válasz igen. 1. megoldás. Teljes indukció.
2. megoldás. Válasszuk ki a házaspárok legnagyobb csoportját, ahol a férjek is-
merik egymást. A többi házaspár megy a másik szobába. Utóbbi szobából vá-
lasztva két házaspárt, és az előbbiből egy harmadik párt; a férjek nem ismerik
egymást, így az asszonyok ismerősök.
1952. Teljes indukcióval bizonyítunk. n = 2-re könnyű. 2n + 2 személy esetén vá-
lasszuk ki közülük A-t és B-t úgy, hogy egymást nem ismerik. Ha van két közös
ismerősük, akkor ez a 4 személy jó. Ellenkező esetben hagyjuk el a 2n + 2 fős
társaságból A-t és B-t, s alkalmazhatjuk az indukciós feltevést.
1953. Ha mindenkinek legalább 4 ismerőse van, akkor igaz az állítás. Ha van olyan A,
akinek legfeljebb 3 ismerőse van, akkor ő a társaság legalább 5 tagját nem ismeri.
Ezek viszont mind ismerik egymást (közülük bármelyik kettőhöz válasszuk A-t,
s alkalmazzuk a feladat feltételét).
Megjegyzés. Az állítást úgy erősítve, hogy az 5 fő ismeri egymást — már ez nem
lesz igaz. Ellenpélda a következő: állítsunk 9 főt körbe, s csak a szomszédok nem
ismerik egymást.
1954. Ha van olyan, aki legalább 4 másikat nem ismer, akkor ez utóbbiak közül bár-
mely kettőnek ismernie kell egymást; azaz máris találtunk 4 megfelelő embert.
33. Gráfok 365

Így feltehetjük, hogy mindenkinek legalább 5 ismerőse van. De nem lehet min-
5·9
denkinek pontosan 5 ismerőse, mert ez ismeretséget jelentene. Kell tehát 6
2
ismerőssel rendelkezőnek is lennie; legyen ez A, és A-nak ismerőse B. Ha B az
A ismerősei közül legalább 3-at ismer, akkor ezek közül kettő A-val és B-vel jó
négyes. Ha nem, akkor B az A ismerősei közül legalább hármat nem ismer. Ez
a három és A jó négyes.
Megjegyzés. Teljes indukcióval
 könnyen be lehet bizonyítani a következő tételt.
n+1
Ha egy szobában ember van, és bármely három között van kettő, akik
2
ismerik egymást; akkor van a szobában n ember, akik közül bármely
 kettő ismeri
n+1
egymást. Jelöljük f (n)-nel azt a legkisebb egészt, amit helyébe téve még
2
mindig igaz marad az állítás. Könnyű ellenőrizni, hogy f (2) = 3, f (3) = 6 és
f (4) = 9. A feladatban f (4) ≤ 9-et kellett bizonyítani.
1956. Eljárást adunk arra, hogyan lehet a társaságból legalább k + 1 személyt leültetni
egy kör alakú asztal köré úgy, hogy mindegyikük ismerje a szomszédait.
Válasszunk ki valakit a társaságból, és jelöljük 1-gyel. Ezután jelöljük (i +1)-gyel
az i-vel jelölt tag egyik, eddig még nem számozott ismerősét, majd ismételjük
meg ezt a műveletet addig, amíg el nem akadunk.
Legyen a legutolsó sorszám n, az n jelű személy legkisebb sorszámú ismerősének
sorszáma m. Ekkor m, m + 1, m + 2, . . . , n leültethető egy kör alakú asztalhoz
úgy, hogy mindenki ismerje a szomszédjait. Másrészt n minden ismerősének van
sorszáma, hiszen n-nél akadtunk el, és ezek mind az asztalnál ülnek. Így legalább
k + 1 személyt ültettünk le, amivel a feladat állításánál többet is bizonyítottunk.
1957. Tekintsünk egy G gráfot, melynek csúcsai a városok, élei pedig kétféle színű-
ek. A kék él jelentse a hajóutat, a piros pedig a repülőutat a két város között.
Mivel bármely két csúcs össze van kötve, ezért G teljes gráf, azaz a kék élek ál-
tal meghatározott részgráfjának komplementere a piros élek által meghatározott
részgráf. Mivel egy gráf és komplementere közül legalább az egyik összefüggő,
ezért az országot vagy hajóval, vagy repülővel be lehet járni.
1958. Legfeljebb 100 kézfogás lehetséges.
1959. Egy ember legfeljebb hat másikkal fogott kezet, hiszen a társával, ill. saját magá-
val nem fog kezet. Emiatt Barna úr a kérdésére úgy kaphat különböző válaszokat,
ha a válaszok rendre: 0, 1, 2, 3, 4, 5, 6. Jelöljük A-val azt az embert, aki 6 másik-
kal fogott kezet! Ez azt jelenti, hogy csak házastársával nem fogott kezet. Tehát
aki a 0-t mondta, az nem lehet más, mint a házastársa. (Mivel Barna úr nem
válaszolt a kérdésre, A nem lehet Barna úr!) Ezután ezt a házaspárt küldjük át a
szomszéd szobába, és újra tegyük fel a kérdést. Ekkor a 0, 1, 2, 3, 4 válaszokat
kapjuk. Ismét látható, hogy a 0-t és 4-et válaszolók házaspárok. Őket átküldjük
366 Megoldások, útmutatások

a szomszédba. Az újabb válaszok 0, 1, 2. A 0-t és 2-t válaszolók házastársak, az


1-et válaszoló Barnáné. Így Barnáné három jelenlevővel fogott kezet.
1960. A vendégségben jelen levő 10 ember közül mindenki ismerte házastársát, ezért
nem mutatkozott be neki, és önmagának sem. Így mindenki maximálisan 8 új
ismeretséget köthetett. Mivel a feladat szerint a 10 jelenlevő 9 különböző isme-
retséget kötött, ezért volt olyan, aki 8, aki 7, 6, 5, 4, 3, 2, 1, 0 ismeretséget kötött.
Kovács kézfogásainak száma természetesen ezek közül valamelyikkel megegye-
zett, hiszen őrá is érvényes, amit az első mondatban mondtunk.
A 8 ismeretséget kötő házastársa a 0 ismeretséget kötő volt, hiszen a többiek
biztosan megismerkedtek vele.
Hasonlóan láthatjuk be lépésről lépésre, hogy a 7, illetve 1; a 6, illetve 2; és
az 5, illetve 3 ismeretséget kötők házastársak. Például a 7 ismeretséget kötő
önmagán és az x emberrel megismerkedő házastársán kívül a 0 ismeretséget
kötőnek nem mutatkozott be, tehát a fennmaradó 7 ember mindegyikét most
kellett megismernie. Így a 8 − 0 és 7 − x páron kívül mindenki legalább két
emberrel megismerkedett. Emiatt x csak 1-gyel lehet egyenlő.
A megmaradó kilencedik jelenlevő, Kovácsné már csak négy ismeretséget köt-
hetett.

34. Halmazrendszerek
 
n
1961. Az állítás igazolható teljes indukcióval, vagy úgy is, hogy felhasználjuk az +
      0
n n n
+ + + ··· + = 2n összefüggést. Elegáns útja a bizonyításnak a
1 2 n
következő: ha az n-elemű halmaz {a1 , a2 , . . . , an }, akkor ennek egy részhalmazát
egyértelműen kijelöli egy n hosszúságú 0 − 1 sorozat. Ha a sorozatban a k-adik
helyen 1-es áll, akkor ak eleme a részhalmaznak; ha pedig 0 áll a k-adik helyen,
akkor nem. Így a részhalmazok száma ugyanannyi, mint az n hosszúságú 0 − 1
sorozatok száma: 2n .
1962. Ha egy részhalmaz a kiválasztottak között van, akkor a komplementere nem lehet
ezek között. Így a kiválasztott részhalmazok száma legfeljebb fele az összes
részhalmaz számának, azaz legfeljebb 2n−1 . Ez a határ el is érhető: valamely
x ∈ H esetén minden Ai olyan, hogy x ∈ Ai .
1963. Soroljuk X részhalmazait párokba úgy, hogy minden részhalmaz a komplemen-
terével legyen egy párban. Mivel a párok száma 2n−1 > m, ezért van olyan pár,
amelynek egyik tagja sem szerepel az Ai halmazok között. Legyen egy ilyen pár
két tagja B és C. Ha az Ai halmazok mindegyikének van közös eleme B-vel, ak-
kor a B halmaz megfelelő. Feltehetjük tehát, hogy például A1 a B-vel diszjunkt,
s így részhalmaza C-nek.
34. Halmazrendszerek 367

Ebben az esetben a C halmaz lesz megfelelő. Valóban, minden egyes Ai hal-


maznak van közös eleme A1 -gyel, ezért az A1 -et tartalmazó C-vel is.
1964. Legyen S páros számainak halmaza E, a páratlan számok halmaza O. Minden
Ai -ből két halmazt készítünk: Ei = Ai ∩E, Oi = Ai ∩O. (Az Ei halmazok között
lehetnek megegyezők, s ugyanígy az Oi halmazok között is.) Ekkor Ei ∩ Ej = ∅
1
és Oi ∩ Oj = ∅. Így az Ei halmazok száma ≤ 2|E| , az Oi halmazok száma
2
1 |O|
≤ 2 . Az Ei és Oi halmazokból tudjuk felépíteni az Ai halmazok rendszerét,
2
1 1 1
ezért m ≤ 2|E| · 2|O| = 2|E|+|O| = 2n−2 .
2 2 4
1966. Ai -hez rendeljünk egy tetszőleges xi ∈ H \ Ai elemet. Ha Ai  = Aj , akkor xi  = xj ;
hiszen az előbbi xi minden Aj -nek eleme kell hogy legyen, ahol j  = i. Az így
kiválasztott xi -k száma legfeljebb n, ezért m ≤ n.
Megjegyzés. Vizsgálhatnánk a következő kérdést:
A1 , A2 , . . . , Am ⊂ H , |H | = n, és Ai ∪ Aj ∪ Ak = H , ha i = j = k. Ekkor
mekkora lehet m legnagyobb értéke?
1967. A feltétel miatt mindegyik Ai -nek van olyan eleme, amely a többi Aj halmaznak
nem eleme. Ezért az Ai halmazok száma legfeljebb n.
1968. Általában igaz, hogy n elemű halmaz esetén a feladatbeli részhalmazok száma
legfeljebb 2n−1. Ezt teljes indukcióval igazoljuk. Legyen m elemű a legnagyobb
kiválasztott M halmaz, amely nem az alaphalmaz. Ekkor bármely kiválasztott
halmaz M-nek vagy komplementerének egy részhalmaza, vagy maga a teljes hal-
maz. Az indukciós feltevésből ezek száma legfeljebb (2m − 1)+(2(n − m) − 1)+
+ 1 = 2n − 1.
Legyen a 100 elemű halmaz az 1, 2, . . . , 100 számok halmaza. Válasszuk ki
összes egyelemű részhalmazát, valamint az {1, 2, . . . , n} alakú halmazokat, ahol
n = 2, 3, . . . , 100. Az így kiválasztott 199 részhalmaz teljesíti a feladat feltételeit.
Tehát a feladat kérdésére a válasz: 199.
1969. Ha valamely i-re |Ai | > k + 1, akkor Ai helyett vegyük annak egy k + 1 elemű
Ai részhalmazát, s ha |Ai | ≤ k + 1, akkor legyen Ai = Ai . Ha Ai = Aj , akkor
Ai = Aj , hiszen |Ai ∩ Aj | ≤ k. Az így kapott A1 , A2 , . . . , Am rendszer halmazai
k+1  
 n
legfeljebb k + 1 eleműek, ezért m ≤ .
i=0
i
Második bizonyítás. Legyen g(n, k) az állításban szereplő m maximális értéke
adott n és k mellett. Ekkor teljesül a g(n+1, k +1) ≤ g(n, k +1)+g(n, k) rekurzió.
k+1  
 n
Ezt felhasználva teljes indukcióval bizonyítható g(n, k) ≤ . n = 1, n = 2
i=0
i
esetén könnyen látható az egyenlőtlenség. Az indukciós lépésben a rekurziót is
368 Megoldások, útmutatások

felhasználva
k+2  
 k+1  
   k+2    
n n n+1 n n
g(n + 1, k + 1) ≤ + = + + =
i=0
i i=0
i 0 i=1
i i−1
k+2  
n+1
= .
i=0
i
Az {Ai : Ai ⊂ H, |H | = n, |Ai | ≤ k + 1} halmazrendszer mutatja, hogy az
állításban szereplő felső korlát pontos.
1970. |A1 ∩ A2 ∩ . . . ∩ Ak | ≤ |Ai |.
 
k
1971. Az Ai ∩ Aj 1-elemű halmazok, mind különbözők, számuk ; ezért
2
k(k − 1)
|A1 ∪ A2 ∪ . . . ∪ Ak | ≥ .
2
1972. A feladat egy más megfogalmazása:
Egy n lakosú városban klubokat szerveznek úgy, hogy bármely két klubnak le-
gyen, és bármelyik háromnak már ne legyen közös tagja. Legfeljebb hány klubot
lehet így szervezni?
Ennek megoldása:
Legyen a klubok száma m. A feltétel második része azt jelenti, hogy egy ember
legfeljebb két klubba járhat. Rendeljük hozzá minden klub-párhoz valamelyik
közös tagjukat. (A feltétel első fele szerint
  van közös tagjuk.) Mivel egy ember
m
nem járhat kettőnél több klubba, az kiválasztott ember mind különböző.
 2
m
Mivel a városban n ember lakik, így ≤ n.
  2
m
Megfordítva, ha ≤ n teljesül, akkor lehetséges az m darab klub megszerve-
  2  
m m
zése: embert szétosztunk a klub-párok között, a maradék n − embert
2 2
pedig például egy klubba sem járatjuk. Ezzel elérjük, hogy bármely két klubnak
legyen pontosan egy, de semelyik háromnak nelegyen közös tagja.
m m(m − 1)
Keressük a legnagyobb m értéket, amelyre ≤ n. ≤ n,
2 2
1
2 1 1 1
m2 − m ≤ 2n, m − ≤ 2n + , m ≤ + 2n + . A legnagyobb ilyen m,
1 
2 √ 4 2 4
1   1 + 8n + 1 
az m = + 2n + = .
2 4 2
1973. (Megjegyzés. A feladat azonos a következővel: Egy 9 fős társaságban mindenki-
nek van (legalább) 7 ismerőse. Mutassuk meg, hogy ekkor bármely 4 személynek
van közös ismerőse.)
34. Halmazrendszerek 369

A négy részhalmaz a 9 elemű halmaznak összesen 4 · 7 = 28 elemét reprezen-


tálja (némelyiket többször). Nem lehet, hogy mindegyik elem legfeljebb 3-szor
szerepeljen, mivel 9 · 3 < 28.
1974. (2n+1 − 1)-féleképpen választhatunk ki néhány halmazt, és mindegyik esetben
vehetjük ezek unióját. Az uniónak 2n − 1 lehetséges értéke van. A skatulyaelv
egyszerű alkalmazásával adódik az állítás.
1977. Bármely két halmaznak egy közös eleme van. Lássuk be, hogy ez a közös elem
mindig ugyanaz. A1 -nek van olyan x eleme, melyben 45 másik halmaz metszi,
ugyanis 1 + 44 · 45 = 1981 < 1985. Ha van olyan Ai halmaz, mely x-et nem
tartalmazza, az előbbi 46 halmaz mindegyikét más-más elemben metszi. Ez azt
jelentené, hogy |Ai | ≥ 46. Ezért a feladat kérdésére a válasz: 1985 · 44 + 1.
1980. Ha található a tagok között olyan, aki legalább 7 ülésen volt jelen, akkor az
ezeken részt vevő további 7 · 9 ember a feltételek miatt szükségképpen mind
különböző. Ez viszont legalább 7 · 9 + 1 = 64 tagot jelent. Ha pedig mindenki
legfeljebb 6 ülésen vett részt, akkor az ülések összlétszáma csak úgy lehetett 400,
400
ha legalább > 66 tagja volt a bizottságnak.
6
1981. Az osztály minden tanulója szakköri tag, hiszen mindegyikük még közös szak-
körbe is jár valaki mással. Ugyanezért, ha valaki csak egy szakkörbe jár, akkor
ennek a szakkörnek mindenki tagja. Ezért feltehetjük, hogy mindenki legalább
két szakkörbe jár, azaz — a második feltétel miatt — mindenki pontosan két
szakkörbe jár.
Tekintsünk valakit az osztályból, s legyen S1 és S2 az a két szakkör, amelynek
ő tagja. Ha S1 -ben vagy S2 -ben ott van az egész osztály, akkor készen vagyunk.
Különben van olyan g1 és g2 gyerek, hogy g1 ∈ S1 , g1 ∈ S2 és g2 ∈ S2 , g2 ∈ S1 .
Legyen S az a szakkör, ahová g1 és g2 is jár.
Ha egy g gyerek jár S1 -be is, S2 -be is, akkor más szakkörbe nem jár. Ha g ∈ S1 ,
g1 ∈ S2 , akkor g ∈ S, hiszen a g2 gyerekkel csak itt járhat közös szakkörbe.
Ugyanígy; ha g ∈ S2 , g2 ∈ S1 , akkor g ∈ S, hiszen a g1 gyerekkel csak itt
járhat közös szakkörbe. Tehát az S, S1 , S2 szakkörökben az osztály mindegyik
diákja szerepel, s mindenki kétszer. Emiatt valamelyik szakkörbe az osztálynak
legalább a kétharmada jár.
1982. Két esetet vizsgálunk. Ha bármely két tudós beszél közös nyelvet, akkor bár-
melyikük 8 másikkal beszél; és mivel legfeljebb 3 nyelven tud, ezért található a
többiek között legalább kettő (sőt három is), akikkel azonos nyelven beszél.
Ha van két tudós, A és B, akik nem beszélnek közös nyelven; akkor válasszuk
harmadiknak sorba a többi hét tudóst. Ezek mindegyike a feltétel szerint vagy
A-val, vagy B-vel beszél közös nyelvet. A és B együttvéve legfeljebb 2 · 3 =
= 6 nyelven beszél, ezért van legalább két tudós, akik azonos nyelven beszélnek
mondjuk A-val.
370 Megoldások, útmutatások

1983. A négy pénztáros közül bármely kettőt választjuk is ki, lesz olyan zár a páncél-
szekrényen, melyhez nekik nincs kulcsuk; de bármely más pénztárosnak már van
(hiszen ketten nem tudják kinyitni, de bármely három pénztáros már igen). Ezért
a zárak száma legalább annyi, ahányféleképp a négy pénztáros közül kettőt ki
tudunk választani, azaz legalább 6.
Ennyi kulccsal megoldható, hogy teljesüljenek a feltételek. Legyenek a pénztá-
rosok A, B, C és D; a kulcsok sorszámai 1., 2.,
1. 2. 3. 4. 5. 6.
3., 4., 5. és 6. legyenek. 6-nál több zár esetén is
elvégezhető a kívánt módon a kulcsok megfelelő A + + +
szétosztása: az előbbi módon osztjuk szét az első B + + +
6 zár kulcsait, a többi zárhoz pedig mindenkinek C + + +
adjunk kulcsot. D + + +

1984. A bizottság bármelyik két tagja a többi három távollétében nem nyithatja ki a
szekrényt; ezért bármelyik tag-párhoz található legalább egy olyan zár, amelyhez
egyiküknek sincs kulcsa. Egy másik tag-párhoz nem tartozhat ugyanez a zár,
amit ők sem tudnak kinyitni. Ezért legalább annyi zár kell, ahányféleképp a
bizottságból két embert kiválaszthatunk, azaz legalább 10 zárra van szükség.
Mindegyik bizottsági tagot legalább annyi kulccsal kell ellátni, ahány különböző
tag-pár képezhető a többi négy tagból — vagyis legalább 6 kulccsal —, mert
bármelyik tagnál kell kulcsnak lennie a többi négyből képezhető 6 párhoz tar-
tozó 6 különböző nyithatatlan zár
mindegyikéhez. 1. 2. 3. 4. 5. 6. 7. 8. 9. 10.
10 zár elegendő a feltétel teljesíté- A - - - - + + + + + +
séhez. Legyenek a bizottság tagjai B - + + + - - - + + +
A, B, C, D és E; a kulcsok sorszá- C + - + + - + + - - +
mai 1., 2., 3., . . . , 10. legyenek. D + + - + + - + - + -
E + + + - + + - + - -
 
10
1985. A zárak száma legalább = 120, s ennyi zár elegendő is.
3
1986. Először megmutatjuk, hogy bármely kijelölt R részhalmaznak pontosan m eleme
van. Mivel R valódi részhalmaz, azért létezik benne nem lévő p elem. Minden
q ∈ R elemnek megfeleltetve az a) szerint egyértelműen létező, p-t és q-t tar-
talmazó S kijelölt részhalmazt, kölcsönösen egyértelmű megfeleltetést kapunk R
elemei és a p-t tartalmazó, R-t metsző kijelölt részhalmazok közt. Tehát minden
kijelölt részhalmaz m elemet tartalmaz.
A b) feltételből következik, hogy minden p elemet pontosan m+n kijelölt részhal-
maz tartalmaz. Rögzítsük most H -nak egy r elemét. Az r-től különböző elemek
mindegyike — a) miatt — pontosan egy, r-t tartalmazó részhalmazhoz tartozik
hozzá. Az r-t m + n részhalmaz tartalmazza, és ezek mindegyikében m − 1 darab
r-től különböző elem van; ezért H -nak összesen h = 1 + (m + n)(m − 1) eleme
van.
34. Halmazrendszerek 371

A kijelölt részhalmazok száma legyen k. Számoljuk össze azokat a (p, R) pá-


rokat, ahol p a H -nak egy eleme, R egy kijelölt részhalmaz, és p ∈ R. Mivel
h elemünk van, és mindegyik m + n részhalmazban van benne, ezért az ilyen
párok száma h(m + n). A számolást viszont úgy is végezhetjük, hogy a k kijelölt
részhalmaz mindegyike m elemet tartalmaz, ezért a párok száma k · m. A kétféle
számolás eredményének meg kell egyeznie, ezért
h(m + n) [1 + (m + n)(m − 1)](m + n)
k · m = h(m + n), ahonnan k = = .
m m
Megjegyzés. Érdekes kérdés, hogy léteznek-e a feltételeknek eleget tevő H hal-
mazok. A kérdésre pontos, teljes válasz nem ismert. Ha m = n = 2, akkor a
megfelelő H halmaz egy 5-elemű halmaz; a kijelölt részhalmazok pedig ennek
az összes 2-elemű részei.
1987. Legyen adva k darab halmaz. Tegyük fel, hogy már kiválasztottuk az A1 , A2 ,
. . . , An−1 halmazokat (1 ≤ n ≤ k). An -nek válasszunk egy minimálisat a mara-
dékból, azaz olyat, amelynek a még fennmaradó halmazok egyike sem részhal-
maza; ezt megtehetjük, hiszen összesen is csak véges sok halmazunk van. Az An
kiválasztása után hagyjuk el a maradékból természetesen az An -et, valamint az
A1 ∪ An , A2 ∪ An , . . . , An−1 ∪ An közül azokat, amelyek eredetileg ott voltak.
Az így megmaradó halmazok közül válasszuk ki hasonlóan An+1 -et stb. Tetsző-
leges Ai kiválasztásával így összesen legfeljebb 1 + (i − 1) = i darab halmazt
„fogyasztottunk el” a készletből, tehát az első n darab halmaz kiválasztása során
n(n + 1)
összesen legfeljebb 1+2+· · ·+n = darabot. Ez azt jelenti, hogy biztosan
2
n(n + 1)
kiválasztható ezen a módon n darab halmaz, ha k > .
2
Az így kiválasztott halmazok eleget tesznek a feladat követelményének: tételez-
zük fel ugyanis, hogy létezne olyan Ai , Aj , Al , melyre Al = Ai ∪ Aj és például
i < j . Mivel ekkor Ai és Aj is részhalmaza Al -nek, azért i, j < l, azaz Al -et az
(Ai és) Aj megtalálása után választhattuk csak ki. Ez azonban lehetetlen, mivel
Aj kiválasztásakor a maradékból kiselejteztük (Ai ∪ Aj )-t.
1989. Egy n-elemű halmaznak összesen 2n részhalmaza van, ezért a részhalmaz-párok
száma 2n · 2n = 4n . A részhalmazpárokat rendezzük 4-es csoportokba: (Ai , Aj ),
(Ai , Aj ), (Ai , Aj ), (Ai , Aj ). Ezeknek a csoportoknak a száma: 4n /4 = 4n−1 . Egy
ilyen csoportban a metszetek elemszámának összege n. Emiatt a kérdéses összeg:
n · 4n−1 .
1990. Álljon H azokból a K részhalmazokból, amelyekre K ⊇ f (K). Ez a H nem üres,
mert A eleme, hiszen A ⊇ f (A) biztosan teljesül. Legyen a H -beli halmazok
metszete az M halmaz. Mit tudunk az f (M)-ről?
Ha K tetszőleges H -beli halmaz, akkor K ⊇ M miatt fönnáll f (K) ⊇ f (M). Eb-
ből pedig K ⊇ f (K) alapján (ez volt a H -beli halmazok definiáló tulajdonsága)
K ⊇ f (M). Tehát f (M)-et minden H -beli halmaz tartalmazza, így metszetük,
372 Megoldások, útmutatások

M is: M ⊇ f (M). Ugyanakkor M ⊇ f (M)-ből f (M) ⊇ f (f (M)) következik,


tehát (definíció szerint) f (M) H -beli. Az M minden, H -beli halmaznak része,
tehát M ⊆ f (M). Ezt az előbbi M ⊇ f (M) eredményünkkel összevetve M =
= f (M); ami azt jelenti, hogy megtaláltuk a keresett részhalmazt.
1991. Legyen az n-elemű halmaz {1, 2, 3, . . . , n}. Tekintsük a halmaz egy partícióját,
és minden részhalmazban az elemeket rendezzük nagyság szerint csökkenő sor-
rendbe, majd ezeket a sorozatokat első elemük szerint növekedve írjuk egymás
után. Például, ha n = 6 és az egyik partíció {1, 5}, {2, 3}, {4, 6}, akkor ehhez a
325164 sorozat tartozik. Ily módon megadjuk az n elem egy permutációját, kü-
lönböző partíciókhoz különböző permutációkat. Van olyan permutáció is, mely-
hez nem tartozik partíció, pl.: 326451.
Megjegyzés. Az n-elemű halmaz partícióinak számát a Tn Bell-féle szám jelöli.
1992. Belátjuk, hogy f (n + 2) = f (n + 1) + f (n). Ehhez tekintsük az Mn+2 -nek azokat
a kövér részhalmazait, melyek Mn+1 -nek nem kövér részhalmazai, azaz azokat,
amelyek az n + 2 elemet tartalmazzák. Ezeknek a halmazoknak a száma f (n +
+ 2) − f (n + 1). Egy ilyen halmaznak az 1 nyilván nem lehet eleme, mivel akkor
nem lenne kövér. Képezzük most egy ilyen T halmazhoz a T  halmazt úgy, hogy
előbb minden elemből levonunk 1-et, majd ezután töröljük (n + 1)-et. Ekkor T 
kövér részhalmaza Mn -nek, továbbá a T → T  leképezés megfordítható, így
f (n + 2) − f (n + 1) = f (n).
1993. A Bs ⊃ Bs−1 ⊃ . . . ⊃ B2 ⊃ B1 sorozatot láncnak nevezzük. A H halmaz
n hosszúságú láncainak száma n!. Ha |Ai | = ki , akkor Ai ki ! · (n − ki )! db n
hosszúságú láncban szerepel, s ha i = j , akkor Ai és Aj nem lehet ugyanabban
a láncban, tehát


m n  n 

n! ≥ ki !(n − ki )! ≥ m · !· n− !
i=1
2 2
 
n
azaz m ≤ .
[ n2 ]
 
n
Megjegyzés. Az m = érték elérhető, ha vesszük az n elemű H halmaz
[ n2 ]
n!
összes elemű részhalmazát.
2
1994. A H halmaz elemeit helyezzük el valamilyen sorrendben egy n oldalú szabályos
sokszög csúcsaiba. Számoljuk meg, hogy egy ilyen permutációra hány db Ai
jelenik meg úgy, hogy elemei egymást követik a sokszög csúcsaiban. k < n/2 és
Ai ∩Aj = ∅, |Ai | = k miatt könnyű látni, hogy legfeljebb k darab. Így leszámlálva
Ai -ket multiplicitásukkal együtt, ez a szám M ≤ k · n!.
Ezt az M számot másképp megszámlálva egy Ai -t hányszor számolhatunk? Adott
k egymást követő helyre Ai elemeit k! módon, a megmaradó n − k helyre az
34. Halmazrendszerek 373

elemeket (n − k)!-féleképpen írhatjuk. Ezt a k helyet n módon jelölhetjük ki.


Ezért M = m · n · k!(n
 − k)!.
 
k n n−1
Eddigiekből m ≤ = .
n k k−1
Látható, hogy az egyenlőtlenség éles: valamely x ∈ H elemet rögzítve a H \ x
halmaz k−1 elemű részeihez hozzávéve az x elemet,
 egy megfelelő tulajdonságú

n−1
halmazrendszert kapunk; s a halmazrendszer halmazból áll.
k−1
1995. Erősebb állítást bizonyítunk:
[Bollobás, 1964] Ha A1 , A2 , . . . , Am és B1 , B2 , . . . , Bm olyan, hogy
m
1
Ai ∩ Bi = ∅ és Ai ∩ Bj = ∅, ha i = j ; akkor |Ai |+|Bi | ≤ 1.
i=1 |Ai |
Következmény. Ha A1 , A2 , . . . , Am és B1 , B2 , . . . , Bm olyan,  hogy  |Ai | ≤ a,
a+b
|Bi | ≤ b és a tétel metszetfeltételei is teljesülnek, akkor m ≤ .
b
Legyen X = (∪i=1 Ai ) ∪ (∪i=1 Bi ), |Ai | ≤ ai , |Bi | ≤ bi , |X| = n. Tekintsük X egy
m m

π permutációját. Azt mondjuk, hogy ez i-típusú, ha Ai minden eleme megelőzi


Bi elemeit. Belátjuk, hogy π legfeljebb egy típusú lehet. Tegyük fel, hogy π i-
és j -típusú. Legyen xi , ill. xj a legnagyobb eleme π-ben Ai -nek, ill. Aj -nek. Ha
xi ≤ xj , akkor Ai ∩ Bj = ∅, ami ellentmondás.
Számoljuk
  meg, hogy hány permutáció i-típusú. Ez a szám
n n!
· (n − ai − bi )! · ai ! · bi ! = ai +bi  . Ezeket összegezve a kívánt állítást
ai + bi ai
kapjuk.
1996. Képzeljük el, hogy sikerült megadni 16 megfelelő H1 , H2 , . . . , H16 részhal-
mazt. Ekkor az első 10 000 természetes szám bármelyike ezek közül pontosan
nyolcnak eleme. Valamennyi i ≤ 10 000 pozitív egészre jegyezzük fel az i szá-
mot tartalmazó halmazok sorszámát; megjegyzésünk szerint ezzel a 16-elemű
I = {1, 2, . . . , 16} halmaznak egy nyolcelemű Ci részhalmazát kapjuk, minden
i-re mást és mást.
A feladat állításának igazolásához először ilyen Ci -ket állítunk elő úgy, hogy
tetszés szerint kiválasztjuk I -nek 10 000 darab,
 páronként
 különböző nyolcelemű
16
részhalmazát. (Ez megtehető, mert 10 000 < = 12 870.) Definiáljuk ezután
8
a Hj halmazt mindazoknak a k számoknak az összességeként, amelyekre Ck


tartalmazza j -t. Mivel Ck -nek nyolc eleme van, ezért mindegyik k pontosan
nyolc Hj -ben van benne. Ha pedig k ∗ = k, akkor Ck ∗ = Ck miatt k ∗ és k nem
ugyanabban a nyolc Hj halmazban van, tehát a k-t tartalmazó nyolc halmaz
egyetlen közös eleme a k.
1997. Legyen H = {x1 , x2 , . . . , xn }, s legyen Xi = {xi }, i = 1, 2, . . . , n. Tegyük fel,
hogy A1 , A2 , . . . , Am független metszőrendszer. Megmutatjuk, hogy például X1
374 Megoldások, útmutatások

előállítható az A1 , A2 , . . . , Am halmazokból néhánynak a metszeteként. (Felhasz-


náljuk közben a de Morgan-azonosságokat: A ∪ B = A ∩ B és A ∩ B = A ∪ B.)
     
X1 = H \ {X2 X3 ··· Xn } = X2 X3 ··· Xn =
" " " " " " " " "
= X2 X3 ··· Xn = ( Ai ) ( Ai ) ··· ( Ai ) =
 " " " i∈I2  
i∈I3 " "i∈In "
= ( Ai ) ( Ai ) · · · ( Ai ) = (Ai2 A i3 ··· A in )
i∈I2 i∈I3 i∈In
Mivel X1 egyetlen elemből álló halmazt jelentett, így az unió tagjainak valame-
lyike egyenlő X1 -gyel.
Látható, hogy az A1 , A2 , . . . , Am és az A1 , A2 , . . . , Am rendszerek egyszerre ren-
delkeznek az (1) tulajdonsággal, és emiatt a (2) tulajdonsággal is.
"
1998. Xk = Ai , |Xk | = 1, k = 1, 2, . . . , n. Az I1 , I2 , . . . , In ⊆ {1, 2, . . . , m} rendszer
i∈Ik
Sperner-rendszer (egyik halmaz
 sem
 tartalmazza részként valamely másikat), így
m
a Sperner-lemma miatt n ≤ m ! , azaz m ≥ c1 · log2 n. Megmutatjuk, hogy az
2 
m
alsó korlát pontos; vagyis, ha m ! ≥ n, akkor van egy legfeljebb m halmazból
2
álló A1 , A2 , . . . , Am független
 metszőrendszer az n elemű H halmazon.
m
Tekintsük az A m × m ! -es 0 − 1 mátrixot, melynek oszlopai az {1, 2, . . . , m}
m 2
halmaz egy-egy -elemű részét reprezentálják. Az A mátrix soraival megadott
2  
m
m db halmazból néhányat választva az m
! elemű halmaz bármely elemét
2
kimetszhetjük.
A felső korlát igazolása. Legyen x ∈ H , s tekintsük azokat az Ai -ket, melyek
szükségesek az (1) és (2) szerinti előállításhoz. Ai -kből hagyjuk el az x elemet.
Az így kapott halmazokat jelölje Ai . Ezeknek az Ai -knek a száma s. Most bár-
mely s−1 db Ai metszete = ∅, míg az összes Ai metszete = ∅. Így minden Ai -hez
hozzárendelhetünk egy olyan elemet a H \ {x} halmazból, mely a többi Aj -nek
nem eleme. Ezért s ≤ n − 1. Így leszámolva az A1 , A2 , . . . , Am halmazokat, azt
kapjuk, hogy m ≤ n(n − 1).
Megadunk olyan független metszőrendszert  n  a H = {x1 , x2 , . . . , xn } halmazon,
mely c2 · n2 halmazból áll. Legyen r = , R = {x1 , x2 , . . . , xr }, Ai = R \ {xi },
 2
i = 1, 2, . . . , r; Ai,j = {xi } Aj , i = r + 1, r + 2, . . . , n, j = 1, 2, . . . , r.
Az Ai,j halmazok független metszőrendszert alkotnak, s a halmazok száma több,
1
mint n2 − 1.
4
1999. Legyen M egy maximális elemszámú halmaz, mely még egyik Ai -t sem tartal-
mazza. |M| = r.
34. Halmazrendszerek 375

A maximalitásból adódik, hogy minden x ∈ H \ M-hez van Ax ⊂ M, |Ax | = 2,


hogy x ∪ Ax = Ai valamely i-re.
Ha x = y, akkor Ax = Ay , különben a metszetfeltétel
 nem teljesül. Így n − r ≤
 
2
r 1 1 1 1
≤ , r 2 + r ≥ 2n, r + ≥ 2n + , r + ≥ 2n + ; mivel r egész szám,
2 √ 2 4 2 4
így r ≥ [ 2n].
2000. Jelölje φx mindazoknak az Ai halmazoknak a számát, melyek az x elemet tartal-
mazzák. Nyilvánvaló, hogy
 
n+1 
φx = |Ai ∩ Aj | = n + |Ai ∩ Aj |,
x∈Aj i=1 i=j
innen

φx ≤ n + nk = n(k + 1) (1)
x∈Aj

Összegezzünk minden j -re. Ekkor a bal oldalon



n+1    
φx = φx 1= φx2
j =1 x∈Aj x∈A x∈Aj x∈A
adódik. Ezt a számtani és négyzetes közép közötti egyenlőtlenséggel megbecsül-
ve # 2  n+1 2
 1  n2 (n + 1)2
x∈A φx
φx2 ≥ |A| = |Aj | = .
x∈A
|A| |A| j =1 |A|
Az (1) egyenlőtlenség jobb oldalából az összegezés után azt kapjuk, hogy
n(n + 1)(k + 1).
Tehát
n2 (n + 1)2
≤ n(n + 1)(k + 1).
|A|
Átrendezve:
n(n + 1)
|A| ≥ .
k+1
Mivel az ellenkező irányú egyenlőtlenséget feltettük, itt az egyenlőségnek kell
állnia. Ahhoz, hogy ilyen halmazrendszer létezzen, szükséges természetesen
k + 1 | n(n + 1).
Tartalom

Bevezető . . . . . . . . . . . . . . . . . . . . . . . . . . . . . . . . . . . . . . . . . . . . . . . . . . . . . . . . . . . . . . . . 5
FELADATOK . . . . . . . . . . . . . . . . . . . . . . . . . . . . . . . . . . . . . . . . . . . . . . . . . . . . . . . . . . 7
1. Fejtörők . . . . . . . . . . . . . . . . . . . . . . . . . . . . . . . . . . . . . . . . . . . . . . . . . . . . . . . . . . . . 7
2. Páros vagy páratlan? . . . . . . . . . . . . . . . . . . . . . . . . . . . . . . . . . . . . . . . . . . . . . . . . 19
3. Párbaállítás . . . . . . . . . . . . . . . . . . . . . . . . . . . . . . . . . . . . . . . . . . . . . . . . . . . . . . . . . 20
4. Miért nem négyzetszám? . . . . . . . . . . . . . . . . . . . . . . . . . . . . . . . . . . . . . . . . . . . . 23
5. Négyzetszámok . . . . . . . . . . . . . . . . . . . . . . . . . . . . . . . . . . . . . . . . . . . . . . . . . . . . . 25
6. Két szomszédos egész szám szorzata . . . . . . . . . . . . . . . . . . . . . . . . . . . . . . . . . 28
7. Diofantoszi egyenletek . . . . . . . . . . . . . . . . . . . . . . . . . . . . . . . . . . . . . . . . . . . . . . 29
8. Prímszámok . . . . . . . . . . . . . . . . . . . . . . . . . . . . . . . . . . . . . . . . . . . . . . . . . . . . . . . . 33
9. Oszthatósági feladatok . . . . . . . . . . . . . . . . . . . . . . . . . . . . . . . . . . . . . . . . . . . . . . . 36
10. Különféle számelméleti feladatok . . . . . . . . . . . . . . . . . . . . . . . . . . . . . . . . . . . . . 43
11. Számok reciprokainak összege . . . . . . . . . . . . . . . . . . . . . . . . . . . . . . . . . . . . . . . 50
12. Számok és számjegyek . . . . . . . . . . . . . . . . . . . . . . . . . . . . . . . . . . . . . . . . . . . . . . 52
13. Racionális és irracionális számok . . . . . . . . . . . . . . . . . . . . . . . . . . . . . . . . . . . . . 54
14. Egész együtthatós polinomok . . . . . . . . . . . . . . . . . . . . . . . . . . . . . . . . . . . . . . . . 57
15. Kombinatorika a számelméletben . . . . . . . . . . . . . . . . . . . . . . . . . . . . . . . . . . . . . 58
16. Számkonstrukciók . . . . . . . . . . . . . . . . . . . . . . . . . . . . . . . . . . . . . . . . . . . . . . . . . . . 60
17. Melyik szám a nagyobb? . . . . . . . . . . . . . . . . . . . . . . . . . . . . . . . . . . . . . . . . . . . . 66
18. Egyenletek és egyenletrendszerek . . . . . . . . . . . . . . . . . . . . . . . . . . . . . . . . . . . . 68
19. Egyenlőtlenségek . . . . . . . . . . . . . . . . . . . . . . . . . . . . . . . . . . . . . . . . . . . . . . . . . . . 72
20. Különféle algebrai feladatok . . . . . . . . . . . . . . . . . . . . . . . . . . . . . . . . . . . . . . . . . 81
21. Függvényegyenletek . . . . . . . . . . . . . . . . . . . . . . . . . . . . . . . . . . . . . . . . . . . . . . . . . 90
22. Vektorok a geometriában . . . . . . . . . . . . . . . . . . . . . . . . . . . . . . . . . . . . . . . . . . . . 95
23. Területátalakítások . . . . . . . . . . . . . . . . . . . . . . . . . . . . . . . . . . . . . . . . . . . . . . . . . . 101
24. Geometriai konstrukciók . . . . . . . . . . . . . . . . . . . . . . . . . . . . . . . . . . . . . . . . . . . . . 114
25. Invariáns tulajdonságok . . . . . . . . . . . . . . . . . . . . . . . . . . . . . . . . . . . . . . . . . . . . . . 118
26. Feladatok a sakktáblán . . . . . . . . . . . . . . . . . . . . . . . . . . . . . . . . . . . . . . . . . . . . . . 126
27. Skatulyaelv . . . . . . . . . . . . . . . . . . . . . . . . . . . . . . . . . . . . . . . . . . . . . . . . . . . . . . . . . 132
28. Matematikai játékok . . . . . . . . . . . . . . . . . . . . . . . . . . . . . . . . . . . . . . . . . . . . . . . . . 141
29. Különféle kombinatorikai feladatok . . . . . . . . . . . . . . . . . . . . . . . . . . . . . . . . . . . 149
30. Konstrukciók . . . . . . . . . . . . . . . . . . . . . . . . . . . . . . . . . . . . . . . . . . . . . . . . . . . . . . . 157
31. Teljes indukció . . . . . . . . . . . . . . . . . . . . . . . . . . . . . . . . . . . . . . . . . . . . . . . . . . . . . 159
32. Kombinatorika a geometriában . . . . . . . . . . . . . . . . . . . . . . . . . . . . . . . . . . . . . . . 165
378 Tartalom

33. Gráfok . . . . . . . . . . . . . . . . . . . . . . . . . . . . . . . . . . . . . . . . . . . . . . . . . . . . . . . . . . . . . 170


34. Halmazrendszerek . . . . . . . . . . . . . . . . . . . . . . . . . . . . . . . . . . . . . . . . . . . . . . . . . . . 172

MEGOLDÁSOK . . . . . . . . . . . . . . . . . . . . . . . . . . . . . . . . . . . . . . . . . . . . . . . . . . . . . . . . 177
1. Fejtörők . . . . . . . . . . . . . . . . . . . . . . . . . . . . . . . . . . . . . . . . . . . . . . . . . . . . . . . . . . . . 177
2. Páros vagy páratlan? . . . . . . . . . . . . . . . . . . . . . . . . . . . . . . . . . . . . . . . . . . . . . . . . 186
3. Párbaállítás . . . . . . . . . . . . . . . . . . . . . . . . . . . . . . . . . . . . . . . . . . . . . . . . . . . . . . . . . 187
4. Miért nem négyzetszám? . . . . . . . . . . . . . . . . . . . . . . . . . . . . . . . . . . . . . . . . . . . . 191
5. Négyzetszámok . . . . . . . . . . . . . . . . . . . . . . . . . . . . . . . . . . . . . . . . . . . . . . . . . . . . . 193
6. Két szomszédos egész szám szorzata . . . . . . . . . . . . . . . . . . . . . . . . . . . . . . . . . 197
7. Diofantoszi egyenletek . . . . . . . . . . . . . . . . . . . . . . . . . . . . . . . . . . . . . . . . . . . . . . 198
8. Prímszámok . . . . . . . . . . . . . . . . . . . . . . . . . . . . . . . . . . . . . . . . . . . . . . . . . . . . . . . . 203
9. Oszthatósági feladatok . . . . . . . . . . . . . . . . . . . . . . . . . . . . . . . . . . . . . . . . . . . . . . . 208
10. Különféle számelméleti feladatok . . . . . . . . . . . . . . . . . . . . . . . . . . . . . . . . . . . . . 215
11. Számok reciprokainak összege . . . . . . . . . . . . . . . . . . . . . . . . . . . . . . . . . . . . . . . 223
12. Számok és számjegyek . . . . . . . . . . . . . . . . . . . . . . . . . . . . . . . . . . . . . . . . . . . . . . 225
13. Racionális és irracionális számok . . . . . . . . . . . . . . . . . . . . . . . . . . . . . . . . . . . . . 227
14. Egész együtthatós polinomok . . . . . . . . . . . . . . . . . . . . . . . . . . . . . . . . . . . . . . . . 231
15. Kombinatorika a számelméletben . . . . . . . . . . . . . . . . . . . . . . . . . . . . . . . . . . . . . 233
16. Számkonstrukciók . . . . . . . . . . . . . . . . . . . . . . . . . . . . . . . . . . . . . . . . . . . . . . . . . . . 237
17. Melyik szám a nagyobb? . . . . . . . . . . . . . . . . . . . . . . . . . . . . . . . . . . . . . . . . . . . . 243
18. Egyenletek és egyenletrendszerek . . . . . . . . . . . . . . . . . . . . . . . . . . . . . . . . . . . . 244
19. Egyenlőtlenségek . . . . . . . . . . . . . . . . . . . . . . . . . . . . . . . . . . . . . . . . . . . . . . . . . . . 250
20. Különféle algebrai feladatok . . . . . . . . . . . . . . . . . . . . . . . . . . . . . . . . . . . . . . . . . 260
21. Függvényegyenletek . . . . . . . . . . . . . . . . . . . . . . . . . . . . . . . . . . . . . . . . . . . . . . . . . 269
22. Vektorok a geometriában . . . . . . . . . . . . . . . . . . . . . . . . . . . . . . . . . . . . . . . . . . . . 278
23. Területátalakítások . . . . . . . . . . . . . . . . . . . . . . . . . . . . . . . . . . . . . . . . . . . . . . . . . . 282
24. Geometriai konstrukciók . . . . . . . . . . . . . . . . . . . . . . . . . . . . . . . . . . . . . . . . . . . . . 285
25. Invariáns tulajdonságok . . . . . . . . . . . . . . . . . . . . . . . . . . . . . . . . . . . . . . . . . . . . . . 293
26. Feladatok a sakktáblán . . . . . . . . . . . . . . . . . . . . . . . . . . . . . . . . . . . . . . . . . . . . . . 300
27. Skatulyaelv . . . . . . . . . . . . . . . . . . . . . . . . . . . . . . . . . . . . . . . . . . . . . . . . . . . . . . . . . 313
28. Matematikai játékok . . . . . . . . . . . . . . . . . . . . . . . . . . . . . . . . . . . . . . . . . . . . . . . . . 325
29. Különféle kombinatorikai feladatok . . . . . . . . . . . . . . . . . . . . . . . . . . . . . . . . . . . 334
30. Konstrukciók . . . . . . . . . . . . . . . . . . . . . . . . . . . . . . . . . . . . . . . . . . . . . . . . . . . . . . . 345
31. Teljes indukció . . . . . . . . . . . . . . . . . . . . . . . . . . . . . . . . . . . . . . . . . . . . . . . . . . . . . 348
32. Kombinatorika a geometriában . . . . . . . . . . . . . . . . . . . . . . . . . . . . . . . . . . . . . . . 352
33. Gráfok . . . . . . . . . . . . . . . . . . . . . . . . . . . . . . . . . . . . . . . . . . . . . . . . . . . . . . . . . . . . . 361
34. Halmazrendszerek . . . . . . . . . . . . . . . . . . . . . . . . . . . . . . . . . . . . . . . . . . . . . . . . . . . 366

You might also like